Está en la página 1de 471

M QUINAS E LCTRICAS ROTATIVAS :

Introduccin a la Teora General

Jos Manuel Aller


U NIVERSIDAD S IMN B OLVAR
Departamento de Conversin y Transporte de Energa

M QUINAS E LCTRICAS ROTATIVAS :


I NTRODUCCIN A LA T EORA G ENERAL
Jos Manuel Aller
2006 EDITORIAL EQUINOCCIO
Todas las obras publicadas bajo nuestro sello
han sido sometidas a un proceso de arbitraje.
Valle de Sartenejas, Baruta, Edo. Miranda
Apartado postal 89000, Caracas 1080-A, Venezuela
Telfono (0212)9063160/3162/3164, fax (0212)9063159
Hecho el depsito de ley
Reservados todos los derechos

Coordinacin editorial: Carlos Pacheco


Produccin:
Composicin grfica: Jos Manuel Aller
Correccin:
ISBN 980-237-223-4
Depsito legal LF: 2442004600958

Al profesor Gastn Pesse, quien dedic muchos aos para


ensearnos su visin de las mquinas elctricas
y
a todos aquellos estudiantes que durante tantos aos han
contribuido y enriquecido este libro con sus sugerencias.

ndice general

PARTE I

Fundamentos generales de las mquinas elctricas

C APTULO I
Conversin de energa elctrica
I.1
Conceptos bsicos . . . . . . . . . . . . . .
I.2
Convertidor electromecnico elemental . . .
I.3
Curvas caractersticas . . . . . . . . . . . . .
I.4
Balance energtico . . . . . . . . . . . . . .
I.5
Sumario . . . . . . . . . . . . . . . . . . . .
I.6
Ejemplos resueltos . . . . . . . . . . . . . .
I.7
Ejercicios propuestos . . . . . . . . . . . . .

13

.
.
.
.
.
.
.

15
15
21
25
29
31
31
44

.
.
.
.
.
.
.
.

49
50
59
64
68
70
74
75
80

C APTULO III
Circuitos acoplados magnticamente
III.1
Definiciones bsicas . . . . . . . . . . . . . . . . . . . . . . . . . . . . .
III.2
Ecuaciones de tensin . . . . . . . . . . . . . . . . . . . . . . . . . . . .

87
87
90

C APTULO II
Fundamentos de conversin
II.1
Energa y coenerga en el campo . . . . .
II.2
Balance energtico . . . . . . . . . . . .
II.3
Ecuaciones internas del convertidor . . .
II.4
Ecuaciones de potencia . . . . . . . . .
II.5
Generalizacin de las ecuaciones . . . .
II.6
Sumario . . . . . . . . . . . . . . . . . .
II.7
Ejemplos resueltos . . . . . . . . . . . .
II.8
Ejercicios propuestos . . . . . . . . . . .

.
.
.
.
.
.
.
.

.
.
.
.
.
.
.
.

.
.
.
.
.
.
.

.
.
.
.
.
.
.
.

.
.
.
.
.
.
.

.
.
.
.
.
.
.
.

.
.
.
.
.
.
.

.
.
.
.
.
.
.
.

.
.
.
.
.
.
.

.
.
.
.
.
.
.
.

.
.
.
.
.
.
.

.
.
.
.
.
.
.
.

.
.
.
.
.
.
.

.
.
.
.
.
.
.
.

.
.
.
.
.
.
.

.
.
.
.
.
.
.
.

.
.
.
.
.
.
.

.
.
.
.
.
.
.
.

.
.
.
.
.
.
.

.
.
.
.
.
.
.
.

.
.
.
.
.
.
.

.
.
.
.
.
.
.
.

.
.
.
.
.
.
.

.
.
.
.
.
.
.
.

.
.
.
.
.
.
.

.
.
.
.
.
.
.
.

.
.
.
.
.
.
.

.
.
.
.
.
.
.
.

.
.
.
.
.
.
.

.
.
.
.
.
.
.
.

.
.
.
.
.
.
.

.
.
.
.
.
.
.
.

III.3
III.4
III.5
III.6

Coeficientes de acoplamiento y dispersin


El transformador como circuito acoplado
Sumario . . . . . . . . . . . . . . . . . .
Ejercicios propuestos . . . . . . . . . . .

.
.
.
.

.
.
.
.

C APTULO IV
Mquinas elctricas rotativas
IV.1
Caractersticas comunes . . . . . . . . . . .
IV.2
Bobinas ortogonales . . . . . . . . . . . . .
IV.3
Mltiples pares de polos . . . . . . . . . . .
IV.4
La mquina generalizada . . . . . . . . . . .
IV.5
Clculo del par elctrico . . . . . . . . . . .
IV.6
Par elctrico y fuerzas magnetomotrices . . .
IV.7
El campo magntico rotatorio . . . . . . . .
IV.8
La mquina trifsica . . . . . . . . . . . . .
IV.9
Transformacin de coordenadas . . . . . . .
IV.10
Transformacin de coordenadas dq . .
IV.11
Ecuaciones generales en coordenadas dq .
IV.12
Sumario . . . . . . . . . . . . . . . . . . .
IV.13
Ejemplo resuelto . . . . . . . . . . . . . . .
IV.14
Ejercicios propuestos . . . . . . . . . . . . .

.
.
.
.

.
.
.
.
.
.
.
.
.
.
.
.
.
.

.
.
.
.

.
.
.
.
.
.
.
.
.
.
.
.
.
.

.
.
.
.

.
.
.
.
.
.
.
.
.
.
.
.
.
.

.
.
.
.

.
.
.
.
.
.
.
.
.
.
.
.
.
.

.
.
.
.

.
.
.
.
.
.
.
.
.
.
.
.
.
.

.
.
.
.

.
.
.
.
.
.
.
.
.
.
.
.
.
.

.
.
.
.

.
.
.
.
.
.
.
.
.
.
.
.
.
.

.
.
.
.

.
.
.
.
.
.
.
.
.
.
.
.
.
.

.
.
.
.

.
.
.
.
.
.
.
.
.
.
.
.
.
.

.
.
.
.

.
.
.
.
.
.
.
.
.
.
.
.
.
.

.
.
.
.

.
.
.
.
.
.
.
.
.
.
.
.
.
.

.
.
.
.

.
.
.
.
.
.
.
.
.
.
.
.
.
.

.
.
.
.

.
.
.
.
.
.
.
.
.
.
.
.
.
.

.
.
.
.

.
.
.
.
.
.
.
.
.
.
.
.
.
.

.
.
.
.

.
.
.
.
.
.
.
.
.
.
.
.
.
.

.
.
.
.

91
92
98
98

.
.
.
.
.
.
.
.
.
.
.
.
.
.

103
103
106
107
110
113
117
121
122
124
126
129
130
131
134

PARTE II Mquinas elctricas rotativas


C APTULO V
Mquinas de conmutador
V.1
Principio de operacin . . . . . . . . . . . . . . . . . .
V.2
Ecuaciones de las mquinas de conmutador . . . . . . .
V.3
Caractersticas de operacin de las diferentes conexiones
V.4
Control de velocidad . . . . . . . . . . . . . . . . . . .
V.5
Valores nominales y bases . . . . . . . . . . . . . . . .
V.6
Reaccin de armadura . . . . . . . . . . . . . . . . . .
V.7
Saturacin de la mquina de corriente continua . . . . .
V.8
La conmutacin . . . . . . . . . . . . . . . . . . . . . .
V.9
Prdidas en las mquinas de corriente continua . . . . .
V.10
Controladores electrnicos de velocidad . . . . . . . . .
V.11
Mquinas especiales de corriente continua . . . . . . . .
V.12
Sumario . . . . . . . . . . . . . . . . . . . . . . . . . .
V.13
Ejemplos resueltos . . . . . . . . . . . . . . . . . . . .
V.14
Ejercicios propuestos . . . . . . . . . . . . . . . . . . .

139

.
.
.
.
.
.
.
.
.
.
.
.
.
.

.
.
.
.
.
.
.
.
.
.
.
.
.
.

.
.
.
.
.
.
.
.
.
.
.
.
.
.

.
.
.
.
.
.
.
.
.
.
.
.
.
.

.
.
.
.
.
.
.
.
.
.
.
.
.
.

.
.
.
.
.
.
.
.
.
.
.
.
.
.

.
.
.
.
.
.
.
.
.
.
.
.
.
.

.
.
.
.
.
.
.
.
.
.
.
.
.
.

.
.
.
.
.
.
.
.
.
.
.
.
.
.

.
.
.
.
.
.
.
.
.
.
.
.
.
.

141
141
153
154
163
165
166
169
170
173
176
181
184
185
192

C APTULO VI
La mquina de induccin
201
VI.1
Principio de funcionamiento . . . . . . . . . . . . . . . . . . . . . . . . . 202
VI.2
Modelo de la mquina de induccin . . . . . . . . . . . . . . . . . . . . . 203
VI.3
Vectores espaciales . . . . . . . . . . . . . . . . . . . . . . . . . . . . . . 207

VI.4
VI.5
VI.6
VI.7
VI.8
VI.9
VI.10
VI.11
VI.12
VI.13
VI.14
VI.15
VI.16

Modelo en rgimen permanente . . . .


Ecuaciones de la mquina de induccin
Caracterstica par-deslizamiento . . . .
Puntos de operacin . . . . . . . . . .
El punto nominal . . . . . . . . . . . .
Sistema en por unidad . . . . . . . . .
Determinacin de los parmetros . . . .
Condiciones de operacin . . . . . . .
Caractersticas normalizadas . . . . . .
Diagrama de crculo . . . . . . . . . .
Sumario . . . . . . . . . . . . . . . . .
Ejemplos resueltos . . . . . . . . . . .
Ejercicios propuestos . . . . . . . . . .

.
.
.
.
.
.
.
.
.
.
.
.
.

.
.
.
.
.
.
.
.
.
.
.
.
.

.
.
.
.
.
.
.
.
.
.
.
.
.

.
.
.
.
.
.
.
.
.
.
.
.
.

.
.
.
.
.
.
.
.
.
.
.
.
.

.
.
.
.
.
.
.
.
.
.
.
.
.

.
.
.
.
.
.
.
.
.
.
.
.
.

.
.
.
.
.
.
.
.
.
.
.
.
.

C APTULO VII
Operacin de la mquina de induccin
VII.1
Arranque de motores de induccin . . . . . . . . . . .
VII.2
El rotor de jaula de ardilla . . . . . . . . . . . . . . .
VII.3
Corriente de arranque . . . . . . . . . . . . . . . . . .
VII.4
Rgimen desequilibrado de las mquinas de induccin
VII.5
Armnicas temporales en la mquina de induccin . .
VII.5.1 Sistema de terceras armnicas 3e . . . . . . . .
VII.5.2 Sistema de quintas armnicas 5e . . . . . . . .
VII.5.3 Sistema de sptimas armnicas 7e . . . . . . .
VII.5.4 Sistema armnico de orden h he . . . . . . .
VII.6
Armnicas espaciales en la mquina de induccin . . .
VII.7
La mquina de induccin bifsica . . . . . . . . . . .
VII.8
Anlisis transitorio . . . . . . . . . . . . . . . . . . .
VII.9
Control de velocidad . . . . . . . . . . . . . . . . . .
VII.9.1 Control tensin-frecuencia . . . . . . . . . . . .
VII.9.2 Control por campo orientado . . . . . . . . . . .
VII.9.3 Control directo de par . . . . . . . . . . . . . . .
VII.10 Sumario . . . . . . . . . . . . . . . . . . . . . . . . .
VII.11 Ejemplos resueltos . . . . . . . . . . . . . . . . . . .
VII.12 Ejercicios propuestos . . . . . . . . . . . . . . . . . .
C APTULO VIII
La mquina sincrnica
VIII.1
Descripcin de la mquina sincrnica .
VIII.2
Modelo de la mquina sincrnica . . .
VIII.3
Transformacin a vectores espaciales .
VIII.4
Transformacin a coordenadas rotricas
VIII.5
Transformacin de Park . . . . . . . .
VIII.6
Rgimen permanente . . . . . . . . . .
VIII.7
Diagrama fasorial . . . . . . . . . . . .
VIII.8
Potencia y par elctrico . . . . . . . . .

.
.
.
.
.
.
.
.

.
.
.
.
.
.
.
.

.
.
.
.
.
.
.
.

.
.
.
.
.
.
.
.

.
.
.
.
.
.
.
.

.
.
.
.
.
.
.
.

.
.
.
.
.
.
.
.

.
.
.
.
.
.
.
.

.
.
.
.
.
.
.
.
.
.
.
.
.

.
.
.
.
.
.
.
.
.
.
.
.
.
.
.
.
.
.
.

.
.
.
.
.
.
.
.

.
.
.
.
.
.
.
.
.
.
.
.
.

.
.
.
.
.
.
.
.
.
.
.
.
.
.
.
.
.
.
.

.
.
.
.
.
.
.
.

.
.
.
.
.
.
.
.
.
.
.
.
.

.
.
.
.
.
.
.
.
.
.
.
.
.
.
.
.
.
.
.

.
.
.
.
.
.
.
.

.
.
.
.
.
.
.
.
.
.
.
.
.

.
.
.
.
.
.
.
.
.
.
.
.
.
.
.
.
.
.
.

.
.
.
.
.
.
.
.

.
.
.
.
.
.
.
.
.
.
.
.
.

.
.
.
.
.
.
.
.
.
.
.
.
.
.
.
.
.
.
.

.
.
.
.
.
.
.
.

.
.
.
.
.
.
.
.
.
.
.
.
.

.
.
.
.
.
.
.
.
.
.
.
.
.
.
.
.
.
.
.

.
.
.
.
.
.
.
.

.
.
.
.
.
.
.
.
.
.
.
.
.

.
.
.
.
.
.
.
.
.
.
.
.
.
.
.
.
.
.
.

.
.
.
.
.
.
.
.

.
.
.
.
.
.
.
.
.
.
.
.
.

.
.
.
.
.
.
.
.
.
.
.
.
.
.
.
.
.
.
.

.
.
.
.
.
.
.
.

.
.
.
.
.
.
.
.
.
.
.
.
.

.
.
.
.
.
.
.
.
.
.
.
.
.
.
.
.
.
.
.

.
.
.
.
.
.
.
.

.
.
.
.
.
.
.
.
.
.
.
.
.

.
.
.
.
.
.
.
.
.
.
.
.
.
.
.
.
.
.
.

.
.
.
.
.
.
.
.

.
.
.
.
.
.
.
.
.
.
.
.
.

212
215
217
220
222
224
226
233
236
239
250
253
261

.
.
.
.
.
.
.
.
.
.
.
.
.
.
.
.
.
.
.

267
267
269
273
276
288
288
288
290
291
294
297
309
316
316
320
322
325
326
337

.
.
.
.
.
.
.
.

343
345
348
351
353
354
359
359
363

VIII.9
VIII.10
VIII.11
VIII.12
VIII.13
VIII.14
VIII.15
VIII.16
VIII.17
VIII.18
VIII.19

Convenciones de la mquina sincrnica . . . . . . . . . . .


Valores nominales de la mquina sincrnica . . . . . . . . .
Lugares geomtricos . . . . . . . . . . . . . . . . . . . . .
Circuito equivalente de la mquina sincrnica . . . . . . . .
Curvas en V . . . . . . . . . . . . . . . . . . . . . . . . . .
Medicin de las reactancias permanentes . . . . . . . . . .
Anlisis de la mquina sincrnica considerando la saturacin
La mquina sincrnica en el sistema elctrico . . . . . . . .
Sumario . . . . . . . . . . . . . . . . . . . . . . . . . . . .
Ejemplos resueltos . . . . . . . . . . . . . . . . . . . . . .
Ejercicios propuestos . . . . . . . . . . . . . . . . . . . . .

.
.
.
.
.
.
.
.
.
.
.

.
.
.
.
.
.
.
.
.
.
.

C APTULO IX
Rgimen transitorio de la mquina sincrnica
IX.1
Transitorios electromagnticos . . . . . . . . . . . . . . . . . .
IX.1.1 Solucin mediante autovalores-autovectores . . . . . . . .
IX.1.2 Solucin mediante la transformada de Laplace . . . . . . .
IX.2
Cortocircuito brusco de la mquina sincrnica . . . . . . . . . .
IX.3
Interpretacin fsica de las inductancias transitorias . . . . . . .
IX.4
Tensin de armadura en circuito abierto . . . . . . . . . . . . .
IX.5
Sistema adimensional de unidades . . . . . . . . . . . . . . . .
IX.6
Anlisis transitorio con resistencias . . . . . . . . . . . . . . .
IX.7
Constantes de tiempo en circuitos acoplados magnticamente .
IX.8
Anlisis transitorio aproximado . . . . . . . . . . . . . . . . .
IX.9
Pequeas oscilaciones de la mquina sincrnica . . . . . . . . .
IX.10
Efecto del devanado amortiguador . . . . . . . . . . . . . . . .
IX.11
Anlisis subtransitorio aproximado . . . . . . . . . . . . . . .
IX.12
Determinacin de las inductancias transitorias y subtransitorias
IX.13
Rgimen desequilibrado de la mquina sincrnica . . . . . . . .
IX.14
Estabilidad de la mquina sincrnica . . . . . . . . . . . . . . .
IX.15
Diagrama de bloques de la mquina sincrnica . . . . . . . . .
IX.16
Sumario . . . . . . . . . . . . . . . . . . . . . . . . . . . . . .
IX.17
Ejemplo resuelto . . . . . . . . . . . . . . . . . . . . . . . . .
IX.18
Ejercicios propuestos . . . . . . . . . . . . . . . . . . . . . . .
ndice alfabtico

.
.
.
.
.
.
.
.
.
.
.

.
.
.
.
.
.
.
.
.
.
.
.
.
.
.
.
.
.
.
.

.
.
.
.
.
.
.
.
.
.
.

.
.
.
.
.
.
.
.
.
.
.
.
.
.
.
.
.
.
.
.

.
.
.
.
.
.
.
.
.
.
.

.
.
.
.
.
.
.
.
.
.
.
.
.
.
.
.
.
.
.
.

.
.
.
.
.
.
.
.
.
.
.

.
.
.
.
.
.
.
.
.
.
.
.
.
.
.
.
.
.
.
.

.
.
.
.
.
.
.
.
.
.
.

.
.
.
.
.
.
.
.
.
.
.
.
.
.
.
.
.
.
.
.

.
.
.
.
.
.
.
.
.
.
.

366
368
371
373
376
378
380
386
389
390
399

.
.
.
.
.
.
.
.
.
.
.
.
.
.
.
.
.
.
.
.

407
408
409
411
412
415
417
418
422
427
428
430
435
437
440
443
445
452
452
456
461
467

Prefacio

En los ltimos veintisis aos, en el Departamento de Conversin y Transporte de Energa de la


Universidad Simn Bolvar se ha desarrollado un mtodo eficiente y sistemtico para la docencia
de los cursos de Conversin de Energa Elctrica 1 . Se fundamenta en la experiencia aportada
originalmente por el Profesor Gastn Pesse Vidal despus de ms de 45 aos de fructfera labor
universitaria, as como por el trabajo sistemtico de los profesores de la seccin de Conversin de
Energa Elctrica. Esto ha permitido amplar la visin de los ingenieros electricistas, facilitando
la incorporacin de los nuevos desarrollos en electrnica, computacin, sistemas de control y
las nuevas tcnicas para el control electrnico de potencia.
A diferencia de los mtodos convencionales para el anlisis de las mquinas elctricas, el mtodo que se desarrolla en este texto permite el estudio de los convertidores elctromecnicos
mediante una modelacin generalizada, donde las diferencias se establecen fundamentalmente a partir de la configuracin de las fuentes de alimentacin. Este libro utiliza ampliamente
el lgebra lineal con la finalidad de simplificar las operaciones matemticas necesarias para el
anlisis permanente y transitorio de las mquinas elctricas. Se incorpora al mismo tiempo la
visin fsica de los fenmenos involucrados, para permitir una comprensin ms completa de
cada tema.
El objetivo general consiste en ofrecer al futuro ingeniero electricista aquellos fundamentos tericos y conceptuales necesarios para comprender los principios, analizar y evaluar las diferentes
condiciones de operacin de las mquinas elctricas convencionales y su interrelacin con el
sistema elctrico de potencia. El texto se orienta fundamentalmente al anlisis de los convertidores, pero en algunos casos se desarrollan ideas generales que podran servir de pie a cursos
1

Mquinas elctricas y controladores electrnicos de potencia.

posteriores donde se desarrollen los temas relativos al diseo y construccin de los convertidores
electromecnicos.
El actual perfil profesional del ingeniero electricista est en continuo cambio, cada da el desarrollo tecnolgico aumenta aceleradamente. Nuevas mquinas y aplicaciones aparecen en el
horizonte. Es necesario preparar a las nuevas generaciones para que puedan afrontar estos retos.
Por esta razn es necesario incorporar nuevas herramientas y conceptos que flexibilicen el conocimiento de estas tecnologas que se encuentran en permanente evolucin. Para cumplir con este
cometido es indispensable romper con aquellos esquemas conceptuales que eran vlidos cuando
las mquinas elctricas cumplan una funcin mucho ms restringida. La investigacin metdica
y las continuas asesoras profesionales permiten el desarrollo de esta visin conceptual de las
mquinas elctricas, dentro de los alcances y limitaciones impuestos por el nivel acadmico al
que va dirigido este texto2.
La necesidad permanente de actualizar conocimientos, y la definicin constante de nuevas metas
y objetivos hacen indispensable la revisin peridica de este material. Con este espritu ha sido
concebido. Es un deseo que el contenido de este texto ayude a simplificar la difcil labor del docente en esta rea, y el an ms complejo proceso de aprendizaje a los estudiantes de ingeniera
elctrica. El estudio de este tema requiere una fuerte conceptualidad fsica y matemtica, debido
a que los fenmenos de conversin electromecnica de la energa deben interpretarse espacial y
geomtricamente al mismo tiempo. Se ha incluido un nmero importante de ilustraciones, grficos y diagramas para simplificar la comprensin de aquellas ideas que tienden a ser difciles
para el estudiante por integrar aspectos espaciales y temporales. En esta edicin se incorporan
ejemplos resueltos que ilustran los conceptos desarrollados, ejercicios propuestos que permiten
ampliar las habilidades necesarias para cumplir con los objetivos de cada tema y un sumario
en cada captulo que expone de forma concisa las ideas fundamentales. En varios temas se han
incluido pequeos programas desarrollados mediante herramientas de clculo de alto nivel3 que
permiten obtener resultados prcticos de los modelos y constituyen en s mismos una poderosa
herramienta de aprendizaje.
Este libro comienza presentando las bases fundamentales que permiten un anlisis sistemtico
de las mquinas elctricas: la ley de Lorentz, el principio de los trabajos virtuales y el anlisis
de circuitos acoplados magnticamente. Posteriormente se desarrollan en detalle los principios
bsicos de conversin electromecnica, el planteamiento de las ecuaciones diferenciales que
rigen su comportamiento y las transformaciones necesarias para su solucin eficaz y eficiente.
Las mquinas de conmutador, de induccin y sincrnicas se presentan a partir de estas ideas y se
obtienen en modelos para el anlisis en rgimen permanente y transitorio de estos convertidores.
2
3

Fundamentalmente estudiantes no graduados de ingeniera elctrica


Matlab, Scilab y Octave.

10

Algunos temas importantes tales como armnicos, saturacin, desequilibrios, limitaciones de


diseo, ensayos de laboratorio, estimacin paramtrica y valores esperados se han incluido a
travs de todo el texto.
Quisiera terminar el prefacio a la presente edicin agradeciendo a los innumerables colaboradores que han contribuido con su realizacin durante todos estos aos, especialmente a las generaciones de ingenieros electricistas que permanente, entusiasta y desinteresadamente han revisado,
discutido y hecho sugerencias sobre todos los temas desarrollados. Desde su primera publicacin en Internet en el ao 2002, han incrementado notablemente los comentarios y propuestas
que han ido enriqueciendo el material. Tambin quiero agradecer al profesor Jos Restrepo por
su invaluable ayuda con la edicin del libro en LATEXy al profesor Alexander Bueno por el gran
apoyo que me ofreci en todo momento.

Prof. Jos Manuel Aller


Valle de Sartenejas, 2007

11

12

Parte I

Fundamentos generales de las mquinas


elctricas

13

CAPTULO I

Conversin de energa elctrica

En la historia del desarrollo de la humanidad se han buscado muchas fuentes de energa para movilizarse, construir viviendas, arar, segar, procesar los alimentos e iluminar. Hombres y bestias
fueron las primeras fuentes de energa, incluso la esclavitud fue ampliamente justificada durante
milenios con esta finalidad. La lea y el carbn desempearon un papel protagnico durante la
revolucin industrial, con la invencin de la mquina de vapor. El desarrollo de la electricidad a
finales del siglo XIX permiti el desarrollo de la industria moderna y requiri la conversin de
diversas fuentes de energa en energa elctrica y viceversa. En la actualidad el desarrollo de la
electrnica y en especial de la electrnica de potencia, permite el control efectivo y eficiente de
los procesos de conversin de energa elctrica.
En este captulo analizaremos los conceptos fundamentales involucrados en la conversin de
energa, los principios bsicos que permiten la conversin electromecnica de energa y las tcnicas matemticas para analizar el comportamiento de los convertidores electromecnicos de
energa.
I.1

C ONCEPTOS

BSICOS

La energa es uno de los conceptos ms importantes en el estudio de las mquinas elctricas.


La energa es la capacidad de realizar un trabajo. La energa se presenta en la naturaleza en
diferentes formas. El objetivo de las mquinas elctricas consiste en convertir la energa de una
forma en otra.
En la tabla 1.1 se presenta un resumen de las densidades de energa que pueden ser almacenadas
en diversos procesos fsicos.
Se puede observar que los sistemas elctricos y magnticos no son buenos acumuladores de
energa porque las mximas densidades de energa que se pueden obtener con los materiales

15

Captulo I Conversin de energa elctrica


1.
2.
3.
4.
5.
6.
7.
8.
9.
10.

Gravitacin (100 m)
Energa Cintica (5.000 rpm)
Campo Magntico (2W b/m)
Campo Elctrico (6,5 MV /m)
Batera de plomo cido Pb + 2O PbO2
Calor de reaccin del combustible fsil
Calor de combinacin H + H H2
Energa de Ionizacin
Fisin U 235
Fusin Deuterio + Tritio He + 17,6 MeV

0,0098
0,053
0,0016
0,006
0,16
44
216
990
83.000
340.000

MJ/kg
MJ/kg
MJ/litro
MJ/litro
MJ/kg
MJ/kg
MJ/kg
MJ/kmol
MJ/kg
MJ/kg

Tabla 1.1 Densidades de energa que pueden ser almacenadas en diversos procesos fsicos
existentes en la actualidad, son relativamente pequeas al compararse con la energa por unidad
de peso que puede ser almacenada en una batera o en los combustibles fsiles. Por esta razn
es necesario realizar la conversin electromecnica de la energa para obtener energa elctrica
en grandes cantidades. La conversin electromecnica de energa permite transmitir, consumir,
modificar o transformar la energa electromagntica de una forma en otra, pero no es posible
almacenarla en grandes cantidades1 .
El segundo concepto fsico importante en los fenmenos de conversin de energa es la fuerza.
La fuerza se manifiesta en un sistema fsico mediante la presencia de interacciones entre la
materia. Aun cuando parece que las fuerzas pueden ser de muy diferentes formas y tipos, se
conocen en la actualidad slo cuatro fuerzas:
1. Interacciones gravitacionales entre masas (gravitones)
2. Interacciones elctricas entre las cargas (electrn-protn-fotn)
3. Interacciones nucleares dbiles (bosones intermedios)
4. Interacciones nucleares fuertes (protn-neutrn-pin)
Si se asocia a las fuerzas nucleares fuertes de cohesin protn-protn por intercambio de piones
entre protones y neutrones el valor unitario, las interacciones nucleares dbiles de las partculas
nucleares con rareza se encuentran en el orden de 10 14 . Las fuerzas gravitacionales se encuentran, en la misma base de comparacin, en el orden de 10 37 . Las fuerzas de atraccin y
repulsin de cargas elctricas por intercambio de fotones estn en el rango de 102 .
El tercer concepto bsico es el de campo. La palabra campo posee la interpretacin geomtrica
de extensin, superficie o espacio. Sin embargo, en fsica el concepto de campo consiste en la
descripcin del espacio donde se produce algn tipo de fuerza. El campo gravitatorio es la zona
del espacio donde una masa ejerce su influencia atrayendo a otras masas. El campo elctrico
se define exactamente igual, pero considerando las interacciones entre las cargas elctricas. El
campo magntico se define a travs de las fuerzas entre dipolos magnticos. La medicin de
un campo se realiza colocando en un punto del espacio una partcula de prueba (masa, carga o
1

Existen algunas excepciones como pueden ser los voltmetros electrostticos y ciertos sensores de posicin que
utilizan el campo elctrico en el proceso de conversin de energa.

16

I.1 Conceptos bsicos


dipolo magntico) y se mide la fuerza ejercida sobre ella. El cociente entre la fuerza en dicho
punto y la magnitud de inters de la partcula es la intensidad del campo en el punto. Por ejemplo,
si en un punto en la superficie de la tierra se mide la fuerza de atraccin gravitatoria sobre la
masa de prueba m, el dinammetro indicar F = mg, donde g es la aceleracin de gravedad en
el punto donde se realiza la medida, y su direccin apunta hacia el centro de la tierra. El campo
gravitatorio es el cociente entre la fuerza y la masa. En otras palabras la aceleracin de gravedad
en cada punto determina el valor de la intensidad del campo gravitatorio. De igual forma, el
campo elctrico es el cociente entre la fuerza elctrica sobre una partcula cargada, y el valor de
la carga de esa partcula E = Fq .
Para el fenmeno elctrico se plantea una ecuacin de equilibrio de fuerzas en funcin del campo
elctrico E y el campo magntico B de un sistema dado. Esta ecuacin de equilibrio se conoce
como relacin de Lorenz:
F = q (E + v B)

(1.1)

Donde:
F

es el vector de la fuerza resultante sobre la partcula cargada

es la carga elctrica de la partcula

es el vector intensidad del campo elctrico

es el vector velocidad

es el vector densidad de campo magntico

Figura 1.1 Carga elctrica en un campo elctrico


En la ecuacin 1.1 todas las cantidades vectoriales deben estar referidas a un sistema de referencia nico. Adems, el campo elctrico E y el campo magntico B deben ser producidos
externamente a la carga q. Para que ocurra una interaccin electromagntica sobre la carga q
es necesaria la existencia de otras cargas. La figura 1.1 ilustra esta idea. En el punto que ocupa
la carga q, el campo elctrico E1 se debe a las otras cargas presentes en el sistema y no a s
misma. En estas condiciones existe una interaccin elctrica entre la carga puntual q y el campo
elctrico E1 producido por las cargas distribuidas en las dos placas.

17

Captulo I Conversin de energa elctrica


En un convertidor electromagntico de energa es necesario analizar el mecanismo de creacin
de campo elctrico E y magntico B. Para este fin se recurre a las ecuaciones de Maxwell y a las
condiciones de contorno impuestas por el equipo.
Para determinar la solucin del campo electromagntico, se parte de las siguientes premisas:
1. Las partculas elctricas q se desplazan en campos elctricos E y magnticos B.
2. Estos campos son producidos externamente a las cargas, por otras partculas cargadas.
Con las premisas anteriores, las leyes de Maxwell expresadas en su forma diferencial para un
punto cualquiera del espacio son:

B
t

E =

H = J +

(1.2)

D
t

(1.3)

E =

(1.4)

B = 0

(1.5)

Y las relaciones constitutivas debidas al medio material:


B = H

(1.6)

D = E

(1.7)

J = E

(1.8)

Donde , y pueden ser tensores que dependen del tipo de material y orientacin, pero que
en los casos ms simples son cantidades escalares.
Las ecuaciones 1.2 a 1.5 se pueden escribir en forma integral:
I
I

E dl =

H dl =
I

18

J dS +

D dS =
I

B dS

v dv

B dS = 0

D dS

(1.9)

(1.10)
(1.11)

(1.12)

I.1 Conceptos bsicos


En general, cuando se analizan casos prcticos de los convertidores electromecnicos de energa, la variacin de la densidad del campo elctrico B con respecto al tiempo es despreciable
comparada con la densidad de corriente J. Este trmino representa las corrientes capacitivas
debidas a las variaciones del campo elctrico y se conoce como corrientes de desplazamiento.
Las corrientes de desplazamiento son importantes cuando el campo elctrico es muy intenso2 o
cuando su variacin es muy rpida3 . Ninguna de estas condiciones es frecuente en las mquinas
elctricas convencionales en condiciones normales de operacin.
Para resolver las ecuaciones de Maxwell en un problema concreto, se define a las corrientes
como las variables independientes. A partir de ellas se calcula el campo magntico B con las
ecuaciones 1.3 y 1.5, el campo elctrico E de la ecuacin 1.2 y las fuerzas electromotrices por
integracin lineal del campo elctrico en la trayectoria de inters. Las condiciones de contorno
del sistema fsico relacionan las fuerzas electromotrices con las corrientes que han sido previamente consideradas como variables independientes. Este proceso de clculo se utilizar en el
prximo captulo para obtener el modelo de un sistema electromecnico simple, pero es totalmente general. La ecuacin 1.4 no se utiliza en este anlisis ya que se supone que en el medio
no se encuentran disponibles cargas libres, es decir la densidad de carga es cero.

Figura 1.2 Efecto del cambio del sistema de referencia sobre el campo elctrico
En la figura 1.2 se ilustra un par de conductores idnticos. El primero se desplaza a una velocidad v diferente de cero, en la presencia de los campos E1 y B1 . En el segundo conductor el
observador se mueve a la misma velocidad v y considera por esta razn que el conductor est en
reposo. En esta condicin el observador detecta el campo E2 .
Si se introduce una partcula en cada uno de los conductores anteriores cuya carga es q1 , en el
primer sistema la fuerza sobre la partcula, de acuerdo con la relacin de Lorenz 1.1, es:
F1 = q1 (E1 + v B1 )

(1.13)

Si la velocidad es constante, las fuerza F1 es nula y de la ecuacin 1.13 se deduce:


E1 = v B1
2
3

(1.14)

Alta tensin.
Alta frecuencia.

19

Captulo I Conversin de energa elctrica

Figura 1.3 Conductor en movimiento en presencia de campos elctricos y magnticos


En el sistema II, como la velocidad relativa es cero, el observador slo puede atribuir la fuerza
actuante sobre la partcula q1 al campo elctrico E2 :
E2 =

F2
q1

(1.15)

Como los conductores son idnticos en los dos sistemas, a excepcin de su sistema de referencia,
se puede establecer la transformacin de Lorenz mediante las expresiones 1.13 y 1.15, debido a
que F1 = F2 :
E2 = E1 + v B1

(1.16)

La ecuacin 1.16 permite calcular el campo elctrico equivalente de un sistema de referencia solidario a los conductores del convertidor electromecnico de energa, conociendo vectorialmente
el campo elctrico y el campo magntico, del sistema fijo y externo al conductor.
En la figura 1.3 se ha esquematizado un segmento conductor al cual se le aplica entre sus extremos el campo elctrico E. El circuito se encuentra inmerso en un campo magntico uniforme
B. La densidad de corriente J que circula por el conductor depende de la superposicin de los
campos elctricos aplicados sobre l y de la conductividad del material, segn la relacin
constitutiva 1.8, tambin conocida como ley de Ohm:
J = E = Eaplicada Einducida

(1.17)

El campo elctrico producido por el movimiento del conductor a la velocidad v en un campo


magntico B se calcula segn la ecuacin 1.14, y por lo tanto la expresin 1.17 queda:

J = E = Eaplicada v B

(1.18)

La expresin anterior determina la densidad de corriente J por el conductor. Una vez conocida
la densidad de corriente se puede evaluar el campo elctrico o magntico en cualquier punto del

20

I.2 Convertidor electromecnico elemental

Figura 1.4 Convertidor electromagntico elemental


espacio utilizando las ecuaciones de Maxwell 1.2 a 1.5. Conocidos los campos se pueden evaluar
las fuerzas sobre cualquier partcula elctrica cargada o sobre cualquier dipolo magntico. De
esta forma queda resuelto el problema de la conversin electromecnica de la energa.
I.2

C ONVERTIDOR

ELECTROMECNICO ELEMENTAL

En general las mquinas elctricas tienen por finalidad transformar la energa mecnica en energa elctrica y viceversa. Cuando la conversin es de energa mecnica en energa elctrica se
dice que la mquina est funcionando como generador y en el caso contrario opera como motor.
Tal vez la mquina elctrica ms simple es la que se representa en la figura 1.4. Este dispositivo es un convertidor electromagntico elemental y est constituido solamente por un conductor
rectilneo, movindose ortogonalmente a un campo magntico uniforme.
En la figura 1.4, el conductor longitudinal se mueve en el interior de un campo magntico B:
E
e
B
v

es el vector intensidad de campo elctrico


es la fuerza electromotriz
es el vector densidad de campo magntico
es el vector velocidad del conductor lineal

Las variables anteriores se relacionan a partir de la ecuacin 1.13, considerando que no existe
campo elctrico externo:
E = vB

(1.19)

Si en la ecuacin 1.19, se supone que el campo magntico B es uniforme en todos los puntos del
conductor y la velocidad v es constante, la fuerza electromotriz e de todo el conductor es:
e=

Z l
0

E dl

(1.20)

Si al conductor anterior se le conecta una resistencia entre sus extremos, circularn cargas por
el conductor y se producir una corriente de valor:

21

Captulo I Conversin de energa elctrica

Figura 1.5 Corriente circulando por un conductor

i=

e
R

(1.21)

En el conductor de la figura 1.5 se produce una fuerza Fe , que se opone al movimiento. Esta
fuerza puede calcularse a partir de la relacin de Lorenz 1.1, expresada como funcin de la
corriente i por el conductor:
Fe = l i B

(1.22)

La fuerza calculada en la expresin anterior muestra que el sistema se opone a la extraccin


de energa. Para obtener la energa, es necesario forzar el movimiento del conductor. Si no
acta ninguna otra fuerza que mantenga el movimiento, y si la velocidad es diferente de cero,
el sistema tendr un movimiento retardado de aceleracin negativa. El conductor convertir la
energa que estaba inicialmente almacenada en su masa, en prdidas en la resistencia R del
circuito externo. En estas condiciones, la velocidad decae exponencialmente a cero.
Para mantener una velocidad constante en el conductor de la figura 1.5, es necesario aplicar una
fuerza externa al conductor que se oponga a Fe . Esta fuerza es de origen mecnico y se denomina
Fm . En la figura 1.5 se observa el equilibrio de fuerzas necesario para mantener constante la
velocidad v del conductor.
El sistema mecnico entrega potencia al sistema elctrico para mantener la velocidad v, la potencia mecnica instantnea entregada por el sistema externo se calcula mediante la relacin
siguiente:
Pm = Fm v

(1.23)

y la potencia elctrica instantnea en el conductor es:


Pe = e i
22

(1.24)

I.2 Convertidor electromecnico elemental

Figura 1.6 Conductor alimentado por una fuente de tensin V


Si se realiza un balance de potencia, considerando que las cantidades vectoriales son ortogonales
entre s, se obtiene el siguiente resultado:
Pm = Fm v = Fe v = i B v l = i E l = i e = Pe

(1.25)

La ecuacin 1.25 demuestra que la conversin de energa mecnica en energa elctrica ha sido
completa. En el proceso no hay prdidas debido a que la potencia disipada en la resistencia del
circuito es externa a la mquina.
Aadiendo una fuente de tensin al conductor anterior con el conductor inicialmente en reposo,
tal como se ilustra en la figura 1.6, la fuente de tensin V hace circular una corriente i por
el circuito. Esta corriente produce, segn la ecuacin 1.22 una fuerza elctrica Fe . Si no acta
ninguna otra fuerza sobre el conductor, este comienza a moverse con aceleracin.
Cuando el conductor se mueve en un campo magntico, se origina a su vez un campo elctrico
E. Como se puede apreciar en la figura 1.6, la fuente de tensin produce una corriente que se
opone al campo elctrico E inducido por el movimiento. La corriente se puede calcular como:
i=

V e
R

(1.26)

De esta forma, en la medida que aumenta la fuerza electromotriz e inducida por el movimiento
del conductor, disminuye la corriente en el circuito. Al decrecer la corriente, se reduce la fuerza
elctrica sobre el conductor. El proceso contina hasta que la fuerza elctrica Fe se hace cero. En
esta condicin la tensin aplicada por la batera V es igual a la fuerza electromotriz e, inducida
por el movimiento del conductor en el campo magntico y la corriente i se anula.
La velocidad del conductor en que la fuerza elctrica es cero, debido al equilibrio entre la tensin aplicada y la fuerza electromotriz inducida por el movimiento, se define como velocidad
sincrnica del conductor. En esta situacin:
e = V = l vs B

(1.27)

23

Captulo I Conversin de energa elctrica


Donde vs es la velocidad sincrnica y se calcula de la expresin anterior como:
vs =

V
l B

(1.28)

Una vez que el conductor alcanza la velocidad sincrnica (V = e ; i = 0), si se aplica una fuerza
resistente al conductor, el sistema comienza a retardarse y la fuerza electromotriz inducida e
disminuye, aumenta la corriente en el conductor debido a que la tensin V de la batera supera
a la fuerza electromotriz e. La aceleracin o retardo del sistema se puede calcular aplicando
convenientemente la segunda ley de Newton:
a=

1
Fe + Fm
dv
= F =
dt
M
M

(1.29)

Donde:
F

es la sumatoria de fuerzas aplicadas

Fe

es la fuerza elctrica sobre el conductor

Fm

es la fuerza mecnica resistente

es la masa del conductor

Cuando la fuerza mecnica Fm equilibra a la fuerza elctrica Fe , la aceleracin es cero y en ese


instante se cumple que:


V B l v0
Fm = Fe = l B i = l B
R

(1.30)

De la ecuacin 1.30 se obtiene la velocidad de operacin v0 en funcin de la fuerza mecnica


resistente:

v0 =

m R
V FBl
Bl

(1.31)

La velocidad v0 corresponde a la operacin de la mquina cuando las fuerzas elctricas y mecnicas sobre el conductor estn en equilibrio. Si en este momento se elimina la fuerza resistente
Fm , el conductor se acelera en la direccin de la fuerza elctrica Fe hasta alcanzar nuevamente
la velocidad sincrnica.
La exposicin anterior permite resumir en seis ecuaciones los principios que rigen la conversin
electromecnica de energa:

24

E = vB

(1.32)

f = iB

(1.33)

I.3 Curvas caractersticas

e=

Z l
o

F=

E dl = E l = v B l

Z l
o

f dl = f l = i B l
i=

V e
R

1
Fe + Fm
dv
= Fa =
dt
M
M

(1.34)

(1.35)

(1.36)

(1.37)

En el sistema de ecuaciones representado por las expresiones 1.32 a 1.37 se destacan los siguientes puntos:
1. La ecuacin 1.34 calcula una variable elctrica (e) en funcin de una variable mecnica
(v) y el campo (B).
2. La ecuacin 1.35 determina una variable mecnica (F) en funcin de una variable elctrica
(i) y el campo (B).
3. Las expresiones 1.34 y 1.35 dependen del conductor y del campo en el cual est inmerso,
por esta razn se denominan las ecuaciones internas del convertidor electromecnico.
4. Las ecuaciones 1.36 y 1.37 representan las relaciones entre el conductor mquina elctrica y el resto del universo. Estas ecuaciones se denominan ecuaciones de ligazn, ecuaciones de borde, ecuaciones de contorno o ecuaciones de frontera.
I.3

C URVAS

CARACTERSTICAS

Para representar la curva caracterstica de la fuerza elctrica sobre el conductor en funcin de


la velocidad, se puede utilizar la ecuacin 1.30:

Fe = i B l =


V e
V B l (B l)2
Bl =

v
R
R
R

(1.38)

La ecuacin 1.38 representa la fuerza elctrica Fe como una recta en funcin de la velocidad v
del conductor. Cuando el conductor se encuentra en reposo (v = 0), la fuerza elctrica es igual
al trmino independiente en velocidad. Si la fuerza elctrica es cero, la velocidad corresponde
a la velocidad sincrnica de la mquina. Si se opone una fuerza constante de valor conocido,
como se observa en la figura 1.7, se determina un punto de equilibrio v0 en la interseccin de
las caractersticas elctrica y mecnica. En este caso v0 corresponde a la velocidad en la cual la
fuerza elctrica Fe equilibra a la fuerza mecnica Fm , y constituye un punto de operacin estable
debido a que cualquier perturbacin en la velocidad mecnica del sistema tender a ser restituida

25

Captulo I Conversin de energa elctrica

Figura 1.7 Curva caracterstica de la mquina


a las condiciones previas por las fuerzas actuantes sobre el conductor. Esta interseccin es un
punto de operacin de rgimen permanente para la mquina.
En la figura 1.7 se han marcado dos zonas (1) y (2). En la zona (1), si la mquina arranca
en contra de una fuerza mecnica resistente constante, se acelera hasta alcanzar el punto de
operacin permanente o punto de equilibrio v0 interseccin de las caractersticas. Esto ocurre
debido a que esta zona de operacin, la fuerza elctrica Fe , siempre es superior a la fuerza
mecnica Fm .
Si el sistema se encuentra originalmente en vaco, es decir, operando a velocidad sincrnica, sin
carga mecnica y repentinamente se aade una fuerza mecnica resistente, la fuerza elctrica es
inferior a la mecnica y ocurre un proceso de retardo en la zona (2) de la figura 1.7. La velocidad
disminuye desde la sincrnica hasta la velocidad de operacin v0 en el punto de equilibrio.
La fuerza mecnica Fm depende en general, para un accionamiento fsico, de la velocidad del
conductor. En la figura 1.8 se muestra la curva caracterstica de la mquina elctrica anterior,
pero sometida a una fuerza mecnica dependiente de la velocidad.
En este caso, al igual que en el anterior, v0 es un punto de equilibrio estable ya que si se aumenta
un diferencial la velocidad del conductor por encima de v0 , se origina una fuerza retardadora que
hace regresar el conductor a la anterior condicin de operacin. Por el contrario, si la velocidad
del conductor disminuye en un diferencial, se produce una fuerza acelerante que incrementa la
velocidad del conductor hasta alcanzar el punto de equilibrio en v0 .
Al producirse un cambio en la tensin de la batera que alimenta al convertidor, la velocidad
sincrnica de la mquina tambin vara, debido a que esta velocidad se determina cuando existe
equilibrio entre la tensin de la batera y la fuerza electromotriz inducida en el conductor. En
la figura 1.8 es posible definir una familia de curvas de acuerdo a como se vare la tensin de
la fuente. Mediante la variacin de la tensin de la batera se puede controlar la velocidad de
operacin de la mquina.

26

I.3 Curvas caractersticas

Figura 1.8 Fuerza mecnica variable con la velocidad

Figura 1.9 Efecto de la variacin de la tensin de alimentacin

27

Captulo I Conversin de energa elctrica

Figura 1.10 Efecto de la variacin del campo B del convertidor


Tambin se puede controlar la mquina elemental variando la densidad de flujo magntico B. La
variacin del campo produce un cambio en la pendiente de la curva caracterstica de la mquina,
ya que como se observa en la ecuacin 1.38, esta variacin altera la pendiente de la caracterstica
de forma cuadrtica y el punto de corte en el eje de la fuerza (v = 0), de forma lineal. En la
figura 1.10 se ilustra esta situacin y como es posible cambiar el punto de operacin de la
mquina mediante variaciones del campo magntico B.
De los dos mtodos analizados para controlar el punto de operacin de la mquina, la variacin del campo magntico tiene un inconveniente. Cuando el campo se reduce demasiado, la
velocidad sincrnica aumenta considerablemente y se puede producir un fenmeno denominado
embalamiento. El embalamiento es una aceleracin sbita debida a la prdida del campo en una
mquina elctrica sin carga. Si la velocidad sube a niveles peligrosos, puede ocurrir deterioro de
la mquina por fallas elctricas y mecnicas. En las mquinas elctricas rotativas este problema
es muy grave como se observa del siguiente ejemplo:
Una mquina de 3.600 rpm con un radio de 50 cm gira a una velocidad angular
de:

= 2

n
rad
= 377
f
s

La aceleracin centrpeta que aparece sobre los conductores de la periferia del


rotor de la mquina se calcula como:
ac = 2 r = 71.061

28

m
s2

I.4 Balance energtico

Figura 1.11 Modos de operacin del convertidor


Esta aceleracin es aproximadamente 7.252 veces superior a la de gravedad, por
lo tanto sobre cada gramo de material en la periferia aparece una fuerza de 7 kg
tratando de mover el material conductor de sus ranuras. Como la aceleracin vara
con el cuadrado de la velocidad angular, si se duplica la velocidad angular, la
aceleracin aumenta 4 veces.

I.4

BALANCE

ENERGTICO

En el balance de potencias desarrollado en la ecuacin 1.25 se lleg a la conclusin de que todo


el proceso es conservativo sobre la base de que la potencia elctrica desarrollada por la mquina
es igual a la potencia mecnica entregada por el sistema externo.
En general, todas las mquinas elctricas son reversibles y su funcionamiento depende del sentido en que se transmite la potencia. Si la energa fluye del sistema elctrico al mecnico, la
mquina funciona como motor. Si el flujo de energa es del sistema mecnico al elctrico, el
convertidor es un generador. Cuando el sistema elctrico y mecnico inyectan energa a la mquina, y esta energa se consume totalmente como prdidas internas, esta condicin se denomina
freno. La mquina se puede alimentar indistintamente con energa elctrica o con energa mecnica. En la figura 1.11 se presenta un grfico de la caracterstica fuerza-velocidad de la mquina
analizada anteriormente, con los diferentes modos de operacin factibles para este convertidor.
En la figura 1.12 se muestra un esquema donde se realiza el balance energtico de la mquina
en las tres condiciones de operacin posibles: motor, generador y freno.
En la zona (1), la velocidad del conductor es menor que la velocidad sincrnica, la fuerza electromotriz inducida es menor que la tensin aplicada externamente y la corriente tiene signo
contrario a la fuerza electromotriz. En estas condiciones el conductor se desplaza en el mismo
sentido de la fuerza elctrica, es decir, esta fuerza realiza trabajo positivo y por lo tanto se est
transformando energa elctrica en mecnica. La mquina est actuando como un motor. En esta
zona se satisfacen las siguientes condiciones:

29

Captulo I Conversin de energa elctrica

Motor
(1)

Pm

Pe

Pe

Pe

prdidas

Generador
(2)
prdidas

Freno
(3)

prdidas

Pm

Pm

Figura 1.12 Balance de potencia en los diversos modos de operacin

e > 0
e < V
i > 0

En la zona (2), la velocidad del conductor es mayor que la velocidad sincrnica y la fuerza
electromotriz es mayor que la tensin aplicada, por esta razn la corriente y la fuerza elctrica
invierten su sentido. Para encontrar un punto de equilibrio la fuerza mecnica tambin debe
invertir su sentido original. La fuerza mecnica ahora est entregando energa y el sistema se
comporta como un generador. Las condiciones que imperan en esta zona de trabajo son:

e > 0
e > V
i < 0

En la zona (3), tanto la velocidad, como la fuerza electromotriz son negativas. La fuerza mecnica est aplicada en el mismo sentido de la velocidad negativa en este caso, por lo tanto el
sistema mecnico entrega energa a la mquina. Simultneamente, la fuente de tensin entrega
potencia elctrica a la carga. En esta condicin toda la potencia entregada por el sistema mecnico y por el sistema elctrico se consume en la resistencia interna del conductor y se produce
un gran calentamiento de la mquina. Este estado se conoce con el nombre de frenado elctrico
y se caracteriza por las siguientes condiciones de operacin:

30

I.5 Sumario

e < 0
e < V
i > 0
I.5

S UMARIO

1. La conversin de energa es necesaria para utilizar los diferentes recursos disponibles en la


naturaleza. El campo magntico permite acumular energa con una densidad mayor que la
del campo elctrico, esto ha favorecido el desarrollo de las mquinas elctricas basadas en
campo magntico. El campo magntico acumula cantidades muy pequeas de energa al
ser comparado con las densidades obtenidas en otros procesos fsicos, esto hace necesaria
la conversin de energa para poder obtener electricidad a partir de estos procesos (Ver
Tabla 1.1).
2. Energa, fuerza y campo son conceptos fsico-matemticos de gran utilidad en los procesos que involucran conversin de energa.
3. La ley de Lorenz 1.1, las leyes de Maxwell 1.2 a 1.5 y las relaciones constitutivas de la
materia 1.6 a 1.8, conforman un marco matemtico que permite determinar el comportamiento de los convertidores electromecnicos de energa.
4. El convertidor electromecnico elemental est constituido por un conductor rectilneo movindose ortogonalmente a una velocidad v en un campo magntico B, en estas condiciones aparece en cada punto del conductor un campo elctrico constante de valor E = v B.
Cuando este conductor se conecta a un circuito elctrico externo, se obtiene una mquina
elctrica que es capaz de comportarse como motor, generador o freno.
5. El sistema formado por las ecuaciones internas y las relaciones con el exterior del convertidor, determinan completamente el comportamiento elctrico y mecnico del convertidor
electromecnico. Las ecuaciones internas definen la fuerza electromotriz e, y la fuerza
elctrica Fe sobre el conductor. Las relaciones externas son la ecuacin de Kirchoff para
el sistema elctrico y la segunda ley de Newton para el sistema mecnico.
6. La ecuacin caracterstica, permite obtener el punto de operacin que est determinado
por aquella velocidad donde se alcanza el equilibrio entre las fuerzas actuantes.
I.6

E JEMPLOS

RESUELTOS

Ejemplo 1: Conductor movindose en un campo uniforme


En la figura 1.13 se muestra el diagrama esquemtico de un convertidor electromecnico de
energa constituido por una fuente de tensin V = 1,0V y un conductor de masa M = 0,1 kg, que
se mueve ortogonalmente hacia un campo magntico uniforme B = 1,0 T . La resistencia de los
conductores est distribuida y depende de la longitud del camino que conecta la fuente con el

31

Captulo I Conversin de energa elctrica

Figura 1.13 Conductor movindose en un campo uniforme


conductor mvil (R = 1 + 2x ). Al movimiento del conductor se opone una fuerza mecnica
Fm = 1,0 N . En estas condiciones determine:
1. Las ecuaciones diferenciales completas que rigen el comportamiento del convertidor electromecnico.
2. La trayectoria descrita por el conductor mvil, si en el instante inicial t = 0, la posicin
de este elemento es x(0) = 1,0 m y parte de la condicin de reposo4 .
3. La trayectoria del conductor utilizando mtodos analticos de solucin suponiendo que
ahora la resistencia es concentrada y de valor constante 5 5 .
Solucin:
1.- Es necesario determinar tanto las ecuaciones internas 6 , como las relaciones con el mundo
externo7 . Las ecuaciones internas del convertidor son:
e=

Z l
0

Fe =

E dl = v B l

Z l
0

f dl = i B l

(1.39)

(1.40)

Las ecuaciones que relacionan al convertidor electromecnico con el mundo externo son:
i=

V e
R

Fe Fm = M a
4
5
6
7

(1.41)

(1.42)

Debido a la no-linealidad existente en el modelo matemtico del convertidor utilice un programa para resolver
numricamente este problema.
Las condiciones iniciales coinciden con las indicadas en el punto 2 de este problema.
Fuerza electromotriz y fuerza elctrica.
Ecuacin de la malla y segunda ley de Newton.

32

I.6 Ejemplos resueltos


Sustituyendo las ecuaciones internas 1.39 y 1.40 en las relaciones con el mundo externo 1.41 y
1.42 se obtiene:
i=

V vBl
R(x)

(1.43)

Fe Fm = i B l Fm = M a

(1.44)

Reemplazando el resultado de la expresin 1.43 en la ecuacin 1.44 se obtiene la ecuacin


diferencial que determina el comportamiento dinmico del conductor mvil:
Ma =

V vBl
V B l v (B l) 2
B l Fm =
R(x)
R(x)

(1.45)

2.- La resistencia de los conductores est distribuida y depende de la posicin x, la ecuacin


diferencial que define el comportamiento dinmico del conductor mvil es:
V B l x (B l) 2
+ Fm = 0 ;
M x
1 + 2x

x(0) = 1,0 m
x(0)
= 0,0 ms

(1.46)

Para resolver el problema planteado en la ecuacin 1.46 es necesario utilizar un mtodo numrico debido a la dependencia de la posicin en los coeficientes que acompaan a las derivadas de
esta variable de estado. La ecuacin 1.46 se puede descomponer en un sistema de dos ecuaciones
diferenciales de primer orden:
(

u =

1
M

V Blu(Bl)2
(1+2x)

x = u

Fm

x(0) = 1,0 m
x(0)
= 0,0 ms

(1.47)

El sistema de ecuaciones planteado en 1.47 puede ser integrado numricamente. En el listado se


reproduce un cdigo fuente MATLAB8 que permite realizar esta operacin. En la figura 1.14 se
observa el resultado de esta integracin, donde se ha representado la posicin de la pieza mvil
en funcin del tiempo para los datos de este problema:
En el listado 2 se presenta un programa que resuelve el mismo problema en el entorno de cdigo
abierto y libre distribucin Scilab 3.1.1, que est disponible9 para varios sistemas operativos
entre los cuales se puede destacar Windows, Linux, MacOS y Unix.
El cdigo incluido en el listado 3 resuelve este ejemplo utilizando el programa Octave10 , entorno
similar a Matlab, pero cuyo cdigo es abierto y de distribucin libre.
3.- Si la resistencia R no cambia con la posicin x, la ecuacin diferencial que determina el
comportamiento dinmico del convertidor es lineal:
8
9
10

Un cdigo parecido puede ser adaptado para resolver el problema utilizando herramientas de licencia libre y
cdigo abierto como pueden ser Octave y Scilab.
Las diferentes versiones y distribuciones pueden ser descargadas desde el enlace http://www.scilab.org.
http://www.octave.org.

33

Captulo I Conversin de energa elctrica

Algoritmo 1 Rutina para la solucin del problema utilizando el entorno MATLAB


%****************************************************************************
% Programa para el l ulo de la traye toria de un ondu tor
% que se mueve en un ampo magnti o uniforme. Matlab
%****************************************************************************
global m l B Fm V % Traspaso de variables a la fun in ondu tor
% Defini in de los parmetros y variables de entrada
m=0.1; l=1.0; B=1.0; Fm=.1; V=1;
% Condi iones ini iales de las variables de estado
y0=[0 1; % u(0)= 0 m/s x(0)=1.0 m
Ta=0:.001:10; % Defini in de tiempos y pasos de integra in
% Integra in de las variables de estado por un mtodo Runge-Kutta
% on paso variable
[T,X=ode23(' ondu tor',Ta,y0);
% Grfi o de las variables de estado
[AX,H1,H2=plotyy(T,X(:,1),T,X(:,2)) xlabel('tiempo (s)','FontName','times')
set(get(AX(1),'Ylabel'),'String','velo idad u(t) (m/s)','FontName','times')
set(get(AX(2),'Ylabel'),'String','posi ion x(t) (m)','FontName','times')
set(H2,'LineStyle',':')
grid
%*****************************************************************************
% E ua iones diferen iales del problema 1
fun tion pX= ondu tor(t,X)
global m l B Fm V % Traspaso de variables a la fun in ondu tor
% Conversin de las variables de estado a defini iones nemot ni as
u=X(1); x=X(2);
% Cl ulo de las derivadas de las variables de estado
pu=((V*B*l-(B*l)^2*u)/(1+2*x)-Fm)/m;
px=u;
% Asigna in de las variables de estado al ve tor de salida de la fun in
pX=[pu;px;
%*****************************************************************************

0.7

4.5

velocidad u(t) (m/s)

x(t)

u(t)
0.5

3.5

0.4

0.3

2.5

0.2

0.1

1.5

posicin x(t) (m)

0.6

1
10

tiempo (s)

Figura 1.14 Velocidad y posicin del conductor (solucin numrica utilizando Matlab 7.0)

34

I.6 Ejemplos resueltos

Algoritmo 2 Rutina para la solucin del problema utilizando el entorno SCILAB


// E ua iones diferen iales del problema 1 programado en el entorno S ilab
//
// Defini in de la fun in ondu tor
fun tion pX= ondu tor(t,X)
global m l B Fm V // Traspaso de variables a la fun in ondu tor
// Conversin de las variables de estado a defini iones nemot ni as
u=X(1); x=X(2);
// Cl ulo de las derivadas de las variables de estado
pu=((V*B*l-(B*l)^2*u)/(1+2*x)-Fm)/m;
px=u;
// Asigna in de las variables de estado al ve tor de salida de la fun in
pX=[pu;px;
endfun tion
//********************************************************************
// Programa para el l ulo de la traye toria de un ondu tor
// que se mueve en un ampo magnti o uniforme. S ilab 3.1.1
//********************************************************************
global m l B Fm V // Traspaso de variables a la fun in ondu tor
// Defini in de los parmetros y variables de entrada
m=0.1; l=1.0; B=1.0; Fm=.1; V=1;
// Condi iones ini iales de las variables de estado
y0=[0;1; // u(0)= 0 m/s x(0)=1.0 m
Ta=0:.001:10; // Defini in de tiempos y pasos de integra in
// Integra in de las variables de estado por el mtodo Runge-Kutta
X=ode(y0,0,Ta, ondu tor);
// Grfi o de las variables de estado
subplot(121)
plot2d(Ta,X(1,:)',frameflag=6)
xtitle('velo idad [m/s','t [s','u(t)')
xgrid(2)
subplot(122)
plot2d(Ta,X(2,:)',frameflag=6)
xtitle('posi ion [m','t [s','x(t)')
xgrid(2)
//*********************************************************************

35

Captulo I Conversin de energa elctrica

velocidad [m/s]

posicin [m]

0.7

4.5

4.0

0.6

3.5
0.5
3.0
x(t)

u(t)

0.4
2.5

0.3
2.0
0.2
1.5
0.1

1.0

0.0

0.5
0

5
t [s]

10

5
t [s]

Figura 1.15 Resultados del ejemplo obtenidos en el entorno Scilab-3.0

36

10

I.6 Ejemplos resueltos

Algoritmo 3 Rutina para la solucin del problema utilizando el entorno OCTAVE


#
#
#
#
#
#
#
#

#
#
#
#

***********************************************************************
Programa para el l ulo de la traye toria de un ondu tor
que se mueve en un ampo magnti o uniforme. O tave
***********************************************************************
global m l B Fm V # Traspaso de variables a la fun in ondu tor
Defini in de los parmetros y variables de entrada
m=0.1; l=1.0; B=1.0; Fm=.1; V=1;
Condi iones ini iales de las variables de estado
y0=[0;1; # u(0)= 0 m/s x(0)=1.0 m
Ta=linspa e(0,10,200); # Defini in de tiempos y pasos de integra in
Integra in de las variables de estado por el mtodo Runge-Kutta
X=lsode(' ondu tor',y0,Ta);
Grfi os de la velo idad y posi in
gset nokey
gset term posts ript olor
plot(Ta,X(:,1))
grid(); xlabel('tiempo [s');ylabel('velo idad [m/s')
gset output "ej_1_1a_o tave.ps"
replot
plot(Ta,X(:,2))
xlabel('tiempo [s');ylabel('posi ion [m/s')
gset output "ej_1_1b_o tave.ps"
replot
***********************************************************************
fun tion pX= ondu tor(X,t)
global m l B Fm V # Traspaso de variables a la fun in ondu tor
Conversin de las variables de estado a defini iones nemot ni as
u=X(1); x=X(2);
Cl ulo de las derivadas de las variables de estado
pu=((V*B*l-(B*l)^2*u)/(1+2*x)-Fm)/m;
px=u;
Asigna in de las variables de estado al ve tor de salida de la fun in
pX=[pu;px;
endfun tion

37

Captulo I Conversin de energa elctrica

0.7

0.6

velocidad [m/s]

0.5

0.4

0.3

0.2

0.1

0
0

10

tiempo [s]

4.5

posicin [m/s]

3.5

2.5

1.5

1
0

10

tiempo [s]

Figura 1.16 Resultados del ejemplo utilizando el entorno Octave 2.1.50

38

I.6 Ejemplos resueltos

V Bl
(B l) 2
x
+ Fm = 0
M x +
R
R

x(0) = 1,0 m
x(0)
= 0,0 ms

(1.48)

Sustituyendo los valores de los parmetros M y l, as como de las fuentes V , B y Fm en la


expresin 1.48, se obtiene:

x + 2x = 1 ;

x(0) = 1,0 m
x(0)
= 0,0 ms

(1.49)

La expresin 1.49 se puede resolver ms fcilmente si se sustituye la definicin de la velocidad


u:

u + 2u = 1 ; u(0) = 0,0

m
s

(1.50)

Aplicando la transformada de Laplace, se obtiene el siguiente resultado:


sU (s) + 2U (s) =
1
1
U (s) =
=
s(s + 2) 2

1
s

1
1

s s+2

(1.51)


(1.52)

Antitransformando la expresin 1.52 se obtiene la velocidad u(t):


u(t) =

m
1
1 e2t
2
s

(1.53)

La posicin se obtiene integrando la solucin 1.53:


x(t) = x(0) +

Z t
0



1 2t 1
1
t+ e
u( )d = 1 +
2
2
2

(1.54)

En la figura se puede observar esta solucin obtenida numricamente con el programa anterior.
Ejemplo 2: Rueda de Faraday
En la figura 1.18 se muestra el diagrama esquemtico de un convertidor electromecnico de
energa constituido por una rueda metlica cuyos radios conductores unen el eje con la periferia.
En cada momento uno de los conductores del dispositivo se encuentra en presencia de un campo
magntico uniforme B = 1 T . Entre los extremos del conductor activo se aplica una fuente de
tensin V = 1V . La resistencia equivalente entre el eje y el punto de contacto perifrico es de
0, 1 . La longitud de cada uno de los radios es de 1,0 m. La mquina mueve un ventilador cuyo
par mecnico es proporcional al cuadrado de la velocidad angular Tm = km2 . Si el convertidor
39

Captulo I Conversin de energa elctrica

0.6

10

u(t)

0.4

0.3

x(t)

posicin x(t) (m)

velocidad u(t) (m/s)

0.5

0.2

0.1

0
10

tiempo (s)

Figura 1.17 Velocidad y posicin del conductor en funcin del tiempo (solucin analtica)
gira a la velocidad sincrnica del sistema, se obtiene un par mecnico de 0, 1 Nm. Se puede considerar que el nmero de radios de la rueda es prcticamente infinito, de tal forma que siempre
existe un radio bajo el campo magntico uniforme. La masa de la rueda se puede considerar
distribuida y tiene por valor 0, 2 kg. Con estos parmetros determine:
1. Las ecuaciones diferenciales completas que rigen el comportamiento del convertidor electromecnico.
2. El punto de operacin (m , i) cuando se acopla el ventilador al eje de la rueda.
3. Determine la velocidad angular y la corriente en funcin del tiempo, si el dispositivo parte
del reposo en el instante inicial, en vaco y cargado con el ventilador.
Solucin:
1.- Al igual que en el ejemplo anterior, en este caso tambin es necesario determinar las ecuaciones internas y las relaciones con el mundo externo. Como el movimiento de los conductores es
circular, el anlisis dinmico se realiza sobre el balance de par sobre el eje mecnico del dispositivo. Para comprender el problema es necesario analizar en detalle el diagrama del conductor
activo en un instante determinado. En la figura 1.19 se han representado esquemticamente las
consideraciones fundamentales.
En la figura 1.19 se puede observar que a la distancia r del eje de giro, el mdulo de la velocidad
de giro es u = m r, y el vector sale del plano del papel. Con esa velocidad y el campo B se
obtiene en ese mismo punto el campo elctrico E. La circulacin de la corriente i(t) por todo el

40

I.6 Ejemplos resueltos

Figura 1.18 Diagrama esquemtico de la rueda de Faraday

Figura 1.19 Diagrama esquemtico del conductor activo

41

Captulo I Conversin de energa elctrica


conductor produce en cada punto del mismo un diferencial de fuerza elctrica dFe = i Bdr, y
un diferencial de par elctrico d e = r dFe . Con las consideraciones anteriores, las ecuaciones
internas del convertidor son:

e=

Z l
0

Te =

E dr =
Z l
0

Z l
0

d e =

u B dr =

Z l
0

r dFe =

Z l
0

Z l
0

1
m rBdr = m Br2
2

l

1
r i B dr = iBr2
2

1
= m Bl 2
2
0

(1.55)

l

(1.56)

1
= iBl 2
2
0

Las ecuaciones externas del convertidor son:


V = Ri + e

(1.57)

Te Tm = J m

(1.58)

La inercia de una masa distribuida en una rueda es 12 Mr2 . Sustituyendo los resultados de las
expresiones 1.55 a 1.57 en la ecuacin diferencial 1.58 se obtiene la ecuacin diferencial que
determina el comportamiento dinmico del convertidor analizado:
1
J m =
2

V 21 m Bl 2
R

Bl 2 km2

(1.59)

Reagrupando los trminos en velocidad angular de la ecuacin diferencial 1.59 se obtiene:


J m +

1 V Bl 2
1 B2 l 4
m + km2 =
4 R
2 R

(1.60)

Para determinar el valor del coeficiente k del ventilador es necesario calcular la expresin de
la velocidad sincrnica en funcin de los parmetros y variables conocidas, porque un dato del
problema es que a la velocidad sincrnica del sistema, el ventilador requiere 0,1 Nm de par
mecnico. Para determinar la velocidad sincrnica es necesario eliminar de la expresin 1.60
la contribucin del par mecnico11 y considerar el punto de equilibrio en rgimen permanente
pm = 0, as se obtendra:

ms =

2V
rad
= 2,0
2
Bl
s

(1.61)

Como se conoce que a esta velocidad el ventilador requiere del par mecnico se puede determinar el coeficiente k de la bomba:
k=
11

Condicin de vaco km2 = 0.

42

Nm.s2
0,1 Nm
Tm
=
0,025
=
2 m (2,0 ms )2
rad 2

(1.62)

I.6 Ejemplos resueltos


En valores numricos la ecuacin diferencial 1.60 quedara de la forma siguiente:

m + 25m + 2,5 2 m = 50

(1.63)

Para poder resolver la ecuacin diferencial 1.63 y obtener el comportamiento dinmico del convertidor es necesario incluir la condicin inicial del problema m (0) = m0 .
2.- El punto de operacin se determina directamente de la solucin de rgimen permanente de
la ecuacin diferencial 1.63, de esta forma:
2,5

m + 25m 50 =

0 m =

1,708 rad
11,708 s

La solucin negativa no se considera en este caso, debido a que el ventilador se utiliza para
impulsar aire y para esto debe girar en sentido positivo. Cuando la velocidad angular de la
mquina es conocida se puede determinar la corriente de operacin:
V e V 12 m B l 2 1,0 21 1,708 1 12
=
=
= 1,459 A
i=
R
R
0,1
3.- Si el convertidor se encuentra en vaco, la ecuacin diferencial que determina el comportamiento del sistema es lineal. En la parte 1 de este problema fue determinada la velocidad angular
sincrnica ms = 2,0 rad
s , que corresponde en este caso de excitacin constante a una solucin
particular de la ecuacin diferencial. Es necesario superponer la solucin homognea y determinar a partir de las condiciones iniciales, el coeficiente indeterminado correspondiente. De esta
forma, la solucin homognea es:

m + 25m = 0 mh = Ae25t
Y la solucin general es:

m (t) = mh (t) + mp(t) = Ae25t + 2,0

rad
s

Recordando que el sistema parte del reposo, se obtiene la siguiente solucin en la condicin de
vaco:

m (t) = 2,0 (1 e25t )

rad
s

Cuando el ventilador est acoplado al convertidor, la solucin analtica es posible, sin embargo
la solucin numrica puede ser til en este caso. Un algoritmo semejante al utilizado en el
ejemplo N. 1 puede ser adaptado para resolver este nuevo problema. La funcin que calcula las
derivadas debera ser programada con la siguiente ecuacin:

m = 50 25m 2,5 2 m
En la figura 1.20 se han representado las dos soluciones de este problema, en vaco y en carga.

43

Captulo I Conversin de energa elctrica

1.5

1.5
carga

0.5

0.5

0.05

0.1

0.15
0.2
tiempo (s)

0.25

0.3

velocidad angular w(t) vaco (rad/s)

velocidad angular w(t) carga (rad/s)

vaco

0
0.35

Figura 1.20 Grfico de la respuesta dinmica del convertidor en las dos condiciones de operacin, vaco y cargado con el ventilador.
I.7

E JERCICIOS

PROPUESTOS

1. Repetir el ejemplo N. 1 con las siguientes variantes:


a) Considerando que la fuerza mecnica es constante de valor 0,1N.


b) Cambiando la tensin de la fuente V = 1, 0, 21 , 2 V.


c) Asumiendo la densidad de campo magntico B = 14 , 12 , 34 , 54 T.
2. Repetirel ejemplo N. 1 suponiendo
la fuente de voltaje es variable en el tiempo
 1 1 que
v(t) = 2V sen t, donde: = 10
, 2 , 1 rad
s .
3. Repetir el ejemplo N. 1 suponiendo que el conductor se desplaza inclinado en el ngulo
, tal como se ilustra en la figura 1.21. La resistencia del elemento mvil es proporcional
a su longitud medida entre los puntos de contacto con los conductores riel.
4. Repetir el ejemplo N. 2 con las siguientes variantes:
a) Considerando que la carga mecnica es constante de valor 0, 1 Nm.

44

I.7 Ejercicios propuestos

Figura 1.21 Diagrama esquemtico del ejercicio N. 3




b) Cambiando la tensin de la fuente V = 1, 0, 21 , 2 .


c) Asumiendo la densidad de campo magntico B = 14 , 21 , 34 , 54 T .
5. Repetirel ejemplo N. 2 suponiendo
la fuente de voltaje es variable en el tiempo
 1 1 que
rad
v(t) = 2V sen t, donde: = 10 , 2 , 1 s .
6. Repetir el ejemplo N. 2 suponiendo que el campo magntico aumenta linealmente desde
el centro de la rueda hacia la periferia, siendo 0,5 T en el eje y 1,0 T en el otro extremo.
7. Un conductor semicircular de radio 0,5 m rota en un campo magntico que vara sinusoidalmente en el tiempo a una frecuencia de 100 Hz. El conductor tiene una resistencia de
2 y est alimentado por una fuente de corriente alterna de 10 V y de la misma frecuencia.
Si se desprecia la inductancia del conductor, y se considera operacin en rgimen permanente del convertidor, determine:

a) El par elctrico del convertidor en funcin de su velocidad angular.


b) La velocidad de operacin cuando se acciona una carga mecnica al 75 % del par
mximo como motor.
c) La tensin inducida sobre el conductor a una velocidad de 628 rad
s .
d) La corriente necesaria en el arranque, en funcin de la posicin inicial del conductor.

8. Un conductor rectilneo de longitud l se mueve perpendicularmente a un campo magntico uniforme de magnitud B. El conductor posee una resistencia R y est excitado
mediante una fuente de tensin continua V , que se utiliza para acelerar la mquina hasta la velocidad de operacin. Esta velocidad se establece cuando el conductor vence una
fuerza mecnica uniforme y constante que se opone al movimiento del conductor. En estas
condiciones determine:

45

Captulo I Conversin de energa elctrica


a) La ecuacin mecnica caracterstica y la velocidad de operacin de la mquina.
b) Las ecuaciones diferenciales completas del convertidor electromecnico.
c) La mxima velocidad que puede adquirir el convertidor cuando se debilita la densidad de campo magntico B.
d) Las condiciones que se deben establecer sobre las funciones forzantes para obtener
la operacin del convertidor en la condicin de freno, si originalmente la mquina se
encuentra a velocidad constante como motor.

46

Bibliografa

A SIMOV, I., Understanding Physics: Light, Magnetic and Electricity, George Allen & Unwin,
vol. 2, London, 1966.
Encyclopaedia Britannica, William Benton Publisher, vol. 9, pp. 802-806, Chicago, 1964.
H ALLIDAY, D. & R ESNICK , R., Fsica, Parte I y II, John Wiley & Sons, Mxico, 1974.
H AYT, W. H. J R ., Teora electromagntica, McGraw-Hill, quinta edicin, segunda edicin en
espaol, Mxico, 1991.
J OHNK , C. T. A., Engineering Electromagnetic Fields & Waves, Wiley International Edition,
New York, 1975.
M AXWELL , J. C., A Treatise on Electricity and Magnetism, Dover Publications, Unabridged
Third Edition, Volume one and two, New York, 1954.

47

Captulo I Conversin de energa elctrica

48

CAPTULO II

Fundamentos de conversin

En el captulo anterior se analiz el comportamiento dinmico de un convertidor electromecnico elemental. El planteamiento de estas ecuaciones fue una tarea realizable con nociones
bsicas de clculo numrico aplicado a unas condiciones geomtricas simples. Las ecuaciones
de Maxwell en su forma diferencial y la relacin de Lorenz se aplican infinitesimalmente y estn sujetas a condiciones de contorno que no siempre pueden ser integradas directamente, al
menos mediante herramientas analticas. Los convertidores electromagnticos prcticos estn
constituidos por muchos conductores y materiales inmersos en campos elctricos y magnticos que pueden ser muy difciles de modelar mediante la aplicacin directa de las leyes fsicas
fundamentales.
Un vendedor en una ferretera no est obligado a registrar la ganancia unitaria de cada tornillo
que vende, ni a calcular las ganancias totales de un determinado mes sumando cada una de estas ganancias parciales. Desde hace muchos siglos los comerciantes confan en los principios
de contabilidad general para conocer la utilidad obtenida en la actividad econmica que realizan. Un mtodo similar a los balances contables permite determinar el comportamiento de los
convertidores electromecnicos: el balance de energa1 .
Este captulo presenta las herramientas fundamentales para el anlisis prctico de los convertidores electromecnicos de energa. Balances de energa, balances de coenerga y el principio de
los trabajos virtuales permiten la determinacin de las fuerzas elctricas involucradas en la conversin electromecnica de energa. El mtodo desarrollado en este captulo permite generalizar
una tcnica que puede ser aplicada al anlisis de cualquier convertidor electromecnico.
1

Tal vez no sea una casualidad que Lavoasier, quien descubriera en el siglo XVIII el principio de conservacin de
la masa y figura fundamental en el desarrollo terico de la Qumica, fuese contador de profesin. Ya en el siglo
XX Einstein ampli el mbito de este principio al incluir la energa en el balance.

49

Captulo II Fundamentos de conversin

Figura 2.1 Mquina elctrica y algunos de sus posibles ejes


II.1

ENERGA

Y COENERGA EN EL CAMPO

Un convertidor electromecnico de energa es una mquina elctrica. En general una mquina


elctrica posee varios ejes o puertos por los cuales fluye la energa. Estos ejes pueden ser de dos
tipos: elctricos o mecnicos. Esquemticamente se representan en la figura 2.1.
En los ejes elctricos de la mquina, las interacciones se analizan conociendo las corrientes
y tensiones. En los ejes mecnicos las variables que determinan la condicin de operacin de
la mquina son las velocidades y fuerzas, si el movimiento es lineal, o el par2 y la velocidad
angular, si el movimiento es rotativo.
La mquina elctrica ms simple requerira al menos un eje elctrico y un eje mecnico. El
esquema bsico de esta mquina se ilustra en la figura 2.2: We es el incremento de energa
elctrica que entra en el convertidor por el eje elctrico, Wm es el variacin de energa mecnica
que sale por el eje mecnico y Wc es el aumento de energa que se almacena en los campos
elctrico y magntico de la mquina.
En las mquinas elctricas, no toda la energa introducida en los ejes elctricos se entrega en
los ejes mecnicos o viceversa. Es necesario que parte de la energa elctrica se almacene en los
campos electromagnticos del convertidor. En un balance de la energa en la mquina elctrica
es necesario tener en cuenta la parte de la energa que fluye hacia y desde los campos elctricos
y magnticos. En la figura 2.2 esta energa se representa por Wc .
En el siguiente ejemplo se compara la capacidad de acumular energa que tienen los campos
elctrico y magntico respectivamente:
Como se estudia en Teora Electromagntica, la energa acumulada en el campo
elctrico viene dada por la expresin:
1 D2 1 2
1
= E
Wcelct = D E =
2
2
2
2

En algunos textos se utiliza la palabra torque, pero este vocablo no se ha incorporado an al idioma espaol.

50

II.1 Energa y coenerga en el campo

Figura 2.2 Mquina elctrica con un eje elctrico y un eje mecnico

pero la resistencia dielctrica del aire es aproximadamente 3 106 Vm , y consideranF


do que la permitividad del aire es igual a la del vaco, es decir = 8,85 1012 m
,
la mxima densidad de energa del campo elctrico en el aire a presin atmosfrica,
sin que se produzca arco disruptivo es:
Wcelct = 39,82

J
m2

La energa almacenada en el campo magntico es:


1
1 B2
Wcmag. = B H =
2
2
La permitividad del aire es 0 = 4 107 , y considerando una densidad de flujo
de 1,0 Wm2b , que es un valor frecuentemente encontrado en dispositivos de conversin,
se obtiene una energa de:
Wcmag. = 3,98 105

J
m3

Como se puede observar, los dispositivos magnticos que utilizan densidades de


flujo B conservadoras, pueden contener 104 veces la densidad de energa mxima
disponible en el campo elctrico de una mquina electrosttica. Por esta razn las
mquinas que utilizan el campo magntico en la conversin de la energa elctrica
son mucho ms pequeas que una mquina equivalente que utilice campo elctrico.
Del principio de conservacin de la energa se determina:

51

Captulo II Fundamentos de conversin

We = Wc + Wm

(2.1)

La energa acumulada en el campo no puede ser medida, pero es posible calcularla por la diferencia entre la energa elctrica y la mecnica:
Wc = We Wm

(2.2)

La energa elctrica se determina a partir de la integral de la potencia elctrica en el tiempo.


Esta energa puede ser calculada directamente en el eje elctrico de la mquina a partir de las
medidas de tensin y corriente instantnea:
We =

Z t
0

Pe ( )d =

Z t
0

v( ) i( )d

(2.3)

Transformando las variables de la expresin anterior se puede reescribir esta ecuacin en una
forma ms conveniente. Considerando que el sistema es conservativo, es decir, no existen prdidas en elementos resistivos, la tensin v(t) aplicada a la mquina y la fuerza electromotriz
inducida son iguales, y por lo tanto:
v(t) = e(t) =

d
dt

(2.4)

En este caso, a partir de 2.3 y 2.4 se determina que:


We =

Z t
0

v( ) i( )d =

Z t
d
0

dt

i( )d =

Z (t)
(0)

i(x, )d

(2.5)

La ecuacin 2.5 indica que para obtener la energa elctrica que fluye por la mquina es necesario
conocer solamente la dependencia de la corriente i(x, ) con respecto al flujo y a la posicin
x del convertidor.
Para determinar la variacin de la energa mecnica es necesario conocer la velocidad y la fuerza
en funcin del tiempo:
Z
Z
t

Wm =

Pm ( )d =

)d
F( ) x(

(2.6)

Realizando cambio de variables sobre la ecuacin 2.6, se obtiene:


Wm =

Z t
0

dx
F( ) d =
d

Z x(t)
x(0)

F(x, )dx

(2.7)

Para analizar las relaciones anteriores se puede utilizar como ejemplo el electroimn que se
ilustra en la figura 2.3. All se ha representado un grfico de la relacin existente entre los
enlaces de flujo y la corriente i, para dos condiciones extremas de la posicin relativa del yugo
del electroimn. Para la misma corriente i, al disminuir la distancia x, disminuye la reluctancia
y se incrementan los enlaces de flujo .
En el grfico i, la regin sombreada representa la integral de la corriente i( ) con respecto a
para una posicin x fija. Como se ha determinado en la ecuacin 2.5, esta regin representa la

52

II.1 Energa y coenerga en el campo

Figura 2.3 Diagrama i de un electroimn elemental


variacin de la energa elctrica en un circuito magntico que se energiza manteniendo constante
la posicin del yugo (x).
En un sistema conservativo, la energa es una funcin de estado. Esto quiere decir que en estos
sistemas el incremento de energa acumulada no depende de la trayectoria utilizada para alcanzar un determinado estado, sino del valor de las variables en los estados iniciales y finales del
proceso.
Para determinar la energa acumulada en el campo, es necesario calcular la diferencia entre
las energas elctrica y mecnica del sistema despus del proceso. Si el sistema mecnico est
detenido, no existe variacin en la energa mecnica del convertidor y por lo tanto toda la energa
elctrica que entra en la mquina se convierte en energa acumulada en el campo, entonces:
We =

Z (t)
(0)

i(x, )d = Wc , si x = cte

(2.8)

La ecuacin 2.8 se puede integrar por partes y se obtiene:


(t)

Wc = i(x, ) | (0)

Z i(t)
i(0)

(x, i)di

(2.9)

En la ecuacin 2.9, el trmino integral de define como coenerga en el campo y se expresa como

Wc . En la figura 2.4 se observa que la coenerga es el rea bajo la caracterstica i.

En la figura 2.4 se observa que un sistema electromecnico donde la posicin x es constante


cumple la siguiente relacin:

i = Wc + Wc
(2.10)
De las definiciones anteriores de energa y coenerga en el campo magntico se destacan las
siguientes observaciones:
1. Para la energa, el enlace de flujo es la variable independiente, y la corriente i es la
variable dependiente.

53

Captulo II Fundamentos de conversin

Figura 2.4 Energa y coenerga en el campo

Figura 2.5 Convertidor electromecnico lineal


2. Para la coenerga, la corriente i es la variable independiente y el enlace de flujo es la
variable dependiente.
Si el sistema fsico es lineal, es decir, si la relacin entre los enlaces de flujo y la corriente i
del convertidor electromecnico es proporcional, la energa y la coenerga son iguales, esto se
puede observar en la figura 2.5.
En la figura 2.6, se ilustra un electroimn cuyo yugo est conectado a un sistema mecnico
constituido por un resorte unido slidamente en un extremo al propio yugo y en el otro a un
sistema en reposo. Los valores de la posicin del yugo y de los enlaces al inicio del proceso, en
el instante de tiempo t son:
x(0) = x0 x(t) = x f
(0) = 0 (t) = f
54

II.1 Energa y coenerga en el campo

Figura 2.6 Electroimn en un sistema mecnico


Para calcular el incremento de energa acumulada en el campo hasta el instante de tiempo t es
necesario considerar que en el proceso real vara la potencia elctrica y la potencia mecnica.
Es posible realizar un experimento terico para determinar la energa acumulada en el campo.
Dicho experimento comprende dos fases:
1. Desplazamiento de la pieza mvil desde x(0) a x(t) con el circuito elctrico desenergizado,
es decir, con i(0) = 0. En estas condiciones la fuerza elctrica Fe es cero y no es necesario
consumir energa mecnica para desplazar el yugo a la posicin final x(t).
2. Se fija la posicin final de la pieza mvil y se incrementan los enlaces de flujo desde el
valor 0 hasta f .
En las condiciones del experimento terico anterior, para determinar la variacin de la energa en
el campo de la mquina es suficiente evaluar la integral de la corriente con respecto a los enlaces
de flujo cuando la pieza mvil est en su posicin final x f . La trayectoria real depende de la
mquina y de las condiciones de frontera o ligazn, pero en cualquier caso es posible evaluar
la energa almacenada en el campo. En la figura 2.7 se presenta grficamente el experimento
terico realizado para la determinacin de la energa en el campo. De esta forma, la evaluacin
se reduce a determinar el rea sombreada en la figura.
Mediante el convertidor electromecnico ilustrado en la figura 2.6, se puede realizar un anlisis
ms complejo de los procesos involucrados. Considerando que inicialmente el yugo se encuentra
fijo en la posicin x, al cerrar el interruptor, la corriente aumenta exponencialmente cuando el
sistema tiene un comportamiento lineal:

= Li

(2.11)

La ecuacin de mallas correspondiente a la red elctrica es:


v = Ri+e = Ri+

d
dt

(2.12)

55

Captulo II Fundamentos de conversin

Figura 2.7 Determinacin de la energa en el campo


Sustituyendo la expresin 2.11 en la ecuacin 2.12 se obtiene la ecuacin diferencial que rige el
comportamiento de la corriente elctrica en el circuito:
v = Ri+L

di
dt

(2.13)

La solucin en el tiempo de la ecuacin diferencial 3.14 es una corriente exponencial cuyo valor
en rgimen permanente es:
v
(2.14)
if =
R
El conocimiento de la trayectoria de la corriente en funcin del tiempo no es necesaria por las
consideraciones realizadas previamente relativas a las funciones de estado.
Una vez que la corriente i aumenta hasta su valor final i f , con la posicin x1 fija, se permite
el movimiento de la pieza hasta una segunda posicin x2 . Despus que finalizan los procesos
transitorios, el sistema alcanza el rgimen permanente en la segunda posicin con una corriente
i igual a la primera, debido a que en rgimen permanente no varan los enlaces de flujo. En la
figura 2.8 se muestra la trayectoria seguida por la corriente.
En la figura 2.8 se han marcado dos trayectorias tentativas de la corriente cuando la pieza mvil pasa de la posicin x1 a la x2 . Para determinar la trayectoria correcta (A) o (B), se debe
recordar que:
V e
i=
(2.15)
R
El paso de x1 a x2 requiere del incremento de los enlaces de flujo y por lo tanto la derivada
de estos enlaces (e), es positiva durante el proceso transitorio, por esta razn inicialmente la
corriente i disminuye y la trayectoria se ajusta al caso (A). Cuando la pieza alcanza la posicin
final, el enlace en rgimen permanente no vara y la corriente regresa a su valor inicial.
El proceso seguido por el convertidor ilustrado en la 2.8 es el siguiente:
Originalmente el sistema est desenergizado, la pieza mvil se encuentra en la posicin
inicial x1 y al cerrar el interruptor que alimenta el magneto, aumenta la corriente hasta

56

II.1 Energa y coenerga en el campo

Figura 2.8 Trayectoria de la corriente en una energizacin con desplazamiento

Figura 2.9 Movimiento de apertura del yugo


el valor i f . En ese momento se permite la reduccin de la posicin del yugo hasta x2
por efecto de la fuerza electromagntica y finalmente se abre el interruptor del circuito
elctrico para desenergizar el sistema. El rea sombreada en la figura 2.8 representa la
energa elctrica que el convertidor cede al sistema mecnico.
Otra posibilidad es que el dispositivo mvil se encuentre inicialmente en la posicin x2 ,
se energice el circuito, se desplace la pieza mvil hasta la posicin x1 y finalmente se
desenergice el circuito. En este caso, la trayectoria se representa en la figura 2.9. Al desplazar la pieza mvil desde la posicin inicial a la posicin final, es necesario reducir los
enlaces de flujo y por esta razn se induce en el circuito elctrico una fuerza electromotriz negativa que aumenta transitoriamente la corriente, para regresar nuevamente al valor
primitivo i f , cuando cesa la variacin de los enlaces de flujo.
Ahora bien, si en la primera condicin analizada, se desea desplazar el yugo desde la posicin x2
a x1 , manteniendo constante la corriente, es necesario mover la pieza muy lentamente, para que

57

Captulo II Fundamentos de conversin

Figura 2.10 Desplazamiento del yugo a diferentes velocidades

Figura 2.11 Apertura del yugo a velocidad cero e infinita


varen los enlaces de flujo, pero su derivada sea prcticamente cero. A medida que el dispositivo
se cierra con mayor velocidad, las trayectorias se muestran en la figura 2.10.
La trayectoria D corresponde a un yugo que se cierra a velocidad infinita, es decir la pieza pasa
de la posicin x1 a la x2 en un tiempo cero. En esta situacin lmite, ni el flujo ni el tiempo
han variado al pasar de la posicin x1 a x2 y por lo tanto la derivada del enlace de flujo con
respecto al tiempo tiene un valor finito que permite que la corriente en el circuito elctrico vare
instantneamente desde i0 a ix , como se observa en la figura 2.10.
Si la pieza mvil sigue la segunda trayectoria, es decir, se mueve de la posicin x2 a x1 y todo
esto a velocidad prcticamente cero, el recorrido se efecta a corriente constante. En la figura
2.11 se puede observar el proceso cuando el yugo se desplaza a una velocidad tericamente
infinita.
Si la velocidad de la pieza es tericamente infinita, la corriente crece considerablemente debido
a que la fuerza electromotriz es negativa y se superpone a la tensin aplicada por la fuente.

58

II.2 Balance energtico

Figura 2.12 Balance energtico del electroimn


Cuando la saturacin del circuito magntico es muy intensa, los picos de corriente que aparecen
en la operacin del electroimn pueden ser de gran magnitud.
II.2

BALANCE

ENERGTICO

Mediante el diagrama de la figura 2.12 se puede realizar un balance energtico del proceso
descrito en la seccin anterior.
La operacin del electroimn se divide en tres trayectorias:
1. Trayecto O A: Desde que se cierra el interruptor, energizando el circuito elctrico con el
yugo en la posicin x1 .
2. Trayecto A B: Cuando se permite el movimiento mecnico de la pieza hasta alcanzar la
posicin x2 .
3. Trayecto B O: Representa la apertura del interruptor para desenergizar el sistema.
A partir de la ecuacin 2.5 se puede calcular el incremento de energa elctrica por tramos de la
siguiente forma:
WeOA =
WeAB =
WeBO =

Z 1

i(x1 , )d = OAD

(2.16)

i(x, )d = DABC

(2.17)

i(x2 , )d = BCO

(2.18)

Z 2
1

Z 2
2

La energa acumulada en el campo viene determinada por:


WcOA = OAD

(2.19)

59

Captulo II Fundamentos de conversin


La ecuacin 2.19 determina la energa acumulada en el campo, debido a que en este proceso la
posicin se mantiene constante y la variacin de energa mecnica Wm es nula. Toda la energa
elctrica se almacena en el campo del convertidor. De las otras trayectorias se deduce que:
WcAB = WcB WcA = OBC OAD

(2.20)

WcBO = BCO = WeBO

(2.21)

El clculo del incremento de energa mecnica, se obtiene de las diferencias entre los incrementos de energa elctrica y energa acumulada en el campo del convertidor durante todo el
trayecto:
Wm = We Wc
(2.22)
Donde:

We = WeOA + WeAB + WeBO = OAD + DABC BCO

Wc = WcOA + WcAB + WcBO = OAD + OBC OAD BCO

(2.23)
(2.24)

De las ecuaciones 2.22, 2.23 y 2.24 se obtiene:


Wm = (OAD + DABC BCO) (OAD + OBC OAD BCO)
Wm = DABC + OAD OBC = OABO

(2.25)

La expresin 2.25 indica que el incremento en la energa mecnica en el proceso es igual al rea
encerrada en la trayectoria OABO, que es precisamente la regin sombreada en el esquema de la
figura 2.12. En este caso, la energa mecnica realiza un trabajo positivo porque la fuerza sobre el
yugo y el desplazamiento tienen la misma direccin. Si inicialmente el convertidor tiene el yugo
muy cerca del electroimn y se alejan estas dos piezas, el trabajo mecnico realizado es negativo,
ya que en este caso la fuerza sobre la pieza mvil tiene direccin opuesta a su desplazamiento.
En la figura 2.13 se muestra esta condicin. El rea sombreada corresponde al incremento de la
energa mecnica, y el sentido del recorrido determina el signo del trabajo realizado, negativo
segn las agujas del reloj y positivo en el sentido contrario.
En la figura 2.14 se representa el proceso electromecnico descrito anteriormente pero el movimiento de acercamiento del yugo se ha realizado a una velocidad terica infinita. En este caso
los enlaces de flujo no pueden variar instantneamente y de acuerdo con la ecuacin 2.5, el incremento de energa elctrica en este tramo es cero. Recordando la expresin 2.1, se determina
para los procesos electromagnticos que mantienen constante el enlace de flujo:
Wm = Wc , si = cte.

(2.26)

Por esta razn, si el dispositivo se desplaza manteniendo constante el enlace de flujo, no se incrementa la energa elctrica y toda la energa mecnica empleada en el movimiento es suministrada
por el campo de la mquina.
Para calcular la fuerza Fe , se reducen los incrementos de energa mecnica y de energa en el
campo a valores diferenciales. Recordando que la energa acumulada en el campo de la mquina

60

II.2 Balance energtico

Figura 2.13 Trabajo mecnico negativo

Figura 2.14 Clculo de la energa mecnica en un desplazamiento rpido del yugo

61

Captulo II Fundamentos de conversin


depende de los enlaces de flujo y de la posicin de la pieza mvil:
Wc = Wc (x, )

(2.27)

El trabajo mecnico se define en su forma diferencial como:


dWm = Fe dx

(2.28)

A partir de las ecuaciones 3.27 y 3.29 se obtiene:


dWm = Fe dx = dWc (x, ) , si = cte.

(2.29)

El diferencial total de la energa en el campo es:


dWc (x, ) =

Wc
Wc
dx +
d
x

(2.30)

Como el enlace se considera constante, el segundo trmino de la sumatoria de la ecuacin 2.30


es nulo y por lo tanto se deduce de 2.29 y de 2.30 que:
Fe dx =

Wc (x, )
dx , si = cte.
x

(2.31)

Por identificacin de trminos en la ecuacin 2.31 se puede calcular la fuerza sobre la pieza
mvil en un proceso a enlace de flujo constante como:
Fe =

Wc (x, )
, si = cte.
x

(2.32)

La ecuacin anterior, tambin denominada principio de los trabajos virtuales, indica que para
calcular la fuerza Fe sobre la pieza mvil, es necesario conocer la variacin de la energa del
campo en funcin del desplazamiento, cuando se mantiene constante el enlace de flujo . Cuando en el convertidor, la energa acumulada en el campo es independiente de la posicin, la fuerza
elctrica es cero.
Si el convertidor electromecnico analizado anteriormente, mantiene una caracterstica lineal
entre el enlace de flujo y la corriente, la energa en el campo se puede evaluar mediante la
siguiente expresin:
1
1 2
1
Wc = i = L(x) i2 =
(2.33)
2
2
2 L(x)
En la ecuacin anterior, L(x) representa la inductancia en funcin de la posicin de la pieza
mvil. La inductancia de una bobina se determina a partir del nmero de vueltas N y de la
permeanza del circuito magntico como:
L(x) = N 2 (x)
62

(2.34)

II.2 Balance energtico


Para el electroimn en anlisis, la permeanza del circuito magntico es:

o A
2(x + d)

(x) =

(2.35)

Donde:

es la permeabilidad del vaco 4 107 H


m

es el rea efectiva del magneto

es la separacin del yugo

es la distancia entre el yugo y el circuito electroimn

Sustituyendo la expresin 2.35 en 2.34 y este resultado en 2.33 se obtiene:


Wc (x) =
y aplicando 2.32 a 2.36:
Fe =

1 2(x + d) 2

2 0 A N 2

(2.36)

Wc (x, )
2
=
x
0 A N 2

(2.37)

El mismo electroimn permite analizar lo que sucede si el movimiento se realiza muy lentamente. Si el yugo se desplaza a una velocidad prcticamente cero, la corriente se mantiene constante
porque no se induce fuerza electromotriz debido a que los enlaces de flujo cambian muy lentamente y su derivada con respecto al tiempo es prcticamente nula. En la figura 2.15 se muestra
la situacin anterior. En este caso, la energa mecnica se puede evaluar mediante las diferencias
de la coenerga en el campo entre la posicin x1 y la posicin x2 . En la figura 2.15 se observa
que para la condicin descrita:

Wm = Wc , si i = cte.

(2.38)

La coenerga en el campo se calcula de la siguiente forma:

Wc =

Z i(t)
i(0)

(x, i)di

(2.39)

La coenerga en el campo depende de la posicin de la pieza mvil y de la corriente, por lo tanto:

Wc (x, i)
Wc (x, i)
dWm = Fe dx = dWc =
dx +
di
x
i

(2.40)

Durante el proceso, la corriente i no vara y por esta razn se puede determinar a partir de 2.40
que:

Wc (x, i)
si i = cte.
(2.41)
Fe =
x
63

Captulo II Fundamentos de conversin

Figura 2.15 Clculo de la energa con desplazamientos muy lentos del yugo
La fuerza elctrica originada en el convertidor electromagntico depende de la variacin de la
energa en el campo en funcin del desplazamiento cuando el movimiento se realiza manteniendo constantes los enlaces de flujo. Si el movimiento se realiza manteniendo constante la
corriente, la fuerza elctrica depende de la variacin de la coenerga en funcin de la posicin.
Para calcular o medir una fuerza se utiliza el principio de los trabajos virtuales. Este mtodo
consiste en evaluar las variaciones de la energa o coenerga en el campo ante un desplazamiento
diferencial. Cualquiera de los dos mtodos analizados anteriormente, permite calcular las fuerzas
que aparecen sobre el sistema. Sin embargo, dependiendo de la forma como se presenten los
datos del convertidor, es ms fcil para determinar la fuerza utilizar los conceptos de energa o de
coenerga. En los sistemas lineales el clculo puede ser realizado con igual facilidad por ambos
mtodos. Cuando el sistema no es lineal, la facilidad o dificultad del clculo de fuerzas por uno
u otro mtodo depende de cules sean las variables independientes y cules las dependientes. Si
se conoce el enlace de flujo en funcin de las corrientes, el clculo por medio de la coenerga
simplifica el problema. Si la corriente se expresa como funcin de los enlaces, la energa es el
mejor mtodo para determinar la fuerza que aparece en la mquina.

II.3

ECUACIONES

INTERNAS DEL CONVERTIDOR

En la figura 2.16 se representa una mquina elctrica constituida por un electroimn alimentado
por una bobina y una pieza mvil sobre la que actan dos fuerzas, la fuerza elctrica Fe producida por la interaccin electromagntica del dispositivo y una fuerza externa Fm de naturaleza
mecnica.
En general la fuerza elctrica no tiene por qu ser igual a la fuerza mecnica. En el sistema
mecnico ilustrado en la figura 2.17, las tensiones de las cuerdas no estn necesariamente equilibradas.
En el ejemplo de la figura 2.17, la fuerza F1 es diferente a la fuerza F2 , ya que:
F1 = (m + M) a
64

(2.42)

II.3 Ecuaciones internas del convertidor

Figura 2.16 Electroimn sometido a fuerzas internas y externas

Figura 2.17 Sistema mecnico elemental sin equilibrio de fuerzas

65

Captulo II Fundamentos de conversin


F2 = m a

(2.43)

El razonamiento anterior es vlido tambin para el electroimn de la figura 2.16. La fuerza


mecnica en el extremo del yugo se determina mediante la segunda ley de Newton:
Fm = Fe + M x + x

(2.44)

Donde:
Fe

es la fuerza elctrica

M x

es la fuerza producida por la aceleracin de la pieza mvil

es la fuerza producida por el rozamiento de la pieza


es el coeficiente de roce

La ecuacin 2.44 se puede escribir mediante la expresin 2.41 como:

W (x, i)
+ M x + x
Fm = c
x

(2.45)

La ecuacin del equilibrio elctrico en la mquina es:


v = Ri+e = Ri+

d (x, i)
dt

(2.46)

Si se conoce la relacin entre los enlaces de flujo (x, i) o la corriente i( , x), el sistema queda
completamente definido ya que se puede evaluar la energa o la coenerga en el campo:
Wc =

i( , x)d

(2.47)

Z i

(i, x)di

(2.48)

Wc =

La expresin 2.45 determina el comportamiento dinmico del sistema ilustrado en la figura 2.16
si se conoce la fuerza mecnica Fm .
Si el sistema es lineal, la relacin entre los enlaces de flujo y la corriente viene expresada mediante la ecuacin 2.11. En esa ecuacin, la inductancia L depende de la posicin del yugo, es
decir L = L(x). Por esta razn:
i = i( , x) =

1
(i, x) = (x) (i, x)
L(x)

Donde:
(x)

66

es la inductancia inversa L1 .

(2.49)

II.3 Ecuaciones internas del convertidor


Mediante la ecuacin 2.49, la dinmica del electroimn queda completamente determinada.
Como el sistema es lineal:

Wc =

Z i
0

(i, x)di =

Z i
0

1
L(x) i di = L(x) i2
2

(2.50)

Sustituyendo la ecuacin 2.50 en la ecuacin 2.45 se obtiene:

W
1 dL(x) 2
i + M x + x
Fm = c + M x + x =
x
2 dx

(2.51)

La ecuacin 2.51 representa el equilibrio de fuerzas sobre la pieza mvil. La ecuacin que
representa el circuito elctrico del sistema es:
v = Ri+

d
dL(x) dx
di
(L(x) i) = R i +
i + L(x)
dt
dt
dt
dt

(2.52)

Definiendo (x) como:

(x)

dL(x)
dt

(2.53)

la ecuacin elctrica de la mquina, a partir de 2.52 y 2.53, es:


v = R i + (x) x i + L(x)

di
dt

(2.54)

En la expresin anterior, el primer sumando representa la cada de tensin en la resistencia de la


bobina, el segundo representa la fuerza electromotriz inducida en la bobina por el movimiento
del yugo y el tercer sumando representa la fuerza electromotriz inducida por variacin de la
corriente en la bobina. De forma compacta, la ecuacin 2.54 se puede escribir como:
v = R i + eG + eT

(2.55)

Donde:
e

es la fuerza electromotriz total compuesta por eG y eT

eG

es el trmino que depende de la velocidad de la pieza mvil de la mquina, denominado trmino de generacin

eT

es el trmino que depende de la variacin de la corriente en la mquina,


denominado trmino de transformacin

Cuando la corriente es cero, puede existir fuerza electromotriz de transformacin, pero no de


generacin como se observa en la ecuacin 2.54.
En conclusin, las ecuaciones internas de la mquina se pueden escribir, en funcin de la coenerga:
1
(2.56)
Fm = (x) i2 + M x + x
2
67

Captulo II Fundamentos de conversin


o, en funcin de la energa:
Fm =

1 d(x) 2
+ M x + x
2 dx

(2.57)

y la ecuacin elctrica 2.54.


Las variables que definen el estado del sistema en las ecuaciones 2.56, 2.57 y 2.54 son la corriente i, la posicin x y la velocidad x.
Realizando el cambio de variables x =
u, las ecuaciones
anteriores se pueden expresar de la siguiente forma:

1
2
Fm = 2 (x) i + M u + u
di
(2.58)
v = R i + (x) u i + L(x)
dt

x = u

Representando el sistema de ecuaciones diferenciales 2.58 en la forma cannica x = A(x)x + Bu,


se obtiene:
di
1
1
[R i + (x) i u] + L(x)
v(t)
dt = L(x)


(2.59)
u = M1 12 (x) i2 u + M1 Fm (t)

x = u
Para determinar la solucin de este sistema de ecuaciones diferenciales no lineales, es necesario
conocer:
1. Las condiciones iniciales de las variables de estado i(0), u(0) y x(0).
2. Las condiciones de borde o ligazones externas. En el presente caso definidas por las excitaciones en el tiempo de la fuerza mecnica Fm (t) aplicada al yugo y la tensin v(t)
aplicada a la bobina del electroimn.
II.4

ECUACIONES

DE POTENCIA

La potencia utilizada por el convertidor electromecnico en el eje mecnico de la mquina de la


figura 2.16 se puede calcular a partir de la fuerza mecnica y de la velocidad del yugo:
1
Pm = Fm x = (x) i2 x + M x x + x2
2

(2.60)

La potencia absorbida por el eje elctrico es:


Pe = v i = R i2 + (x) x i + L(x)

di
i = R i2 + eG i + eT i
dt

(2.61)

Para que la mquina anterior pueda trabajar en un rgimen continuo, con corriente y velocidad
constante, despreciando las prdidas de friccin ( = 0), y las prdidas por efecto Joule en los
conductores (R = 0), mediante las ecuaciones 2.60 y 2.61 se observa que:
1
Pm = eG i
2
68

(2.62)

II.4 Ecuaciones de potencia

Figura 2.18 Balance energtico de una mquina elctrica en rgimen continuo


Pe = eG i

(2.63)

Las expresiones 2.60 y 2.61 indican que en las condiciones anteriores, la mquina absorbe permanentemente por el eje elctrico el doble de la potencia mecnica que est utilizando. La
diferencia entre estas dos potencias slo puede ser almacenada en el campo. En la figura 2.18
se representa esta situacin. De toda la potencia que es inyectada en el eje elctrico, el 50 % se
convierte en energa mecnica y el otro 50 % se almacena en el campo. Como la corriente es
constante, el trmino de transformacin (eT i) es cero y el campo no puede devolver al sistema
la energa que le ha sido entregada en el proceso de conversin.
Si una mquina elctrica se mantiene todo el tiempo operando en esta situacin, acumula de
forma indefinida energa en el campo. Esto no es factible para un sistema fsico real. La solucin
del problema planteado consiste en permitir la variacin de la corriente. Con la variacin de la
corriente aparece el trmino de transformacin (eT i) que compensa el trmino de generacin
( 21 eG i). Por esta razn no es posible construir un mquina que funcione slo con corriente continua. En todas las mquinas elctricas es necesaria la variacin de las corrientes para permitir
una operacin en rgimen permanente.
La argumentacin anterior se puede cuestionar debido a que son muy frecuentes en la industria
las Mquinas de corriente continua. Sin embargo en este caso el trmino corriente continua
se aplica a la fuente utilizada para alimentar el convertidor. Las mquinas de corriente continua
requieren de un dispositivo inversor electromecnico las escobillas y el colector que permite
la variacin de las corrientes en los devanados de la mquina.
Tambin parecen contradecir esta argumentacin los principios de funcionamiento de las mquinas homopolares y los convertidores magneto-hidrodinmicos 3. En ambos casos, estas mquinas
funcionan con corriente continua, pero la corriente no siempre circula por el mismo material. Si
un observador se mueve solidario con el medio conductor, el disco en el caso homopolar y el
fluido en la mquina magnetohidrodinmica, puede medir la variacin de las corrientes al aproximarse y alejarse del punto de inyeccin. En otras palabras, estas mquinas son equivalentes
a las de corriente continua, pero si en ellas el proceso de variacin de las corrientes se realiza
de forma discreta mediante el colector y las escobillas, en las homopolares y magnetohidrodinmicas el proceso de variacin de las corrientes se lleva a cabo de forma continua mediante un
proceso de acercamiento y alejamiento del punto de inyeccin de la corriente.
3

Ver figura 2.19.

69

Captulo II Fundamentos de conversin

(a) Convertidor homopolar

(b) Bomba magneto-hidrodinmica

Figura 2.19 Mquinas de corriente continua


Por lo tanto en ningn caso conocido, la experiencia contradice la necesidad terica de variacin
de la corriente para el funcionamiento en rgimen permanente de los convertidores electromecnicos de energa.

II.5

G ENERALIZACIN

DE LAS ECUACIONES

En una mquina con dos ejes elctricos y un eje mecnico, como la ilustrada en la figura 2.20,
se satisface la siguiente relacin para la evaluacin de la fuerza elctrica sobre la pieza mvil:
Fe =

Wc (x, 1 , 2 )
x

(2.64)

Para demostrar la validez de la ecuacin 2.64 se debe recordar que en un sistema mecnico de
este tipo, si se vara la posicin x, el intercambio energtico se produce entre los ejes elctricos
y el eje mecnico. Si la posicin permanece fija, el intercambio energtico se realiza entre los
ejes elctricos nicamente. La ecuacin 2.64 mantiene la validez en el clculo de la fuerza
en un sistema con dos ejes elctricos, ya que la ecuacin 2.33 se demostr para el caso en el
que los enlaces de flujo se mantienen constantes. Si el enlace de flujo es constante, las fuerzas
electromotrices son cero y no puede entrar energa hacia el campo desde ninguno de los ejes
elctricos. Por esta razn se cumplen las mismas condiciones en la expresin 2.64 que en la
2.33. De todo esto se concluye que es completamente general su aplicacin.
Cualquiera que sea el nmero de ejes elctricos o mecnicos de un convertidor electromecnico, para calcular la fuerza elctrica se puede utilizar una expresin similar a la ecuacin 2.64,
siempre y cuando el movimiento se realice slo en uno de los ejes mecnicos y se mantengan
constantes todos los enlaces de flujo en los ejes elctricos. La expresin generalizada para el
clculo de la fuerza elctrica es:
Fer =
70

Wc (x1 , x2 , ..., xr , ..., xn, 1 , 2 , ..., m)


xr

(2.65)

II.5 Generalizacin de las ecuaciones

Figura 2.20 Mquina con dos ejes elctricos y un eje mecnico


La ecuacin 2.65 determina la fuerza elctrica que aparece sobre el eje mecnico r. Para este
fin, se calcula la derivada parcial de la energa en el campo con respecto a la posicin del eje
r, manteniendo constantes las posiciones de los otros ejes mecnicos y los enlaces de flujo de
todos los ejes elctricos.
En el sistema de la figura 2.20, si la posicin x se mantiene constante, la energa acumulada en
el campo es igual a la energa elctrica:
dWc = dWe , si x = cte.

(2.66)

La energa elctrica se puede calcular como:


dWc = dWe = i1 d 1 + i2 d 2 , si x = cte.

(2.67)

Si se conoce cmo varan las corrientes con los enlaces de flujo y con la posicin, el problema
queda resuelto, es decir:

i1 = f1 (x, 1 , 2)
(2.68)
i2 = f2 (x, 1 , 2)
En los casos lineales se puede establecer:

1 = L11 i1 + L12 i2
2 = L21 i1 + L22 i2

(2.69)

Matricialmente la expresin 2.67 se puede escribir como:


[ ] = [L] [i]
Donde:
[ ] =

1
2

; [i] =

i1
i2

; [L] =

(2.70)

L11 L12
L21 L22


71

Captulo II Fundamentos de conversin


Empleando lgebra matricial, se puede determinar la corriente [i] en funcin de los enlaces [ ]:
[i] = [L]1 [ ] = [] [ ]

(2.71)

La expresin 2.71 en forma explcita es:




 

i1
11 (x) 12 (x)
1
=
i2
21 (x) 22 (x)
2

(2.72)

Para calcular la energa en el campo, es necesario variar cada uno de los parmetros en forma
sucesiva, desde su valor inicial a su valor final, mientras todas las otras variables de estado se
mantienen constantes. Para evaluar la energa acumulada en el campo, se realiza el siguiente
procedimiento:
Wc =

Z (x,1 ,2 )
(0,0,0)

dWc =

Z (x,0,0)
(0,0,0)

dWc +

Z (x,1 ,0)
(x,0,0)

dWc +

Z (x,1 ,2 )
(x,1 ,0)

dWc

(2.73)

La primera integral de la sumatoria de la ecuacin 2.73 es cero, debido a que los enlaces de flujo
son cero mientras se mueve el yugo de la mquina. Como no existe variacin de los enlaces,
no existen fuerzas electromotrices y por lo tanto no se inyecta potencia elctrica desde los ejes
elctricos hacia el campo. Al no existir enlaces de flujo, para realizar el desplazamiento mecnico x no es necesario consumir ni suministrar energa. Para la evaluacin de los dos trminos
restantes de la ecuacin 2.73, se sustituyen las ecuaciones 2.67 y 2.72:
Wc =

Z (x,1 ,0)

(11 1 + 12 2 )d 1 + (21 1 + 22 2 )d 2 +
(x,0,0)
Z (x,1 ,2 )
+
(11 1 + 12 2 )d 1 + (21 1 + 22 2 )d 2
(x,1 ,0)
1
1
11 12 + 21 1 2 + 22 22
2
2

=
(2.74)

En el clculo de las integrales de la ecuacin 2.74 se asume que 12 es igual a 21 , condicin de


simetra siempre vlida para los sistemas fsicos.
Generalizando el clculo anterior mediante el lgebra de matrices, se tiene:
dWc = dWe = [i]t [d ] , si x = cte.

(2.75)

De la ecuacin 2.72 y recordando la propiedad sobre la traspuesta de un producto de matrices:


[i]t = [ ]t []t

(2.76)

Se obtiene la energa acumulada en el campo como:


Wc =

72

Z (x,1 ,2 )
(0,0,0)

[ ]t [(x)]t [d ] =

1 t
[ ] [(x)]t [ ]
2

(2.77)

II.5 Generalizacin de las ecuaciones

Figura 2.21 Electroimn con yugo rotativo

Si se deriva parcialmente la ecuacin 2.77 con respecto a la posicin x, se encuentra la fuerza


elctrica Fe que acta sobre la pieza mvil:
Fe =


Wc (x, [ ])
1
d
[(x)]t [ ]
= [ ]t
x
2
dx

(2.78)

1 t
[i] [L(x)]t [i]
2

(2.79)

Por un razonamiento semejante, pero aplicado a la coenerga, se puede deducir que:

Wc =

La fuerza elctrica sobre la pieza se puede calcular como:


Wc (x, [i]) 1 t d
1
Fe =
[L(x)]t [i] = [i]t [ (x)]t [i]
= [i]
x
2
dx
2

(2.80)

Las ecuaciones 2.78 y 2.80 son vlidas para un nmero cualquiera de ejes elctricos, pero para
un eje mecnico solamente. La mayora de las mquinas elctricas poseen un solo eje mecnico,
pero si existen ms, es necesario calcular las derivadas parciales de la energa o de la coenerga,
segn sea el caso, con respecto a cada una de las variables que definen la posicin de cada eje
mecnico (x1 , x2 , x3 , ..., xn).
Si el eje mecnico es rotativo o giratorio, como se representa en la figura 2.21, la matriz de
inductancia se define en funcin del ngulo y no se calculan fuerzas sino pares elctricos y
mecnicos.
Las ecuaciones del convertidor en este caso son:
Te =

1 t
[i] [ ( )]t [i]
2

Donde:
[ ( )] =

(2.81)

d
[L( )]
d
73

Captulo II Fundamentos de conversin


Las ecuaciones de equilibrio elctrico y mecnico de un convertidor electromecnico lineal con
mltiples ejes elctricos y un eje mecnico son:
[i] = [R] [i] + [e] =
d
= [R] [i] + [ ] =
dt
d
d [i]
= [R] [i] + [L(x)] x [i] + [L(x)]
=
dx
dt
d [i]
= [R] [i] + [ (x)] x [i] + [L(x)]
dt
1
Fm = [i]t [ (x)]t [i] + M x + x
2

(2.82)
(2.83)

En las ecuaciones 2.82 y 2.83 se observa que la informacin que determina la dinmica y el
comportamiento de la mquina elctrica est contenida en la matriz [L(x)]. A partir de esta
matriz, se obtiene la matriz [ (x)], y con estas dos matrices y los elementos de ligazn con los
sistemas elctricos y mecnicos externos, se formulan las ecuaciones completas del convertidor.
II.6

S UMARIO

1. Para el anlisis de convertidores electromecnicos resulta de utilidad la aplicacin de los


balances de energa o coenerga como mtodo para la determinacin de las fuerzas elctricas involucradas en el proceso. Los mtodos de anlisis directo a partir de las leyes
de Maxwell y la relacin de Lorenz pueden ser muy difciles de aplicar en las mquinas
reales.
2. Los convertidores electromecnicos pueden poseer varios ejes o puertos elctricos y mecnicos. Al menos un eje elctrico y uno mecnico son indispensables para definir que una
mquina es un convertidor electromecnico. Los ejes o puertos son las puertas de entrada
y salida de la energa. Los puertos elctricos estn definidos por la fuerza electromotriz y
la corriente. Los puertos mecnicos lo estn por la velocidad angular o lineal y por el par
o la fuerza.
3. El principio de los trabajos virtuales permite la evaluacin de la fuerza o el par en un
sistema electromecnico. Conocida la energa o coenerga en funcin de las variables
de estado, la derivada parcial de la energa o coenerga con respecto a la posicin
desplazamiento virtual, determinan la fuerza o el par de acuerdo con las expresiones
2.65 y 2.80.
4. Las ecuaciones internas de fuerza electromotriz se obtienen por derivacin con respecto al
tiempo de los enlaces de flujo. En los sistemas lineales, la matriz de inductancia determina
el comportamiento electromagntico completo del convertidor.
5. Cuando se incluyen las condiciones de contorno que unen la mquina con los sistemas
elctricos y mecnicos, utilizando las leyes de Kirchoff y la segunda ley de Newton, se
completa el conjunto de ecuaciones diferenciales que definen el comportamiento dinmico
del convertidor.

74

II.7 Ejemplos resueltos


II.7

EJEMPLOS

RESUELTOS

Ejemplo 1: Determinacin de las ecuaciones internas del convertidor


De un convertidor electromecnico se conocen las relaciones no lineales existentes entre los
enlaces de flujo y la corriente por sus respectivas bobinas:

1 = k1 x i21 + k2 i1 i2 + k3 y i22
2 = k3 y i21 + k2 i1 i2 + k1 x i22
Determine las ecuaciones internas de esta mquina.
Solucin:
El convertidor est constituido por dos ejes elctricos (bobinas 1 y 2) y dos ejes mecnicos (x, y).
Existen por tanto cuatro ecuaciones internas, dos de fuerza electromotriz y dos de fuerza, una
por cada puerto.
Las ecuaciones de fuerza electromotriz en cada una de las bobinas se obtienen derivando los
enlaces de flujo con respecto al tiempo:
d 1
di1 2 dx
di2
di1
di2 2 dy
= k1 (2x i1
+ i1 ) + k2 (i1
+ i2 ) + k3 (2y i2
+ i2 )
dt
dt
dt
dt
dt
dt
dt
d 2
di1 2 dy
di2
di1
di2 2 dx
= k3 (2y i1
+ i1 ) + k2 (i1
+ i2 ) + k1 (2x i2
+ i2 )
dt
dt
dt
dt
dt
dt
dt
El clculo de la fuerza elctrica en cada uno de los ejes mecnicos requiere la determinacin de
la coenerga en funcin de las posiciones x y y de dichos ejes. Una vez determinada la expresin
de la coenerga es necesario derivarla parcialmente con respecto a cada posicin mecnica para
obtener la fuerza e lctrica instantnea en los puertos mecnicos:

Wc (x, y, i1, i2 ) =
=
=

Z (x,y,i1 ,i2 )
(0,0,0,0)
Z (x,y,i1 ,0)

(x,y,0,0)
Z (x,y,i1 ,0)
(x,y,0,0)

+
=

1 di1 + 2 di2 =

1 di1 +

Z (x,y,i1 ,i2 )
(x,y,i1 ,0)

2 di2 =

(k1 x i21 )di1 +

Z (x,y,i1 ,i2 )
(x,y,i1 ,0)

(k3 y i21 + k2 i1 i2 + k1 x i22 )di2 =

k2
k1
k1 3
x i1 + k3 y i21 i2 + i1 i22 + x i32 .
3
2
3

Las fuerzas elctricas instantneas en cada eje son:

75

Captulo II Fundamentos de conversin

Fex

Wc k1 3 3
= (i1 + i2 )
=
x
3

Wc
Fey =
= k3 i21 i2
y
Ejemplo 2: Ecuacin del par de un convertidor rotativo
La relacin entre las corrientes y los enlaces de flujo de un convertidor electromecnico rotativo
se puede expresar de la siguiente forma:


i1
i2

1
3 cos2
3 cos2
2



1
2

Determine el par elctrico instantneo de este convertidor y la condicin necesaria para obtener
un par elctrico promedio diferente de cero, si el enlace 1 es constante y 2 vara sinusoidalmente
con el tiempo a una frecuencia 2 .
Solucin:
En este problema existe una relacin lineal entre las corrientes en las dos bobinas del convertidor y sus respectivos enlaces de flujo. Aun cuando existe la posibilidad de invertir la matriz y
obtener una relacin explcita entre los enlaces y las corrientes, en este caso resulta ms simple
determinar el par elctrico mediante el concepto de energa en el campo:

1
1 2
Wc ( , 1 , 2) =
2

1
3 cos2
3 cos2
2



1
2

Wc ( , 1 , 2 )
=




1
1
0
23 sen2
1 2
=
=
23 sen2
0
2
2
= 23 sen2 1 2 .

e =

Conocida la expresin del par elctrico instantneo, es posible integrarla en un perodo para
obtener la condicin necesaria de par promedio diferente de cero:
1 2
(23 sen2 1 2 sen(2t + 2 )) d =
2 0
Z
23 1 2 2
(sen2 sen(2t + 2 )) d =
=
2
0
Z
23 1 2 2 1
(cos(2 2t 2 ) cos(2 + 2t + 2 )) d .
=
2
2
0

he i =

76

II.7 Ejemplos resueltos


Las integrales entre 0 y 2 de las funciones sinusoidales son cero; para que la expresin anterior
pueda tener promedio diferente de cero es indispensable que alguno de los dos argumentos del
integrando sea independiente de , en otras palabras:
2 (2t) = constante.
Derivando la expresin anterior se obtiene la condicin necesaria, pero no suficiente, para obtener un par promedio diferente de cero en este convertidor:
1
d
= mec = 2 .
dt
2
En otras palabras, es necesario que la velocidad angular de giro del eje mecnico del convertidor
est sincronizado a la mitad de la velocidad angular del segundo eje elctrico, para poder obtener
un par promedio diferente de cero.
Ejemplo 3: Modelacin del convertidor por inspeccin
Un convertidor electromecnico como el ilustrado en la figura 2.22 tiene dos bobinas ortogonales ( , ) en un estator cilndrico y un rotor de polos salientes f , con un devanado a lo largo
de la pieza magntica. Se desconocen las dimensiones, los materiales de construccin y sus
caractersticas. Determine:
1. El modelo matemtico completo del convertidor mediante inspeccin del diagrama ilustrado en la figura 2.22. Puede considerar que las variaciones de la reluctancia son aproximadamente sinusoidales. El nmero de vueltas de las bobinas son diferentes, N = N 6= N f .
2. Los ensayos necesarios para determinar los parmetros del modelo desarrollado.
Solucin:
1. El modelo matemtico completo del convertidor mediante inspeccin del diagrama ilustrado en la figura 2.22. Puede considerar que las variaciones de la reluctancia son aproximadamente sinusoidales. El nmero de vueltas de las bobinas y son iguales, N =
N 6= N f .
El modelo de la mquina determinado por inspeccin implica la representacin matemtica de las inductancias propias y mutuas en funcin de la posicin angular del rotor .
En la figura 2.23 se muestra la dependencia de las inductancias propias del estator L y
L , mutuas entre las bobinas y del estator L , los acoplamientos mutuos entre las
bobinas del estator y rotor, L f y L f , as como la inductancia propia del rotor L f . El
perodo de repeticin de las inductancias propias y mutuas del estator dependen del doble
del ngulo debido a que con cada giro de de la posicin del rotor, las reluctancias
se repiten. Los acoplamientos estator-rotor tienen un comportamiento diferente porque la
polaridad relativa entre las bobinas invierten su signo con un giro de en la posicin del

77

Captulo II Fundamentos de conversin

Figura 2.22 Convertidor rotativo


rotor. La inductancia del rotor es constante debido a que el estator es cilndrico y por lo
tanto la permeanza de la bobina rotrica f , es independiente de la posicin angular .4
Las expresiones analticas de estas inductancias obtenidas por inspeccin se pueden representar de la siguiente forma aproximada 5:
L ( ) Le1 + Le2 cos 2 +

L ( ) Le1 + Le2 cos 2( ) +


2

L ( ) Le3 + Le4 cos 2( + ) +


4
L f ( ) Le f cos +
L f ( ) Le f sin +
L f ( ) = L f = cte.

Una vez conocidas las funcionalidades de las inductancias con la posicin, las ecuaciones
del convertidor electromecnico se obtienen directamente:

L ( ) L ( ) L f ( )
i
i
v
Re 0 0
v = 0 Re 0 i + p L ( ) L ( ) L f ( ) i
0 0 Rf
vf
if
Lf
if
L f ( ) L f ( )

Un observador ubicado en el rotor no percibe cambios del entrehierro a medida que la pieza gira. En el estator
sucede lo contrario, el observador detecta mnimo entrehierro cada vez que el eje magntico positivo o negativo
del rotor pasa por el frente.
Las funciones de las inductancias con respecto al ngulo son peridicas y es posible utilizar series de Fourier para representarlas mediante senos y cosenos. En este caso se ha truncado la serie en la primera armnica
espacial.

78

II.7 Ejemplos resueltos

Figura 2.23 Inductancias del convertidor

L ( ) L ( ) L f ( )
i
i
1
d 2

d
L ( ) L ( ) L f ( ) i = Tm + J 2 +
Te = i
2

dt
dt
if
Lf
if
L f ( ) L f ( )

2. Los ensayos necesarios para determinar los parmetros del modelo desarrollado.
Es necesario determinar las inductancias propias y mutuas del modelo para lo cual hay
que disear un ensayo que permita reproducir el grfico 2.23. Las inductancias propias
se miden aplicando corriente en la bobina y midiendo el enlace de flujo que se produce.
La medida directa del flujo requiere dispositivos invasores dentro del entrehierro, por esta
razn es preferible aplicar una tensin alterna en la bobina de prueba y medir la corriente
que consume para una posicin fija . El enlace de flujo en la bobina se obtiene por integracin de la fuerza electromotriz inducida:
d (t)
di(t)
e(t) = v(t) R i(t) =
= L( )
dt
dt

v(t) = 2V cos t ; i(t) = 2I cos( t )

V cos t = RI cos( t ) + L( )I sin( t )


q
L( )
)
V cos t = R2 + ( L( ))2 I cos( t + arctan
R
r
1 V2
X
R
1p 2
2=
2
Z

R
=
tan
(2.84)
L( ) =

R
=
I2


La expresin 2.84 determina la inductancia propia para una posicin fija del rotor midiendo la resistencia de la bobina R, la tensin efectiva aplicada V , la corriente efectiva I
y la frecuencia angular = 2 f .

79

Captulo II Fundamentos de conversin

Las impedancias mutuas estator-estator o estator-rotor deben medirse aplicando tensin


alterna en la bobina excitadora y y midiendo la fuerza electromotriz inducida en la bobina
de prueba x, para cada posicin angular fija:

x = Lxy ( ) iy vx = Lxy ( )

diy
dt

La corriente iy se obtiene alimentando la bobina y con una fuente de tensin vy tal como
se hizo en el caso de inductancia propia:

iy (t) = 2Iy cos( t y )


Vy
Ly ( )
Iy = q
; y ( ) = arctan
Ry
R2y + ( Ly ( ))2

2Iy Lxy ( ) cos( t y )


2
q
Vx
1
Vx Zy ( ) Vx
R2y + ( Ly ( ))2 =
=
Lxy ( ) =

Vy
Vy Iy

II.8

2Vx cos( t x ) =

EJERCICIOS

(2.85)

PROPUESTOS

1. En la figura 2.24 se ha representado un convertidor electromecnico compuesto por un


electroimn y su yugo. El electroimn tiene una bobina de 1.000 vueltas, alimentada con
una fuente de corriente alterna de 100V efectivos y su resistencia es de 5 . En el yugo
existe otra bobina de 500 vueltas que se encuentra en cortocircuito y posee una resistencia de 10 . El yugo tiene una masa de 250 g y est conectado mediante un resorte de
104 Nm a un sistema inercial. En la posicin de reposo del resorte, el yugo se encuentra a
5 mm del electroimn. La seccin transversal del material electromagntico es de 25 cm2
y la longitud media del camino magntico sin considerar el entrehierro es de 48 cm. La
permeabilidad relativa del material magntico es 2.000. El material se considera lineal en
todo el rango de la densidad de flujo. En estas condiciones determine:

a) La relacin entre los enlaces de flujo y las corrientes en funcin de la posicin del
yugo.
b) Las ecuaciones dinmicas completas del convertidor.
c) La solucin en rgimen permanente, considerando que la inercia mecnica del sistema elimina las vibraciones mecnicas del yugo posicin de equilibrio.
d) La potencia de prdidas del convertidor en rgimen permanente.

2. El convertidor electromecnico que se muestra en la figura 2.25 est compuesto por un


condensador, una fuente de tensin alterna de 100V pico, una pieza mvil que entra en el

80

II.8 Ejercicios propuestos

Figura 2.24 Diagrama esquemtico del ejercicio N. 1

Figura 2.25 Diagrama esquemtico del ejercicio N. 2


interior del condensador y un resorte que lo conecta a un sistema inercial, cuya posicin
de equilibrio es x0 = 5 cm. La masa de la pieza mvil es de 10 g. La constante k del resorte
N
es de 0,3 cm
. La separacin de las placas cuadradas es de 1,0 cm, siendo cada uno de sus
lados de 10 cm. La permitividad relativa r de la pieza mvil es 5,0. La frecuencia de la
fuente es de 60 Hz. En estas condiciones determine:

a) La fuerza elctrica sobre la pieza mvil utilizando el concepto de coenerga.


b) La fuerza elctrica sobre la pieza mvil utilizando el concepto de energa.
c) Las ecuaciones dinmicas completas del convertidor.
d) La solucin analtica de rgimen permanente.
e) La amplitud de la tensin si la pieza mvil se desplaza 5 cm de su posicin de equilibrio.6
6

Sugerencia: Q = C e ; C = Ad ; D = ; e = Edl ; D = E.
R

81

Captulo II Fundamentos de conversin

Figura 2.26 Diagrama esquemtico del convertidor electromecnico del ejercicio N. 4


3. La mquina que se muestra en la figura 2.22 posee dos bobinas en el estator con Ne vueltas
cada una y un devanado en el rotor con N f vueltas. Las dos bobinas del estator tienen la
misma resistencia Re y se alimentan mediante dos fuentes sinusoidales de tensin desfasadas 2 y cuya frecuencia es e . El rotor tiene una resistencia R f y se alimenta mediante
una fuente de corriente constante de valor Ir . Las dimensiones y los parmetros del convertidor electromecnico de energa son bien conocidos. Determine:
a) Las ecuaciones completas7 del convertidor expresadas en forma cannica (px =
Ax + Bu).
b) La condicin necesaria para obtener par promedio diferente de cero cuando el equipo
se encuentra operando a velocidad constante en rgimen permanente (m = cte).
c) Repita la pregunta anterior si el rotor est en cortocircuito (vr = 0).
d) Repita el ejercicio considerando que la bobina del rotor est abierta (ir = 0).
4. En el diagrama de la figura 2.26 se tiene un electroimn y una pieza magntica que puede atravesar el entrehierro a diferentes velocidades. Utilizando diagramas de energa y/o
coenerga en el campo, determine el trabajo realizado por el sistema en las siguientes
condiciones:
a) Cuando la pieza cruza el entrehierro a una velocidad extremadamente reducida (prcticamente cero).
b) Cuando la pieza cruza el entrehierro a una velocidad extremadamente rpida (prcticamente infinita).
c) Cuando la pieza cruza el entrehierro a una velocidad intermedia.
d) Determine las ecuaciones diferenciales que rigen el comportamiento del convertidor.

5. Repita el ejemplo N. 1, figura 2.24, considerando una caracterstica de magnetizacin


B(H) no lineal, tal como se ilustra en la figura 2.27. En estas nuevas condiciones determine:
7

Internas y externas.

82

II.8 Ejercicios propuestos

Figura 2.27 Caracterstica B(H) del material magntico del convertidor


a) La relacin entre los enlaces de flujo y las corrientes en funcin de la posicin del
yugo.
b) Las ecuaciones dinmicas completas del convertidor.
c) La solucin en rgimen permanente, considerando que la inercia mecnica del sistema elimina las vibraciones mecnicas del yugo (posicin de equilibrio).
d) La potencia de prdidas del convertidor en rgimen permanente.

83

Captulo II Fundamentos de conversin

84

Bibliografa

A DKINS, B., The General Theory of Electric Machines, Chapman and Hall, London, 1957.
F ITZGERALD , A. E., K INGSLEY, C. J R . & K USKO, A., Electric Machinery: The Processes,
Devices, and Systems of Electromechanical Energy Conversion, McGraw-Hill, Third Edition,
New York, 1971.
NASAR , S. A. & U NNEWEHR , L. E., Electromechanics and Electric Machines, John Wiley &
Sons, New York, 1983.
S AY, M. G., Introduction to the Unified Theory of Electromagnetic Machines, Pitman Press,
London, 1971.
S EN G UPTA , D. P. & LYNN , J. W., Electrical Machine Dynamics, Ed. Macmillan Press LTD,
London, 1980.
T HALER , G. & W ILCOX , M., Mquinas elctricas, Editorial Limusa, Mxico, 1979.
W HITE , D. C. & WOODSON , H. H., Electromechanical Energy Conversion, John Wiley &
Sons, New York, 1959.

85

Captulo II Fundamentos de conversin

86

CAPTULO III

Circuitos acoplados magnticamente

En los dos captulos precedentes se desarrollaron las bases generales para el anlisis de los
convertidores electromecnicos de energa mediante las ecuaciones que describen el comportamiento fsico y el equilibrio energtico de estos sistemas. Los balances energticos presentados
en el captulo II constituyen un mtodo simple que permite el anlisis cualitativo de las mquinas elctricas. Esta metodologa requiere la determinacin de las inductancias propias y mutuas
entre las diferentes bobinas para poder obtener las ecuaciones diferenciales que rigen el comportamiento de estos dispositivos. En este captulo se estudia las relaciones electromagnticas
existentes entre las diferentes bobinas de un circuito acoplado magnticamente, as como el comportamiento transitorio de estos circuitos, considerando que los devanados se encuentran fijos
en el espacio. El medio magntico se considera con permeabilidad , constante y homognea.
En todo el captulo se asume linealidad entre el flujo magntico y las corrientes.
III.1

D EFINICIONES

BSICAS

En primer lugar se considera el diagrama de la figura 3.1 en la cual se han representado n


circuitos magnticamente acoplados. En el circuito k se coloca una fuente de tensin vk , que
inyecta en esa bobina la corriente ik .
Las lneas de la figura 3.1, representan la distribucin del flujo cuando se excita la bobina k. El
flujo total que enlaza la bobina k se representa por kk y se puede descomponer en dos flujos:
Donde:

mk

es el flujo que enlaza a las otras bobinas

lk

es el flujo que enlaza solamente a la bobina k

87

Captulo III Circuitos acoplados magnticamente

Figura 3.1 Representacin del flujo propio


De esta forma, se establece:

kk = mk + lk

(3.1)

En la figura 3.2 se representa el caso contrario, donde todas las bobinas estn excitadas, menos
la bobina k.
El flujo mutuo que enlaza la bobina k, debido a la excitacin de las otras bobinas, se denomina
MK y comprende n 1 componentes:

MK =

k j

(3.2)

j6=k

En la ecuacin 3.2, k j representa el flujo mutuo producido por la bobina j que enlaza a la bobina
k. Por superposicin, el flujo magntico total enlazado por la bobina k es:

k = kk + MK = lk + mk + MK = lk + mk +

k j

(3.3)

j6=k

Los enlaces de flujo correspondientes son:

k = Nk k = lk + mk +

k j

(3.4)

j6=k

Si los enlaces de flujo de la ecuacin 3.4 se expresan en funcin de la permeanza magntica y


de las corrientes de excitacin de las bobinas, se obtiene:

88

III.1 Definiciones bsicas

Figura 3.2 Representacin de los flujos mutuos

lk = Nk lk = Plk Nk2 ik

(3.5)

mk = Nk mk = Pmk Nk2 ik

(3.6)

k j = Nk k j = Pk j Nk N j i j

(3.7)

Se pueden definir las siguientes inductancias:


Llk = Plk Nk2 =

Nk lk
ik

(3.8)

Lmk = Pmk Nk2 =

Nk mk
ik

(3.9)

Lk = (Plk + Pmk ) Nk2 =

Nk kk
ik

(3.10)

Donde Llk es la inductancia de dispersin, Lmk es la inductancia de magnetizacin y Lk es la


inductancia propia. Las inductancias mutuas se definen como:
Mk j = Pk j Nk N j =

Nk k j
ij

(3.11)

M jk = Pjk N j Nk =

N j jk
ik

(3.12)

89

Captulo III Circuitos acoplados magnticamente


Como las permeanzas Pk j y Pjk son iguales, se demuestra que:
Mk j = M jk

(3.13)

Si se expresa la ecuacin 3.4 en trminos de las inductancias definidas en 3.8, 3.9, 3.10, 3.11 y
3.12 se obtiene para la bobina k:

k = Lk ik +

Mk j i j

(3.14)

j6=k

La ecuacin 3.14 se puede escribir en forma matricial para todas las bobinas del sistema:

1
2
..
.
k
..
.

L1 M12
M21 L2
..
..
.
.
Mk1 Mk2
..
..
.
.

M1k
M2k
..
..
.
.
Lk
.
..

Mn1 Mn2 Mnk

M1n
M2n
..

.
Mkn
..
..
.
.
Ln

i1
i2
..
.
ik
..
.
in

(3.15)

La ecuacin 3.15 en forma compacta se escribe as:


[ ] = [L] [i]

III.2

ECUACIONES

(3.16)

DE TENSIN

La tensin instantnea aplicada en la bobina k del sistema acoplado magnticamente de las


figuras 3.1 y 3.2 es:
vk = Rk ik + pk
En la ecuacin 3.17 el operador p se refiere a la derivada con respecto al tiempo
bobinas acopladas se cumple:
[v] = [R] [i] + [L] p [i] = ([R] + [L] p) [i]
Donde:
[R] es una matriz diagonal
[L] est definida por la ecuacin 3.15

90

(3.17)
d
dt .

Para las n

(3.18)

III.3 Coeficientes de acoplamiento y dispersin


III.3

C OEFICIENTES

DE ACOPLAMIENTO Y DISPERSIN

Multiplicando las ecuaciones 3.11 y 3.12 trmino a trmino, se obtiene:


M 2jk =

Nk N j jk k j
ik i j

(3.19)

De la ecuacin 3.10 se puede deducir que:


Lj
Nk
Lk N j
=
=
;
ik
kk i j
jj

(3.20)

Sustituyendo 3.20 en 3.19 se obtiene:


M 2jk = Lk L j

jk k j
kk j j

(3.21)

El cociente de los flujos representa la fraccin del flujo total propio de la bobina k que enlaza a
la bobina j. Estos coeficientes son constantes y se definen como:
kk =

jk
k j
; kj =
kk
jj

(3.22)

En 3.22, kk y k j se denominan factores de acoplamiento e indican la cantidad de flujo existente


entre las dos bobinas. A medida que decrece la separacin entre las bobinas, se incrementa el
valor del coeficiente de acoplamiento. El valor mximo terico para un acoplamiento perfecto
es la unidad.
Reemplazando las definiciones de 3.22 en la ecuacin 3.21 se obtiene:
M 2jk = k j kk L j Lk M jk =

k j kk

L j Lk

(3.23)

En la ecuacin 3.23, a la media geomtrica de los factores de acoplamiento se le denomina


coeficiente de acoplamiento entre la bobina j y la bobina k, k jk y puede variar entre los valores
cero y uno. Otro coeficiente ampliamente utilizado es jk o coeficiente de dispersin y queda
definido por:

jk = 1 k2jk

(3.24)

Como:
M jk = k jk

L j Lk

(3.25)

Por lo tanto, sustituyendo 3.25 en la ecuacin 3.24 se obtiene:

jk = 1

M 2jk
L j Lk

(3.26)

91

Captulo III Circuitos acoplados magnticamente

Figura 3.3 Transformador de dos devanados


III.4

EL

TRANSFORMADOR COMO CIRCUITO ACOPLADO

En la figura 3.3 se presenta un transformador de dos devanados. Cada bobina posee una inductancia propia de valor L1 y L2 respectivamente, una inductancia mutua M y una resistencia propia
en cada bobina, R1 y R2 .
Aplicando la ecuacin 3.18, se obtiene:


v1
v2

R1 0
0 R2



i1
i2

L1 M
M L2

 

i1
p
i2

(3.27)

Despejando la derivada de las corrientes con respecto al tiempo, se transforma la ecuacin diferencial 3.27 a su forma cannica:
p [i] = [L]1 [R] [i] + [L]1 [v]

(3.28)

En forma explcita el sistema representado en la ecuacin 3.28 es:

i1
i2

L1 M
M L2

1  

 

R1 0
i1
v1

+
0 R2
i2
v2

(3.29)

Considerando que los dos devanados del transformador son idnticos se obtiene L1 = L2 = L y
R1 = R2 = R. Con estas condiciones y evaluando la matriz inversa de la ecuacin 3.29 se obtiene:
p

i1
i2

R
= 2
L M2

L M
M L



i1
i2

1
+ 2
L M2

L M
M L



v1
v2

(3.30)

Los valores propios de la matriz caracterstica del sistema de ecuaciones diferenciales lineales
de primer grado se pueden calcular a partir de:
det {[A] [I]} = 0
Reemplazando la matriz caracterstica de la ecuacin 3.30 en 3.31:

92

(3.31)

III.4 El transformador como circuito acoplado


det

R
2
L M2

L M
M L



=0

(3.32)

Calculando el determinante de la ecuacin 3.32 se obtiene:


2RL
R2
=0
+ 2
+ 2
L M2
L M2
2

(3.33)

El polinomio de segundo grado en 3.33, tambin denominado polinomio caracterstico, posee


dos races que corresponden a los autovalores de la matriz caracterstica [A]:

1 =

R
1
=
L+M
M

(3.34)

2 =

1
R
=
LM
F

(3.35)

Donde:

M
F

es la constante de tiempo de magnetizacin [s]


es la constante de tiempo de fuga o dispersin[s]

Como L y M son valores positivos, 2 es mucho mayor que 1 si el coeficiente de acoplamiento


mutuo k12 es cercano a la unidad. De la ecuacin 3.25 se obtiene para el transformador de la
figura 3.3:

M = k12 L1 L2 = k12 L

(3.36)

A partir de 3.34, 3.35 y 3.36, se determinan M y F como:

M =

L+M
L
= (1 + k12 )
R
R

(3.37)

F =

L
LM
= (1 k12 )
R
R

(3.38)

Para resolver el sistema de ecuaciones diferenciales 3.27, se determina la solucin homognea


a partir de los autovalores y autovectores de la matriz caracterstica, calculados mediante las
expresiones 3.34 y 3.35. La solucin completa se obtiene superponiendo a la solucin homognea la solucin particular y determinando los coeficientes constantes a partir de las condiciones
iniciales del problema. La solucin homognea del problema es:

i1h (t) = Ae1t + Be2t


i2h (t) = Ce1t + De2t

(3.39)

93

Captulo III Circuitos acoplados magnticamente


Los coeficientes indeterminados A, B, C y D no son arbitrarios, se obtienen a partir de los autovectores de la matriz caracterstica. Para calcular los autovectores es necesario resolver el
sistema de ecuaciones:
{[A] i [I]} [Vi ] = [0]

(3.40)

Aplicando la ecuacin 3.40 para el primer autovalor 1 :


"

R
L2RL
+ L+M
M 2

RM
L2 M 2

RM
L2 M 2

R
L2RL
+ L+M
M 2

#

A
C

0
0

0
0

(3.41)

El sistema 3.41 se puede reducir a:


RM
2
L M2

1 1
1 1



A
C

(3.42)

Del sistema 3.42 se observa que A = C. Del autovalor 2 de la matriz caracterstica, se determina
el segundo autovector:
RM
L2 M 2

1 1
1 1



B
D

0
0

(3.43)

A partir de 3.43 se obtiene que B = D. Sustituyendo los autovectores correspondientes en la


ecuacin 3.39:

i1h (t) = Ae L+M t + Be LM t


i2h (t) = Ae L+M t Be LM t

(3.44)

Si el sistema no est alimentado por fuentes forzantes y se sustituyen en 3.44 las condiciones
iniciales i1 (0) = I e i2 (0) = 0:

I = A+B
0 = AB

(3.45)

1
I
2

(3.46)

La solucin del sistema 3.45 es:


A=B=

Reemplazando el resultado 3.46 en la ecuacin 3.44, se obtiene la siguiente solucin:

94

III.4 El transformador como circuito acoplado

Figura 3.4 Corrientes en el transformador

Figura 3.5 Circuito equivalente del transformador de dos bobinas

I R t I R t
e L+M + e LM
2
2
I R t I R t
i2h (t) =
e L+M e LM
2
2
i1h (t) =

(3.47)

En la figura 3.4 se observa el diagrama en el tiempo de las corrientes en el primario y secundario


del transformador.
El circuito de la figura 3.5 satisface la ecuacin 3.27 para el transformador de la figura 3.3. Para
obtener las ecuaciones homogneas de este circuito equivalente es necesario cortocircuitar los
dos puertos del transformador.
En la figura 3.6(a) se presenta una interpretacin en el circuito equivalente del transformador,
de la constante de tiempo de magnetizacin. Si se unen los puntos a y b de la figura, entre estos
puntos y tierra, la constante de tiempo del circuito es:

95

Captulo III Circuitos acoplados magnticamente

(a) Circuito de magnetizacin

(b) Circuito de fuga

Figura 3.6 Constante de tiempo del circuito magntico

M =

1
2 LF + M
1
2R

LF + 2M L + M
=
R
R

(3.48)

En la figura 3.6(b) se presenta el circuito equivalente para la constante de tiempo de fuga. En


este caso se desprecia la inductancia mutua M del circuito equivalente:

F =

LM
LF
=
R
R

(3.49)

Una forma ms directa para calcular la respuesta transitoria y permanente de sistemas acoplados
magnticamente consiste en aplicar la Transformada de Laplace. Si al sistema 3.27 se le aplica
esta transformacin, se obtiene el siguiente sistema de ecuaciones algebraicas:


V1 (s)
V2 (s)

R1 0
0 R2



I1 (s)
I2 (s)

L1 M
M L2

 

I1 (s)
s
I2 (s)

(3.50)

Agrupando el vector de corrientes y sustituyendo los valores del transformador de la figura 3.4:

96

V1 (s)
V2 (s)

R + sL
sM
sM
R + sL



I1 (s)
I2 (s)

(3.51)

III.4 El transformador como circuito acoplado


A partir de la ecuacin 3.51, se puede determinar la transferencia transitoria de tensiones en
el secundario de un transformador. Si el transformador se encuentra en vaco, la corriente del
circuito secundario i2 es cero y por tanto I2 (s) es cero tambin. En estas condiciones:
V1 (s) = (R + sL) I1 (s)

(3.52)

V2 (s) = sM I1 (s)

(3.53)

Dividiendo la ecuacin 3.53 por la ecuacin 3.52 se obtiene la funcin de transferencia operacional entre las tensiones secundaria y primaria del transformador:
V2 (s)
sM
=
V1 (s) R + sL

(3.54)

Si se aplica un escaln de tensin en la bobina primaria, la tensin secundaria se calcula a partir


de la ecuacin 3.54 como:
M
V
R + sL

(3.55)

R
M Rt
V e L = k12V e L t
L

(3.56)

V2 (s) =
Antitransformando la ecuacin 3.55:
v2 (t) =

La ecuacin 3.56 se ha determinado haciendo uso de la definicin del coeficiente de acoplamiento mutuo de la ecuacin 3.19. En la figura 3.7 se representa la respuesta al impulso del
transformador de dos devanados con el secundario en vaco.
Si se aplica al transformador una tensin sinusoidal en el primario en lugar de un escaln, para
el tiempo mayor o igual que cero, se tiene:
v1 (t) = V sin t V1 (s) =

V
s2 + 2

, t 0

(3.57)

Sustituyendo la ecuacin 3.57 en 3.54 se obtiene:


V2 (s) =

MV
s
2
L s + 2

(3.58)

Reagrupando la ecuacin 3.58 en fracciones parciales y antitransformando:

MV

R Rt
R
L + sin t +

e
cos t
v2 (t) = 
R2
L
L
2
L +
L2

(3.59)

La ecuacin 3.59 representa una respuesta sinusoidal en rgimen permanente superpuesta a un


decaimiento exponencial similar al obtenido en la ecuacin 3.56, cuando se aplica un escaln de
tensin al primario del transformador.

97

Captulo III Circuitos acoplados magnticamente

Figura 3.7 Respuesta en el tiempo del transformador al escaln de tensin


III.5

SUMARIO

1. Las mquinas elctricas estn constituidas en general por varios circuitos acoplados magnticamente. Su comportamiento electromagntico puede ser estudiado mediante la tcnica de autovectores y autovalores o a travs de la Transformada de Laplace, si el convertidor
es lineal, o cuando se linealiza su comportamiento en torno a un punto de operacin.
2. Si la mquina no es lineal y es necesario evaluar su comportamiento a grandes perturbaciones, las ecuaciones diferenciales deben ser integradas por mtodos numricos tales como
los algoritmos de Simpson, Euler, Euler Modificado, Regla Trapezoidal, Runge Kutta de
varios rdenes o mediante mtodos de prediccin y correccin de error como el de Adams
o el de Adams-Merson.
III.6

EJERCICIOS

PROPUESTOS

1. El circuito magntico acoplado que se ilustra en la figura 3.8 posee tres bobinas con 100,
200 y 300 vueltas respectivamente. El circuito magntico que cierra las tres bobinas tiene
una longitud media de 60 cm, un rea de 25 cm2 y una permeabilidad relativa r de 1.000.
Las tres bobinas estn constituidas por conductores de cobre del mismo dimetro (2 mm2
y 1,75 108 m). El flujo de dispersin de cada bobina es proporcional a su respectivo
nmero de vueltas y el coeficiente de acoplamiento entre la bobina de 100 y 200 vueltas
es 0,95. Determine:

a) Las resistencias e inductancias propias y mutuas de este transformador.


b) Los respectivos coeficientes de acoplamiento y dispersin.

98

III.6 Ejercicios propuestos

Figura 3.8 Circuito magntico con tres bobinas acopladas

Figura 3.9 Diagrama esquemtico del autotransformador


c) Las constantes de tiempo del circuito si la bobina de 100 vueltas est en cortocircuito
y las otra dos bobinas se conectan en serie pero en contrafase (polaridad opuesta),
excitadas con una fuente sinusoidal de 50V efectivos.
d) La corriente resultante en cada bobina si se excita en el instante inicial la bobina de
100 vueltas con 12V continuos, mientras que las otras dos bobinas estn cortocircuitadas independientemente.
2. En la figura 3.9 se representa un transformador de dos devanados conectado como autotransformador. Se desea hacer un anlisis lo ms detallado posible de la operacin en
rgimen permanente y transitorio de este convertidor. La carga del autotransformador es
un condensador y se excita mediante un escaln de tensin continua en la entrada. Determine las corrientes y tensiones permanentes y transitorias tanto en la entrada como en
condensador.

99

Captulo III Circuitos acoplados magnticamente

100

Bibliografa

H ARLOW, J. H., Electric Power Transformer Engineering, CRC Press LLC, Boca Raton, Florida, 2004.
K ELLY, O. & S IMMONS, S., Introduction to Generalized Machine Theory, McGraw and Hall,
New York, 1968.
K ULKARNI, S. V. & K HAPARDE, S. A., Transformer Engineering: Design and Practice, Marcel Dekker, Inc., New York, 2004.
M C LYMAN , W. T., Transformer and Inductor design Handbook, Marcel Dekker, Inc., New
York, 2004.
MIT, Circuitos magnticos y transformadores, Editorial Revert, Madrid, 1965.
R AS , E., Transformadores, de potencia, de medida y de proteccin, Ediciones Tcnicas Marcombo, tercera edicin, Madrid, 1975.

101

Captulo III Circuitos acoplados magnticamente

102

CAPTULO IV

Mquinas elctricas rotativas

Las mquinas elctricas se han desarrollado en un frentico proceso evolutivo que comienza a
mediados del siglo XIX y que an contina en la actualidad. Innumerables patentes de conocidos
inventores tales como Edison y Tesla, entre muchos otros, realizaron contribuciones significativas que lograron eficacia y eficiencia en la conversin electromecnica de energa. Despus
de todo este proceso, las mquinas convencionales presentan caractersticas comunes que permiten generalizar la descripcin matemtica de su comportamiento mediante las herramientas
discutidas en los dos captulos anteriores. Los modelos analticos de las mquinas elctricas convencionales pueden ser notablemente simplificados cuando se realizan las hiptesis apropiadas
y se utilizan transformaciones de las coordenadas de las variables de estado, a sistemas de coordenadas donde el comportamiento de estos convertidores se independiza de la posicin angular
del rotor.
IV.1

C ARACTERSTICAS

COMUNES

Las mquinas elctricas rotativas convencionales presentan generalmente las siguientes caractersticas comunes:
1. Poseen un eje mecnico a travs del cual se realiza el intercambio de energa.
2. Tienen una pieza esttica o inmvil denominada estator.
3. Disponen de una pieza mvil denominada rotor en el caso particular de las mquinas
cilndricas.
4. Generalmente son cilndricas.

103

Captulo IV Mquinas elctricas rotativas

Figura 4.1 Conductor en condiciones ptimas de operacin

Figura 4.2 Configuracin cilndrica de los conductores en una mquina


5. El flujo en el entrehierro de la mquina es peridico.
Para obtener la mayor fuerza elctrica posible, al estudiar el conductor en presencia de un campo
magntico, resulta conveniente que su velocidad de desplazamiento y el campo magntico se
encuentren perpendiculares entre s. Adems, de esta forma la fuerza electromotriz e aparece
disponible en el sentido del conductor, tal como se puede observar en la figura 4.1.
La mayor parte de las mquinas elctricas convencionales son cilndricas porque en esta geometra se obtiene una disposicin de todos los conductores en la cual la velocidad, el campo
magntico y los conductores son perpendiculares entre s. En la figura 4.2 se muestra un diagrama de este tipo de configuracin.
En la superficie de revolucin o manto del cilindro, se encuentran los conductores dispuestos en
forma axial y simtrica. La simetra evita vibraciones en la mquina, pero adems es necesario
que la corriente se distribuya uniformemente por todos los conductores.
En una mquina cilndrica se garantiza la periodicidad del flujo porque la divergencia de la
densidad de campo magntico es nula ( B = 0). En otras palabras, todo el flujo que penetra la
superficie cilndrica sale de ella como se ilustra en la figura 4.3.
Ahora bien:

104

Z 2
0

(4.1)

IV.1 Caractersticas comunes

Figura 4.3 Flujo entrando y saliendo de un cilindro


Donde:
d = B ds

(4.2)


ds = r l nd

(4.3)

d = Bn r l d

(4.4)

De la figura 4.3 se puede deducir que:

Y sustituyendo 4.3 en 4.2:

Como todo el flujo que penetra en el cilindro es igual al que sale de l:


Z 2
0

d = 0

Z 2
0

B ds = 0

(4.5)

A partir de la ecuacin 4.5 se determina que el diferencial de flujo en un perodo de revolucin


del cilindro es cero. Por lo tanto, la distribucin del campo magntico B en funcin del ngulo,
es peridica y existe alternancia en el signo del campo. Por otra parte, a partir de la expresin
4.5 se determina que para anular la integral en un perodo completo, las reas positiva y negativa
de la funcin densidad del campo magntico B en funcin del ngulo tienen que ser iguales, tal
como se observa en la figura 4.4.
Como la distribucin de la densidad de campo B en funcin del ngulo es peridica, se puede
descomponer en series de Fourier espaciales. En la figura 4.5 se ha representado la primera
armnica o componente fundamental del campo, suponiendo que ste presenta simetra impar.
Para calcular el valor de la densidad de campo correspondiente a la primera armnica en el punto
de la figura 4.5, se tiene:
B1 ( ) = B1 max cos

(4.6)

105

Captulo IV Mquinas elctricas rotativas

Figura 4.4 Distribucin de la densidad de campo B en un cilindro

Figura 4.5 Primera armnica de la densidad de campo B


Donde B1 max es la amplitud del campo B1 . En la figura 4.5 se ha representado la distribucin
del campo alrededor del cilindro.
IV.2

B OBINAS

ORTOGONALES

Cualquier distribucin sinusoidal en el espacio de la densidad de campo, puede ser obtenida a


partir de la suma vectorial de dos componentes ortogonales tales como B y B de la figura 4.6.
Dado que B1 es sinusoidal, la distribucin de los campos B y B tambin deben ser sinusoidales. En las mquinas elctricas convencionales se distribuyen los conductores en la periferia de
la mquina para que al inyectar las corrientes i e i que se muestran en la figura, la configuracin espacial del flujo en la periferia del cilindro resulte aproximadamente sinusoidal. En esta
figura se han representado dos bobinas colineales con los ejes y respectivamente, cuyo eje
magntico coincide con la amplitud de la distribucin espacial del campo. Cuando por cualquiera de las bobinas circula corriente, se produce un campo en toda la periferia de la mquina, cuya
amplitud se encuentra orientada segn su respectivo eje.
En general B es un vector que representa la magnitud y direccin de la primera armnica del
campo en el cilindro segn el eje . B es el vector que representa la amplitud y direccin de la
primera armnica del campo segn el eje . Por lo tanto:

106

IV.3 Mltiples pares de polos

Figura 4.6 Distribucin espacial del campo en el cilindro

B1 max = B + B

(4.7)

Tanto B como B tienen un solo grado de libertad, es decir, solamente puede variar en magnitud
o signo, pero no en direccin. Las dos componentes poseen el mismo perodo espacial y se
encuentran en cuadratura, segn la posicin relativa de los devanados. Para obtener cualquier
valor de B o B , es suficiente con ajustar las corrientes i e i . Estas consideraciones son
vlidas tanto para el rotor como para el estator de las mquinas elctricas rotativas.

IV.3

M LTIPLES

PARES DE POLOS

Cuando se analiza la distribucin del flujo en la mquina, se observa que en una zona de los 2
radianes, el campo es positivo el flujo sale de la superficie y en el resto del cilindro es negativo
el flujo penetra en la superficie del cilindro. La zona del cilindro en el cual hay salida del flujo
se define como polo norte y la regin por la cual penetra el flujo a la superficie se define como
polo sur. En la figura 4.7 se ilustra el polo norte y sur de un cilindro elemental excitado por dos
conductores.
Las mquinas elctricas pueden ser diseadas de tal manera que en el desarrollo de 2 radianes
existan varios polos norte y varios polos sur. En la figura 4.8 se muestra un ejemplo de un cilindro
en el cual existen dos polos norte y dos polos sur alternados entre s. Esta situacin corresponde
a una mquina con dos pares de polos, pero se puede repetir con cualquier cantidad de pares
de polos. Como en las mquinas elctricas cada par de polos se repite exactamente igual, es
suficiente analizar el primer par de polos y extender los resultados obtenidos a la totalidad de la
mquina.
En una mquina con mltiples pares de polos se definen ngulos elctricos y mecnicos. Los
ngulos mecnicos o ngulos fsicos son los que se han utilizado en todo el anlisis y son ngulos

107

Captulo IV Mquinas elctricas rotativas

Figura 4.7 Polo norte y sur de un cilindro

Figura 4.8 Cilindro con dos pares de polos

108

IV.3 Mltiples pares de polos

Figura 4.9 Definicin de ngulos elctricos y mecnicos

reales. Para definir los ngulos elctricos se acota un paso polar de la mquina, es decir la zona
comprendida por un par de polos y se define este ngulo mecnico como 2 radianes elctricos.
En la figura 4.9 se ilustra este concepto con una mquina de tres pares de polos. Entre 0 y 23
radianes mecnicos se definen 2 radianes elctricos.
Si se define como p el nmero de pares de polos de la mquina, entonces:

e = p m

(4.8)

Mediante la ecuacin 4.8 se puede estudiar la configuracin y operacin de una parte de la


mquina, recordando que en el resto se repite el proceso tantas veces como nmero de pares
de polos p tenga el convertidor. Para calcular el par, es necesario recordar que cada uno de los
elementos de repeticin produce un par idntico, por lo tanto, el par total en el eje mecnico de
la mquina real se calcula como:

Ttotal = p Te

(4.9)

El rotor y el estator de una mquina deben tener siempre el mismo nmero de pares de polos,
porque en caso contrario no es posible producir par promedio neto diferente de cero. En la figura
4.10 se muestra un ejemplo de esta situacin. En la mquina (a) los polos norte y sur intentan
alinearse para reducir al mnimo posible la longitud de los enlaces de flujo y, por lo tanto, la
energa almacenada en el campo. En la mquina (b) se producen pares iguales y opuestos, y por
esta razn el par total sobre el eje es nulo.
En los anlisis de los captulos posteriores, se considera siempre un par de polos extendido a 2
radianes elctricos. No se utiliza un ndice especfico para diferenciar los ngulos elctricos de
los ngulos mecnicos. En los casos en que es necesario, se indica en las expresiones el nmero
de pares de polos p de la mquina en estudio.

109

Captulo IV Mquinas elctricas rotativas

Figura 4.10 Mquina con iguales (a) y diferentes (b) nmeros de pares de polos en el rotor y en
el estator

Figura 4.11 Partes de una mquina elctrica rotativa


IV.4

LA

MQUINA GENERALIZADA

Las mquinas elctricas rotativas poseen caractersticas comunes entre s y en general se asemejan al modelo representado en la figura 4.11. En algunas ocasiones el elemento interior de la
mquina es fijo y el exterior, mvil. Incluso pueden ser mviles los dos elementos; pero lo ms
caracterstico de las mquinas elctricas rotativas es la existencia de dos superficies cilndricas
con movimiento relativo entre una y otra.
El flujo puede ser descompuesto en dos componentes ortogonales y . Para representar el flujo
producido en el rotor se inyectan corrientes en las bobinas r y r , fijas en el rotor. El flujo del
estator se obtiene inyectando corrientes en las bobinas e y e fijas en el estator. Estos devanados
no tienen necesariamente existencia fsica, pero pueden reproducir los campos en el interior de
la mquina. La posicin relativa entre el rotor y el estator queda determinada mediante el ngulo
, medido entre los ejes magnticos e y r respectivamente.
La mquina elctrica generalizada posee cuatro ejes elctricos e , r , e y r por los cuales se
inyectan las corrientes y un eje mecnico o eje de giro. El flujo en el entrehierro de la mquina cambia su distribucin cuando varan las corrientes i r , i r , i e e i e . En la figura 4.12 se
representa el esquema de las bobinas ortogonales de la mquina generalizada.

110

IV.4 La mquina generalizada

Figura 4.12 Esquema de la mquina generalizada


Definiendo a Tm como el par mecnico en el eje de la mquina, las ecuaciones de la mquina en
forma matricial compacta son:

d
[v] = [R] [i] + [ ( )] [i] + [L( )] [i]
dt
1 t
Tm = [i] [ ( )] [i] + J +
2

(4.10)

En el sistema de ecuaciones 4.10, es el coeficiente de friccin y J es la inercia del eje de


rotacin. Las variables de estado de este sistema de ecuaciones diferenciales son las corrientes
[i], el ngulo y la velocidad angular ddt , denominada tambin m .
Para poder plantear el sistema 4.10, es necesario determinar las matrices de resistencias [R],
inductancias en funcin del ngulo [L( )], as como la derivada con respecto al ngulo de la
matriz de inductancias, tambin denominada matriz de par [ ( )].
La matriz de resistencias
La matriz de resistencias de la mquina elctrica generalizada es diagonal porque todas las resistencias son propias de cada bobina y no existen resistencias mutuas debido a que los devanados
estn aislados galvnicamente:

0
0
Re 0
0 Re 0
0

[R] =
0
0 Rr 0
0
0
0 Rr

(4.11)

111

Captulo IV Mquinas elctricas rotativas


La matriz de inductancias
Si la mquina posee un rotor cilndrico y homogneo, al girar no se modifica la permeanza del
camino magntico, por esta razn la inductancia propia del estator no depende de la posicin
del rotor. La inductancia propia del estator es constante e independiente del ngulo . Esta
inductancia se puede calcular como:
Le = Ne2 Pe

(4.12)

Si el estator es cilndrico, la inductancia propia del rotor es constante por el mismo razonamiento
anterior. Si todos los devanados del estator poseen el mismo nmero de vueltas y lo mismo
ocurre con las bobinas del rotor, los trminos de la diagonal de la matriz de inductancia son:

Le ? ? ?
? Le ? ?

[L] =
? ? Lr ?
? ? ? Lr

(4.13)

Las inductancias mutuas entre los devanados y del estator son cero porque estas bobinas
son ortogonales y el flujo que se produce en una de ellas no puede enlazar a la otra. La misma
situacin sucede con los devanados del rotor:

Le 0 ? ?
0 Le ? ?

[L] =
? ? Lr 0
? ? 0 Lr

(4.14)

La inductancia mutua entre la bobina del estator y del rotor es mxima cuando ambos devanados se encuentran alineados, es decir con = 0. Para representar este valor de la inductancia
mutua se debe utilizar un trmino en cos .
Una situacin semejante se presenta entre el eje del estator y el eje del rotor. La inductancia
mutua entre las bobinas del rotor y del estator es mxima cuando = 2 ; esto se representa
mediante un trmino en sen . La inductancia mutua entre el devanado del rotor y del estator
es mxima cuando = 2 ; por esta razn esta inductancia se puede representar mediante un
trmino sen .
De esta forma y recordando que la matriz de inductancias es simtrica, se obtiene:

Le
0
Ler cos Ler sen

0
Le
Ler sen Ler cos

[L] =

Ler cos Ler sen


Lr
0
Ler sen Ler cos
0
Lr

(4.15)

Matriz de par
La matriz de par [ ( )] se calcula derivando la matriz de inductancias de la mquina con respecto
al ngulo :

112

IV.5 Clculo del par elctrico

[ ( )] =

d
[L]
d

(4.16)

De esta forma se obtiene:

0
0
Ler sen Ler cos

0
0
Ler cos Ler sen

[ ( )] =
Ler sen Ler cos

0
0
Ler cos Ler sen
0
0r

IV.5

C LCULO

(4.17)

DEL PAR ELCTRICO

A partir de las matrices 4.15 y 4.17 se puede calcular el par elctrico de la mquina:
t
ie
Le
0
Ler cos Ler sen

1
ie
0
Le
Ler sen Ler cos
Te =
Lr
0
2 ir Ler cos Ler sen
ir
Ler sen Ler cos
0
Lr

Efectuando los productos matriciales en la ecuacin 4.18 se obtiene:

ie
ie

ir
ir

(4.18)




Te = Ler sen ie ir ie ir + cos ie ir + ie ir

(4.19)

Te = Ler {A sen + B cos }

(4.20)

Si las corrientes del estator o del rotor son cero, todos los trminos del par elctrico en la ecuacin 4.19 se anulan y no se produce par. Si se inyectan corrientes constantes en todas las bobinas
del rotor y del estator, el par elctrico que se obtiene es de la forma:

En la ecuacin 4.20 se observa que para cada valor de la posicin del rotor existe un par
elctrico, pero el promedio de ese par en un giro completo de la mquina es cero. sta es una
razn que refuerza el concepto de la imposibilidad de que una mquina elctrica pueda funcionar
en rgimen permanente con corriente continua en todos sus devanados.
Calculando el par elctrico promedio de la mquina bifsica en un perodo se obtiene:
Z


Ler T 
sen ie ir ie ir + cos ie ir + ie ir d
hTe i =
(4.21)
T 0
El ngulo en la expresin 4.21, considerando que el rotor gira a velocidad angular constante
m , se puede expresar como:

= m t + 0

(4.22)

Sustituyendo la expresin 4.22 en 4.21 se obtiene:

113

Captulo IV Mquinas elctricas rotativas

Ler
hTe i =
T

Z T


sen (mt + 0 ) ie ir ie ir +
0

+cos (mt + 0 ) ie ir + ie ir d m t

(4.23)

Si se expresan las corrientes en forma de cosenos:

ie =
ie =
ir =
ir =

Recordando que:

2Ie cos (e t + e )

2Ie cos e t + e
2Ir cos (r t + r )

2Ir cos r t + r

(4.24)

1 T
sen cos d = 0
(4.25)
T 0
Los nicos trminos que pueden producir par promedio diferente de cero son los productos de
cosenos, por lo tanto:
Z

Ler
hTe i =
T

Z T
0

cos (mt + 0 ) ie ir + ie ir

(4.26)

Si las corrientes estatricas y rotricas son peridicas, es posible expresarlas mediante series de
Fourier. Utilizando expansin de las funciones en series de cosenos:

ie =

Ikecos (ket e )

(4.27)

k=1

ir =

I jr cos ( jr t r )

(4.28)

j=1

Los trminos del par son de la forma:

k=1

j=1

cos (mt + 0 ) Ike cos (ket e ) I jr cos ( jr t r )

(4.29)

Recordando la propiedad trigonomtrica:


cos cos cos

1
[cos ( + + ) + cos ( + ) +
4
+ cos ( + ) + cos ( + + )]

(4.30)

Se puede aplicar esta propiedad al trmino genrico del par elctrico 4.29. El trmino genrico
queda entonces as:

114

IV.5 Clculo del par elctrico

cos (m t + 0 ket jr t e r )

(4.31)

Para que un trmino igual al 4.31 tenga un promedio diferente de cero en un perodo, es necesario
que se anule la dependencia del tiempo en el argumento de la funcin coseno:

m ke jr = 0

(4.32)

La ecuacin 4.42 es fundamental en el anlisis de las mquinas elctricas rotativas y se conoce


como condicin necesaria para par promedio. En la ecuacin 4.42, m es la velocidad mecnica
del sistema, e representa la frecuencia angular de las corrientes inyectadas en las bobinas del
estator y r es la frecuencia angular de las corrientes inyectadas en el rotor.
Los tipos ms comunes de mquinas elctricas convencionales se diferencian por el mecanismo
que utilizan para dar cumplimiento a la ecuacin 4.42. Las mquinas sincrnicas, de induccin
y de corriente continua utilizan diferentes mecanismos de excitacin de sus bobinas rotricas y
estatricas, pero siempre deben satisfacer la condicin necesaria de par promedio para permitir
la conversin de energa.
La mquina sincrnica
A las mquinas sincrnicas se les inyecta corriente continua en las bobinas rotricas, por esta
razn:

r = 0

(4.33)

Aplicando la condicin necesaria de par promedio 4.42 con la restriccin 4.43 para las mquinas
sincrnicas se obtiene:

m ke = 0

(4.34)

La ecuacin 4.44 justifica el nombre de estas mquinas, ya que las mquinas sincrnicas slo
pueden producir par promedio diferente de cero cuando la velocidad mecnica coincide con la
velocidad angular de las corrientes inyectadas en el estator. En otras palabras, la mquina debe
girar en sincronismo con las corrientes estatricas.
Las mquinas de corriente continua son un caso particular de mquina sincrnica, donde la
igualdad de frecuencias entre las corrientes rotricas en este caso y la velocidad mecnica se
obtiene mediante un inversor mecnico constituido por un colector y un juego de carbones que
conmuta las corrientes en las bobinas del rotor, con una frecuencia igual a la velocidad mecnica
de giro.
La mquina de induccin
En la mquina de induccin se permite un grado de libertad adicional. En esta mquina se puede obtener par promedio diferente de cero en un amplio rango de velocidades mecnicas. Las

115

Captulo IV Mquinas elctricas rotativas


corrientes que circulan por el rotor se ajustan por el fenmeno de induccin electromagntica
y cumplen la condicin 4.42. En la mquina de induccin se fija la frecuencia de las corrientes
en el estator e , se produce un campo electromagntico en el entrehierro de la mquina que gira
mecnicamente con la frecuencia angular de estas corrientes. Como el rotor gira a la velocidad mecnica m , los conductores del rotor cortan el campo magntico producido en el estator
con una velocidad que es la diferencia entre e y m . La diferencia porcentual entre estas dos
velocidades se conoce como deslizamiento de la mquina:
s=

e m
100
e

(4.35)

La velocidad angular e se conoce como velocidad sincrnica de la mquina de induccin.


La mquina de corriente continua
En la figura 4.13 se muestra una mquina de corriente continua simplificada. Esta mquina posee
un devanado estatrico por el cual se inyecta corriente continua y una armadura en el rotor
alimentada mediante una fuente de corriente continua y un colector que permite la inversin de
las corrientes en la armadura. Para calcular el par elctrico que produce esta mquina se utiliza
la expresin deducida en el captulo 2 para los sistemas lineales:
Te =

1 t
[i] [ ( )] [i]
2

(4.36)

Desarrollando explcitamente la ecuacin 4.46 se obtiene:



1
ie ir
Te =
2

0
Msen
Msen
0



ie
ir

(4.37)

En la expresin anterior, M es la inductancia mutua entre el estator y el rotor. Realizando las


operaciones matriciales en la ecuacin 4.47:
Te = M ie ir sen

(4.38)

El colector o conmutador mecnico de la mquina de corriente continua permite alternar la


polaridad de la tensin de alimentacin de la bobina del rotor Vr al mismo tiempo que gira el
rotor. En la figura 4.13 se observa tambin la corriente que circula por la armadura (rotor) de la
mquina.
El par promedio en el eje de la mquina se calcula como:
Z

Z 2
1
MIe Ir sen d
MIe Ir sen d +
hTe i =
2
0

(4.39)

Resolviendo las integrales de la ecuacin 4.49 se obtiene:


hTe i =
116

2M
Ie Ir = k Ie Ir

(4.40)

IV.6 Par elctrico y fuerzas magnetomotrices

Figura 4.13 Diagrama esquemtico de una mquina elemental de corriente continua


La expresin anterior determina el par elctrico promedio en la mquina de corriente continua.
El coeficiente k depende de la construccin fsica de los devanados del rotor y del estator.
IV.6

PAR

ELCTRICO Y FUERZAS MAGNETOMOTRICES

En la figura 4.14 se representa el diagrama de una mquina elctrica cilndrica con un estator y
un rotor. En el estator y rotor se producen las fuerzas magnetomotrices Fe y Fr respectivamente,
cuya amplitud y direccin se representa vectorialmente en la figura. Estas fuerzas magnetomotrices se encuentran separadas en un ngulo una de la otra. La suma de las fuerzas magnetomotrices del rotor y del estator produce la fuerza magnetomotriz resultante en el entrehierro
de la mquina Ft . Para calcular el par elctrico de una mquina en funcin de las fuerzas electromotrices, se determina la coenerga en el campo y luego se deriva con respecto a la posicin
angular :

Wc (F, )
Te =
(4.41)

De la figura 4.14 se deduce:

Ft2 = F2r + F2e + 2Fr Fe cos

(4.42)

Si la permeabilidad del material magntico es muy grande, es decir r tiende a infinito, toda la
energa est concentrada en el entrehierro y la coenerga se puede calcular de la siguiente forma:

Wc = Wc = volumen hwc i

(4.43)

117

Captulo IV Mquinas elctricas rotativas

Figura 4.14 Par elctrico a partir de las fuerzas magnetomotrices


En la ecuacin 4.43, hwc i representa la energa promedio en el campo por unidad de volumen.
De esta forma:



1
Wc = 2 r l
HB
(4.44)
2
Donde:
r

es el radio medio del entrehierro [m].

es el espesor del entrehierro [m].

es la longitud activa de la mquina [m].

Como la densidad de campo magntico B en el entrehierro es igual a 0 H:




1

2
0 H
Wc = 2 r l
2

(4.45)

La primera armnica de la intensidad de campo magntico H es sinusoidal y su valor promedio


es:
Z

2
1 2
1 2
(Hmax sen )2 d = Hmax
(4.46)
H =
2 0
2
Sustituyendo 4.46 en 4.45:
1 2

Wc = 2 r l Hmax
(4.47)
2
En la ecuacin 4.47 es necesario expresar la amplitud de la intensidad de campo magntico de
primera armnica en funcin de las fuerzas magnetomotrices. En la figura 4.15 se representa
una mquina con un devanado en el estator. Como la permeabilidad del hierro es infinita, toda
la fuerza magnetomotriz se utiliza para que el flujo cruce el entrehierro. Aplicando la ley de
Ampre a esta mquina, se tiene:
F = NI =

118

H dl =

Haire dlaire +

Hhierro dlhierro

(4.48)

IV.6 Par elctrico y fuerzas magnetomotrices

Figura 4.15 Fuerzas magnetomotrices e intensidades de campo magntico


El segundo trmino integral es cero, ya que:
B
=0
0 hierro

Hhierro =

(4.49)

Sustituyendo 4.49 en 4.48:


F = NI =

H dl =

Haire dlaire

(4.50)

En la figura 4.15 tambin se representa la distribucin de la intensidad del campo magntico en


funcin de la posicin de la trayectoria de Ampre. De esta forma se obtiene:
F = NI =

H dl =

Haire dlaire = 2 Haire

(4.51)

Despejando de la ecuacin 4.51 la intensidad de campo magntico en funcin de la fuerza electromotriz:


F
(4.52)
H=
2
Sustituyendo la ecuacin 4.52 en la ecuacin 4.47 se obtiene:

Wc =

rl 0 2
F
8

(4.53)

Reemplazando la ecuacin 4.42 en la ecuacin 4.53:

Wc =


rl 0 2
Fr + F2e + 2Fr Fe cos
8

(4.54)

119

Captulo IV Mquinas elctricas rotativas

(a) Distribucin trapezoidal

(b) Distribucin triangular

Figura 4.16 Diferentes distribuciones de conductores y campos en las mquinas


Para calcular el par elctrico se utiliza la ecuacin 4.31:

Wc (F, )
rl 0
Te =
=
Fr Fe sen

(4.55)

Mediante la ecuacin 4.55 se puede calcular el par elctrico en funcin de las fuerzas magnetomotrices de la mquina. La fuerza magnetomotriz depende de las corrientes y del nmero de
vueltas de las bobinas. Si se conocen las dimensiones de la mquina, las corrientes y el nmero
de conductores de cada bobina, es posible utilizar la ecuacin 4.55 para determinar el par.
Si la distribucin de las corrientes en la mquina no es puntual, se puede utilizar la misma tcnica
para calcular la intensidad de campo magntico H pero se tiene en cuenta que:
I

H dl =

Z Z

J ds

(4.56)

En la figura 4.16 se muestran dos distribuciones diferentes de los conductores en la superficie de


una mquina, as como su respectiva distribucin de intensidades de campo magntico H. Cuando el entrehierro es constante, la densidad de campo magntico B posee la misma distribucin
que la intensidad de campo magntico H.

120

IV.7 El campo magntico rotatorio

Figura 4.17 Corrientes inyectadas en la mquina generalizada


IV.7

EL

CAMPO MAGNTICO ROTATORIO

Cuando se analizaron las bases de la mquina elctrica generalizada, se utilizaron dos grados de
libertad para la representacin del campo magntico, uno dado por la bobina y el otro por la
bobina . Mediante este esquema se puede determinar el campo en cualquier punto del plano.
En la figura 4.17 se muestran dos corrientes i e i que pueden ser inyectadas en las bobinas
y de la mquina.
En el instante inicial t = 0 la corriente i vale cero e i es I, por lo tanto el campo resultante
apunta en la direccin negativa del eje . Cuando el tiempo se incrementa y llega al instante

2 , la corriente i se anula, mientras que la corriente i es +I, el campo en estas condiciones


apunta en la direccin positiva del eje . En el instante el flujo se orientar segn la direccin
positiva del eje , ya que la corriente i tiene como valor +I y la corriente i es cero. Para
el instante de tiempo 2 , la corriente i es cero, la corriente i vale I y el vector del campo
apunta nuevamente en la direccin negativa del eje , repitindose de esta forma las condiciones
iniciales. En la figura 4.18 se representa la situacin anterior.
El anlisis anterior seala que las corrientes que varan en el tiempo, producen un campo magntico que gira en el espacio. Aun cuando los campos de cada eje tienen igual amplitud, el
desfasaje en el tiempo y en el espacio origina un campo magntico rotatorio. La frecuencia de
giro del campo magntico en el espacio es igual a la frecuencia de variacin de las corrientes en
el tiempo.
Si la bobina no es idntica a la bobina , o las corrientes inyectadas a la mquina en cada eje
difieren en amplitud, el campo no es circular sino elptico. Los campos elpticos tambin son
considerados campos magnticos rotatorios o rotantes. Las mquinas trifsicas tambin funcionan mediante el principio del campo magntico rotatorio.

121

Captulo IV Mquinas elctricas rotativas

Figura 4.18 Campo magntico rotatorio

IV.8

LA

MQUINA TRIFSICA

La mquina trifsica dispone de tres devanados repartidos simtricamente en la periferia del cilindro. En la figura 4.19 se representa la configuracin esquemtica de este tipo de mquinas as
como las tres corrientes que se han inyectado en las bobinas a, b, y c. En la figura se representan
las corrientes a, b, c y las fuerzas magnetomotrices que estas corrientes producen en el tiempo
inicial (t = 0) como fasores. En el instante inicial las corrientes que circulan por las tres bobinas
son:

ia (0) = Imax
1
ib (0) = Imax
2
1
ic (0) = Imax
2

(4.57)

Para demostrar que el campo magntico originado por las corrientes de la figura 4.19 es rotatorio,
se expresan estas corrientes como:
ia (t) = I cos( t )

2
)
3
4
ic (t) = I cos( t )
3

ib (t) = I cos( t

122

(4.58)

IV.8 La mquina trifsica

Figura 4.19 Corrientes y fuerzas magnetomotrices de la mquina trifsica


Si es la direccin de un punto cualquiera en el entrehierro medido a partir del eje magntico
de la bobina a, se obtiene:
F( ,t) = N ia cos + N ib cos( +

4
2
) + N ic cos( + )
3
3

(4.59)

Sustituyendo las expresiones 4.58 en la ecuacin 4.59 se obtiene:


F( ,t) = N I {cos( t ) cos +
2
4
+ cos( t ) cos( + ) +
3
3
4
2
+ cos( t ) cos( + ) }
3
3

(4.60)

Aplicando las propiedades trigonomtricas para el producto de cosenos se obtiene:


F( ,t) =

NI
{cos( t + ) + cos( t ) +
2
2
+ cos( t + + ) + cos( t ) +
3
2
+ cos( t + ) + cos( t }
3

(4.61)

En la ecuacin anterior los trminos primero, tercero y quinto de la sumatoria de cosenos suman
cero porque el desfasaje entre ellos es de 23 . Con esta consideracin se obtiene:
3
F( ,t) = N Icos( t )
2

(4.62)

Esta expresin permite obtener la fuerza magnetomotriz en el espacio y en el tiempo. Si se fija


la posicin, es decir, el ngulo es constante, la ecuacin 4.62 indica que en esa posicin la

123

Captulo IV Mquinas elctricas rotativas


fuerza magnetomotriz vara sinusoidalmente en el tiempo. Si se congela el tiempo en un instante
especfico, la expresin 4.62 determina una distribucin sinusoidal de la fuerza magnetomotriz
en el espacio. La ecuacin 4.62 demuestra que en una mquina elctrica trifsica, alimentada
por tres corrientes balanceadas y desfasadas 23 en el tiempo, produce un campo magntico rotatorio similar al producido por dos devanados ortogonales a los cuales se les inyecten corrientes
sinusoidales desfasadas 2 .
IV.9

T RANSFORMACIN

DE COORDENADAS

El sistema de ecuaciones diferenciales 4.10, que modela el comportamiento de la mquina elctrica, no es lineal. La dependencia en de este modelo dificulta notablemente la solucin de
cualquier problema. La transformacin de las ecuaciones diferenciales a nuevos sistemas de
coordenadas simplifica en muchos casos este modelo.
Un nuevo sistema de coordenadas se puede definir mediante una matriz de transformacin aplicada a las variables en coordenadas primitivas y . Las tensiones y corrientes en el nuevo
sistema transformado son:


(4.63)
v e e, r r = [Awxyz ] [vwxyz ]


i e e, r r = [Awxyz ] [iwxyz ]
(4.64)
Donde:

Awxyz

es la matriz de transformacin

v e e, r r

son las tensiones en coordenadas primitivas

vwxyz

son las tensiones en las nuevas coordenadas

i e e, r r

son las corrientes en coordenadas primitivas

iwxyz

son las corrientes en las nuevas coordenadas

La potencia en coordenadas primitivas se puede calcular mediante la expresin:


t 


p = i e e, r r v e e, r r

(4.65)

En la expresin 4.65, el asterisco () indica que el vector de corrientes se debe conjugar en caso
de ser complejo y el superndice t representa una trasposicin del vector de corrientes para que
el producto matricial con el vector de tensiones sea conformable. Sustituyendo en la ecuacin
4.65 las definiciones 4.63 y 4.64, se obtiene:
p = [iwxyz ]t [Awxyz ]t [Awxyz ] [vwxyz ]

(4.66)

Para que la transformacin utilizada [Awxyz ] sea invariante en potencia es necesario que:
[Awxyz ]t [Awxyz ] = [I]

(4.67)

En la ecuacin 4.67, [I] es la matriz identidad. De esta expresin se obtiene:


[Awxyz ]t = [Awxyz ]1

124

(4.68)

IV.9 Transformacin de coordenadas


Una matriz que satisface la condicin 4.68 se denomina hermitiana o hermtica. La ecuacin
4.68 indica que si en la matriz de transformacin de coordenadas, su conjugada traspuesta es
idntica a la matriz inversa, dicha transformacin es conservativa en potencia. En otras palabras,
una transformacin hermitiana permite calcular las potencias en las variables transformadas sin
necesidad de regresar a las coordenadas primitivas.
Las ecuaciones de los ejes elctricos de la mquina se pueden escribir como:

 
 


 

v , = R , + L , p + , i ,

Transformando las coordenadas en la ecuacin 4.69 se obtiene:


 




[Awxyz ] [vwxyz ] = R , + L , p + , [Awxyz ] [iwxyz ]
Despejando de 4.70 el vector de tensiones se obtiene:
n


[vwxyz ] =
[Awxyz ]1 R , [Awxyz ] +


+ [Awxyz ]1 L , [Awxyz ] p +

d
+ [Awxyz ]1 L ,
[Awxyz ] +
dt
o

1 

, [Awxyz ] [iwxyz ]
+ [Awxyz ]

(4.69)

(4.70)

(4.71)

La ecuacin 4.71 se puede escribir utilizando las siguientes definiciones:




[Rwxyz ] [Awxyz ]1 R , [Awxyz ]


[Lwxyz ] [Awxyz ]1 L , [Awxyz ]


[wxyz ] [Awxyz ]1 , [Awxyz ]

(4.72)
(4.73)
(4.74)

Como la matriz de transformacin puede depender en general de la posicin angular , se obtiene:


d
d
d
[Awxyz ]
[Awxyz ] =
dt
d
dt

(4.75)

y definiendo:
 d

[Awxyz ]
[Hwxyz ] [Awxyz ]1 L ,
d
Se puede escribir la ecuacin 4.71 como:


[vwxyz ] = [Rwxyz ] + [Lwxyz ] p + [[wxyz ] + [Hwxyz ]] [iwxyz ]

(4.76)

(4.77)

En la ecuacin 4.77, el segundo trmino de la sumatoria, corresponde a las fuerzas electromotrices de transformacin y el trmino tercero a las fuerzas electromotrices de generacin. Este
ltimo trmino se descompone en dos partes, por un lado la matriz de par [wxyz ] y por otro la

125

Captulo IV Mquinas elctricas rotativas

Figura 4.20 Transformacin de coordenadas de del rotor a dq del rotor


matriz [Hwxyz ] que reproduce los trminos de generacin originados por el movimiento relativo
de los ejes transformados con respecto a los ejes reales. La matriz [Hwxyz ] determina los trminos
no-holonmicos debidos a la transformacin de coordenadas.
La ecuacin dinmica de la mquina se expresa como:
Tm =

t 


1
i ,
, i , + J +
2

(4.78)

Transformando la ecuacin 4.78 a las nuevas coordenadas:




1
Tm = [iwxyz ]t [Awxyz ]t , [Awxyz ] [iwxyz ] + J +
2

(4.79)

y sustituyendo la ecuacin 4.74 en 4.79:


1
Tm = [iwxyz ]t [wxyz ] [iwxyz ] + J +
2

(4.80)

Las ecuaciones 4.77 y 4.80 representan a la mquina elctrica en un nuevo sistema de coordenadas. Mediante una seleccin apropiada de la matriz de transformacin [Awxyz ], es posible
encontrar una solucin ms simple al sistema de ecuaciones diferenciales que definen el comportamiento de la mquina.

IV.10

T RANSFORMACIN

DE COORDENADAS

dq

Una transformacin til en el anlisis de las mquinas elctricas rotativas consiste en proyectar
las coordenadas del rotor en ejes colineales con los ejes del estator. Estos nuevos ejes se denominan directo dr y cuadratura qr ; esta transformacin permite anular el movimiento de las bobinas

126

IV.10 Transformacin de coordenadas dq


del rotor y las inductancias entre el estator y el rotor son constantes en el sistema de coordenadas transformadas. En la figura 4.20 se ha representado un diagrama con la transformacin
propuesta. En esta transformacin, las tensiones y corrientes correspondientes a las coordenadas
primitivas del rotor son referidas a nuevas tensiones y corrientes inyectadas en bobinas fijas en
el espacio. Los ejes del estator permanecen inalterados en las nuevas coordenadas. La matriz de
transformacin de coordenadas se puede particionar de la siguiente forma:




[Aee ] [0]
A dq =
(4.81)
[0] [Arr ]

Las coordenadas del estator no cambian en la transformacin, por esta razn la submatriz [Aee ]
debe ser unitaria:


1 0
[Aee ] =
(4.82)
0 1
Para determinar [Arr ] se debe recordar que:




ir r = [Arr ] idr qr

(4.83)

La matriz [Arr ] corresponde a la proyeccin de los ejes r y r sobre los ejes dr y qr solidarios
con el estator. Esta transformacin es una rotacin inversa que anula la rotacin del rotor de la
mquina. De la figura 4.20 se deduce que la transformacin de coordenadas es:
[Arr ] =

cos sen
sen cos

(4.84)

La matriz obtenida en la ecuacin 4.84 es hermitiana y su traspuesta conjugada es igual a su


inversa:
1

[Arr ]

cos sen
sen cos

1

1
=
2
cos + sen2

cos sen
sen cos

= [Arr ]t

(4.85)

Definida
la transformacin
rr ], es posible determinar las matrices transforma

  decoordenadas[A


das R dq , L dq , dq y H dq .

Matriz de resistencias en coordenadas dq




La matriz de resistencia R dq en las nuevas coordenadas es:





1 

R dq = A dq
R , A dq =



1 
[I] [0]
[I] [0]
Re [I] [0]
=
[0] [Arr ]
[0] Rr [I]
[0] [Arr ]t

Efectuando el triple producto matricial de la ecuacin 4.86 se obtiene:






Re [I] [0]
R dq =
[0] Rr [I]

(4.86)

(4.87)

127

Captulo IV Mquinas elctricas rotativas


Como se observa en la ecuacin 4.87, la transformacin aplicada no modifica la matriz original
de resistencias. Esto es de esperar, debido a que las resistencias no dependen de la posicin del
rotor y no existe acoplamiento galvnico entre las bobinas.
Matriz de inductancias en coordenadas dq


Si se aplica la transformacin a la matriz de inductancia L , se obtiene:



1 


L , A ,dq =
L ,dq = A ,dq

Le 0 Ler 0



0 Le 0 Ler
Le [I] Ler [I]
=
=
Ler 0 Lr 0
Ler [I] Lr [I]
0 Ler 0 Lr

(4.88)

En la ecuacin 4.88 se observa que la matriz de inductancias transformadas es independiente de


la posicin angular del rotor. Esto es debido a la rotacin en sentido inverso de la transformacin,
que con los ejes del rotor convierte las inductancias solidarias en inductancias que giran en contra
de la posicin angular del rotor y por tanto mantienen una posicin constante con respecto a los
ejes y del estator.
Matrices de generacin en coordenadas dq


Aplicando el mismo procedimiento a la matriz de par , se obtiene:

0
0
0 Ler


 
1 
 0
0 Ler
0

,dq = A ,dq
, A ,dq =
0
Ler 0
0
Ler 0
0
0

(4.89)





Igual que con la matriz de inductancia L ,dq , la matriz de par ,dq es independiente


del ngulo . La matriz de trminos de generacin no-holonmicos H ,dq se puede calcular
como:

0 0
0
Ler
 d 
 
1 
 0 0 Ler 0


A ,dq =
(4.90)
L ,
H ,dq = A ,dq
0 0
0
Lr
d
0 0 Lr
0


La matriz de generacin G ,dq se define de la siguiente forma:

0
0
0
0
 
 
 0

0
0
0

G ,dq = ,dq + H ,dq =


0
Ler 0 Lr
Ler 0 Lr 0

128

(4.91)

IV.11 Ecuaciones generales en coordenadas dq

Figura 4.21 Modelo esquemtico de la mquina generalizada


IV.11

E CUACIONES

GENERALES EN COORDENADAS

dq

Las ecuaciones de tensin para la mquina en coordenadas transformadas dq son:



Re + Le p
0
Ler p
0
ve
v
0
Re + Le p
0
Ler p
e =

vd Ler p
Ler
Rr + Lr p
Lr
r
vqr
Ler
Ler p
Lr Rr + Lr p

ie
i
e
id
r
iqr

(4.92)

La ecuacin 4.92 representa a la mquina elctrica en coordenadas dq. La construccin de


una mquina como sta es posible fsicamente mediante la incorporacin de un par de conmutadores como los que se ilustran en la figura 4.21. El colector permite que las inductancias propias
y mutuas vistas desde el estator sean independientes de la posicin del rotor. Las escobillas o
carbones que recolectan la
neutralizan el efecto del giro de forma anloga a lo que
 corriente,

realiza la transformacin A ,dq .

Los trminos de la ecuacin 4.92 se pueden identificar fcilmente en el modelo de la figura 4.21.
Es necesario destacar que los signos negativos tienen su origen en el sentido de giro de la mquina, las convenciones de polaridad y la posicin relativa de los ejes , , d y q. Para completar las
ecuaciones que definen el comportamiento de la mquina elctrica en las coordenadas dq, es
necesario calcular el par elctrico:
1
ie
0
0
0 Ler

1
i
0
0
L
0
er
e
Te =

0
Ler 0
0
2 idr
Ler 0
0
0
iqr

ie
i

e = Ler i id i iq
e r
e r
id
r
iqr

(4.93)

129

Captulo IV Mquinas elctricas rotativas


La ecuacin de balance del par mecnico es:

Tm = Ler ie idr ie iqr + J +

(4.94)

La condicin necesaria para la existencia del par elctrico requiere que existan al menos dos
corrientes, una en el estator y otra en el rotor, y que esas corrientes se encuentren en ejes ortogonales del modelo de la mquina generalizada.
IV.12

S UMARIO

1. Las mquinas elctricas convencionales tienen varios elementos comunes que permiten
realizar modelos analticos generalizados. En general poseen dos estructuras bien definidas y cilndricas denominadas rotor y estator. Los conductores estn colocados en la
periferia de los cilindros y la distribucin de los campos producidos por las corrientes
que por ellos circulan es peridica. Es indispensable que el nmero de pares de polos del
rotor y del estator sean idnticos para permitir la existencia de par neto para una posicin
angular dada.
2. La distribucin peridica del campo alrededor de la periferia del cilindro estator o rotor,
permite representar esta funcin mediante series de Fourier. Cada una de las armnicas de
la distribucin del campo puede ser representada por una bobina cuyo eje magntico se
encuentra orientado en direccin paralela a la amplitud de dicho componente del campo.
De esta forma es posible utilizar el lgebra vectorial para realizar la superposicin de
diferentes componentes del campo desplazados espacialmente. Este tipo de representacin
es vlida para cualquier instante de tiempo.
3. Los elementos comunes de las mquinas elctricas convencionales permiten modelar estos
convertidores utilizando dos bobinas ortogonales, simtricas y fijas que representan todos
los grados de libertad del estator y dos bobinas ortogonales, simtricas y ubicadas en la
posicin , que representan los grados de libertad del rotor. Estas cuatro bobinas pueden
modelar mquinas sincrnicas de rotor y estator liso, mquinas de induccin y mquinas
de corriente continua. Por su principio de funcionamiento estas mquinas garantizan la
condicin necesaria pero no suficiente de par promedio diferente de cero, representada
por la ecuacin 4.42.
4. Cuando se combinan campos magnticos ortogonales, desplazados en el espacio y producidos por corrientes balanceadas y sinusoidales desfasadas 2 en el tiempo, se obtienen
campos magnticos rotatorios. Estos campos permiten el giro de una distribucin espacial del campo alrededor de la mquina a travs del tiempo. Es posible obtener campos
magnticos rotatorios con dos, tres o ms bobinas, siempre y cuando se mantengan las
condiciones necesarias de simetra.
5. Las ecuaciones de la mquina generalizada en coordenadas primitivas son dependientes
de la posicin angular del rotor con respecto al estator. Transformar las coordenadas del
rotor a ejes colineales con las coordenadas del estator permite independizar las matrices
de parmetros del modelo del ngulo . De esta forma es posible obtener un sistema

130

IV.13 Ejemplo resuelto


de ecuaciones diferenciales no lineal, pero con parmetros constantes en el tiempo, cuya
solucin numrica es mucho ms simple que el modelo original y la solucin analtica
es posible considerando como hiptesis que la velocidad mecnica es aproximadamente
constante
6. Para representar los trminos no-holonmicos de la transformacin de coordenadas es necesario incluir en el modelo un sistema de contactos deslizantes que obtengan las fuerzas
electromotrices en el eje d y q respectivamente generadas en conductores en movimiento que han sido representados por bobinas fijas en el espacio. Este modelo matemtico
puede construirse fsicamente mediante un colector electromecnico. En los conductores
conectados a este colector se inyectan corrientes que producen campos fijos en el espacio
y se inducen fuerzas electromotrices por el movimiento relativo entre dichos conductores
y los campos resultantes en el espacio.
IV.13

E JEMPLO

RESUELTO

La mquina esquematizada en el diagrama ilustrado en la figura 4.22, posee un entrehierro g =


2 mm, un radio del rotor r = 10 cm, una longitud axial l = 15 cm; el estator tiene un nmero de
vueltas Ne = 200 vueltas y el rotor Nr = 150 vueltas; la resistencia del estator es de Re = 1 y la
del rotor Rr = 2 ; el coeficiente de acoplamiento estator-rotor es ker = 0,85; la tensin aplicada
a la bobina del estator es de 50V efectivos a la frecuencia de 60 Hz y el rotor se encuentra en
cortocircuito. Utilizando estos datos y el esquema de la mquina, determine:
1. Las inductancias propias y mutuas de las bobinas del rotor y estator del convertidor.
2. Las ecuaciones de tensin y par elctrico en coordenadas primitivas.
3. Las ecuaciones de tensin y par elctrico de la mquina si se transforma la bobina del
rotor a ejes solidarios con el estator.
4. Las ecuaciones de tensin y par elctrico de la mquina si se transforma la bobina del
estator a ejes solidarios con el rotor.
5. El par elctrico y la corriente por el rotor de la mquina en rgimen permanente, cuando
la velocidad del convertidor es de 3.500 rpm.
Solucin:
1. Las inductancias propias y mutuas de las bobinas del rotor y estator del convertidor:
El estator est constituido por dos bobinas concentradas cuyos ejes magnticos se encuentran desfasados 3 . Cada bobina concentrada produce una fuerza magnetomotriz sinusoidal
de primera armnica cuya magnitud es:
|F | =

4
Ni

131

Captulo IV Mquinas elctricas rotativas

Figura 4.22 Diagrama esquemtico y datos del ejemplo N. 1

Como las dos bobinas estn separadas 3 , la fuerza magnetomotriz resultante ser:
Fe =

4 Ne
4 Ne
4 Ne
ie +
ie = 3
ie 0
2
6 2
6
2

La distribucin de la fuerza magnetomotriz en el entrehierro de la mquina producida por


la excitacin de las dos bobinas del estator ser entonces:
Fe ( , ie ) =

2
3 Ne ie cos

Conocida la fuerza magnetomotriz, se puede calcular el enlace de flujo sobre cada uno de
los grupos de bobinas del estator:
Be ( , i e ) =

Ne
e1 =
2

Ne
e2 =
2

23
2
3

1
3 Ne ie 0 cos
g

Be ( , ie)ds =

3 2
N 0 r l ie
2 g e

Be ( , ie )ds =

3 2
N 0 r l ie
2 g e

Como los dos grupos de bobinas del estator estn en serie, la inductancia del estator es:
Le =

132

3 2
N 0 rl = 0,36 H
g e

IV.13 Ejemplo resuelto


La bobina rotrica es concentrada; aplicando un procedimiento similar al realizado con la
inductancia del estator se obtiene:
Lr =

4 2
N 0 r l = 0,27 H
g r

La inductancia mutua se determina directamente de las inductancias propias y del coeficiente de acoplamiento entre ambas bobinas:

Ler = ker Le Lr = 0,265 H


2. Las ecuaciones de tensin y par elctrico en coordenadas primitivas:
  
 


 
  
ie
ie
0
sen
ve
Re 0
ie
Le
Ler cos

p
Ler
=
+
Ler cos
Lr
ir
sen
0
ir
vr
0 Rr
ir
Ler
Te =
2

ie
ir

t 

0
sen
sen
0



ie
ir

= Ler ie ir sen

3. Las ecuaciones de tensin y par elctrico de la mquina si se transforma la bobina del


rotor a ejes solidarios con el estator:

ve
ie
Re + Le p
Ler p
0
vdr = Ler p
Lr idr
Rr + Lr p
(4.95)
Ler
Lr Rr + Lr p
vqr
iqr
Te = Ler ie iqr

4. Las ecuaciones de tensin y par elctrico de la mquina si se transforma la bobina del


estator a ejes solidarios con el rotor:

Ler p
vde
ide
Re + Le p Le
vqe = Le
Re + Le p
Ler iqe
Ler p
0
Rr + Lr p
vr
ir
Te = Ler iqe ir

5. El par elctrico y la corriente por el rotor de la mquina en rgimen permanente, cuando


la velocidad del convertidor es de 3.500 rpm:
En rgimen permanente, el sistema de ecuaciones 4.95 se pueden expresar de la siguiente
forma:
1


je Ler
0
Ve
Ie
Re + je Le
0
Idr = je Ler
Rr + je Lr
Lr
(4.96)
Ler
Lr
Rr + je Lr
0
Iqr
La expresin 4.96 permite determinar las corriente Ie , Idr e Iqr conocida la tensin Ve , las
velocidades angulares , e y los parmetros de la mquina Re , Rr , Le , Lr y Ler . De esta

133

Captulo IV Mquinas elctricas rotativas


forma se obtienen los siguientes resultados:
Ve = (50 + j 0)V

rad
3.500
= 366,52
= 2
60
s


3.600
rad
e = 2
= 376,99
60
s


I e = 0,3493 j0,7668 A
Idr = 0,4668 + j0,5447 A
Iqr = 0,2044 + j0,1165 A
Te (t) = Ler ie iqr = 0,05255 [cos(2et 3,7662) 0,869]
hTe i =
IV.14

E JERCICIOS

1
2

Z 2
0

Te (et) d et = +0,0457 Nm

PROPUESTOS

1. Una mquina de induccin bifsica en el estator y bifsica en el rotor, tiene sus devanados
distribuidos uniformemente en la periferia del convertidor. Cada fase del estator y rotor,
posee Ne = 500 y Nr = 300 vueltas respectivamente. El entrehierro es uniforme y de dimensin g = 3 mm. La longitud axial es l = 30 cm. El dimetro del rotor es D = 20 cm. El
coeficiente de acoplamiento entre las bobinas del estator y rotor es ker = 0,95. La resistencias de las bobinas son Re = 1 y Rr = 0,5 respectivamente. Se cortocircuitan las
bobinas del estator y se alimenta el rotor con fuentes de corriente independientes, ideales,
sinusoidales, desfasadas 2 entre s con valor efectivo Ir = 20 A. Determine:
a) Inductancias de la mquina en coordenadas primitivas.
b) Ecuaciones que describen el comportamiento del convertidor en rgimen permanente
en las condiciones de operacin enumeradas anteriormente.
c) Corriente en cada una de las bobinas del estator.
d) Par elctrico medio durante el arranque (velocidad = 0).
2. La figura 4.23 representa el corte transversal de una mquina donde se indican las corrientes en las diferentes fases tanto del estator como del rotor. En el estator las bobinas se
encuentran linealmente distribuidas en la periferia, los conductores del rotor en cambio
estn concentrados. Determine:

a) El par elctrico en la posicin ilustrada en la figura.

134

IV.14 Ejercicios propuestos

Figura 4.23 Diagrama esquemtico del ejercicio N. 2


b) El par elctrico si el rotor gira

con respecto a la posicin ilustrada en la figura 4.23.

c) El par elctrico promedio si las corrientes del rotor y del estator son constantes.
d) El par elctrico promedio si la corriente del estator es constante y la del rotor es
sinusoidal y sincronizada con la velocidad mecnica del convertidor.

3. La figura 4.24 representa tres mquinas diferentes. Se desea hacer un anlisis lo ms detallado posible de la operacin en rgimen permanente y transitorio de estos convertidores.
Las mquinas (a) y (b) son casi iguales pero tienen una excitacin diferente en el estator.
La mquina (c) es de campo cruzado, excitada con corriente alterna en el estator. Determine:

a) Las ecuaciones en coordenadas primitivas.


b) Transforme el rotor a ejes dq y exprese las ecuaciones en estas coordenadas. Analice
el rgimen permanente de los tres convertidores en este sistema de coordenadas.
c) Transforme el estator a coordenadas dq y exprese las ecuaciones de la mquina en
estas coordenadas. Analice el rgimen permanente de los tres convertidores en este
sistema de coordenadas.
d) El par elctrico desarrollado por cada convertidor.
e) Transforme a coordenadas primitivas todas las corrientes transformadas obtenidas
previamente.

4. Una mquina de rotor y estator cilndrico tiene dos bobinas ortogonales en el estator y una
en el rotor. El dimetro del rotor es de 15 cm, la longitud axial de la mquina es de 20 cm
y el entrehierro es de 1,5 mm. Las bobinas del estator tienen 200 vueltas y se alimentan
con tensiones sinusoidales de 110V efectivos, 60 Hz, desfasadas una de otra 2 . El material
135

Captulo IV Mquinas elctricas rotativas

Figura 4.24 Esquemas de los convertidores del ejemplo N. 3


ferromagntico del convertidor tiene una permeanza relativa de 1.000. La bobina del rotor
tiene 1.000 vueltas y por ella circula una corriente de 0,5 A. El mximo acoplamiento entre
las bobinas del rotor y del estator es de 90 % y la dispersin en la bobina rotrica es el
doble que en cada una de las bobinas del estator. Conocidos todos estos datos:

a) Calcule todos los parmetros del modelo de la mquina y las ecuaciones completas
que determinan su comportamiento dinmico.
b) Convierta las ecuaciones del estator a coordenadas dq y calcule el par elctrico de
la mquina, cuando el rotor gira a velocidad sincrnica y se encuentra adelantado 6
con respecto al eje magntico de la fase a.
c) Calcule las corrientes del estator en rgimen permanente si las bobinas del estator se
encuentran en cortocircuito.

5. Determine la expresin general del campo magntico rotatorio para una mquina compuesta de m devanados desfasados espacialmente 2m , a los cuales se les inyecta un sistema
m-fsico de corrientes simtricas, balanceadas y de secuencia positiva.

136

Bibliografa

A DKINS, B., The General Theory of Electric Machines, Chapman and Hall, London, 1957.
A DKINS, B. & H ARLEY, R. G., The General Theory of Alternating Current Machines, Chapman and Hall, London, 1975.
E LSGOLTZ , L., Ecuaciones diferenciales y clculo variacional, Editorial MIR, Mosc, 1977.
G OLDSTEIN , H., Classical Mechanics, Adison-Wesley, Cambridge, Mass., 1953.
K ARNOPP, D. & ROSENBERG , R., System Dynamics: A Unified Approach, John Wiley & Sons,
New York, 1975.
K REIDER, D. L., K ULER , R. G. & O STBERG , D. R., Ecuaciones diferenciales, Fondo Educativo Interamericano, Mxico, 1973.
K RON , G., The Application of Tensors to the Analysis of Rotating Electrical Machinery,
General Electric Review, Schenectady, New York, 1942.
NARA , H. R., Mecnica vectorial para ingenieros: Dinmica, Editorial Limusa, vol. II, Mxico,
1977.
P IPES , L. A. & H ARVILL , L. R., Applied Mathematics for Engineers and Physicists, McGrawHill, Third Edition, New York, 1970.
S AY, M. G., Introduction to the Unified Theory of Electromagnetic Machines, Pitman Press,
London, 1971.
W HITE , D. C. & WOODSON , H. H., Electromechanical Energy Conversion, John Wiley &
Sons, New York, 1959.
W HITTAKER, E. T., Analytical Dynamics, Dover Publications, New York, 1944.

137

Parte II

Mquinas elctricas rotativas

139

CAPTULO V

Mquinas de conmutador

En el captulo se analiz la transformacin de coordenadas e e r r a coordenadas e e dr qr .


En las mquinas con conmutador mecnico, esta transformacin se realiza fsicamente, el colector convierte los ejes r y r del rotor en ejes dr y qr . La mquina de corriente continua es
un caso particular de convertidor electromecnico que utilizan conmutador, son ampliamente
utilizadas para el control de par y velocidad en los procesos industriales porque tienen una alta
velocidad de respuesta, al mantener siempre en ortogonalidad los campos estatricos y rotricos1 . El desarrollo vertiginoso de la electrnica de potencia ha ido reemplazando poco a poco el
uso de estas mquinas por convertidores electromecnicos que no utilizan el conmutador mecnico. Sin embargo, estos sistemas2 utilizan los mismos principios de funcionamiento y control
que fueron desarrollados para las mquinas de conmutador y por este motivo es muy importante
comprender cabalmente su funcionamiento.
V.1

P RINCIPIO

DE OPERACIN

Una mquina de conmutador est constituida bsicamente por un estator, un rotor y un colector
acoplado slidamente al rotor. El colector permite conectar galvnicamente los conductores del
circuito rotrico o armadura a la fuente de tensin continua, mediante un juego de carbones3
o escobillas4 solidarios con el estator de la mquina. En la figura 5.1 se presenta el diagrama
1
2
3

Observe la expresin 4.55 con el ngulo = 90 entre las fuerzas magnetomotrices del estator y rotor. En este
caso la expresin del par es mxima para unas corrientes y dimensiones de la mquina determinadas.
Mquinas sincrnicas de imn permanente, mquinas de reluctancia variable, motores de induccin, etc.
Esta denominacin se debe al hecho de estar fabricados con carbn. Este material, adems de ser conductor
elctrico, proporciona una superficie suave que lubrica el contacto con las delgas de cobre, evitando de esta
forma que se desgaste rpidamente.
Otro mtodo para obtener un contacto elctrico deslizante es mediante una escobilla fabricada con hilos de
cobre; este dispositivo se utiliza frecuentemente cuando es necesario hacer circular corrientes constantes por una

141

Captulo V Mquinas de conmutador

(a) Modelo elemental

(b) Diagrama esquemtico

Figura 5.1 Mquina elemental de colector

Figura 5.2 Alineamiento de fuerzas electromotrices en la mquina


esquemtico de la mquina de corriente continua y un modelo constructivo simple para fines
demostrativos.
El principio de operacin de las mquinas de corriente continua se fundamenta en la inyeccin
de corriente continua, tanto en el circuito rotrico como estatrico. Estas corrientes producen
las fuerzas magnetomotrices Fr en el rotor y Fe en el estator, que intentan alinearse. Cuando se
alcanza el alineamiento, cesa el par elctrico. Si en ese preciso instante se invierte el sentido de la
corriente inyectada en el circuito rotrico, la fuerza magnetomotriz del rotor cambia de sentido
180 y aparece un nuevo par de alineamiento. En la figura 5.2 se representa esta situacin.
Analizando los diagramas de la figura 5.2 se pueden indicar las siguientes observaciones:
1. Las fuerzas magnetomotrices en el semiplano positivo, producen par positivo en el sentido
de las agujas del reloj.
bobina rotrica. En el caso de las mquinas de colector es ms beneficioso el empleo de carbones, que producen
un desgaste menor de las delgas.

142

V.1 Principio de operacin

Figura 5.3 Conmutador y forma de la corriente del rotor en un perodo de revolucin


2. Las corrientes que circulan por el rotor deben producir la fuerza magnetomotriz en el
plano positivo, para que el par siempre resulte positivo.
Para invertir el sentido de la fuerza magnetomotriz del rotor se utiliza el conmutador. En la figura
5.3 se observa que la corriente tiene como perodo de repeticin una revolucin del rotor de la
mquina de corriente continua. Al girar el rotor, la escobilla (1), se conecta con la delga (4) y
la escobilla (2) se conecta con la delga (3). El procedimiento anterior permite la inversin del
sentido de circulacin de la corriente por el rotor mediante el dispositivo mecnico descrito. La
corriente interna en el circuito rotrico es alterna. La corriente inyectada por la fuente es continua. En la prctica, es necesario un conmutador por cada bobina del rotor, pero por simplicidad
en el anlisis se ha supuesto que la mquina posee una sola bobina.
Con la distribucin de la corriente de armadura que se representa en la figura 5.4, la fuerza
magnetomotriz producida en el rotor se encuentra en el semiplano positivo y se produce un par
positivo que intenta alinear esta fuerza magnetomotriz con la fuerza magnetomotriz producida
por el enrollado de campo de la mquina. En esta situacin, los conductores contribuyen al par
en la direccin positiva del movimiento, debido a que los conductores ubicados a la derecha
de la figura producen fuerza tangencial hacia abajo, mientras que los de la izquierda producen
fuerzas tangenciales hacia arriba.
En un alineamiento conductivo semejante al ilustrado en la figura 5.5, existe equilibrio de fuerzas
sobre el mismo brazo y el par resultante es nulo. Este anlisis elemental explica la convenien-

143

Captulo V Mquinas de conmutador

Figura 5.4 Alineamiento de las corrientes por los conductores del rotor para producir par positivo

Figura 5.5 Alineamiento de las corrientes de armadura que no produce par efectivo en el eje
cia de utilizar la distribucin de las corrientes de armadura presentada en la figura 5.4, con la
finalidad de obtener par elctrico significativo en la mquina de corriente continua.
En las mquinas de conmutador, el plano que contiene el eje mecnico y corta diametralmente
al rotor se denomina lnea neutra de la mquina. La lnea neutra divide los puntos del rotor en
los que entra el flujo de aquellos en los cuales el flujo sale.
Para lograr la inversin en el sentido de la corriente, es necesario un dispositivo conmutador
por cada bobina. Esta solucin es muy primitiva, el conmutador puede ser mejorado mediante
una distribucin conveniente de los conductores que permita obtener el resultado deseado. En
la figura 5.6 se representa un abatimiento lineal de la superficie del estator y de los conductores
del rotor. Es conveniente realizar una conexin de los conductores del rotor, de tal forma que
slo se necesite un par de escobillas y no una por cada espira. Esta situacin se puede obtener
conectando las bobinas en serie. La otra condicin que debe cumplirse es que al cambiar de

144

V.1 Principio de operacin

Figura 5.6 Abatimiento lineal de una mquina rotativa de corriente continua

145

Captulo V Mquinas de conmutador

Figura 5.7 Conexin de los conductores del rotor


posicin la espira, en ella debe cambiar el sentido de la corriente, pero no en las otras espiras.
En la figura 5.7 se muestra una forma posible de realizar las conexiones de los conductores del
circuito de armadura.
Los conductores conectados a los terminales (1) y (2) de la figura 5.7 se encuentran en una
situacin diferente al resto de los conductores del circuito rotrico porque son los extremos de
la bobina. Para resolver este inconveniente se conecta un segundo devanado similar al anterior
en las mismas ranuras del rotor y conectados en paralelo.
En la figura 5.8 se observa el abatimiento lineal de estas dos bobinas. Con esta distribucin de
los conductores del devanado de armadura, es suficiente inyectar corriente entre dos delgas separadas 180 elctricos para que la corriente circule en una direccin en una mitad de la periferia
del rotor y en sentido contrario en la otra. Disponiendo de esta forma las bobinas, toda la superficie del rotor puede ser aprovechada para la produccin de par. Si las escobillas se colocan
alineadas convenientemente, se obtendr siempre corriente en un sentido en el polo norte de la
mquina y en sentido contrario en el polo sur. Cuando un conductor atraviesa la lnea neutra, se
invierte el sentido de su corriente y por esta razn el par producido sobre l mantiene la misma
direccin.
En la prctica se utilizan dos esquemas bsicos para bobinar el circuito de armadura de las
mquinas de corriente continua, el devanado imbricado y el devanado ondulado. En la figura 5.9
se muestran dos ejemplos de estos bobinados. En el enrollado imbricado, la bobina se devana
regresando por ranuras adyacentes o muy cercanas los retornos. En el devanado ondulado el
conductor de retorno de bobina adelanta poco ms o menos un paso polar. El anlisis de los
diferentes tipos de devanados es muy complejo y excede los alcances de este texto, pero se

146

V.1 Principio de operacin

Figura 5.8 Armadura de la mquina


puede destacar que en los rotores ondulados se puede utilizar un par de carbones para conectar
todos los pares de polos de la armadura, mientras que los rotores imbricados requieren un par de
carbones por cada par de polos. En mquinas pequeas con mltiples pares de polos el empleo de
bobinas onduladas puede representar un ahorro importante en el proceso de fabricacin, porque
adems se utiliza menor cantidad de cobre en las cabezas de bobina.
En la figura 5.10 se muestran dos etapas del proceso de fabricacin de la armadura de una
mquina de corriente continua. En primer lugar la conexin de los mazos de conductores con
las delgas y en la siguiente se muestra el maquinado final de las delgas realizado en el torno.
En la figura 5.11 se representa la armadura de la mquina de corriente continua mediante capas
de corriente. La capa de corriente puede girar mediante la rotacin de las escobillas que alimentan a las bobinas. La frontera producida por la inversin de las corrientes en la armadura que
contiene a las escobillas de la mquina se conoce como separatriz de la armadura. Este sistema
permite construir fsicamente unos conductores que se mueven en un campo magntico, pero
que al mismo tiempo producen una fuerza electromotriz constante y a 90 del campo estatrico.
En la figura 5.12 se representa un abatimiento lineal de la mquina, los conductores se mueven
hacia la izquierda y el campo magntico originado por el estator de la mquina est fijo. La
fuerza electromotriz inducida en los conductores es:
E = vB

(5.1)

En esta ecuacin, E es la intensidad del campo elctrico sobre cada conductor, v es la velocidad
tangencial de los conductores y B es la densidad de campo magntico producida por el devanado

147

Captulo V Mquinas de conmutador

Figura 5.9 Bobinados de armadura imbricados y ondulados


estatrico. Como todos los conductores se mueven con la misma velocidad tangencial, la fuerza
electromotriz en cada espira es proporcional al campo. Entre las dos escobillas aparece una
fuerza electromotriz igual a la suma de las fuerzas electromotrices de todas las espiras que se
encuentran conectadas en serie entre las dos escobillas. En la figura 5.12 se observa que cada
espira contribuye con:
v = e + e = 2e
(5.2)
Para invertir el sentido de las fuerzas electromotrices manteniendo la direccin de la velocidad,
es necesario invertir el campo. Por esta razn la fuerza electromotriz en las bobinas cambia de
sentido cuando stas cruzan la lnea neutra. En la figura 5.13 se representa esquemticamente
esta situacin.
En la figura 5.13 se definen:
E1

es la fuerza electromotriz resultante en el polo norte

E2

es la fuerza electromotriz resultante en el polo sur

La densidad de campo en el polo norte es prcticamente igual a la del polo sur, por esta razn
las fuerzas electromotrices del rotor E1 y E2 son iguales en magnitud pero contrarias en sentido.
Cuando las fuerzas electromotrices E1 y E2 son diferentes, se produce una corriente circulatoria
en la armadura que puede ocasionar un calentamiento excesivo de la mquina.
Si las escobillas se alinean exactamente con la lnea neutra, la fuerza electromotriz inducida
sobre las bobinas del rotor es mxima. Cuando la lnea neutra y la separatriz no estn alineadas,
ocurre una situacin semejante a la que se muestra en la figura 5.14.
En este caso, la mquina se encuentra girando a la velocidad angular . El par producido en el
sentido del movimiento se denomina motriz. Si el par tiene sentido contrario a la referencia de
posicin o velocidad, se denomina generatriz. En las regiones (2) y (4) de la figura, la mquina
de corriente continua posee par motriz y por lo tanto estas regiones de la mquina trabajan como
motor inventando accionar la carga mecnica en el sentido de las agujas del reloj. En las regiones
(1) y (3) la fuerza es contraria al sentido del movimiento, por lo tanto en estas zonas la mquina

148

V.1 Principio de operacin

(a) Unin de los conductores a las delgas

(b) Armadura en etapa final de fabricacin

Figura 5.10 Proceso de fabricacin de una armadura de corriente continua

149

Captulo V Mquinas de conmutador

Figura 5.11 Separatriz de la armadura

Figura 5.12 Campo elctrico en la superficie de los conductores

150

V.1 Principio de operacin

Figura 5.13 Fuerzas electromotrices inducidas sobre las bobinas

Figura 5.14 Lnea neutra y separatriz desalineadas

151

Captulo V Mquinas de conmutador

Figura 5.15 Flujo magntico producido por las corrientes de la armadura


acta como un generador. Las regiones (2) y (4) son ms extensas que las zonas marcadas con
(1) y (3), el par promedio est dirigido en el sentido del movimiento y el comportamiento neto
de la mquina es como motor. Del anlisis anterior se explica que cuando la separatriz y la lnea
neutra no coinciden, el par resultante se reduce.
Durante la operacin de la mquina, las escobillas permanecen fijas en la separatriz y es conveniente que esta lnea coincida con la lnea neutra. Con esta disposicin, las corrientes que
circulan por los conductores del rotor situadas a un lado de la lnea neutra poseen todas la misma direccin e intensidad.
En la figura 5.15 se puede observar que las corrientes que circulan por el rotor producen una
densidad de campo magntico Br , fijo en el espacio y cuya amplitud se encuentra en cuadratura
con el campo magntico producido por el devanado del estator.
Esta situacin se asemeja a la transformacin de los ejes y del rotor, en ejes d y q. El efecto
fsico del conmutador consiste en referir las corrientes del rotor a ejes ficticios que rotan en sentido contrario con la misma velocidad del rotor. Los ejes transformados parecen estar detenidos
vistos desde el estator de la mquina. Fundamentndose en estas ideas, la mquina de conmutador puede ser analizada mediante una transformacin a coordenadas dq. El conmutador de
estas mquinas es un inversor mecnico de la corriente que circula por los conductores del rotor,
sincronizado con el eje de la mquina. Las conmutaciones suceden con una frecuencia igual a la
de rotacin5 . Si el rotor de la mquina est construido con una sola espira, la fuerza magnetomotriz resultante es perpendicular al plano de la espira. Para un conjunto de conductores como los
ilustrados en la figura 5.16, la fuerza magnetomotriz se encuentra en la direccin de la separatriz
de la mquina.
Para que el par elctrico sea mximo, la fuerza magnetomotriz del rotor debe ser perpendicular
a la fuerza magnetomotriz del estator. Por esta razn, las escobillas se colocan colineales con la
5

Esto es verdad estrictamente para las mquinas con un solo par de polos, en otro caso esta frecuencia ser el
producto de la frecuencia de rotacin por el nmero de pares de polos.

152

V.2 Ecuaciones de las mquinas de conmutador

Figura 5.16 Resultante de la fuerza magnetomotriz del rotor


lnea neutra para permitir que la fuerza magnetomotriz del rotor se encuentre en cuadratura con
la fuerza magnetomotriz del estator, tal como se demostr en la ecuacin 4.55.
V.2

E CUACIONES

DE LAS MQUINAS DE CONMUTADOR

En el captulo 4 se dedujeron las ecuaciones diferenciales que permiten analizar el comportamiento dinmico de las mquinas con conmutador. Estas ecuaciones son:

Re + Le p
0
Ler p
0
ve
ie
v

0
Re + Le p
0
Ler p
e =
ie

vd
Ler p

Ler
Rr + Lr p
Lr
idr
r
vqr
iqr
Ler p
Lr Rr + Lr p
Ler

Tm = Ler i id i iq + J +
e

Las diferentes conexiones de las mquinas de corriente continua convencionales se pueden analizar considerando la existencia de una bobina en el estator orientada en la direccin del eje
y una bobina en el rotor orientada en la direccin del eje d, accesible mediante un par de
escobillas, tal como se ilustra en la figura 5.17.
Con el modelo analtico planteado para la mquina de corriente continua, denominando G al
coeficiente de generacin, que representa la inductancia mutua entre el rotor y el estator, se
obtiene el siguiente sistema de ecuaciones diferenciales:

v
vd

Tm

Re + Le p
0
=
G
Rr + Lr p
= Gid i + J +



i
id

(5.3)

Las mquinas de corriente continua se clasifican normalmente segn la conexin del enrollado
de excitacin o campo. El devanado de excitacin produce un campo magntico ms o menos

153

Captulo V Mquinas de conmutador

Figura 5.17 Representacin bsica de la mquina convencional de corriente continua

uniforme en el cual gira el rotor. Generalmente el devanado de excitacin de las mquinas de


conmutador se encuentra ubicado en el estator6 . Si la corriente de excitacin se obtiene a partir
de la fuente de tensin que alimenta la armadura, la mquina se encuentra en conexin paralelo
o derivacin. Si el campo y la armadura se conectan mediante dos fuentes diferentes, la mquina
se encuentra en conexin independiente. Cuando la corriente de la armadura circula por el devanado de campo, la conexin se denomina serie. Si la mquina tiene dividido el campo en dos
partes, una conectada en serie con la armadura y otra en paralelo, la conexin se conoce como
compuesta. En la figura 5.18 se muestra un diagrama con todas estas conexiones.

V.3

C ARACTERSTICAS

DE OPERACIN DE LAS DIFERENTES CONEXIONES

Si a la armadura de la mquina se le aplica tensin constante de valor Va y al devanado de campo


una tensin constante de magnitud V f ,7 en rgimen permanente las corrientes Ia e I f tambin son
constantes y en el sistema de ecuaciones 5.3 desaparecen los trminos de transformacin:
Vf = R f I f

(5.4)

Va = Gm I f + Ra Ia

(5.5)

Tm = GIa I f + m

(5.6)

Despejando de la ecuacin 5.5 la corriente I f , de la ecuacin 5.6 la corriente Ia y reemplazndolas en la expresin 5.7, se obtiene la ecuacin de equilibrio mecnico de la mquina de corriente
6
7

El rotor tiene asociado el colector para permitir la sincronizacin de la inversin de sus corrientes con la posicin
de esta pieza.
El subndice f se utiliza habitualmente y proviene de los textos en idioma ingls por la inicial de la palabra field=
campo. Utilizar la letra c puede confundir debido a que se identifica con la carga.

154

V.3 Caractersticas de operacin de las diferentes conexiones

Figura 5.18 Conexiones de la mquina de conmutador

155

Captulo V Mquinas de conmutador

Figura 5.19 Par elctrico versus velocidad con excitacin independiente


continua en funcin de las fuentes forzantes:8



GmV f
Va
Rf
V f + m
Tm = G
Ra
Rf

(5.7)

En la figura 5.19 se representa en un grfico el par elctrico de la mquina en funcin de la


velocidad.
En el grfico de la figura 5.19, la velocidad s se define como la velocidad del rotor donde la
tensin aplicada es igual a la fuerza electromotriz inducida en la armadura de la mquina y se
denomina velocidad de sincronismo o velocidad sincrnica. La caracterstica del par elctrico
de la mquina de corriente continua en funcin de la velocidad angular mecnica es igual a
la caracterstica de la fuerza elctrica en funcin de la velocidad tangencial sobre un conductor elemental que se desplaza en la presencia de un campo magntico uniforme analizado en
el captulo 1. Esta semejanza en las caractersticas no es coincidencial, los conductores de la
armadura se encuentran en una disposicin geomtrica similar a la del conductor solitario.
La curva de par elctrico-velocidad puede variar con la tensin aplicada a la armadura o a la
excitacin. Al variar la tensin de armadura se obtiene una familia de caractersticas paralelas
tal como se muestra en la figura 157(a). Si se vara la tensin del campo, cambia la pendiente
de la caracterstica, tal como se puede observar en el grfico de la figura 5.20(b).
Si se conecta la mquina con el campo en derivacin, el sistema de ecuaciones 5.4 representa el
comportamiento de la mquina y la nica diferencia con la mquina de excitacin independiente
es que la tensin de armadura y la tensin del campo son idnticas:


GV 2
Gm
Tm =
1
+ m
(5.8)
Ra R f
Rf
8

Tensin de armadura Va y tensin del campo V f .

156

V.3 Caractersticas de operacin de las diferentes conexiones

Figura 5.20 Efecto de la variacin de las fuentes

Figura 5.21 Par elctrico versus velocidad de la mquina en derivacin

157

Captulo V Mquinas de conmutador

Figura 5.22 Modelo circuital de la mquina de corriente continua en derivacin

En la figura 5.21 se ha representado el par elctrico de la mquina de corriente continua con


conexin en derivacin del circuito de campo; es interesante destacar que en este caso la velocidad sincrnica s es independiente de la tensin, a diferencia de lo que se obtiene para la
mquina de excitacin independiente. La ecuacin de tensin para la armadura de la mquina
es:
Va = Rr ia + Gm I f
(5.9)
En la ecuacin 5.9, el trmino Gm I f es la fuerza electromotriz de generacin producida por el
campo. En la figura 5.22 se representa el modelo circuital equivalente de la mquina de corriente
continua en derivacin.
Si la fuerza electromotriz generada es mayor que la tensin aplicada, la mquina entrega potencia a la fuente y el par elctrico es negativo. En estas condiciones es necesario par mecnico de
accionamiento.
La velocidad sincrnica depende del coeficiente de generacin G y de la resistencia del campo R f . Esta velocidad corresponde a la condicin de vaco9 de la mquina. Para controlar la
velocidad de vaco se pueden intercalar resistencias en el campo.
Para que la mquina pueda generar es necesario que la fuerza electromotriz sea mayor que la
tensin aplicada. El generador en vaco debe satisfacer la siguiente ecuacin diferencial:
V = R f i f + L f pi f = Gm i f
9

(5.10)

Esta condicin se alcanza cuando sobre el rotor no existe carga mecnica ni de prdidas. En el laboratorio se
requiere de un motor que accione la mquina exactamente a la velocidad sincrnica para poder suplir las prdidas
y lograr la condicin de vaco, donde no circula corriente por la armadura.

158

V.3 Caractersticas de operacin de las diferentes conexiones

Figura 5.23 Punto estable de operacin del generador en derivacin autoexcitado sin carga

La ecuacin 5.10 representa los circuitos de campo y armadura. Despejando de esta ltima
expresin la derivada de la corriente en el campo pi f se obtiene:


R f Gm
if
(5.11)
pi f = +
Lf
Lf
La solucin de esta ecuacin diferencial es una exponencial creciente, siempre y cuando se
cumpla que:
R f + Gm
> 0 Gm > R f
(5.12)
Lf
Si no se cumple la condicin 5.12, la corriente del campo y la fuerza electromotriz de generacin
tienden a disminuir. Para que el proceso de autoexcitacin pueda llevarse a cabo, es necesario
que exista un pequeo flujo remanente rem . Si la corriente de campo i f en la ecuacin 5.11
crece, tiende al infinito, a menos que el circuito de campo se sature10 . En caso de saturacin
se obtiene un punto de equilibrio. La no linealidad entre el flujo y la corriente permite que el
generador en derivacin defina un punto estable de operacin. En la figura 5.23 se representa
esta situacin.
Si disminuye la velocidad de accionamiento del generador en derivacin, aparece un punto crtico donde ya no es posible generar debido a que el factor Gm es menor que la resistencia R f y
el sistema se desestabiliza. Cuando el generador entrega potencia elctrica, se cumple:
V = Ra ia + Gm i f = Ra ia + Gm

10

V
Rf

(5.13)

Esto sucede generalmente en todas las mquinas con ncleo ferromagntico, cuando los dominios magnticos
estn completamente alineados con el campo externo, la permeabilidad del medio tiende a la permeablidad
del vaco 0 = 4 107. Las mquinas superconductoras no requieren ncleo magntico para producir flujos
importantes y pueden ser lineales mientras se mantiene este fenmeno fsico.

159

Captulo V Mquinas de conmutador


Despejando la tensin de alimentacin V en la expresin anterior:
V=

Ra i a

1 GRm

= Rcarga ia

(5.14)

La nica solucin estable de la ecuacin 5.14 se obtiene cuando la corriente ia es nula. Considerando que existe un pequeo flujo de remanencia rem en el material ferromagntico, producida
por una corriente equivalente y constante irem :
V = Ra ia + Gm i f + Gm irem

(5.15)

Erem = Gm irem

(5.16)

como:
if =

V
Rf

(5.17)

Despejando de 5.15 y 5.17 la tensin V e igualndola a la cada en la resistencia de carga se


obtiene:
Ra R
Gm R
irem = Rcarga ia
(5.18)
V =
ia
Gm R f
Gm R
En la figura 5.24 se han representado los dos miembros de la ecuacin 5.18. En esta condicin
existe un punto de operacin estable, con corriente de armadura diferente de cero.
En la actualidad, los controladores electrnicos de potencia han reemplazado prcticamente al
generador de corriente continua. Esto es debido a las mejores prestaciones, menor peso, volumen
y costo de estos equipos. Las mquinas de corriente continua se utilizan todava como generadores durante el frenado regenerativo de los sistemas de traccin elctrica, con la finalidad de
recuperar parte de la energa cintica acumulada en las masas en movimiento.
Los motores de corriente continua se utilizan ampliamente para el control de velocidad o para
la traccin de vehculos elctricos y trenes laminadores 11 . Las caractersticas de par-velocidad
de estas mquinas permiten su utilizacin en un gran nmero de aplicaciones. Antiguamente se
utilizaban resistencias para limitar la corriente en la armadura durante el proceso de arranque.
Las mquinas se disean para permitir entre 1, 5 y 2 veces la corriente nominal por la armadura
durante el arranque. En la actualidad el arranque y accionamiento de los motores de corriente
continua se realiza mediante fuentes de corriente continua regulables en tensin, con lo cual las
prdidas en los restatos se eliminan. Esto es de gran importancia en sistemas con paradas y
arranques frecuentes, como ocurre en el caso de un sistema urbano de transporte pblico.
11

Sin embargo, existe una fuerte tendencia para su reemplazo por los motores de corriente continua sin escobillas,
DC Brushless. stos en realidad son mquinas sincrnicas de imn permanente en el rotor, que se alimentan por
el estator con un inversor electrnico sincronizado con la posicin del rotor mediante un decodificador ptico.
Algunas limitaciones a su uso se deben a la posibilidad de desmagnetizacin de los imanes permanentes
durante cortocircuitos en el estator y a los costos de inversin y mantenimiento especializado del controlador de
potencia asociado con este equipo. Por otra parte en plantas donde se utilizan materiales inflamables o corrosivos,
estas mquinas son una necesidad debido a que no producen arcos elctricos durante su operacin.

160

V.3 Caractersticas de operacin de las diferentes conexiones

Figura 5.24 Punto de operacin del generador autoexcitado con remanencia

Figura 5.25 Conexin serie de la mquina de corriente continua


La conexin serie del devanado de campo es una de las ms utilizadas en los sistemas de traccin
elctrica. En este caso, la tensin aplicada se reparte entre la armadura y el campo, y la corriente
de armadura tambin circula por el campo. En la figura 5.25 se muestra el esquema de esta
conexin.
Las ecuaciones dinmicas de la conexin serie son:


v = va + v f = Ra + R f i + La + L f pi + Gm i = (RT + Gm ) i + LT pi

En rgimen permanente se tiene:

(5.19)

Tm = Gi2 + J m + m

(5.20)

V = (RT + Gm ) I

(5.21)

Te = GI 2

(5.22)

161

Captulo V Mquinas de conmutador

Figura 5.26 Caracterstica par-velocidad de una mquina de conmutador serie

Sustituyendo la corriente I de la ecuacin 5.21, en la expresin 5.22 se obtiene:


Te =

GV 2
(RT + Gm )2

(5.23)

En la figura 5.26 se representa la caracterstica de par elctrico para una mquina de corriente
continua con excitacin serie.
La caracterstica par-velocidad tiene la forma de una hiprbola cuadrtica, como se deduce de
la ecuacin 5.23. Esta caracterstica permite variar ampliamente el par resistente manteniendo
la potencia mecnica prcticamente constante12 . El motor serie se utiliza frecuentemente en
traccin elctrica porque permite obtener un elevado par de arranque. Al igual que en el motor
derivacin, es necesario limitar la corriente de arranque.
La mquina de conmutador con excitacin compuesta posee caractersticas combinadas de las
mquinas derivacin y serie. La caracterstica de estas mquinas se parecen ms a uno u otro
tipo, dependiendo del grado de intensidad que proporcione el campo serie y el campo derivacin.
12

Muchas pequeas mquinas utilizadas en el hogar tales como taladros, licuadoras y aspiradoras, entre otras,
utilizan esta conexin. Aun cuando la mayora de ellas se accionan mediante corriente alterna, como la armadura
y el campo serie conducen la misma corriente el par es proporcional al cuadrado de la corriente. Las reactancias
de las bobinas tienen el efecto de limitar la circulacin de corrientes alternas sin producir prdidas. Una licuadora
tiene una alta carga cuando comienza a triturar los alimentos y su velocidad es baja en esta condicin; en la
medida que la carga se procesa, la velocidad aumenta y el par disminuye, tal como sucede en las mquinas de
excitacin en serie. Argumentos similares se pueden utilizar en las otras aplicaciones.

162

V.4 Control de velocidad

Figura 5.27 Diagrama de bloques de la mquina de corriente continua


V.4

C ONTROL

DE VELOCIDAD

Despus de analizar el comportamiento en rgimen permanente de las mquinas de corriente


continua, es posible estudiar el comportamiento transitorio mediante su funcin de transferencia.
La mquina de corriente continua satisface el sistema de ecuaciones diferenciales 5.4 en rgimen
transitorio. De la ecuacin de tensin para el eje se puede obtener la funcin de transferencia
operacional de la corriente i :
1
v
Rbeta v


i =
=
L
1 + p
p
R 1 + Rbeta

(5.24)

La ecuacin del eje d en 5.4 permite obtener la corriente id :


vd Gm i
=

id =
Rd 1 + RLdd p

1
Rd

vd Gm i
1 + d p

A partir de la ecuacin diferencial correspondiente al eje mecnico se obtiene:



1
Te + Tm (m )
Gi id + Tm (m )
m =
=
+Jp
1 + M p

(5.25)

(5.26)

En la figura 5.27 se han representado las funciones de transferencia 5.24, 5.25 y 5.26 en diagrama de bloques, con sus respectivas realimentaciones e interconexiones. Este diagrama contiene
multiplicadores, debido a las no linealidades implcitas entre las variables de estado del modelo.
Por esta razn no es posible reducir este diagrama a una funcin de transferencia. Asumiendo
que la tensin v es constante, la corriente i se estabiliza en un valor continuo despus de
varias constantes de tiempo. En estas condiciones se puede representar el modelo dinmico de
la mquina de corriente continua mediante un solo bloque. Con la corriente i constante, se
puede definir como constante k al producto de esta corriente por el coeficiente de generacin G

163

Captulo V Mquinas de conmutador

Figura 5.28 Diagrama de bloques de la mquina con corriente de campo constante

de la mquina. En la figura 5.28 se observa el diagrama de bloques de la mquina de corriente


continua excitada con una corriente constante en el campo.
Este ltimo diagrama de bloques se puede reducir a una funcin de transferencia cuando el par
mecnico es nulo o constante. Un par mecnico constante no altera la respuesta transitoria del

sistema sino los valores en rgimen permanente. Definiendo la funcin de transferencia T (p)
como el producto de las funciones de transferencia de la figura 5.28:

T (p) =

1
k
1

Rd 1 + d p 1 + M p

(5.27)

La funcin de transferencia entre la velocidad mecnica de la mquina y la tensin aplicada en


el circuito de armadura es:

k
m (p)
T (p)
=
=

vd (p)
1 + kT (p) Rd (1 + d p) (1 + M p) + k2

(5.28)

Transformando al dominio de Laplace la funcin de transferencia 5.28 se obtiene:


k
m (s)
=
2
Vd (s)
Rd d M s + Rd (d + M ) s + Rd + k2

(5.29)

Como todos los trminos del denominador de la funcin de transferencia 5.29 son positivos, los
polos del polinomio tienen parte real negativa. Por esta razn, la respuesta del sistema siempre
es estable. Para reducir los tiempos de respuesta se puede ajustar el valor de la constante k
variando la corriente de campo i . La constante de tiempo de la armadura de la mquina d es
generalmente mucho menor que la constante de tiempo del sistema mecnico M y puede ser
despreciada en la ecuacin 5.29:
m (s)
k
=
Vd (s)
Rd M s + Rd + k2

(5.30)

El polo de la funcin de transferencia 5.30 es:


Rd + k 2
s=
Rd M
164

(5.31)

V.5 Valores nominales y bases

Figura 5.29 Mquina de corriente continua sin friccin


Al aumentar el valor de la constante k,13 el valor del polo se hace ms negativo y la respuesta
de la mquina es ms rpida. Al aumentar la corriente de campo en una mquina de corriente
continua se incrementa considerablemente la velocidad de respuesta.
Otra aproximacin habitual cuando se analiza la dinmica de la mquina de corriente continua,
consiste en despreciar la friccin. En estas condiciones el coeficiente de friccin es cero. En la
figura 5.29 se ilustra el diagrama de bloques correspondiente al sistema sin prdidas mecnicas.
Repitiendo el anlisis realizado anteriormente se obtiene la funcin de transferencia:
k
m (s)


=
2
Vd (s)
Rd J Rkd J + s

(5.32)

Cuando se desprecia la friccin es evidente que para mover el polo del sistema a la izquierda es
necesario incrementar el valor de la constante k y por lo tanto la corriente de campo. Mientras
ms corriente de campo circula por la mquina, los procesos dinmicos o respuestas transitorias
son ms rpidos.
La variable de control en este sistema es la tensin de armadura vd , debido a que la constante de
tiempo de este circuito d es mucho menor que la constante de tiempo mecnica M , fuertemente
dependiente de la inercia J. Para que la respuesta de una mquina sea rpida es necesario que la
inercia sea pequea. Las mquinas de corriente continua son muy rpidas y se utilizan ampliamente para el control par-velocidad en los procesos industriales y en los sistemas de traccin
elctrica.

V.5

VALORES

NOMINALES Y BASES

En las mquinas elctricas es frecuente emplear como potencia base la potencia de salida o
potencia til en el proceso de conversin. Para un motor de corriente continua la potencia de
salida se encuentra disponible en el eje mecnico. En los generadores de corriente continua la
potencia de salida est disponible en los bornes de la armadura. En los datos de placa de una
mquina se especifican las tensiones, corrientes y potencias nominales. El rendimiento de la
mquina en el punto nominal se puede calcular a partir de estos valores:
Pne je = In Vn nmotor
13

(5.33)

Es conveniente recordar que incrementar la constante k es igual a incrementar la corriente de campo i .

165

Captulo V Mquinas de conmutador


Parmetro
Ra
R I2
I p
a
f

Valores tpicos
0,02 0,04 ZB
0,02 PB
0,02 IB
0,005 0,2 s
0,25 0,2 s

Tabla 5.1 Valores tpicos de los parmetros para las mquinas de corriente continua

Pnelectrica = In Vn = Pne je ngenerador

(5.34)

La impedancia base de la mquina de corriente continua es:


ZB =

VB Vn
=
IB
In

(5.35)

En el cuadro 5.1 se han indicado valores tpicos asociados con las mquinas de corriente continua, en funcin de los valores base de la impedancia ZB , potencia PB = Pne je y corriente de
armadura IB = Ina .
V.6

R EACCIN

DE ARMADURA 14

Hasta el momento se ha supuesto que no existe interaccin entre los campos producidos por
los devanados de campo y las corrientes que circulan por la armadura. El flujo que produce el
campo est orientado segn el eje de la mquina y el flujo que se produce en la armadura
est orientado segn el eje d. El campo total en el entrehierro de la mquina se intensifica en
un extremo del polo y se debilita en el otro. En la figura 5.30 se observa un diagrama de esta
situacin. En las zonas polares donde se refuerza el flujo, el material magntico se satura, incrementndose la cada del potencial magntico en el entrehierro produciendo un debilitamiento
del campo resultante en la mquina. Por otra parte, el eje neutro de la mquina se desplaza un
cierto ngulo de la vertical, y si este desplazamiento no es compensado con un desplazamiento
semejante de la separatriz de la mquina, parte de la armadura actuar como un motor y otra parte como generador15 . El rendimiento y las caractersticas nominales de la mquina se reducen
notablemente en estas condiciones.
La reaccin de armadura tambin ocasiona la aparicin de tensiones ms elevadas entre las
delgas debido a que el campo en el entrehierro no se encuentra uniformemente repartido. Como
en las cabezas del polo se presentan densidades de campo mucho mayores, los conductores que
atraviesan esa zona generan fuerzas electromotrices de mayor intensidad, por consecuencia se
incrementan los arcos elctricos durante las conmutaciones del colector. Estos arcos incrementan
las prdidas de la mquina y deterioran prematuramente esta pieza.
Para reducir el efecto negativo ocasionado por la reaccin de armadura se puede colocar en el
eje del estator un devanado adicional de compensacin. Por este devanado se hace circular la
14
15

En algunos textos se denomina tambin reaccin del inducido.


Ver la figura 5.14 para mayor detalle.

166

V.6 Reaccin de armadura

Figura 5.30 Reaccin de armadura en las mquinas de corriente continua


corriente de armadura. Es muy importante que la polaridad de este devanado produzca una fuerza
magnetomotriz igual pero de sentido contrario al de la armadura para neutralizar su efecto. En
la figura 5.31 se muestra la disposicin fsica de la nueva bobina estatrica de compensacin y
su modelacin como mquina generalizada.
Para analizar la mquina de corriente continua, incluyendo el devanado compensador de la reaccin de armadura, se tiene:

Rc + Lc p
0
Lcd p
v
i
v =
i
0
R + L p
0
(5.36)

vd
id
Lcd p
G
Rd + Ld p
El sistema de ecuaciones diferenciales 5.36 se encuentra sujeto, de acuerdo con la figura 5.31, a
las siguientes condiciones de contorno:
i = id
v = v + vd

(5.37)

Aplicando las condiciones de contorno 5.37 al sistema de ecuaciones diferenciales 5.36, se obtiene:
 



id
v
(Rc + Rd ) + (Lc + Ld 2Lcd ) p
G
=
(5.38)
v
i
0
R + L p
Si el nmero de vueltas de la bobina compensadora se iguala al nmero de vueltas de la armadura:
Lc + Ld 2Lcd
(5.39)
=0
167

Captulo V Mquinas de conmutador

(a) Esquema del bobinado de compensacin de la reaccin de armadura

(b) Devanado de compensacin y polos auxiliares de conmutacin

Figura 5.31 Devanado de compensacin de la reaccin de armadura

168

V.7 Saturacin de la mquina de corriente continua

Figura 5.32 Linealizacin por tramos de la curva de magnetizacin


De esta forma, adems de compensar la reaccin de armadura de la mquina se puede mejorar
la respuesta dinmica del sistema.
V.7

S ATURACIN

DE LA MQUINA DE CORRIENTE CONTINUA

En el anlisis desarrollado para la mquina de corriente continua se ha supuesto que el material tiene un comportamiento lineal, exento de saturacin. En otras palabras, se supone que las
inductancias, resistencias y coeficientes de generacin son constantes en el dominio de las variables de inters. En las mquinas reales, esta hiptesis no puede ser mantenida y es necesario
estudiar el efecto de la saturacin.
Cuando se aumenta la corriente de campo en una mquina de corriente continua, inmediatamente
se incrementa el flujo en el entrehierro, pero esta variacin no es lineal. Esto se debe a que
cuanto mayor es la intensidad de campo magntico y ms alineados se encuentran los dominios
magnticos en el hierro, es necesaria mucha ms energa para lograr otra pequea alineacin
que incremente el campo total. La principal consecuencia que tiene la saturacin en la mquina
de corriente continua es que la fuerza electromotriz de armadura ed , no depende linealmente
de la corriente de campo i . La solucin de este problema se puede obtener linealizando la
caracterstica de vaco de la mquina de corriente continua, tal como se observa en la figura
5.32.
La caracterstica de magnetizacin se linealiza mediante asntotas o rectas tangentes a esta curva. Cuando se aproxima la caracterstica mediante dos rectas se obtiene:

G1 m i , si i < i 0
(5.40)
ed =
ed0 + G2 m i , si i i 0
En la ecuacin 5.40, G1 es el coeficiente de generacin no saturado, G2 es el coeficiente de
generacin saturado y ed0 representa la fuerza electromotriz de remanencia que existira en la
armadura de la mquina, si al reducir a cero la corriente de campo la mquina permaneciese
saturada.

169

Captulo V Mquinas de conmutador

Figura 5.33 Corrientes en la armadura antes y despus de una conmutacin


Es suficiente conocer la caracterstica de fuerza electromotriz inducida contra la corriente de
excitacin a una sola velocidad, porque a cualquier otra velocidad existe una relacin siempre
lineal entre la velocidad mecnica y la fuerza electromotriz inducida en los conductores de la
armadura.

V.8

LA

CONMUTACIN

Al analizar la accin del conmutador se determin que la corriente que circula por los conductores de la armadura invierte su sentido de circulacin justo al pasar frente a los carbones. Antes
del proceso de conmutacin se presenta la situacin que se muestra en la figura 5.33.
En la figura 5.33 se observa que en la bobina que converge a la delga (1), la corriente se dirige
hacia esa delga, mientras que en la (2), la corriente se aleja de la delga, ya que est conectada a
una escobilla o carbn en el cual se ha inyectado la corriente I. Las escobillas en la realidad se
encuentran generalmente fijas con respecto al estator o campo de la mquina, pero para explicar
el proceso se puede suponer que el carbn se mueve a una velocidad v y que la armadura se
encuentra fija. En la figura 5.33, en el diagrama de la derecha, se muestra la situacin que se
obtiene cuando la escobilla toca a la siguiente delga del colector. Cuando la escobilla pasa de la
posicin (2) a la (3) en la figura, todas las corrientes a la derecha e izquierda de esa delga no se
alteran, slo en la espira gruesa ocurre inversin de la corriente antes y despus del paso de la
escobilla. De este razonamiento se deduce que en la espira marcada en la figura ocurre todo el
proceso de conmutacin.
La dificultad para la realizacin del proceso de conmutacin se debe a que previamente, la
corriente en la espira tiene una magnitud de + 2I y al finalizar el proceso la corriente es 2I . En
la figura 5.34 se muestra un grfico de la corriente en la espira en funcin del tiempo.
El proceso de cambio de la corriente desde su valor + 2I a 2I depende de la fuerza electromotriz
inducida durante la conmutacin. Este proceso se lleva a cabo durante el tiempo de conmutacin
tc . El tiempo de conmutacin tc se calcula a partir de la velocidad de la mquina n, medida

170

V.8 La conmutacin

Figura 5.34 Conmutacin de la corriente en una espira de la armadura


en revoluciones por minuto, y del nmero de delgas ND del colector:
tc =

60
n ND

(5.41)

Al invertir la corriente en la espira desde + 2I a 2I durante el tiempo tc , se origina una fuerza


electromotriz e en la espira, que intenta oponerse al cambio de la corriente. La fuerza electromotriz en la espira se calcula como:
e = Le

di
I
Le I ND n
Le
=
dt
tc
60

(5.42)

En el momento de la conmutacin, la espira se encuentra muy cercana a la lnea neutra, la


inductancia propia de la espira Le es:
Le = Ne2 Pe = Ne2

0 Ae
2

(5.43)

En la ecuacin 5.43:
Pe

es la permeanza de la espira

Ne

es el nmero de vueltas de la espira

Ae

es el rea de la espira

es el espesor del entrehierro

Si no aparece la fuerza electromotriz descrita en la ecuacin 5.42, el reparto de corrientes entre


las dos delgas que estn siendo tocadas por el carbn depende de la resistencia de contacto. La
resistencia de contacto depende del maquinado de los materiales y de la presin que se ejerce en
el contacto. La corriente que circula por cada delga es inversamente proporcional a la resistencia

171

Captulo V Mquinas de conmutador

Figura 5.35 Produccin del arco elctrico al conmutar una delga por la siguiente
de contacto y por lo tanto directamente proporcional al rea de contacto entre el carbn y la
delga. Por esta razn, a medida que la escobilla se desplaza sobre la delga, la resistencia de
contacto vara aproximadamente de forma lineal y si no existe fuerza electromotriz en la bobina,
la conmutacin se produce de forma ideal, tal como se observa en la figura 5.34. Durante el
proceso de conmutacin, la fuerza electromotriz intenta mantener en circulacin la corriente de
la espira en la misma direccin, esto trae como consecuencia que el proceso de conmutacin real
sea ms lento y la parte de la escobilla que va entrando en la nueva delga tenga una corriente
menor a la que le corresponde a su rea de contacto.
La punta de la escobilla que est abandonando la delga, tiene una densidad de corriente muy
elevada, que ocasiona prdidas Joule significativas y altas temperaturas que pueden deteriorar
las delgas y las escobillas. Cuando la escobilla toca slo la nueva delga, la fuerza electromotriz inducida en la bobina anterior intenta mantener circulando la corriente y por esta razn se
produce el arco elctrico. En la figura 5.35 se muestra un diagrama de esta situacin.
di
En el momento del ltimo contacto entre el carbn y la delga vieja, el dt
aumenta considerablemente, incrementando en forma sustancial la fuerza electromotriz de conmutacin y produciendo el cebado del arco elctrico. Como la temperatura de estabilizacin de la escobilla es elevada,
se facilita la ionizacin del aire y la produccin del arco elctrico. La energa en forma de calor
en el arco es capaz de fundir metales. Esta fusin no ocurre en la operacin normal debido a
que el colector est rotando y el arco sobre cada delga dura tan slo fracciones de milisegundo.
Si se incrementa la corriente de conmutacin, el rea de ionizacin puede ser tan extensa que
se originen arcos entre delga y delga. Si esta situacin se propaga a un nmero importante de
las delgas, se origina el fenmeno conocido como arco de fuego y todo el colector queda en
cortocircuito.

Para contrarrestar el fenmeno de la conmutacin con arco, durante el proceso de diseo de la


mquina se intenta incrementar en lo posible el nmero de delgas para que varias de ellas puedan
ser contactadas por una escobilla simultneamente, obteniendo as una reduccin del problema
de la conmutacin.
Una solucin efectiva para el problema de la conmutacin consiste en equilibrar la fuerza electromotriz que intenta mantener circulando la corriente en la espira, con una fuerza electromotriz
generada localmente sobre la espira que est conmutando. Esta espira se encuentra en una zona
cercana a la lnea neutra, el flujo que la atraviesa en ese momento es mximo y su derivada
es prcticamente nula. En estas condiciones no es posible equilibrar la fuerza electromotriz de
conmutacin.

172

V.9 Prdidas en las mquinas de corriente continua

Figura 5.36 Mquina con polos auxiliares de conmutacin y su representacin


Si se colocan polos auxiliares, semejantes a los presentados en la parte (b) de la figura 5.32 16 para producir un campo magntico slo sobre los conductores que estn conmutando la corriente,
se puede generar una fuerza electromotriz contraria a la de conmutacin y neutralizar la formacin del arco elctrico. De la ecuacin 5.42 se tiene que la fuerza electromotriz de conmutacin
depende de la corriente de armadura y de la velocidad; la fuerza electromotriz inducida en los
conductores que estn conmutando debida a los polos auxiliares depende de la velocidad tangencial de los conductores y del campo magntico auxiliar, por tanto para que las dos fuerzas
electromotrices actuantes se neutralicen, es necesario que la corriente de armadura excite los
campos auxiliares de la mquina.
De esta forma es posible disear la mquina para que en cualquier condicin de carga la conmutacin se realice de forma ideal. En la figura 5.36 se muestra un diagrama de la situacin fsica
de los polos auxiliares y el modelo en coordenadas dq de la mquina.
Si se compara el modelo en coordenadas dq de la figura 5.36 con el modelo de la figura 5.31
para la compensacin de la reaccin de armadura, se observa que el anlisis es idntico. La nica
diferencia consiste en que el trmino Lc + Ld 2Ld no puede ser despreciado, ya que los polos
auxiliares de conmutacin producen slo un campo local y no pueden por tanto compensar el
flujo total de la armadura, como es el objeto de las bobinas de compensacin de reaccin de
armadura.
V.9

P RDIDAS

EN LAS MQUINAS DE CORRIENTE CONTINUA

El rendimiento de una mquina elctrica se define como:

Pent Ppr
Ppr
Psal
Psal
=
=
= 1
Pent
Pent
Psal + Ppr
Pent

(5.44)

En la ecuacin 5.44 se observa que determinando las prdidas en una mquina se puede obtener
su rendimiento17 . Las prdidas de una mquina de corriente continua se pueden dividir en:
16
17

Los polos auxiliares se encuentran ubicados en el espacio existente entre los polos principales del estator.
Este mtodo incluso es preferible a la medida directa del rendimiento (Psal. /Pent ), debido a que generalmente
las mquinas elctricas convencionales tienen un rendimiento elevado, las prdidas son una fraccin reducida de

173

Captulo V Mquinas de conmutador


1. Prdidas debidas al flujo principal

a) Prdidas en el hierro del rotor


b) Prdidas en la cara del polo
c) Prdidas en el cobre del polo

2. Prdidas en carga

a) Prdidas por efecto Joule


b) Prdidas por efecto pelicular

3. Prdidas por rozamiento y resistencia del aire

a) Prdidas por friccin en los rodamientos


b) Prdidas por friccin en las escobillas
c) Prdidas por ventilacin

Las prdidas ocasionadas por flujo principal dependen de la intensidad del campo magntico de
la mquina. En primer lugar existen prdidas en el hierro del rotor, ya ste que gira con respecto
al campo magntico producido por la bobina . El material magntico se magnetiza y desmagnetiza durante cada vuelta del rotor. En estas condiciones se producen prdidas de histresis
que dependen del nmero de revoluciones por minuto y del volumen de hierro involucrado. El
hierro del rotor se encuentra laminado para reducir las corrientes de Foucault, pero an as se
producen prdidas que dependen de la densidad de campo, del nmero de ciclos magnticos por
segundo, del espesor de las chapas del rotor, de la calidad del hierro utilizado y de su volumen.
Para evaluar las prdidas del histresis algunos autores proponen la ecuacin:
Phierro = a f B + b f B2

(5.45)

En la ecuacin 5.45, los coeficientes a y b son constantes, f es la frecuencia en Hz y B es la


densidad de campo en W b/m2 . En la mayora de los casos prcticos, el primer trmino de la
ecuacin 5.45 es despreciable y se puede utilizar la expresin:
Phierro = b f B2

(5.46)

la potencia de entrada y por esta razn los errores en la determinacin de las prdidas tienen un efecto menor
sobre el clculo del rendimiento que cuando los errores de medicin afectan a las potencias que estn siendo
transformadas.

174

V.9 Prdidas en las mquinas de corriente continua


Las prdidas de Foucault se pueden estimar mediante la expresin:
PFoucault = c f 2 B2

(5.47)

Para evaluar las prdidas totales en el hierro a partir de las ecuaciones 5.46 y 5.47 se obtiene:
PT hierro = PHist + PFoucault = b f B2 + c f 2 B2

(5.48)

En la prctica la magnitud de la densidad de campo magntico B es difcil de medir, pero la


fuerza electromotriz que se induce en el rotor en la condicin de vaco es proporcional al campo
y la ecuacin 5.48 se puede escribir como:
PT hierro = k1

V02
+ k2V02
f

(5.49)

Las ranuras del rotor producen variacin de la reluctancia y por lo tanto variacin del flujo. Esta
ondulacin induce corrientes de Foucault en la superficie o cara del polo con una frecuencia de
valor:
n
(5.50)
fd = Q
60
En la ecuacin 5.52, Q es el nmero de ranuras del polo y n es la velocidad de la mquina en
revoluciones por minuto. Para producir el flujo principal es necesario hacer circular corriente
por una bobina fsica con resistencia y por esta razn se producen prdidas por efecto Joule en
el cobre de la bobina de campo. Estas prdidas se calculan como:
Pcobre exc = R I2

(5.51)

Cuando la mquina se encuentra en carga, absorbe o entrega corriente de la fuente. Las corrientes
que circulan por las resistencias de las bobinas de la armadura, por los polos auxiliares, los
devanados de compensacin y por los devanados de excitacin serie, producen prdidas por
efecto Joule. Todas estas resistencias se pueden agrupar en la resistencia de armadura Ra y sus
prdidas se evalan as:
Pcobre arm = Ra Id2
(5.52)
Como la resistencia de las escobillas depende de la corriente, se asume que cada escobilla ocasiona un voltio de cada de tensin y de esta forma las prdidas producidas por la corriente de
armadura se pueden calcular como:
Parm = Ra Id2 + 2Vesc Id

(5.53)

Como la corriente que circula por la armadura es alterna de frecuencia:


frotor =

pn
120

(5.54)

se producen en el rotor las prdidas por efecto pelicular, donde p es el nmero de pares de
polos de la mquina. Estas prdidas debidas al incremento de la resistencia de los conductores
en presencia de campos magnticos variables en el tiempo se amortiguan considerablemente si
en lugar de construir la armadura con un conductor en una ranura profunda se utiliza un haz de

175

Captulo V Mquinas de conmutador


conductores delgados y aislados entre s. En todo caso, para evaluar estas prdidas es necesario
determinar la resistencia de la armadura a cada velocidad de operacin.
Las prdidas mecnicas de la mquina son ocasionadas por friccin o por consumo de los rodetes utilizados para la refrigeracin de la mquina. La friccin se localiza principalmente en
los rodamientos y en el contacto entre las escobillas y el colector. Para determinar las prdidas
debidas a los rodamientos se puede utilizar la ecuacin:
Prod = rod

F
vrod
D

(5.55)

En la ecuacin 5.55, rod es el coeficiente de roce, F es la fuerza actuante sobre el rodamiento,


D es el dimetro al centro de las bolas y vrod es la velocidad tangencial del mun. Para las
escobillas, las prdidas de friccin se determinan como:
Pesc = esc fe Se vc

(5.56)

En la ecuacin 5.56, esc es el coeficiente de roce entre el colector y la escobilla, fe es la fuerza


de presin sobre la escobilla, Se es el rea de la superficie de contacto entre la escobilla y el
colector y vc es la velocidad tangencial del colector. Finalmente las prdidas de ventilacin se
pueden evaluar mediante la expresin:
Pvent = kv Q v2

(5.57)

En la ecuacin 5.57, Q es el caudal de aire impulsado por el ventilador, v es la velocidad en la


periferia del ventilador y kv es un coeficiente constante que depende del tipo de rodete utilizado
y de sus caractersticas constructivas. Como el caudal Q es proporcional a la velocidad de la
bomba, las prdidas de ventilacin dependen del cubo de la velocidad.

V.10

C ONTROLADORES

ELECTRNICOS DE VELOCIDAD

El control de velocidad en las mquinas de corriente continua se realiza mediante la variacin


de la tensin de armadura. Esto permite lograr una gran velocidad de respuesta en el proceso
transitorio. Antes de la aparicin de los controladores electrnicos de potencia, la velocidad de
las mquinas de corriente continua se controlaba intercalando resistencia en el circuito de armadura. Este mecanismo de control produca prdidas considerables de energa durante el proceso
de regulacin. Utilizando dispositivos electrnicos de potencia es posible obtener fuentes de
tensin controlable y de alto rendimiento.
Las fuentes normalmente disponibles son trifsicas de tensin alterna en los sistemas industriales, o continuas en los sistemas de traccin elctrica tales como ferrocarriles, metros, tranvas
y trolebuses. Las fuentes de corriente alterna utilizan rectificadores controlados para obtener
corriente continua con tensin variable y las fuentes de corriente continua utilizan troceadores de tensin o choppers, que no son otra cosa que transformadores de corriente continua a
corriente continua. En la figura 5.37 se muestra un diagrama bsico de estos convertidores.
Con un puente rectificador semejante al de la figura 5.37 se pueden obtener tensiones positivas
y negativas retardando el ngulo de disparo de los tiristores. En este tipo de puentes no es

176

V.10 Controladores electrnicos de velocidad

Figura 5.37 Convertidores de potencia con salida en corriente continua

posible invertir el sentido de la corriente. Para obtener inversin en el sentido de las corrientes
es necesario utilizar un puente rectificador de cuatro cuadrantes, que es equivalente a utilizar dos
rectificadores conectados en oposicin de polaridad sobre la carga.
Aun cuando los puentes rectificadores son fuentes de tensin continua variable, resulta sencillo
convertirlos en fuentes de corriente mediante un lazo de realimentacin. En la figura 5.38 se ha
representado un puente rectificador controlado realimentado en corriente.
El compensador proporcional-integral-derivativo (PID) que se muestra en la figura 5.38, integra
el error que existe entre la corriente que circula por la carg a y la consigna de corriente deseada.
A medida que el error crece, la red de compensacin incrementa su tensin de salida y se ajusta
el ngulo de disparo del puente rectificador. Cuando el error existente entre la corriente real
y la consigna es prcticamente cero, el ngulo de disparo se mantiene constante. Mediante este
mecanismo, la fuente de tensin continua variable se transforma en una fuente de corriente
continua controlada mediante el valor de consigna.
Los sistemas de transporte metropolitanos, interurbanos y los ferrocarriles utilizan fuentes de
tensin continua para evitar las cadas de tensin en las reactancias de los alimentadores. Por
esta razn es necesario un transformador de corriente continua a corriente continua variable.
Este dispositivo se denomina troceador de voltaje o chopper. Un chopper posee la estructura
bsica que se muestra en el diagrama de la figura 5.39. Cuando el transistor de potencia Q de la
figura entra en conduccin, la tensin sobre la carga es igual a la tensin de la fuente:
v0 = V

(5.58)

177

Captulo V Mquinas de conmutador

Figura 5.38 Puente rectificador controlado realimentado en corriente

Figura 5.39 Diagramas bsicos del troceador de tensin para traccin y frenado regenerativo

Si el transistor Q conduce, circula corriente por el motor. Si el transistor deja de conducir, la


corriente que circulaba por el motor tiende a seguir circulando forzada por la inductancia de
alisamiento La que se encuentra en serie con la armadura del motor.
En ese momento entra a conducir el diodo de descarga libre D, puesto que es el nico camino
que tiene la corriente para continuar circulando. Las ecuaciones que rigen el comportamiento
del circuito son:
di
v0 = LT + RT i + E
(5.59)
dt
Cuando el tiristor conduce, a partir de las ecuaciones 5.58 y 5.59 se obtiene:
V = LT

178

di
di
+ RT i + E = LT + (RT + Gm ) i
dt
dt

(5.60)

V.10 Controladores electrnicos de velocidad


La solucin de la ecuacin diferencial 5.60 es:


t
t
V
i(t) =
1 e c + Imin e c
RT + Gm

(5.61)

Donde Imin es el valor de la corriente en el motor en el momento que comienza la conduccin


del transistor Q y la constante de tiempo c est definida por la expresin:

c =

LT
RT + Gm

(5.62)

En la figura 5.39 se muestra el estado cuasiestacionario descrito en la expresin 5.61. Por integracin se puede calcular la tensin promedio en la carga hv0 i:
Z t

Z
Z T
con
1 T
1
tcon
hv0 i =
v0 (t) dt =
V
(5.63)
0 dt =
V dt +
T 0
T 0
T
tcon
Introduciendo la ecuacin 5.59 en la expresin 5.63:
Z T 
 
Z
di
1
1 T
LT + RT i + Gm i dt =
v0 (t) dt =
hv0 i =
T 0
T 0
dt
Z i(T )

Z T
1
LT di +
(RT + Gm ) i dt = (RT + Gm ) hii
=
T i(0)
0

(5.64)

En la ecuacin 5.64, la integracin en el diferencial de corriente di es cero, ya que en el rgimen


cuasiestacionario la corriente en i(0) es igual a la corriente en i(T ). De la ecuacin 5.63 y 5.64
se determina que:
tcon
V
T V
=
(5.65)
hii =
RT + Gm RT + Gm
En la ecuacin 5.65, es la razn de conduccin que se calcula como el tiempo tcon durante el
cual el transistor Q conduce, dividido entre el perodo total T del chopper.
En la mquina de traccin existe una exigencia de par mecnico sobre el eje, esto determina la
corriente necesaria y como la tensin de la fuente y la velocidad de la mquina tambin estn
determinadas, se obtiene el ngulo de conduccin para la condicin de traccin especificada.
Un troceador de tensin permite tambin la posibilidad de frenado regenerativo, es decir, convertir la energa cintica almacenada en la masa del vagn, en energa elctrica para devolverla
a la red. Para obtener esta posibilidad es suficiente con invertir la conexin de armadura de la
mquina. Para realizar el cambio, se invierte la fuerza electromotriz E de la mquina. En la figura 5.39 tambin se muestra un diagrama del circuito utilizado para el frenado regenerativo de
los motores (b). En este circuito, cuando el transistor Q conduce, se produce un cortocircuito de
la fuerza electromotriz sobre la inductancia de alisamiento La . Durante este tiempo la corriente
aumenta de acuerdo con la ecuacin:
di
E = La
(5.66)
dt
La inductancia de alisamiento acumula energa en el campo magntico durante el tiempo en el
cual el transistor Q mantiene la conduccin. Cuando el transistor interrumpe la circulacin de
la corriente, la inductancia de alisamiento mantiene circulando la corriente de la armadura y el

179

Captulo V Mquinas de conmutador

Figura 5.40 Caracterstica par-velocidad motor serie con el campo debilitado


nico camino posible es a travs del diodo de frenado D hacia la fuente. En esta condicin la
red recibe energa y la corriente tiende a decrecer. Durante el proceso, la corriente de armadura
y la corriente de campo no se han alterado, solamente se ha invertido el sentido de la fuerza
electromotriz y por lo tanto el par sobre el eje de la mquina es ahora de frenado, con lo cual se
reduce la velocidad y el vehculo se detiene.
Si en lugar de una fuente de tensin se conecta una resistencia, la energa cintica almacenada
en la inercia del sistema se entrega como prdidas en esta resistencia y el proceso se denomina
frenado reosttico. El frenado reosttico se utiliza frecuentemente en los sistemas de traccin
elctrica, ya que muchas fuentes de corriente continua no son reversibles, no pueden absorber
potencia. Cuando un sistema no es capaz de absorber la potencia del frenado se dice que la red
elctrica no es receptiva. Aun cuando un sistema no sea receptivo, como en la red existen cargas
frenando y acelerando simultneamente, siempre existe una cierta receptividad que puede ser
aprovechada.
Mediante el troceador de tensin es posible acelerar o frenar una mquina, pero cuando el dispositivo se satura porque alcanza el ngulo mximo de conduccin18 , es posible continuar ajustando las caractersticas de traccin de una mquina de corriente continua con excitacin serie
mediante el debilitamiento de la corriente de campo. Esto se consigue conectando resistencias
en paralelo con el campo serie. Al disminuir la corriente de campo, aumenta la corriente de
armadura y se puede ajustar el par, ya que la corriente de armadura se incrementa en una proporcin mayor que la disminucin de la corriente del campo, debido a que la resistencia del circuito
de armadura es pequea. En la figura 5.40 se muestra la curva caracterstica par-velocidad de un
motor serie con el campo debilitado.
18

=1

180

V.11 Mquinas especiales de corriente continua

Figura 5.41 Metadina transformador


V.11

M QUINAS

ESPECIALES DE CORRIENTE CONTINUA

La posibilidad de colocar dos juegos de escobillas en una mquina de corriente continua, una
en el eje d y otra en el eje q, permite el estudio y la fabricacin de algunas mquinas especiales de conmutador. Estas mquinas se pueden utilizar como transductores o servomecanismo en
los procesos de control. Tambin se pueden construir fuentes de corriente o amplificadores de
gran ganancia. El estudio de estos convertidores se puede realizar mediante la transformacin
a coordenadas dq. La familia de mquinas con doble juego de escobillas se denominan las
metadinas o metadinamos. El prefijo griego meta- indica algo que va ms all y por lo tanto las
metadinas o metadinamos son algo ms que dinamos o generadores convencionales de corriente
continua. De la familia de las metadinas se analizan en esta seccin dos representantes, el primero ser la metadina transformador, que permite convertir una tensin constante en una corriente
constante, y el segundo ser la amplidina o amplificador rotativo; muy utilizada hasta hace unos
aos como excitatriz de las mquinas sincrnicas, debido a su elevada ganancia y gran velocidad
de respuesta. La metadina transformador es un mquina de campo cruzado (d, q) que no posee
devanados en el estator. En la figura 5.41 se muestra un diagrama de la mquina.
Las ecuaciones que rigen el comportamiento de esta metadina son:





Gdq
vd
Rd + Ld p
id
=
vq
iq
Gdq Rq + Lq p

Te = Gdq Gdq id iq = 0

(5.67)

Como la mquina es totalmente simtrica en el eje d y en el eje q, y se asume un acoplamiento


perfecto, es decir, se desprecia el flujo de dispersin:
Rd = Rq = R
Ld = Lq = Gdq = L

(5.68)

181

Captulo V Mquinas de conmutador


En rgimen permanente y de acuerdo con las ecuaciones 5.67 y 5.68 se obtiene:

 


Vd
R
Id
L
=
Vq
Iq
L R

(5.69)

Si se alimenta el eje d con una fuente de tensin V y se coloca una carga resistiva en los bornes
del eje q, se obtienen las siguientes condiciones de contorno:
Vd = V
Vq = Rc Iq
Sustituyendo las condiciones 5.70 en el sistema de ecuaciones 5.69 se obtiene:

 


Vd
R
Id
L
=
Vq
Iq
L R + Rc

(5.70)

(5.71)

Si la resistencia R de los devanados d y q es muy pequea, se pueden despreciar las cadas


resistivas en estos devanados; en esta condicin se obtiene:
V L Iq

(5.72)

De la ecuacin 5.72 se puede despejar la corriente Iq :


Iq

(5.73)

La ecuacin 5.73 indica que si se desprecian las cadas en las resistencias de los devanados de
armadura, la metadina transformador convierte la tensin V aplicada en el eje d en una corriente
constante en el eje q. La corriente del eje cuadratura depende slo de la velocidad de la mquina.
La amplidina posee un devanado de compensacin de la reaccin de armadura que se disea para
reducir el valor de las inductancias propias y mutuas. De esta forma se incrementa la velocidad
de respuesta en los procesos transitorios. La ganancia de corriente de armadura a tensin de
campo es muy grande en las amplidinas. La configuracin tpica de una amplidina se ilustra en
la figura 5.42. Las ecuaciones de tensin para una amplidina son:

L c p
0
Lq p
R + L p
v
i
vc

Rc + Lc p
0
Lqc p
L c

ic
=
(5.74)
vd G d
Gcd
Rd + Ld p
Gdq id
vq
iq
Lqc p
Gdq Rq + Lq p
Lq p
Para la amplidina se cumplen las siguientes condiciones de contorno:
vd = 0
v = vq vc
ic = iq = i
182

(5.75)

V.11 Mquinas especiales de corriente continua

Figura 5.42 Circuito de una amplidina


Sustituyendo las condiciones de contorno 5.75 en el sistema de ecuaciones 5.74 se obtiene:

L
p
R
+
L
p
L

q






i
Gdq (Gdq Gdc )
v
2

+
= L q L c p + Rc + Rq +
(5.76)
Rd +Ld p
v
i

G
G
dq

d
+ 2
+ Lc + Lq 2Lcq p
Rd +Ld p

La amplidina se disea con las siguientes consideraciones:


L c L q 0
Lc + Lq 2Lcq 0
Gdq Gdc 0

(5.77)

Con estas condiciones de diseo, el sistema 5.76 queda expresado de la siguiente forma:
#

 "

0
R + L p
i
v
= 2 Gdq G d
(5.78)
v
i
R +L p Rc + Rq
d

En rgimen permanente el operador p tiende a cero y se obtiene:


V = 2


Gdq G d
i + Rc + Rq i
Rd + Ld p
i =

V
R

(5.79)

(5.80)

De las ecuaciones 5.79 y 5.80 se puede observar que a velocidades altas la ganancia VV aumenta

considerablemente. Es importante destacar que la velocidad de respuesta a una perturbacin


es muy alta en una amplidina, debido a que las nicas inductancias involucradas son las de

183

Captulo V Mquinas de conmutador


campo y la de armadura del eje d. Las otras inductancias han sido prcticamente eliminadas
mediante el diseo apropiado de la bobina de compensacin de la reaccin de armadura. Por
estas razones, esta mquina de campo cruzado se utiliz frecuentemente como excitatriz de las
mquinas sincrnicas. En la actualidad ha perdido vigencia a causa de las excitatrices estticas
basadas en puentes rectificadores controlados, que han reducido los costos e incrementado la
velocidad de respuesta y el rendimiento.
V.12

S UMARIO

1. Las mquinas con colector son capaces de invertir mecnicamente el sentido de la corriente en los devanados rotricos en sincronismo con la posicin angular. De esta forma es
posible mantener un par promedio diferente de cero a medida que el rotor gira. Estas mquinas pueden ser analizadas utilizando coordenadas dq y esto corresponde al modelo
de la mquina generalizada desarrollado en el captulo anterior.
2. El alineamiento de la separatriz con la lnea neutra incrementa la capacidad del convertidor puesto que todos los conductores producen par en la misma direccin cuando se
obtiene esta situacin.
3. Las mquinas de conmutador pueden ser diseadas y conectadas en diferentes configuraciones. Las conexiones ms utilizadas son la independiente, paralelo, serie y compuesta.
La mquina con excitacin independiente requiere dos fuentes que controlan independientemente el campo y la armadura. La mquina paralelo o derivacin utiliza la misma fuente
para alimentar el campo y la armadura. En la mquina serie, la corriente de campo y armadura son iguales. Las mquinas compuestas combinan las propiedades serie-paralelo o
serie-independiente, para obtener caractersticas especficas de estas conexiones.
4. La caracterstica par-velocidad de las mquinas con excitacin independiente o paralelo,
son rectas y su corte con el eje de la velocidad define el punto de sincronismo. A esta
velocidad la tensin aplicada por la fuente y la fuerza electromotriz inducida en el rotor
se equilibra, anulando la corriente de armadura y el par elctrico.
5. La autoexcitacin de una mquina de conmutador es posible y el punto de equilibrio
depende de su caracterstica de saturacin. Aun cuando la electrnica de potencia ha relegado la aplicacin industrial de los generadores autoexcitados, en algunas aplicaciones de
traccin se utilizan estos principios para recuperar energa durante el proceso de frenado.
6. La ventaja ms importante de las mquinas de conmutador reside en la rapidez de respuesta, en especial cuando la corriente de campo se mantiene constante y en un valor
elevado.
7. La reaccin de armadura produce saturacin localizada en los polos de la mquina, lo
cual repercute en una reduccin neta del flujo medio. Esta reduccin es equivalente a la
operacin con una corriente de campo menor. La reaccin de armadura puede ser reducida
aumentando la reluctancia en el eje q del convertidor o incluyendo un devanado en el estator que anule el flujo producido por las bobinas del rotor. Esto adems permite incrementar
la velocidad de respuesta de la mquina.

184

V.13 Ejemplos resueltos


8. La conmutacin de las corrientes en las bobinas del rotor presenta un problema importante en las mquinas de conmutador. Durante el proceso de conmutacin se producen
fuerzas electromotrices en las bobinas que tienden a incrementar los arcos entre las delgas y los carbones. Para reducir este problema es posible utilizar carbones que toquen
simultneamente varias delgas y emplear bobinas auxiliares para compensar localmente
el fenmeno.
9. El anlisis de las prdidas de las mquinas es un mecanismo que incrementa la precisin en la determinacin de su eficiencia. Fundamentalmente deben considerarse en esta
evaluacin las prdidas debidas la flujo principal, las prdidas en carga y las prdidas
mecnicas.
10. Los rectificadores controlados y los choppers son dispositivos electrnicos de potencia
que permiten un control eficiente de la tensin y corriente de las mquinas de corriente
continua. Las caractersticas de operacin cuando se incluyen estos sistemas incrementan
en general las prestaciones operativas de la mquina.
11. Las metadinas han ido cediendo su espectro de aplicacin a los dispositivos electrnicos
de potencia. En el pasado sus aplicaciones como transformador de corriente continua y
como amplificador rotativo fueron ampliamente utilizadas.
V.13

E JEMPLOS

RESUELTOS

Ejemplo 1: Punto de equilibrio de un motor derivacin de corriente continua


Una mquina de corriente continua excitacin derivacin tiene acoplada al eje una bomba cuya
caracterstica par-velocidad se puede expresar como una funcin cuadrtica TB = km2 . A velocidad nominal tiene par nominal en el eje. Se conoce la velocidad sincrnica del motor s . De
acuerdo con estos datos determine:
1. Las velocidades de operacin del conjunto si se aplica como fuente de tensin Vn .
2. La corriente de armadura en las mismas condiciones.
3. La resistencia que es necesario incluir en el circuito del campo para obtener la mxima
velocidad con la tensin nominal aplicada.
Solucin:
1. Las velocidades de operacin del conjunto si se aplica como fuente de tensin Vn .
En las mquinas cuyo campo se encuentra conectado en derivacin, la velocidad sincrnica s es independiente de la tensin, tal como se observa en la expresin . De esta forma
se puede establecer la ecuacin de esta mquina una vez que se conoce la velocidad y el
punto nominal:
Te 0
m s
Tn 0
=
Te =
Tn
n s m s
n s
185

Captulo V Mquinas de conmutador


El par de arranque a tensin nominal Vn es:
Te (m = 0) =

s
s Tn
Tn = cVn2 c =
s n
s n Vn2

Si se aplica la tensin Vn a la mquina, la expresin del par elctrico en funcin de la


velocidad m es:
c 2Vn2 0
s m 2 2
T 0
=
Te =
c Vn
0 s
m s
s
Te ( , m ) =

s m
Tn 2
s n

Equilibrando las ecuaciones del par elctrico y el par de la bomba, se pueden obtener las
velocidades de operacin para cada valor del parmetro :

s m
Tn
Tn 2 = km2 = 2 m2
s n
n
m2
2 m
2 s
+

=0
n2 s n s n
s
!
 2 2 
1 n
4s2
4s
m ( ) =
1 1 2 + 2 2
2 s n
n n

(5.81)

En la figura 5.43 se han ilustrado mediante un grfico los puntos de equilibrio calculados
mediante la expresin 5.81:

2. La corriente de armadura en las mismas condiciones.


El balance de tensiones en el circuito de armadura de la mquina es:

Vn E f = Ra Ia Ia =

1 RGf m
Ra

Vn =

m ( )
s

Ra

Vn

(5.82)

La corriente de armadura nominal se obtiene a tensin nominal Vn 19 y velocidad angular


nominal n :
1 ns
Ian =
Vn
(5.83)
Ra
19

Esto equivale a indicar que = 1.

186

V.13 Ejemplos resueltos

Figura 5.43 Puntos de equilibrio entre la mquina de corriente continua con excitacin en derivacin y la bomba

Dividiendo las expresiones 5.82 entre 5.83, se obtiene el siguiente resultado:


Ia ( ) =

1 m(s )
1 ns

Ian

3. La resistencia que es necesario incluir en el circuito del campo para obtener la mxima
velocidad con la tensin nominal aplicada.
Recordando que el par elctrico se determina como:


Vn E f
GVn
G2 m 2
Te = GIa I f = G
If =
If
I = km2 = TB
Ra
Ra
Ra f

(5.84)

Para encontrar la corriente de campo I f que determina la mxima velocidad de equilibrio,


es posible derivar implcitamente la expresin 5.84 con respecto a la corriente de campo
I f , recordando que velocidad m depende de esta corriente:
GVn 2G2 m I f

d m
G2 d m
GVn 2G2 m
d m
Ra
= 2km

I f I 2f
2 2
Ra
Ra
Ra dI f
dI f
dI f
2km + G
Ra I f

(5.85)

Cuando la velocidad m alcanza el mximo, la expresin 5.85 tiene que ser cero; de esta
condicin se obtiene la relacin siguiente:
GVn 2G2 m I f = 0 mmax =

Vn
2GI f

(5.86)

187

Captulo V Mquinas de conmutador

Figura 5.44 Diagrama de conexiones de las mquinas del ejemplo N. 2

Reemplazando la expresin 5.86 en el balance de par 5.84, se obtiene la relacin que


permite determinar la corriente de campo que produce la mxima velocidad de equilibrio:


G2 mmax 2
V2
1 GVn I f
GVn
2
If
I f = kmmax 1
= k n2
Ra
Ra
2
Ra
4I f
1
I f (mmax ) =
G

r
3

kVn Ra
Vn
mmax =
3
2
4kVn Ra

Conocida la corriente de campo que produce la mxima velocidad, se obtiene la respuesta


mediante la siguiente expresin:
R f + Radicional =

Vn
Vn
Radicional =
Rf
I f (mmax )
I f (mmax )

Ejemplo 2: Punto de equilibrio de un grupo generador-motor de corriente continua


Dos motores de corriente continua, excitacin independiente, estn conectados como se muestra
en la figura 5.44 y poseen los siguientes datos de placa:
Pn
5 kW

Vna
230V

Ina
25 A

Vn f
100V

In f
nn
1,0A 2.000 rpm

Las prdidas en el hierro y mecnicas alcanzan los 430W . La tensin de remanencia a velocidad
nominal es de 6V . El par de la bomba depende del cuadrado de la velocidad y a 1.800 rpm
consume 4 kW . Las resistencias adicionales a las excitaciones de ambas mquinas estn convenientemente20 ajustadas para que circule la corriente nominal en la condicin de operacin. En
estas condiciones, determine:
1. Todos los parmetros de la mquina de corriente continua considerando prdidas y flujo
de remanencia.
20

Deben ser calculadas durante la solucin del ejemplo.

188

V.13 Ejemplos resueltos


2. Velocidad de la bomba en la condicin ilustrada en la figura 5.44.
3. Rendimiento del conjunto completo.
4. Valores de las resistencias adicionales para que la bomba gire a 2.000 rpm.

Solucin:

1. Todos los parmetros de la mquina de corriente continua considerando prdidas y flujo


de remanencia:
a) Clculo del flujo de remanencia:

r =

6V

Er
=
n

2.000
60 2

= 2,865 102 W b

b) Clculo del coeficiente de generacin G:



Pn + Pmec + Pf e 5.000 + 438
= 25,97 Nm
=
Tn = GI f n + r Ia =
n
209,4
G=

Tn r Ian
= 1,01 H
I f n Ian

c) Resistencia de armadura:

Va Gn I f + Erem
230 (1,01 209,4 1, 0 + 6)
Ra =
=
= 0,512
Ia
25
d) Resistencia del campo:
Rf =

Vn f
= 100
In f

e) Coeficiente de la bomba:
PBn = k 3 k =

PBn
=
3

4.000
3
1.800
2

60

= 5,97 104

2. Velocidad de la bomba en la condicin ilustrada en la figura:


I1 = I2 + 2 A
EM I2 = km3 + Pmec + Pf e

EM = GI f n + r m
EM = EG Ra I1 Ra I2

189

Captulo V Mquinas de conmutador

Figura 5.45 Circuito equivalente del sistema del ejemplo N. 1

Las cuatro ecuaciones anteriores permiten obtener la siguiente expresin de la velocidad:

m =

k 3 + Pmec + Pf e
437,2
EG 2Ra
4 2
2Ra m
2 = 192,35 5,95 10 m
GI f n + r
m
m GI f n + r

Utilizando el mtodo de Gauss-Seidel para resolver la ecuacin anterior se obtiene el


siguiente resultado:
rad
m = 172
n = 1.644 rpm
s
3. Rendimiento del conjunto completo:

EM = GI f n + r m = 174V
I2 =

km3 + Pmec + Pf e
= 20 A
EM
I1 = I2 + 2 = 22 A

PG = EG I1 + Pmec + Pf e = 4.736W
PB = km3 = 3.070W

T =

PB
= 0,648
PG

4. Valores de las resistencias adicionales para que la bomba gire a 2.000 rpm:
En este caso utilizaremos una aproximacin para realizar un clculo ms rpido pero
posteriormente se podra afinar el resultado. La aproximacin consiste en suponer que
la corriente del campo del motor se mantiene; con este supuesto se facilita el clculo de
EM y posteriormente se recalcula I f 2 . Posteriormente se podra iterar hasta alcanzar la
convergencia, pero en este caso los resultados cambian muy poco y es posible pasar por
alto este tanteo:

190

V.13 Ejemplos resueltos

PB =

km3

= 5,97 10



2.000 3
= 5,485 kW
2
60

PM = PB + Pmec + Pf e = 5,923 kW
PM
= 28,28 Nm
m

Em = GI f n + r m = 211,6V
TM =

I2

TM m
= 28 A
EM

Esta corriente indica que se exceden las condiciones nominales de operacin al trabajar
el sistema en este punto. Durante un perodo de tiempo es posible mantener este punto de
operacin, pero si se mantiene permanentemente, la mquina exceder su temperatura de
diseo y comenzar un deterioro acelerado de sus propiedades dielctricas.


1 EM
If2 =
r = 0,97 A
G m
EG = Ra I1 + Ra I2 + EM = 241V


1 EG
If1 =
r = 1,24 A
G G
Con estos resultados aproximados se podra seguir iterando para mejorar la precisin, pero
para fines prcticos stos son muy cercanos a los valores finales. Con las dos corrientes
de campo y con la tensin del punto medio entre las dos mquinas se determinan las
resistencias totales y de stas, el valor de las resistencias adicionales que requiere cada
campo:
V = EG Ra I1 = EM + Ra I2 = 226V
V
= 182 Rad1 = Rr1 R f = 82
Rr1 =
If1
Rr2 =

V
= 232 Rad2 = Rr2 R f = 132
If2

Ejemplo 3: Clculo de la fuerza electromotriz entre delgas


Determine la fuerza electromotriz inducida entre las delgas si se conocen los siguientes datos
para una espira:

Ne = 25

Ie = 5 A

Ae =102 m2

=3 102 m2

191

Captulo V Mquinas de conmutador


y la mquina posee 80 delgas y gira a 1.800 rpm.
Solucin:
De los datos geomtricos de la mquina se puede determinar mediante la expresin 5.43, la
inductancia de la espira que se encuentra en conmutacin:
Le = (25)2

4 107 102
= 1,3 mH
2 3 103

Si la mquina posee 80 delgas y gira a 1.800 rpm, la fuerza electromotriz inducida entre delgas
se calcula mediante la ecuacin 5.42 como:
1,3 103 10 80 1.800
e=
= 31,4V
60
ste es un valor cercano al lmite, debido a que con tensiones mayores el arco elctrico se puede
automantener.
V.14

E JERCICIOS

PROPUESTOS

1. Una mquina de corriente continua con excitacin serie se acopla mecnicamente a un


generador de corriente continua con excitacin en derivacin de igual potencia, tensin,
velocidad y corriente nominal:
Pn
5 kW

Vn
220V

nn
In
1.800 rpm 30 A

La mquina serie se alimenta con tensin nominal y al generador se le conecta una resistencia en paralelo con su armadura de 1,0 en pu. La tensin de remanencia del generador
es de 5 % a su velocidad nominal y la corriente de campo nominal es un 3 % de la corriente
nominal. Las prdidas mecnicas de las dos mquinas dependen del cuadrado de la velocidad y en el punto nominal representan la tercera parte de las prdidas totales. Determine:

a) Los parmetros de ambas mquinas.


b) La velocidad de operacin del conjunto.
c) La potencia entregada en la resistencia.
d) El rendimiento global.

2. Una mquina de corriente continua con excitacin independiente posee los siguientes datos nominales:
Vn
220V

192

Pn
10 kW

Vf n
110V

Ifn
nn
n
3 A 1.800 rpm 0,90

V.14 Ejercicios propuestos


Esta mquina se acopla mecnicamente a otra mquina idntica, pero en conexin derivacin con una resistencia apropiada en el campo para permitir la generacin de la potencia
elctrica nominal. La tensin de remanencia de estas mquinas es de 5V . Las prdidas
mecnicas en el punto nominal representan el 30 % de las prdidas totales y dependen del
cuadrado de la velocidad. Determine:

a) Los parmetros del circuito equivalente de las dos mquinas.


b) El valor de la resistencia de campo y de carga del generador para no exceder los
valores nominales de la mquina motriz ni del generador.
c) Los nuevos puntos de operacin del conjunto cuando se reduce un 10 % la tensin
del campo de la mquina motriz y cuando se disminuye un 10 % la resistencia de
campo del generador.
d) El rendimiento del conjunto motor-generador en funcin de la potencia de salida del
generador.

3. Dos mquinas similares de corriente continua con excitacin independiente poseen los
siguientes datos nominales:
Mquina
Vn
I
220V
II
220V

Pn
5 kW
5 kW

Vf n
110V
110V

Ifn
nn
1 A 1.800 rpm
1 A 1.750 rpm

n
0,85
0,85

Si las dos mquinas se conectan como motores en derivacin y se acoplan ambas mecnicamente a una bomba centrfuga que a 1.800 rpm consume 10 kW . Determine:

a) La velocidad de operacin del sistema formado por las dos mquinas y la bomba.
b) La corriente manejada por la armadura de cada una de las mquinas.
c) La potencia entregada por cada una de las mquinas en sus respectivos ejes mecnicos.

4. Una mquina de corriente continua excitacin compuesta de 220V , 5 kW , 1.750 rpm y


rendimiento en el punto nominal de 86 %, tiene un devanado serie que produce el 25 %
del flujo nominal cuando por l circula la corriente nominal y un campo derivacin que
produce el resto del flujo cuando se le aplica la tensin nominal entre sus bornes. Las
resistencias del campo serie y de la armadura son iguales. Las prdidas en el campo derivacin son del 2 % de la potencia til. La remanencia a velocidad nominal es del 3 %. Las
prdidas mecnicas en el punto nominal son del 5 % de la potencia til. Determine:

a) Los parmetros de la mquina.

193

Captulo V Mquinas de conmutador


b) El punto de operacin cuando se encuentra accionando una carga mecnica que aumenta su par linealmente con la velocidad y que a 1.750 rpm desarrolla 4 kW .
c) Rendimiento del sistema si se acciona la mquina a 1.800 rpm, con el devanado serie
desconectado y con una carga en la armadura de valor 1,0 pu.

5. Dos mquinas de corriente continua, una con excitacin serie y la otra con excitacin en
paralelo, se encuentran conectadas a la tensin nominal y sus ejes mecnicos estn acoplados. Los datos de placa de ambas mquinas son los siguientes:
Mquina
Vn
Serie
220V
Paralelo 220V

In
Pn
23 A 4 kW
21 A 5 kW

nn
In f
1.750 rpm 23 A
1.750 rpm 1,73 A

Las resistencias de campo y de armadura de la mquina serie son de 0,8 . La resistencia


de armadura de la mquina derivacin es de 0,95 . Las prdidas de ventilacin de ambas
mquinas dependen del cubo de la velocidad. En estas condiciones determine:
a) Los parmetros de ambas mquinas.
b) Las caractersticas par-velocidad de ambas mquinas.
c) La velocidad nominal y el par nominal del conjunto de las dos mquinas.
d) La velocidad si con la carga nominal del conjunto acoplada al eje se debilita el campo serie un 15 %.
6. Se tiene un motor de corriente continua de 10 HP,21220V, 1.000 rpm,excitacin en derivacin, con una resistencia de 100 , una resistencia del inducido de 0,4 y un rendimiento
del 85 %. Determine:
a) Los valores nominales de la corriente del inducido y el par elctrico en el eje.
b) La tensin que habra que aplicar al inducido para reducir la velocidad a 500 rpm
manteniendo la excitacin en condiciones nominales, si se supone que el par de
carga es proporcional al cuadrado de la velocidad.
c) La resistencia a colocar en serie con el inducido para reducir la velocidad a 500 rpm,
manteniendo la tensin de alimentacin en 220V y la excitacin como en las condiciones nominales, suponiendo que el par de carga es proporcional a la velocidad.
7. Se tiene un motor serie de corriente continua que suministra una potencia de 10CV .22 La
tensin de alimentacin es de 200V . Calcular:
21
22

1 HP = 746W .
1CV = 745W.

194

V.14 Ejercicios propuestos


a) Intensidad, si el rendimiento total es del 86 %.
b) Valor de la resistencia interna, si las prdidas por efecto Joule son del 7 %.
c) Fuerza electromotriz del inducido.
d) El par motor til en el eje a 1.000 rpm.
e) Conjunto de prdidas por rozamiento y en el hierro por histresis rotativa.

8. Un generador de corriente continua con excitacin derivacin suministra una intensidad


de 80 A a 200V . El rendimiento elctrico es de 95 %, siendo el reparto de prdidas por
efecto Joule a razn del 3 % en el inducido y del 2 % en el inductor. Calcular:

a) Potencia total generada.


b) Potencia perdida en el inducido.
c) Potencia perdida en el inductor.
d) Intensidad de excitacin.
e) Resistencia del inductor.
f ) Resistencia del inducido.
g) Fuerza electromotriz del inducido.

9. Se tiene un motor de corriente continua con excitacin en derivacin cuya potencia nominal es de 3 kW , con una tensin en bornes de 110V , siendo el rendimiento total del 76 %.
Calcular:

a) Intensidad total.
b) Intensidad de excitacin, sabiendo que la potencia para excitacin es el 5 % de la
consumida.
c) Resistencia del devanado de campo.
d) Intensidad de la armadura.
e) Resistencia de la armadura si sus prdidas por efecto Joule son el 5 % de la potencia
consumida.
f ) Fuerza electromotriz del inducido.
g) Par motor en la polea de transmisin a 1.200 rpm.
h) Valor del restato de armadura para que la intensidad de arranque no exceda el valor
nominal.

195

Captulo V Mquinas de conmutador


10. Un generador de corriente continua con excitacin compuesta suministra 120 A al circuito
exterior, con una tensin en bornes de 120V . Las prdidas por efecto Joule en la armadura,
campo derivacin y campo serie son respectivamente 2,5 %, 2,5 % y 1 %, de la potencia
cedida, respectivamente. Si las nicas prdidas que se consideran son las hmicas en los
distintos devanados, calcular:

a) Prdidas por efecto Joule.


b) Resistencia del devanado de campo serie.
c) Intensidad en el devanado de campo paralelo.
d) Resistencia del devanado de campo en derivacin.
e) Intensidad de la armadura.
f ) Resistencia de la armadura.
g) Fuerza electromotriz de la armadura.

11. Un motor de corriente continua con excitacin independiente mediante un imn permanente, tiene una resistencia elctrica en la armadura de 0,1. La tensin mxima que se
puede aplicar a la armadura es 220V y la corriente mxima que puede pasar por sus devanados es de 100 A.23 La constante de proporcionalidad de la fuerza electromotriz inducida
en el rotor es 0,15 r Vpm . Determinar:
a) La expresin del par elctrico del motor en funcin de la velocidad para diferentes
tensiones de alimentacin.
b) La representacin grfica del par elctrico para tensiones de 50, 100, 150 y 200V .

12. Un generador compuesto de 250 kW, 250V, 1.200 rpm suministra 1.000 A a 250V . La resistencia de la armadura, incluyendo las escobillas, es de 0,0045 . La resistencia del
devanado serie es 0,018 y la del arrollamiento paralelo es 48 . Las prdidas por rozamiento son 6.800W y las prdidas por cargas parsitas son el 1 % de la potencia til.
Calcular:

a) Prdidas totales.
b) Rendimiento.
13. Un generador derivacin tiene una armadura de 0,5 y el campo posee una resistencia
de 400 , conectado en serie con un restato Rh cuya resistencia es variable de 0 a 200 .
Cuando Rh se fija a 100 , el rotor gira a 1.500 rpm y la diferencia de potencial entre bornes es de 100V en circuito abierto. La induccin en el hierro de los campos es de 0,9 T .
23

Esta condicin es vlida en rgimen permanente.

196

V.14 Ejercicios propuestos


Calcular:
a) Fuerza electromotriz de la armadura en estas condiciones.
b) Diferencia de potencial en bornes si el generador suministra 10 A.
c) Velocidad de arrastre necesaria en la armadura para que la tensin vuelva a tener su
valor inicial de 100V .
14. Un motor serie que tiene una resistencia de 1 entre terminales mueve un ventilador para
el cual el par vara con el cuadrado de la velocidad. A 220V el conjunto gira a 300 rpm y
consume 25 A. Debe aumentarse la velocidad a 400 rpm aumentando la tensin. Hallar la
tensin y la corriente para los casos lmites siguientes:
a) Cuando el circuito magntico est saturado, es decir, para flujo constante.
b) Cuando el circuito magntico no est saturado, es decir, cuando el flujo sea directamente proporcional a la corriente.
15. Un motor tipo derivacin de 7, 5 kW, 460V tiene una entrada de 8, 5 kW cuando desarrolla
un par en el eje de 78,3 Nm a 900 rpm. Calcular el porcentaje de reduccin del campo
para aumentar la velocidad a 1.050 rpm con un par en el eje de 60,7 Nm. La resistencia del inducido es de1 , la resistencia del circuito de campo a 900 rpm es de 770 y
las prdidas mecnicas y en el hierro son constantes. Desprecie la reaccin de la armadura.
16. Un motor derivacin de corriente continua de 10 HP, 230V tiene una velocidad a plena
carga de 1.200 rpm. La resistencia de la armadura es de 0,3 y la del campo, 180 . El
rendimiento a plena carga es del 86 %. El motor obtiene la tensin nominal de un generador de corriente continua derivacin, de resistencia de armadura de 0,3 y resistencia
de campo 230 . Las prdidas en el hierro y mecnicas del generador son 500 W . Ambas
mquinas tienen el mismo nmero de polos y conductores y los devanados son ondulados.
Calcular:
a) Velocidad del generador, si ambas mquinas estn funcionando en la zona lineal de
la curva de magnetizacin.
b) Rendimiento del generador.
c) Rendimiento del conjunto generador-motor.
d) Velocidad del motor en vaco, si en estas condiciones su entrada total es de 600W .
e) Valor de la resistencia que hay que aadir a la armadura del motor para reducir su
velocidad a 1.000 rpm cuando entrega el par de plena carga con toda la corriente de
campo24 .
24

Las prdidas en el hierro y mecnicas son las de plena carga.

197

Captulo V Mquinas de conmutador

198

Bibliografa

A LONSO , A. M., Teora de las mquinas elctricas de corriente continua y motores de colector,
Departamento de Publicaciones de la ETSII de Madrid, 1979.
F ITZGERALD , A. E., K INGSLEY, C. J R . & K USKO, A., Electric Machinery: The Processes,
Devices, and Systems of Electromechanical Energy Conversion, McGraw-Hill, Third Edition,
New York, 1971.
KOSTENKO, M. P. & PIOTROVSKI, L. M., Mquinas Elctricas, vol. I, Editorial Mir, segunda
Edicin, Mosc, 1979.
L ANGSDORF, A. S., Theory of Alternating Current Machinery, Tata McGraw-Hill, Second Edition, New York, 1974.
L ANGSDORF, A. S., Principio de las mquinas de corriente continua. Tomo I, Ediciones del
Castillo, S.A., Madrid, 1967.
M C P HERSON , G. & L ARAMORE , R. D., An Introduction to Electrical Machines and Transformers, John Wiley & Sons, Singapur, 1990.
T HALER , G. & W ILCOX , M., Mquinas elctricas, Editorial Limusa, Mxico, 1979.

199

Captulo V Mquinas de conmutador

200

CAPTULO VI

La mquina de induccin

La mquina de induccin1 es el convertidor electromecnico ms utilizado en la industria. Su


invencin se le debe a Tesla, a finales del siglo XIX y nace como solucin al problema de utilizar la corriente alterna. Tesla haba sugerido la idea de las ventajas que posea la corriente
alterna2 sobre la corriente continua cuya dificultad de transmisin para la poca ya comenzaban
a ser evidente. La defensa de la corriente continua era realizada por Edison, famoso inventor
norteamericano que contaba con un inmenso prestigio 3 . Sin embargo, la visin comercial de
Westinghouse impuls las ideas de Tesla para la instalacin de la primera gran central hidroelctrica, que se realiz en las Cataratas del Nigara utilizando generadores de corriente alterna,
que usan transformadores para elevar la tensin, transmiten mediante lneas en alta tensin y la
reducen para alimentar a los motores de induccin que accionan la creciente carga industrial.
Desde ese crucial momento y hasta la actualidad, la mquina de induccin ha ido copando la
inmensa mayora de aplicaciones en la industria, en el comercio y en el hogar. En la figura 6.1 se
muestra un modelo de la mquina diseada por Tesla, cuyo original est expuesto en el museo
Smithsoniano de Washington, DC.
Las razones fundamentales que justifican la aplicacin masiva de la mquina de induccin hoy
en da, residen en su sencillez constructiva y en la robustez que ofrecen estos convertidores
durante la operacin en regmenes de alta solicitacin 4. Estos motores requieren un mantenimiento mnimo, pueden operar convenientemente en ambientes peligrosos y tienen una tasa de
1
2
3

Algunos autores se refieren a este convertidor electromecnico como mquina asincrnica por su capacidad de
accionamiento a velocidades no sincrnicas.
Cuyos niveles de tensin pueden ser variados mediante transformadores y por consiguiente es posible reducir
sustancialmente las prdidas durante la transmisin que se realiza a baja corriente y alta tensin.
En efecto, el prestigio del que disfrutaba Edison, opuesto al desarrollo de la corriente alterna, fue un obstculo
a las ideas de Tesla, quien no gozaba del mismo reconocimiento debido posiblemente a su origen europeo. Esto
constituye un ejemplo interesante de cmo el desarrollo cientfico y tecnolgico finalmente se impone sobre los
prejuicios.
Arranques y paradas frecuentes, operacin continua, sobrecargas, ambientes corrosivos o explosivos, etc.

201

Captulo VI La mquina de induccin

Figura 6.1 Modelo de la mquina de induccin bifsica diseada por Tesla


falla muy reducida. Algunas limitaciones tales como el ajuste de la caracterstica par-velocidad,
la intensidad de las corrientes durante el arranque, la regulacin de velocidad y el rendimiento,
han sido resueltas o mejoradas con diseos ingeniosos5 o incorporando controladores electrnicos de potencia. En la figura se muestra un despiece de la mquina de induccin con rotor de
jaula de ardilla donde se indican las principales partes constitutivas.

VI.1

P RINCIPIO

DE FUNCIONAMIENTO

En el captulo ?? se analizaron las ecuaciones en coordenadas generalizadas de las mquinas


rotativas convencionales; uno de los casos analizados fue la mquina de induccin. En una mquina de induccin convencional toda la energa elctrica fluye hacia o desde el estator. Los
flujos producidos por las corrientes del estator generan un campo magntico rotatorio que corta
a los conductores del rotor y de esta forma se obtiene sobre ellos fuerza electromotriz inducida
que es utilizada para forzar la circulacin de corrientes en el rotor. Al interactuar el campo magntico rotatorio del estator con el campo magntico rotatorio originado por las corrientes que
circulan en el rotor, se produce el par elctrico.
La mquina de induccin se alimenta con corriente alterna en el estator; de esta forma se produce
el campo magntico rotatorio que posee una amplitud constante en el tiempo, pero vara en el
espacio. La velocidad de giro del campo magntico rotatorio est definida por la frecuencia de las
corrientes inyectadas en el estator de la mquina. Para que una mquina de induccin produzca
par elctrico medio diferente de cero, debe satisfacerse la condicin 4.42. Si la mquina no
5

Primero se desarroll el rotor con anillos deslizantes que permita conectar resistencia externa y regular el arranque. Posteriormente se incorpor el rotor de doble jaula y el de barras profundas que permitieron cambiar los
parmetros del rotor de la mquina debido al efecto pelicular sin utilizar contactos deslizantes.

202

VI.2 Modelo de la mquina de induccin

Figura 6.2 Despiece de un motor de induccin industrial con rotor de jaula de ardilla
cumple con esta condicin, el par elctrico medio en un giro completo del rotor ser cero y no
podr transformar energa en rgimen permanente.
La mquina de induccin se utiliza como generador slo en pocas ocasiones, porque la operacin en este rgimen no es eficiente en comparacin con otras alternativas6. Sin embargo,
la mquina de induccin puede regresar energa a la red durante cortos perodos de tiempo en
algunos accionamientos convencionales. En particular puede generar cuando se utilizan en sistemas de traccin tales como ascensores u otras cargas similares, con la finalidad de producir
un frenado regenerativo. En el pasado era frecuente utilizar esta mquina como convertidor de
frecuencia, para lo cual es necesario tener acceso a los devanados del rotor mediante anillos deslizantes, tal como se muestra en la figura 6.3. Algunas centrales elicas utilizan esta mquina
como generador.
VI.2

M ODELO

DE LA MQUINA DE INDUCCIN

En la figura 6.4 se presenta el esquema bsico de las bobinas de una mquina de induccin cuyo
rotor y estator son trifsicos. En general el modelo se puede establecer para un nmero general
de fases en el estator y otro en el rotor. Como la mayora de los motores de induccin de uso
industrial son trifsicos en el estator, se realizar el modelo para un caso particular donde el rotor
y el estator son trifsicos7 . Normalmente las bobinas rotricas se encuentran en cortocircuito y
en el estator se aplica un sistema trifsico y balanceado de tensiones sinusoidales. En los modelos
convencionales de la mquina de induccin se desprecian los efectos que produce el ranurado,
6

La mquina de induccin necesita resistencia en el circuito rotrico para que la fuerza electromotriz inducida se
desfase de la fuerza electromotriz del estator y se pueda producir par. Esto obliga a producir un porcentaje significativo de prdidas en el rotor de la mquina de induccin. Las mquinas sincrnicas no tienen este inconveniente
y por tanto pueden alcanzar rendimientos mayores al escalar el tamao.
El caso general puede ser analizado mediante la misma tcnica.

203

Captulo VI La mquina de induccin

Figura 6.3 Mquina de induccin de rotor bobinado con anillos deslizantes

la distribucin de los devanados, las excentricidades estticas y dinmicas y en ciertos casos las
prdidas en el hierro y las prdidas mecnicas.
Las ecuaciones diferenciales que rigen el comportamiento de la mquina de induccin en el
sistema de coordenadas indicado en la figura 6.4 son:
[v] = [R] [i] + p [ ] = [R] [i] + [L( )] p [i] + [ ( )] [i]
Te Tm =

1 t
[i] [ ] [i] Tm = J +
2

Donde:
[v] =

[ve ]
[vr ]

" 
t #

 "  e e e t #
vea veb vec
i i i
[ie ]
=  r r r t ; [i] =
=  ar br cr t ;
[i
]
va vb vc
ia ib ic
r

" 
t #
ae be ce
t
=  r
[ ] =
a br cr

 

[Ree ] [Rer ]
Re [I] [0]
[R] =
=
[Rre ] [Rrr ]
[0] Rr [I]
 


L e [I] + Lme [S]
Ler [C( )]
[Lee ]
[Ler ( )]
=
[L( )] =
[Lre ( )]
[Lrr ]
L r [I] + Lmr [S]
Ler [C( )]t
 d



d
[Lee ]
[Ler ( )]
[0]
Ler dd [C( )]
d

=
[ ( )] =
d
d
Ler dd [C( )]t
[0]
d [Lre ( )]
d [Lrr ]


204

[e ]
[r ]

(6.1)
(6.2)

VI.2 Modelo de la mquina de induccin

Figura 6.4 Diagrama esquemtico de las bobinas de una mquina de induccin trifsica en el
rotor y estator

205

Captulo VI La mquina de induccin

1 21 21
0 0
1 0 0
1
1

; [S] = 2 1 2
; [0] = 0 0
[I] = 0 1 0
1
1
0 0
0 0 1
2 2 1

cos
cos( + 23 ) cos( + 43 )
[C( )] = cos( + 43 )
cos
cos( + 23 )
cos( + 23 ) cos( + 43 )
cos

sin
sin( + 23 ) sin( + 43 )
d
[C( )] = sin( + 43 )
sin
sin( + 23 )
d
sin( + 23 ) sin( + 43 )
sin

0
0
0

Los parmetros que definen el comportamiento del modelo de la mquina de induccin en el


sistema de coordenadas primitivas son:

Re
Rr
L e
L r
Lme
Lmr
Ler

es la resistencia de cada una de las bobinas del estator


es la resistencia de cada una de las bobinas del rotor
es la inductancia de dispersin del estator
es la inductancia de dispersin del rotor
es la inductancia de magnetizacin del estator
es la inductancia de magnetizacin del rotor
es la inductancia mutua de acoplamiento estator-rotor

La matriz [S] representa los acoplamientos simtricos entre bobinas del estator o rotor, los trminos 1 en la diagonal corresponden a las magnetizaciones de la bobina propia8 y el trmino
12 representa las mutuas entre fases que se encuentran separadas espacialmente 23 o 43 ,9 cuyo
acoplamiento depende entonces del cos 23 = cos 43 = 21 .
La matriz [C( )] determina el comportamiento cclico de los acoplamientos mutuos entre bobinas del rotor y del estator, por esta razn aparece el ngulo como argumento de la funcin
coseno. El acoplamiento entre la fase ae del estator y la fase ar del rotor depende directamente
del cos ; el acoplamiento entre la fase ae del estator y la br del rotor, adems de estar separada en el ngulo entre las referencias de ambos sistemas, tiene una fase adicional de 23 que
corresponde a la separacin espacial entre fases y explica de esta forma la aparicin del trmino cos( + 23 ). De igual forma se puede explicar el trmino cos( + 43 ), correspondiente al
acoplamiento entre la fase ae del estator y la cr del rotor.
El sistema conformado por las seis ecuaciones de tensin planteadas en 6.1 y el balance de
par expresado en la ecuacin 6.2, representan el comportamiento dinmico de la mquina de
induccin10 , pero la dependencia de la posicin angular complica notablemente la solucin
prctica de este modelo y la tcnica de transformacin de coordenadas es conveniente.
8
9
10

Las fases a con a, b con b y c con c del sistema rotrico o estatrico respectivamente.
Lo cual incluye los acoplamientos mutuos entre a y b, a y c, as como b con c.
Dentro del rango de las hiptesis simplificadoras supuestas inicialmente.

206

VI.3 Vectores espaciales


VI.3

V ECTORES

ESPACIALES

Un anlisis de los acoplamientos observados en el modelo de la mquina de induccin en coordenadas primitivas permite destacar que stos estn definidos por matrices simtricas [S] o cclicas
[C( )]. Estas matrices pueden ser diagonalizadas utilizando el mtodo de autovalores y autovectores. Con esta tcnica se puede demostrar que la transformacin de componentes simtricas11
es capaz de realizar el desacoplamiento de ambas matrices. La transformacin de componentes
simtricas hermitiana12 se define como:

1 1
1
1 1 1
xa
xa
x0
4
2
1
x+ = 1
(6.3)
1 e j 3 e j 3 xb = 1 2 xb
4
2
3
3 1 2
j
j
x
x
x
c
c
1 e 3 e 3

1 1
xa
1

xb = 1 e j 43
2
3
xc
1 ej 3

ej 3
4
ej 3

1 1 1
xa
xa
1

xb = 1 2 xb
3 1 2
xc
xc

(6.4)

Al aplicar la transformacin 6.4 a un sistema cclico se obtiene el siguiente resultado:

ya
a b c
xa
yb = c a b xb
yc
b c a
xc

1 1 1
1 1 1
y0
x0
a b c
1
1
2
2

y+ = c a b
x+
1

3 1 2
3
2
y
x
b c a
1

1 1 1
1 1 1
y0
a b c
x0
y+ = 1 1 2 c a b 1 1 2 x+
3 1 2
3 1 2
y
b c a
x

a+b+c
0
0
y0
x0
y+ =
x+
0
0
a + b + c 2
2
y
x
0
0
a + b + c

(6.5)

El desacoplamiento de las matrices simtricas se obtiene como caso particular de las matrices
cclicas donde b = c:

ya
a b b
xa
yb = b a b xb
yc
b b a
xc

y0
a + 2b
0
0
x0
y+ = 0
ab
0 x+
y
0
0
ab
x
11
12

(6.6)

Propuesta por Fortescue y ampliamente utilizada para el anlisis de fallas en sistemas desequilibrados.
Conservativa en potencia.

207

Captulo VI La mquina de induccin

Figura 6.5 Representacin grfica del vector espacial de un sistema trifsico

Esta propiedad caracterstica de transformacin de componentes simtricas permite convertir un


sistema acoplado en tres sistemas independientes. El sistema de secuencia cero solamente se
puede excitar cuando la sumatoria instantnea de las tensiones o de las corrientes es diferente
de cero13 . El sistema de secuencia negativa y de secuencia positiva son similares y uno es el
conjugado del otro. Por estos motivos es posible representar el modelo de la mquina utilizando
solamente la transformacin de secuencia positiva14 y se denomin transformacin a vectores
espaciales. Para conservar la potencia activa en la transformacin se debe definir la siguiente
transformacin:

r h
xa (t)
x
(t)
i
a

2
2
2
4
x(t)=
1 2 xb (t)
(6.7)
1 e j 3 e j 3 xb (t) =
3
3
xc (t)
xc (t)
La transformacin a vectores espaciales permite representar un sistema de tensiones, corrientes
o flujos trifsicos mediante un vector en el espacio, cuya posicin y magnitud dependen del
tiempo. En la figura 6.5 se muestra una representacin grfica con la interpretacin geomtrica
de la transformacin a vectores espaciales para un instante de tiempo dado.

13
14

En los sistemas trifsicos, esto requiere la presencia de un cuarto hilo por donde pueda circular esta componente.
En las mquinas elctricas industriales es poco habitual la conexin del neutro.
Es equivalente utilizar la componente de secuencia negativa. La componente de secuencia cero tiene escasa
utilidad en el anlisis de las mquinas debido a que no puede producir par. Sin embargo, algunos desequilibrios
dependen notoriamente de esta componente.

208

VI.3 Vectores espaciales


Transformando las ecuaciones 6.1 al dominio de los vectores espaciales se obtiene el siguiente
resultado:

 
 

  
Le
Mer e j
ie
Re 0
ve
ie
+p
=
(6.8)

j
ir
0 Rr
vr
ir
Mer e
Lr
Donde:

 
2
1 2
3
r
 
2
=
1 2
3
r
 
2
=
1 2
3
r
 
2
=
1 2
3

ve =
vr
ie
ir

vea veb vec


vra vrb vrc
iea ieb iec
ira irb irc

t
t

t
t

3
3
3
Le = L e + Lme ; Lr = L r + Lmr , Mer = Ler
2
2
2
Los trminos que aparecen en la expresin 6.8 se pueden obtener realizando la transformacin
a vectores espaciales de la matrices que representan el modelo de la mquina en coordenadas
primitivas, tales como:

1. La transformacin de vectores espaciales aplicada a la matriz identidad [I]:

r
r
ya
1 0 0
xa




2
2
1 2 yb =
1 2 0 1 0 xb
3
3
0 0 1
yc
xc

r
 xa
2
2
y =
1 xb = x
3
xc
2. La transformacin aplicada a la matriz simtrica [S]:

r
r
ya


2
2
1 2 yb =
1 2
3
3
yc
r
2
1 2
y =
3

1 21
1 1
2
12 21
3
 23 xa
xb =
2
3
2 xc


(6.9)

12
xa
12 xb
xc
1
3
x
2

(6.10)

209

Captulo VI La mquina de induccin


3. La misma transformacin aplicada a la matriz cclica [C( )], recordando que cos =
e j +e j
:
2

cos
cos( + 23 ) cos( + 43 )
xa

2
y=
1 2 cos( + 43 )
cos
cos( + 23 ) xb =
3
xc
cos( + 23 ) cos( + 43 )
cos

r
1 2
1 2 xa

j


e
2
e 2
1 2 xb

=
1 2
1 +

2
3
2
2
xc

1
2 1

r
xa
n
o




2 1 j
xb = 3 e j x
e
3 3 3 2 + e j 0 0 0
(6.11)
y=
32
2
xc
r

La transformacin a vectores espaciales de la expresin del par elctrico expresado en el balance


de la ecuacin 6.2 queda:
1
1
Te = [i]t [ ] [i] =
2
2

[ie ]
[ir ]



[0]
Ler dd [C( )]
Ler dd [C( )]t
[0]

t 

[ie ]
[ir ]

d
[C( )] [ir ] =
d

1 2
1 2
e j

j
1 2 e 2 1 [ir ] =
= Ler [ie ]t

2j
2j
2 1
2 1
r
 j

j 



e
3
e

=
Ler
ie 1 2
i 1 2
[ir ] =
2
2j
2j e
 j

n
o
n 
 o
e
e j
3

ie ir
ie ir = Mer m ie ir e j = Mer m ie ir e j
= Ler
2
2j
2j
= Ler [ie ]t

(6.12)

El sistema de ecuaciones diferenciales que definen el comportamiento de la mquina de induccin en el sistema de coordenadas correspondiente a los vectores espaciales es:


ve
vr


 
  
Le
Mer e j
ie
Re 0
ie
+p
=
ir
0 Rr
ir
Mer e j
Lr
 o
n 
Tm ( ) = J +
Mer m ie ir e j


(6.13)

El modelo 6.13 simplifica notablemente las expresiones 6.1 y 6.2, al representar las magnitudes trifsicas mediante vectores espaciales. Por una parte el sistema se ha reducido de las siete
ecuaciones diferenciales iniciales a tres15 y la dependencia en la posicin angular se ha simpli15

Esta apariencia ms simple no debe hacer olvidar el hecho de que las nuevas variables son vectores espaciales
variables en el tiempo y no simples variables instantneas, como era en el caso del modelo de la mquina de
induccin en coordenadas primitivas.

210

VI.3 Vectores espaciales

Figura 6.6 Circuito equivalente de la mquina de induccin en vectores espaciales referidos al


sistema de referencia estatrico
ficado a su aparicin en matrices cuya dimensin es 2 2.16 Sin embargo, la dependencia en la
posicin angular puede ser eliminada, si las variables del rotor se refieren al estator utilizando
la siguiente transformacin17 :
xer xr e j
(6.14)
Para aplicar la transformacin 6.14 al modelo de la mquina en vectores espaciales 6.13, se
requiere desarrollar la derivada correspondiente de esta transformacin:
pxer = pxr e j + j xr e j = pxr e j + j xer
pxr e j = pxer j xer

(6.15)

Utilizando las expresiones 6.14 y 6.15 en el modelo 6.13, se obtiene el siguiente modelo de la
mquina de induccin en vectores espaciales referidos al estator:


ve
ver

Re 0
0 Rr


 
  
0
0
ie
Le Mer
ie

+
p e j
Mer Lr
ier
ir
Mer Lr


Mer m ie (ier ) Tm ( ) = J +
(6.16)



ie
ier

El modelo 6.16 es independiente de la posicin angular , que es variable en el tiempo aun en


el caso particular de la operacin en rgimen permanente y esta dependencia es reemplazada
por la velocidad angular cuyo comportamiento temporal vara ms lentamente 18 . Este modelo
puede ser representado mediante el circuito equivalente que se muestra en la figura 6.6. Este
circuito reproduce el comportamiento elctrico de la mquina en rgimen transitorio y es capaz
de calcular el par elctrico evaluando la potencia activa transferida a la fuente dependiente de
corriente del circuito rotrico, tema que ser analizado con mayor profundidad en el captulo 7.
16
17

18

Y que pueden ser invertidas analticamente con relativa sencillez.


Recordemos que el sistema de referencia del estator es independiente del sistema de referencia del estator, pero
ambas referencias se encuentran separadas en el ngulo , por esta razn cuando se multiplica una vector espacial
en el sistema de referencia rotrico por e j , el nuevo vector resultante posee la misma magnitud y su fase ahora
se mide desde el sistema de referencia estatrico.
En efecto, en rgimen permanente la velocidad angular es una constante, mientras que el ngulo cambia constantemente.

211

Captulo VI La mquina de induccin


VI.4

M ODELO

EN RGIMEN PERMANENTE

Se puede obtener un modelo de la mquina de induccin operando en condiciones de rgimen


permanente a partir del modelo transitorio, particularizando las variables correspondientes en
este estado. En rgimen permanente equilibrado, las bobinas del estator de la mquina de induccin se alimentan con un sistema balanceado de tensiones trifsicas de secuencia positiva y las
bobinas del rotor se encuentran en cortocircuito:

2Ve cos et
vae (t) =



2
vbe (t) =
2Ve cos et
3



4
2Ve cos et
(6.17)
vce (t) =
3

var (t) = vbr (t) = vcr (t) = 0

(6.18)

Las tensiones 6.17 y 6.18 expresadas como vectores espaciales son:

r
2V
cos

t
e
e



2
ve =
1 2 2Ve cos et 23

3
2Ve cos et 43
ve =

e jet + e jet


2
1 2 je t
e
+ e jet = 3Ve e jet
Ve 1 2
3
2
e jet + 2 e jet

r
0

2
1 2 0 = 0 = ver
vr =
3
0


(6.19)

(6.20)

Al excitar las bobinas con tensiones trifsicas balanceadas, las corrientes del estator y las del rotor referidas al estator tambin resultarn balanceadas y los correspondientes vectores espaciales
sern:

(6.21)
ie = 3Ie e j(e t+e )

ier = 3Ir e j(et+r )


(6.22)
Por otra parte, la velocidad del rotor en rgimen permanente ser constante = m = cte.
Reemplazando las condiciones 6.19, 6.20, 6.21 y 6.22 en el modelo de la mquina de induccin

212

VI.4 Modelo en rgimen permanente

Figura 6.7 Circuito equivalente de la mquina de induccin en rgimen permanente


descrito en vectores espaciales se obtiene:





j(et+e )
Re 0
3I
e
3Ve e jet
e

=
+
0 Rr
0
3Ir e j(et+r )




j(et+e )
Le Mer
3I
e
e
+
je
+
Mer Lr
3Ir e j(et+r )



3Ie e j(et+e )
0
0

jm
Mer Lr
3Ir e j(e t+r )


Ve
0



 



Re 0
Ie e je
Le Mer
0
0
+ je
=
jm

0 Rr
Mer Lr
Mer Lr
Ir e jr

 
 

je Mer
Ie
Re + je Le
Ve
(6.23)
=
j(e m )Mer Rr + j(e m )Lr
Ir
0


Para determinar un circuito equivalente de la mquina de induccin en rgimen permanente a


partir del sistema de ecuaciones 6.23, es necesario dividir la segunda ecuacin por el deslizamiento19 :
s


Ve
0



e m
e

Re + je Le
je Mer

(6.24)

je Mer
Rr
s + j e Lr

 

Ie
Ir

(6.25)

En la figura 6.7 se presenta el circuito equivalente de la mquina de induccin en rgimen permanente.


El par elctrico en rgimen permanente se calcula sustituyendo en la expresin 13.23 los fasores
espaciales obtenidos en 13.29 y 13.30:
Te = Mer m
19

3Ie e

j(et+e )

3Ir e

j(e t+r )

 o

= 3Mer Ie Ir sin (e r )

(6.26)

El deslizamiento s es una variable de gran importancia en la modelacin de la mquina de induccin y representa


la velocidad relativa entre el campo producido en el estator y la posicin del rotor, en por unidad de la velocidad
de este campo.

213

Captulo VI La mquina de induccin


La ecuacin correspondiente al circuito rotrico en el sistema 6.25 relaciona directamente las
corrientes del estator y del rotor:


Rr
0 = je Mer Ie +
+ je Lr Ir
s


Rr
Rr
j e
s + j e Lr
s + j e Lr
Ie = j
Ir Ie e = j
Ir e jr
e Mer
e Mer
Ie e

j(e r )

=j

Rr
s


+ je Lr
Rr
Ir Ie sin(e r ) =
Ir
e Mer
se Mer

(6.27)

Al sustituir la expresin 6.27 en la ecuacin del par elctrico 6.26, se obtiene el par elctrico en
funcin de la corriente del rotor Ir , el deslizamiento s, la resistencia del rotor Rr y la velocidad
sincrnica s :
Te = 3

Rr 2
I
e s r

(6.28)

La expresin 6.28 se puede obtener directamente del circuito equivalente de la figura 6.7, cuando
se calcula tres veces20 la potencia entregada a la resistencia Rsr y se divide por la velocidad
sincrnica e .
Dentro de las hiptesis del modelo se han despreciado la prdidas en el hierro de la mquina. Es
posible considerar estas prdidas colocando una resistencia en paralelo con la fuerza electromotriz producida por el flujo de magnetizacin. Tambin se puede recordar que las inductancias Le
y Lr estn compuestas de dos partes, dispersin y magnetizacin. Por esta razn, haciendo uso
de sus respectivas definiciones planteadas en el modelo 6.8, se puede establecer lo siguiente:
3
3
Le Mer = L e + Ler Ler = L e
2
2
3
3
Lr Mer = L r + Ler Ler = L r
2
2

(6.29)

Al definir X e e L e , X r e L r y Xm = e Mer , incluir la resistencia de magnetizacin en


paralelo con la reactancia de magnetizacin y separar la resistencia Rsr en dos componentes, una
Rr que representa las prdidas hmicas del circuito rotrico y 1s
s Rr que representa la potencia
transferida al rotor que no se consume en prdidas, se puede obtener el modelo clsico de la
mquina de induccin en rgimen permanente, tal como se muestra en la figura 6.8.
Desde el punto de vista elctrico, el comportamiento de la mquina de induccin en rgimen
permanente depende del deslizamiento s, de la tensin aplicada en el estator Ve y de los parmetros del circuito equivalente (Re , Rr , Rm , X e , X r , Xm ). Una vez que se conocen los parmetros
del modelo, el deslizamiento del rotor y la fuente de alimentacin, se pueden determinar las
corrientes que circulan por la mquina. El anlisis circuital de la mquina de induccin es seme20

Por estar representando un modelo unifilar de la mquina aparece el coeficiente 3 en los clculos de potencia y
par.

214

VI.5 Ecuaciones de la mquina de induccin

Figura 6.8 Modelo clsico de la mquina de induccin


jante al de un transformador con una carga resistiva variable. Esta carga depende exclusivamente
del deslizamiento del rotor.
Aun cuando el modelo clsico de la mquina de induccin es similar al modelo de un transformador, existen algunas diferencias importantes:
1. La reluctancia del circuito magntico de la mquina de induccin es mucho mayor que
la reluctancia de magnetizacin de un transformador. Esto se debe principalmente a la
presencia de entrehierro en la mquina. La corriente de excitacin de una mquina es considerablemente mayor que la de un transformador de igual potencia. Esta corriente puede
alcanzar entre un 30 % y un 50 % de la corriente nominal de la mquina, contrastando con
el 0,5 % a 1,0 % en un transformador convencional.
2. Al ser tan grande la reluctancia de magnetizacin, se incrementan considerablemente los
enlaces de dispersin. Por esta razn las reactancias de dispersin de la mquina son
mayores que estas reactancias para un transformador de similar potencia. Cada una de las
reactancias de dispersin de la mquina pueden superar el 10 %, en comparacin con un
transformador donde se encuentran entre el 1 % y el 6 % aproximadamente.

VI.5

E CUACIONES

DE LA MQUINA DE INDUCCIN

Del modelo clsico de la mquina de induccin mostrado en la figura 6.8, se pueden extraer
varias relaciones de gran utilidad para determinar el comportamiento de la mquina en diferentes
condiciones de operacin. Algunas de estas relaciones son:
1. Potencia de prdidas en el rotor: todas las prdidas elctricas del rotor se encuentran
principalmente en las resistencias de las bobinas del rotor. Estas prdidas se pueden calcular mediante la expresin:
PRr = 3Ir2 Rr
(6.30)
2. Potencia de prdidas en el estator: los conductores del estator poseen resistencia y por
esta razn en estos devanados se producen prdidas. Tambin en el hierro de la mquina

215

Captulo VI La mquina de induccin


se producen prdidas por histresis del material magntico y por induccin de corrientes
parsitas. Todas estas prdidas se pueden calcular mediante la siguiente relacin:
Pper.ext = PRe + Pf e = 3Ie2 + 3

Vm2
Rm

(6.31)

La tensin Vm se puede calcular a partir de la corriente del estator Ie , mediante la siguiente


expresin:
(6.32)
Vm = Ve (Re + jX e )Ie
3. Potencia mecnica en el eje del rotor: de la potencia que entra a la mquina por los ejes
elctricos del estator, una parte se consume en los devanados de estator y otra porcin
en las prdidas del hierro. El resto de la potencia de entrada atraviesa el entrehierro de
la mquina y llega al circuito del rotor. En este circuito se pierde otra porcin en las
resistencias de los conductores. La diferencia entre la potencia de entrada y todas las
prdidas, se encuentra disponible en el eje del rotor como potencia mecnica:




1
1s
2
2
Pe je = Protor PRr = 3Ir Rr
(6.33)
1 = 3Ir Rr
s
s
El balance 6.33 demuestra que la potencia mecnica disponible en el eje es igual a la
potencia que se consume en la resistencia de carga representada en la figura 6.8. La potencia mecnica til disponible en el eje mecnico puede ser menor a la calculada por la
expresin 6.33, debido a que existen prdidas de tipo mecnico tales como la friccin y la
refrigeracin de la mquina mediante ventiladores acoplados al eje mecnico, que reducen
la potencia disponible en el eje.

4. Par elctrico: el par elctrico de la mquina se puede calcular a partir del cociente entre
la potencia mecnica disponible en el eje y la velocidad mecnica del rotor:
Te =

Pe je
1s
1s
1
Protor
= 3Ir2Rr
= 3Ir2 Rr
= 3Ir2Rr
=
m
sm
se (1 s)
e s
e

(6.34)

La ecuacin 6.34 determina el par elctrico mediante la potencia mecnica disponible en el


eje Pe je y la velocidad mecnica del rotor m . Un mtodo alternativo consiste en calcular
el par utilizando la potencia elctrica que atraviesa el entrehierro Protor y la velocidad
sincrnica e a la que se realiza esta conversin.
5. Corriente del rotor: para determinar la potencia en el eje Pe je y el par elctrico Te , es
necesario obtener la corriente del rotor Ie . Para calcular esta corriente es til realizar un
equivalente de Thvenin visto desde el rotor hacia la fuente del estator, tal como se muestra en la figura 6.9. La tensin de Thvenin en el circuito de la figura 6.9 se determina
mediante un divisor de tensin entre la impedancia serie del estator Ze y la impedancia de
magnetizacin Zm :
Zm
Ve
(6.35)
Vth =
Zm + Ze
216

VI.6 Caracterstica par-deslizamiento

Figura 6.9 Equivalente de Thvenin de la mquina de induccin visto desde el rotor


La impedancia de Thvenin del circuito es el resultado del paralelo entre Ze y Zm , en serie
con la impedancia Zr :
Ze Zm
+ Zr = Rth + jXth
(6.36)
Zth =
Ze + Zm
La corriente Ir se obtiene a partir del circuito de Thvenin de la figura 6.9:
Ir = q

Vth
Rth +


Rr 2
+ Xth2
s

(6.37)

Sustituyendo la expresin 6.37 en las ecuaciones 6.33 y 6.34, se determina la potencia en


el eje y el par elctrico en funcin de los parmetros de la mquina, la tensin de Thvenin
y el deslizamiento del rotor:

3Vth2 Rr 1s
s
(6.38)
Pe je =
2
Rth + Rsr + Xth2
3 Rers Vth2
Te =
2
Rth + Rsr + Xth2

VI.6

C ARACTERSTICA

(6.39)

PAR - DESLIZAMIENTO

La ecuacin 6.39 determina el par elctrico de la mquina de induccin. Si la tensin de alimentacin Ve tiene una amplitud constante, la tensin de Thvenin tambin tendr su magnitud
constante, debido a que las impedancias del estator y de magnetizacin son independientes del
deslizamiento del rotor. Si se excluye el deslizamiento, todos los trminos de la ecuacin 6.39
son constantes para una mquina dada, mientras la frecuencia de la red sea constante.

217

Captulo VI La mquina de induccin


Para comprender el comportamiento funcional de esta caracterstica, resulta conveniente realizar
aproximaciones asintticas de la ecuacin 6.39 con respecto a valores extremos del deslizamiento. Cuando el deslizamiento es cero, la velocidad angular del eje rotor es igual a la velocidad
del campo magntico rotatorio. En esta condicin el campo rotante producido en el estator no
corta los conductores del rotor, no se produce fuerza electromotriz en estas bobinas, no circula
corriente y por esta razn no se produce par elctrico medio.
Para deslizamientos muy pequeos s 0 pero diferentes de cero, el trmino Rsr es mucho mayor
que la resistencia y reactancia Thvenin. En este caso es posible despreciar en el denominador
de la expresin 6.39 la resistencia y la reactancia de Thvenin:
q
Rr
2 + X2
>> Rth
th
s
3Vth2
s ; si, s 0
(6.40)
e Rr
En aquellos deslizamientos para los cuales es vlida la expresin 6.40, el comportamiento de la
caracterstica par-deslizamiento es lineal. En la prctica, la ecuacin 6.40 es de gran utilidad debido a que en los puntos de operacin en rgimen permanente, los deslizamientos de la mquina
son lo suficientemente pequeos para satisfacer esta aproximacin con precisin.
Te

Por otra parte, cuando el deslizamiento es grande, el trmino Rsr es despreciable y la caracterstica
par-deslizamiento se puede aproximar a:
3Vth2 Rr
; si, s
Te
2 + X2 )
e s(Rth
th

(6.41)

La expresin 6.41 representa una variacin hiperblica del par elctrico a medida que el deslizamiento aumenta. En valores negativos del deslizamiento, la aproximacin anterior es igualmente
vlida, sin embargo en este caso el par elctrico es negativo.
En la figura 6.10 se ha destacado un punto importante de la caracterstica par-deslizamiento;
este punto corresponde al par mximo de la mquina. El par es mximo cuando la potencia que
atraviesa el entrehierro es mxima. Esto se debe a que la velocidad sincrnica depende de la
frecuencia de las corrientes inyectadas en el estator y por lo tanto es constante. Para calcular la
potencia mxima que puede atravesar el entrehierro se aplica el principio de mxima transferencia de potencia al equivalente de Thvenin de la figura 6.9. La mxima transferencia de potencia
ocurre cuando la impedancia de la carga se Rsr iguala a la impedancia del equivalente de Thvenin Zth . En este circuito la carga es puramente resistiva, mientras que la impedancia de Thvenin
es fuertemente inductiva. En este caso, para transferir la mxima potencia, es necesario que los
mdulos de las impedancias se igualen:
q
Rr
2 + X2
(6.42)
= Zth = Rth
th
s
Despreciando en la ecuacin 6.42 la resistencia de Thvenin Rth , la cual generalmente es muy
pequea en comparacin con la reactancia Xth, y reemplazando esta expresin en la ecuacin

218

VI.6 Caracterstica par-deslizamiento

Figura 6.10 Caracterstica par-deslizamiento de la mquina de induccin

6.39, se puede calcular el par mximo que produce la mquina de induccin:


Te max

3 Vth2
2e Xth

(6.43)

El deslizamiento que produce el par mximo se obtiene de la expresin 6.42:


Rr
sTmax = q
2 + X2
Rth
th

(6.44)

Al examinar la ecuacin 6.39 se observa que la caracterstica par-deslizamiento no es completamente simtrica con respecto al origen. El denominador de esta ecuacin no es indiferente al
signo del deslizamiento. Si la resistencia de Thvenin es nula o despreciable, la caracterstica
entonces es simtrica.
El deslizamiento es la variable que define el punto de operacin de la mquina de induccin.
Conocido este dato se pueden determinar las corrientes, el par elctrico, las potencias de entrada
o salida, las prdidas, el factor de potencia y el rendimiento de la mquina.
En las mquinas con rotor devanado es posible incluir resistencia en serie con el circuito del
rotor. Esta posibilidad se puede utilizar para reducir las corrientes durante el arranque o para incrementar sustancialmente la magnitud del par elctrico durante este proceso. Incluso es posible
aadir suficiente resistencia como para permitir que la mquina arranque con el par mximo:
q
Rr + Radicional
2 + X2 R
(6.45)
= 1 Radicional = Rth
sTmax = q
r
th
2
2
Rth + Xth
219

Captulo VI La mquina de induccin

Figura 6.11 Efecto de la variacin de la resistencia del rotor sobre la caracterstica pardeslizamiento
La magnitud del par mximo no es afectada por la variacin de la resistencia del rotor, pero
la caracterstica par-deslizamiento se modifica considerablemente como se observa en la figura
6.11.
VI.7

P UNTOS

DE OPERACIN

La caracterstica par-deslizamiento indica el valor del par elctrico Te para cualquier deslizamiento s. Para definir el deslizamiento de operacin de la mquina es necesario el conocimiento
de la caracterstica de la carga mecnica. El punto de operacin del sistema formado por la
mquina elctrica y la carga mecnica est definido por la interseccin de las dos caractersticas.
La caracterstica par-velocidad de una bomba puede ajustarse mediante un polinomio de segundo
grado en la velocidad angular mecnica m . Esta caracterstica se representa en funcin del
deslizamiento de la mquina de induccin de la siguiente forma:
Tm (m ) = k1 m2 + k2 m + k3 = k1 (1 s)2e2 + k2 (1 s)e + k3

(6.46)

El punto de operacin de la mquina se obtiene en el deslizamiento sop , que iguala el par elctrico producido por la mquina de induccin con el par mecnico que opone la bomba y se
establece mediante el equilibrio:
Te (sop ) Tm (sop ) = 0
3 Rers Vth2
= k1 (1 s)2e2 + k2 (1 s)e + k3

Rr 2
2
Rth + s + Xth
220

(6.47)
(6.48)

VI.7 Puntos de operacin

Figura 6.12 Puntos de operacin de la mquina de induccin acoplada a una bomba con diferentes valores de la tensin aplicada

En la figura 6.12 se muestra el punto de operacin o punto de equilibrio representado en la


expresin 6.48.
Segn la ecuacin 6.39, el par elctrico de la mquina de induccin depende del cuadrado de
la tensin de Thvenin. Este hecho puede utilizarse para controlar el punto de operacin de la
mquina variando la tensin de alimentacin. En la figura se observa que la reduccin de la
tensin de alimentacin afecta fuertemente el par elctrico de la mquina en todo el rango de
deslizamientos. Si la tensin se reduce durante el proceso de arranque de la mquina, el par de
accionamiento puede ser insuficiente para acelerar la mquina hasta el punto final de operacin.
Para que la mquina pueda acelerar, el par elctrico debe ser mayor que el par de la carga. Si
esta diferencia es muy pequea, la mquina demora mucho tiempo para alcanzar el punto de
operacin permanente:
d m
Te Tm = Tacel. = J
(6.49)
dt
La ecuacin 6.49 determina el proceso dinmico de arranque de la mquina de induccin. En la
medida que el par elctrico Te supera el par mecnico Tm , se incrementa la velocidad del rotor
m . Cuando los pares se igualan en el punto de operacin, la aceleracin se anula y la mquina
elctrica se mantiene accionando a la carga mecnica a esa velocidad. Si vara la carga o la
tensin de la red, la mquina acelera o frena hasta alcanzar el nuevo punto de equilibrio. Algunos
puntos de interseccin de las caractersticas de par elctrico y mecnico no son estables. Si al
aumentar la carga mecnica disminuye el par elctrico, o al disminuir la carga mecnica aumenta
el par producido por la mquina, el punto de operacin es inestable y a la menor perturbacin,
la mquina se detendr o buscar un punto de operacin estable.

221

Captulo VI La mquina de induccin


VI.8

EL

PUNTO NOMINAL

La corriente nominal de una mquina est determinada por la clase de aislamiento de sus bobinas, las prdidas generadas por esta corriente y el sistema de refrigeracin encargado de disipar
al medio ambiente estas prdidas. Los materiales aislantes que recubren los conductores de las
bobinas se degradan ms rpidamente en relacin directa con la temperatura21 . A este fenmeno
se le conoce como envejecimiento. El calor generado por prdidas resistivas en los conductores
crece con el cuadrado de la corriente que circula por las bobinas. La temperatura en el interior
de la mquina y ms concretamente en el aislamiento de las bobinas est determinada por la
capacidad de la mquina para transmitir el calor al medio ambiente. Esta capacidad se conoce
como impedancia trmica y depende de la geometra de la mquina, de los materiales y del
sistema de enfriamiento.
La corriente nominal, por lo tanto, es aquella corriente que al circular por las bobinas produciendo prdidas que incrementan la temperatura interior de la mquina hasta alcanzar el valor
mximo. Con el valor mximo de la temperatura interior, el envejecimiento del material aislante
es tan lento que permite alcanzar a la mquina su perodo de vida til22 , sin que se produzcan
fallas en el mismo.
La corriente del estator Ie se puede obtener a partir del circuito equivalente 6.9 utilizando el
siguiente procedimiento:
Zm
Vth
Zm +Ze Ve
Ir =
=
(6.50)
Zth + Rsr
Rth + Rsr + jXth


Rr
Vm =
+ jX r Ir
(6.51)
s
Im =
Ie = Im + Ir =

Rr
s

Vm
=
Zm

Rr
s

+ jX r
Ir
Zm

Rr
+ jX r + Zm
s + jX r + Zm
Ir =
Ve
Zm
(Zm + Ze )(Zth + Rsr )

(6.52)
(6.53)

La corriente nominal In define el deslizamiento nominal sn de la mquina como se muestra en


la figura 6.13. Una vez definido el deslizamiento nominal, tambin queda determinado el par
elctrico nominal Ten y la potencia nominal en el eje Pe jen . Conocida la temperatura mxima
de operacin tmax , el sistema de enfriamiento determina la corriente nominal y esta corriente
define el deslizamiento nominal correspondiente a una tensin dada. Obtenido el deslizamiento
nominal, tambin quedan definidos el par elctrico nominal y la potencia nominal en el eje.
La tensin nominal de la mquina tiene relacin con las prdidas en el hierro y con la magnitud
de la corriente de magnetizacin. Cuando se aplica la tensin nominal a las bobinas del estator, el
flujo producido en el entrehierro no debe exceder los valores mximos de la densidad de campo
magntico Bmax que tolera el material sin incrementar drsticamente las prdidas en el hierro. Si
21
22

A mayor temperatura la movilidad electrnica facilita la reaccin qumica de los materiales y stos, al incluir
impurezas en las redes cristalinas, degradan las propiedades dielctricas originales.
Entre unos quince y treinta aos de vida media. La mitad de las mquinas en una muestra grande, habr fallado
durante un tiempo equivalente a una vida media.

222

VI.8 El punto nominal

Figura 6.13 Magnitudes de las corrientes del estator Ie y del rotor Ir en funcin del
deslizamiento
la densidad de campo magntico supera este valor, las prdidas en el hierro crecen rpidamente,
aumenta la corriente de magnetizacin Im debido a la saturacin del material y se incrementa la
temperatura interior de la mquina por encima del valor mximo de diseo.
Los valores nominales de la mquina no implican en forma alguna que sta debe funcionar
siempre en esta condicin. Estos valores son simplemente una referencia que indica un punto
de operacin en el cual la mquina puede mantenerse en rgimen permanente durante todo el
perodo diseado de vida til.23 Exceder estos valores incrementa las prdidas y la temperatura
interior de la mquina, pero si la temperatura previa es inferior a la temperatura mxima de
diseo, la inercia trmica de los materiales retarda el proceso. Durante este tiempo es posible
operar la mquina por encima de sus puntos nominales sin reducir la vida til del convertidor24 .
Incluso es posible tolerar un pequeo incremento de la temperatura sobre la temperatura mxima
sin reducir significativamente la vida til de la mquina.
Durante el proceso de arranque, las corrientes pueden alcanzar de tres a seis veces el valor
nominal y esto produce un incremento de las prdidas con el cuadrado de este valor. Las prdidas pueden crecer de nueve a treinta y seis veces su valor con respecto al punto nominal. Si
esta situacin se mantiene indefinidamente, la temperatura se incrementar muy rpidamente y
se envejecer el aislamiento rpidamente. El tiempo de arranque depende de la inercia conectada al eje de la mquina y de la diferencia entre el par elctrico y el par mecnico de la carga.
Cuando el arranque es lento o se realiza mltiples veces, la temperatura mxima se puede exceder. Si esto ocurre frecuentemente, indica que la especificacin nominal de la mquina est
23

24

Es necesario recordar que la vida til es en realidad vida media til, una variable de tipo estadstico. Solamente
la mitad de las mquinas en estas condiciones alcanzara a operar sin fallas durante este tiempo. Probablemente
el fabricante especificar algunas condiciones de mantenimiento mnimo para alcanzar este perodo de vida til.
Esto es una condicin general vlida para todas las mquinas elctricas, incluidos los transformadores.

223

Captulo VI La mquina de induccin


por debajo de los requerimientos de la carga. En estas condiciones es posible que durante los
perodos de operacin en rgimen permanente, la mquina opere por debajo de su especificacin
nominal y sin embargo la temperatura interior exceda la mxima permitida. Por esta razn es
muy importante el ciclo de carga, aceleracin y frenado al que est sometida una mquina en
su especificacin definitiva.

VI.9

S ISTEMA

EN POR UNIDAD

Resulta conveniente utilizar el sistema de valores en por unidad (pu) en la representacin de la


mquina de induccin. Al representar las magnitudes, parmetros y ecuaciones en un sistema
adimensional de unidades, se simplifican y comprenden mucho mejor los clculos y condiciones
de operacin de la mquina. Adems, en por unidad los parmetros del circuito equivalente
varan levemente con el nivel de potencia y tamao de la mquina, diferencindose una de otra
principalmente por sus caractersticas constructivas. Resulta ventajoso indicar cuntas veces es
mayor la corriente de arranque con respecto a la corriente nominal, que utilizar directamente la
informacin en unidades fsicas.
Para definir las bases del sistema en por unidad de un sistema elctrico es necesario especificar
la potencia base y la tensin base. En los transformadores, es necesario definir una tensin base
en un lado del transformador y utilizar la relacin del nmero de vueltas del equipo para definir
la base de tensin del otro lado. Los transformadores, las lneas de transmisin y las grandes
mquinas poseen rendimientos muy altos. Estos elementos del sistema no tienen diferencias
importantes entre las potencias de entrada y salida. Las mquinas de induccin utilizadas industrialmente tienen un rendimiento menor y por tanto existen diferencias entre la potencia de
entrada y salida. Por esta razn hay que definir cul de estas potencias es ms conveniente. Esto
por supuesto, depende de la aplicacin y del enfoque preferido por el analista. En general, entre
las infinitas posibilidades existentes son tres las potencias base ms utilizadas: la potencia aparente nominal del estator SB = Sn , la potencia activa nominal del estator SB = Pen = Sn cos n y
la potencia mecnica nominal en el eje mecnico de la mquina SB = Pe jen .
La seleccin de la tensin base presenta menos problemas en su especificacin; se utiliza habitualmente como base la tensin nominal lnea a lnea, especificada en los datos de placa de la
mquina VB = Vn . Las dems bases deben calcularse partiendo de estas dos definiciones SB y VB .
Una vez seleccionada la potencia base y la tensin base en cada una de las tres alternativas, se
tienen las siguientes bases derivadas:

1. SB = Sn y VB = Vn : en este caso la corriente base IB debe calcularse a partir de la definicin


de potencia aparente en un sistema trifsico balanceado:
SB =

SB
3 VB IB IB =
3VB

(6.54)

La impedancia base del sistema ZB se calcula monofsicamente debido a que el circuito


equivalente representa una fase de la mquina: de esta forma, a partir de la tensin base

224

VI.9 Sistema en por unidad


VB y la corriente base IB , se obtiene:
ZB =

VB

IB

VB

3
SB
3VB

VB2
SB

(6.55)

Segn este sistema de bases, la tensin, corriente del estator y potencia aparente sern
1,0 pu cuando la mquina est operando en el punto nominal. La potencia activa en el
estator tendr el mismo valor del factor de potencia nominal. La potencia en el eje resulta
el producto del factor de potencia nominal por el rendimiento del punto nominal de operacin. Cuando se desea controlar que la corriente del estator no exceda el valor nominal,
este sistema es prctico.

2. SB = Pen y VB = Vn : las expresiones 6.54 y 6.55 determinan la base de las corrientes e impedancias del sistema. Cuando la mquina se encuentra en su punto de operacin nominal,
la tensin y la potencia activa del estator son 1,0 pu respectivamente. La potencia aparente
y la corriente del estator en por unidad son iguales al inverso del factor de potencia nominal. En este sistema la potencia mecnica en el eje en por unidad es igual al rendimiento
del punto nominal. Como la potencia activa nominal en el estator no es una limitacin
operativa de la mquina, este sistema no tiene mucha utilidad prctica.

3. SB = Pe jen y VB = Vn : igual que en los dos sistemas anteriores de bases, las expresiones
6.54 y 6.55 determinan la base de las corrientes e impedancias del sistema. Cuando la
mquina se encuentra operando en su punto de operacin nominal, la tensin y potencia en
el eje del rotor son 1,0 pu respectivamente. La potencia aparente y la corriente del estator
en las condiciones nominales son iguales al producto del inverso del factor de potencia
nominal por el rendimiento en el punto nominal. Este sistema tiene utilidad cuando se
desea analizar la potencia de accionamiento de la carga mecnica.

Los sistemas electromecnicos necesitan adems del clculo de potencias, tensiones, corrientes
e impedancias, el clculo de pares y velocidades. Como el par y la velocidad estn relacionados
por la potencia, es necesario definir una base adicional. En general se escoge la velocidad angular
sincrnica del campo magntico rotatorio como base y de esta forma queda determinado el par
base:
SB
SB
SB
PB = TB B TB =
=
=
(6.56)
B e 2 fe
Si la mquina posee ms de un par de polos, el par base se calcula como el par definido en la
ecuacin 6.56, dividido por el nmero de pares de polos p. Si la potencia base es la potencia del
eje mecnico, el par para la condicin de operacin nominal es 1.0 pu. Cuando se define como
base la potencia aparente de entrada, el par es igual al producto del rendimiento nominal por el
factor de potencia nominal. Si la base de potencia es la potencia activa nominal del estator, en el
punto de operacin nominal el par es igual al rendimiento de la mquina en ese punto.

225

Captulo VI La mquina de induccin


VI.10

D ETERMINACIN

DE LOS PARMETROS

El circuito equivalente de la mquina de induccin est definido por seis parmetros o elementos circuitales25 , tres resistencias que modelan las prdidas y tres reactancias que representan
los flujos de dispersin y magnetizacin de la mquina. El circuito equivalente de la mquina de
induccin es semejante al de un transformador trifsico y por lo tanto la metodologa utilizada
en la determinacin de los parmetros de este circuito puede ser utilizada en este caso con ciertas adaptaciones. Estas variaciones se deben fundamentalmente a la presencia del entrehierro.
En los transformadores, la corriente de magnetizacin es muy pequea en comparacin con la
corriente nominal; por esta razn se puede despreciar esta rama cuando se desea identificar las
reactancias de dispersin. En la mquina de induccin esta aproximacin es ms difcil de sostener. Por otra parte, en los transformadores generalmente se tiene acceso a los circuitos primario
y secundario26 . En la mayora de las mquinas de induccin este acceso no es posible, debido a
que el rotor est en cortocircuito.
Para identificar los parmetros de un transformador se realizan los ensayos de vaco y cortocircuito. El primero con la finalidad de obtener la reactancia y resistencia de magnetizacin y
el segundo para determinar las reactancias de dispersin y resistencias de los conductores. La
separacin de la resistencia del primario y secundario se puede realizar midiendo la cada de
tensin al inyectar corriente continua por una de sus bobinas. La separacin entre las reactancias
de dispersin se obtiene repartiendo proporcionalmente a la reactancia de dispersin total, la
reluctancia del camino magntico en cada bobina. Esto conduce a que en por unidad, las dos
reactancias de dispersin del modelo T del transformador son aproximadamente iguales y en
valores fsicos difieren en la relacin de vueltas al cuadrado. En la mquina de induccin no
sucede lo mismo porque las ranuras y los caminos magnticos de las bobinas del estator y rotor
pueden ser diferentes.
En la mquina de induccin tambin se pueden realizar estos ensayos; a continuacin se describen los ms importantes:
1. Ensayo de vaco: en esta prueba se hace girar la mquina a velocidad sincrnica, preferiblemente por un accionamiento externo. De esta forma el deslizamiento es cero y por
el circuito del rotor no circulan corrientes. La mquina se alimenta a frecuencia y tensin
nominal en el estator. Se miden con la mayor precisin posible las corrientes por las fases,
tensiones de lnea y potencia activa de entrada. Como el circuito es fuertemente inductivo,
durante el ensayo es conveniente utilizar vatmetros especiales para medir bajos factores
de potencia27 . En la figura se presenta el diagrama del montaje experimental requerido
para realizar en ensayo de vaco.
La tensin en la rama de magnetizacin es aproximadamente igual a la tensin de alimentacin, debido a que las corrientes de magnetizacin, aun cuando se encuentran entre
25
26
27

Esto es vlido tanto para el modelo transitorio como para el de rgimen permanente.
En algunas ocasiones esto no es posible, tambin el terciario de algunos transformadores puede no ser accesible
a la medicin.
Estos instrumentos son vatmetros normales que permiten una deflexin de la aguja unas cinco veces mayor que
la de un vatmetro convencional para la misma potencia. Tambin es posible utilizar instrumentos digitales que
no tienen las limitaciones de los electrodinmicos para realizar este tipo de medicin.

226

VI.10 Determinacin de los parmetros

Figura 6.14 Diagrama esquemtico del montaje para realizar el ensayo de vaco
I0
Zm
Xm
Rm

0,2 0,33
2,0 3,0
2,0 3,0
50 100

pu
pu
pu
pu

Tabla 6.1 Magnitudes habituales de los parmetros de magnetizacin de la mquina de induccin


una tercera parte y la mitad de la corriente nominal, no producen una cada significativa en la rama serie del modelo. Con esta simplificacin, la resistencia y reactancia de
magnetizacin se obtienen mediante los siguientes clculos:

S0 = 3V0 I0
(6.57)
P0 = P1 + P2
q
Q0 = S02 P02

V02
V2
; Xm 0
P0
P0
Los rdenes de magnitud habituales se han resumido en la Tabla 6.1.
Rm

(6.58)
(6.59)
(6.60)

2. Prueba de rotor bloqueado: para realizar este ensayo es necesario bloquear el rotor de la
mquina de induccin. Cuando el rotor est detenido, el deslizamiento es 1,0. El circuito
equivalente en estas condiciones es semejante al de un transformador en cortocircuito 28 .
En la identificacin de parmetros del transformador se puede despreciar la rama de magnetizacin, porque la corriente de cortocircuito es mucho mayor que la corriente de magnetizacin. La tensin de la rama de magnetizacin se deprime prcticamente a la mitad
de la tensin de vaco y esto reduce an ms la corriente que circula por ella. En el transformador, la influencia de la rama de magnetizacin durante el ensayo es prcticamente
despreciable. En la mquina de induccin la corriente de rotor bloqueado puede alcanzar
28

Por esta razn algunas veces se denomina incorrectamente a este ensayo como prueba de cortocircuito.

227

Captulo VI La mquina de induccin


entre tres y seis veces la corriente nominal. La corriente de vaco est comprendida entre
la tercera parte y la mitad de la corriente nominal. Durante la prueba de rotor bloqueado la
tensin de la rama de magnetizacin se deprime ms o menos a la mitad y por esta razn
la corriente de la mquina durante este ensayo puede alcanzar a ser entre seis y dieciocho
veces mayor que la corriente de magnetizacin. Desde un punto de vista prctico es posible despreciar esta rama en la estimacin de los parmetros, sin embargo la aproximacin
no es tan precisa como cuando se aplica en el ensayo de cortocircuito de un transformador.
El esquema de medida es similar al ilustrado en la figura 6.14, pero en lugar de hacer girar la mquina de induccin a velocidad sincrnica, es necesario bloquear mecnicamente
el rotor. Como el circuito equivalente en este ensayo es muy inductivo, deben utilizarse
vatmetros de bajo factor de potencia o digitales para mejorar la precisin de la medida.
En la prctica este ensayo no se realiza a valores nominales de tensin, para evitar un
calentamiento excesivo de los devanados debido al incremento de las prdidas con el cuadrado de la corriente y a la falta de ventilacin por estar detenido el rotor. Por otra parte,
hay que utilizar una tensin suficientemente grande que garantice la linealidad del circuito
magntico.
Aun cuando el ensayo a rotor bloqueado se realice con cierta rapidez, la resistencia de
las bobinas cambia apreciablemente con la temperatura y es preciso corregir las medidas.
Para este fin se miden las resistencias del estator cuando la mquina est a temperatura
ambiente, antes de comenzar el ensayo. Esta medida se realiza inyectando corriente continua en la bobina y midiendo la cada de tensin. La corriente inyectada debe ser menor a
un dcimo de la corriente nominal para que el calentamiento sea despreciable. Posteriormente se efecta el ensayo a rotor bloqueado e inmediatamente despus de terminar estas
medidas, se realiza una nueva medida de las resistencias del estator mediante el mismo
mtodo descrito. Las dos medidas de resistencia y el conocimiento del material utilizado
en el bobinado de la mquina permiten deducir la temperatura alcanzada por la mquina
durante el ensayo. Si la mquina est bobinada con cobre recocido en fro, la ecuacin que
determina la variacin de la resistencia en funcin de las temperaturas es la siguiente:
RT 2 234,5 + T2 (C)
=
RT 1 234,5 + T1 (C)

(6.61)

Para determinar los parmetros de la rama serie del circuito equivalente de la mquina,
midiendo potencia, tensin y corriente se utiliza el siguiente procedimiento:

Srb = 3Irb Vrb


(6.62)
q
2 P2
(6.63)
Qrb = Srb
rb
RT Re + Rr =

Prb
Prb
, X T X e + X r = 2
2
3Irb
3Irb

(6.64)

Las resistencias se pueden corregir desde la temperatura de la prueba, a la temperatura


nominal de operacin. Como adems se conoce la resistencia del estator por una medida

228

VI.10 Determinacin de los parmetros


Irb
Re
Rr
X e
X r

3, 0 6, 0
0, 01 0, 03
0, 01 0, 08
0, 07 0, 2
0, 07 0, 2

pu
pu
pu
pu
pu

Tabla 6.2 rdenes de magnitud habituales de los parmetros de rotor bloqueado de la mquina
de induccin
directa, la resistencia del rotor referida al estator se calcula por diferencia:
Rr = RT Re

(6.65)

Con las medidas realizadas, no es posible llevar a cabo una separacin de las reactancias
de fuga del estator y rotor; la prctica ms habitual consiste en dividirlas por igual en
las dos ramas. Sin embargo es necesario recordar que los caminos de fuga del estator y
del rotor son diferentes y dependen principalmente de la forma de la ranura, y esta forma
puede diferir mucho. Los rdenes de magnitud habituales en este ensayo se han resumido
en el cuadro 6.2.

Los ensayos de vaco y rotor bloqueado son una tcnica relativamente simple para determinar
los parmetros del circuito equivalente de la mquina de induccin. Este procedimiento es una
adaptacin del mtodo de clculo de parmetros en transformadores. En estos ensayos se realiza
la medida de la impedancia equivalente de la mquina en dos condiciones de operacin diferentes, deslizamiento: cero y uno. Tambin se mide directamente la resistencia del estator y una vez
conocida sta, slo resta por determinar los otros cinco parmetros. Cada uno de los ensayos
permite establecer dos ecuaciones, una para la parte real y otra para la parte imaginaria de la
impedancia de entrada. En total se dispone de cuatro ecuaciones para la determinacin de cinco
parmetros.
El problema matemtico est indeterminado. La solucin obtenida con tan escasa informacin,
adems de utilizar simplificaciones ms o menos razonables, debe considerar una separacin artificial de las reactancias de dispersin. Este problema se resuelve aplicando ensayos adicionales
a diferentes deslizamientos. Si se hacen varios ensayos, se obtiene un sistema con un mayor nmero de ecuaciones29 . Como los parmetros que se estn determinando son siempre cinco, se
tienen ms ecuaciones que incgnitas. El sistema de ecuaciones obtenido est sobredeterminado. Las medidas realizadas en los ensayos incluyen errores de apreciacin del observador y de
precisin en los instrumentos. Adems, los parmetros de la mquina varan en la prctica dependiendo de variables tales como el grado de saturacin, la temperatura y el efecto pelicular,
entre otras. En esta situacin resulta de gran utilidad la tcnica de estimacin paramtrica por
el mtodo de los mnimos cuadrados.
Del circuito equivalente de la mquina de induccin se puede determinar la impedancia de entrada en funcin de los parmetros de la mquina, la frecuencia de alimentacin y el deslizamiento.
29

Dos ecuaciones por cada medida.

229

Captulo VI La mquina de induccin


La impedancia de entrada vista desde el estator tiene la siguiente forma:
Zent (Re, L e , Rm , Lm , Rr , L r , s, e ) = Ze +
Donde:

Zm Zr
Zm + Zr

(6.66)

Ze = Re + je L e
Rr
Zr =
+ je L r
s
jm Lm Rm
Zm =
Rm + jm Lm

(6.67)
(6.68)
(6.69)

Utilizando el modelo de impedancia de entrada de la mquina, efectuando n ensayos con una


precisin i determinada y variando la velocidad del rotor m o la frecuencia de alimentacin
e , el problema que debe resolverse consiste en minimizar la funcin de costo :
 

n 
Zmed (si , ei ) Zcal (si , ei ) t
Zmed (si , ei ) Zcal (si , ei )

(6.70)
=
Z
(s
,
)
Z
(s
,
)

i
i
ei
i
i
ei
med
med
i=1
Donde:
Zmed (si , ei )
Zcal (si , ei )
si
ei
i
n

es la i-sima impedancia medida en los ensayos


es la i-sima impedancia calculada mediante el modelo
es el deslizamiento de la i-sima medida
es la frecuencia de la i-sima frecuencia de alimentacin
es el factor de ponderacin debido a la precisin de la medida i
es el nmero total de medidas

La ecuacin 6.70 se puede escribir matricialmente como:


= ft f

(6.71)

Donde:
ft =

f1 (X, si , ei ) f2 (X, si , ei ) fn (X, si , ei )

fi (x, si, ei ) =
X=

Re

Zmed (si , ei ) Zcal (X, si , ei )


i Zmed (si , ei )

X e Rm Xm Rr X r

(6.72)
(6.73)
(6.74)

Considerando que la ecuacin 6.71 no es lineal en el caso general, las derivadas primeras de la
funcin de costos con respecto a cada una de las variables de estado X del modelo se calculan
de la siguiente forma:



= G(X) = 2 [A(X)]t f(X)


(6.75)
X
230

VI.10 Determinacin de los parmetros


Donde la matriz A(X) es la matriz Jacobiana del vector de errores ponderados f(X):

f
f1
f1
1

X
X
Xm

 f21 f22
f2

Xm
1
2
A(X) =
=
.
.
.
.
..
..
..
X
.

.
fn
fn
fn

X
X
Xm
1

(6.76)

La matriz Jacobiana es de dimensin n m, donde n es el nmero de medidas y m el nmero


total de parmetros del modelo.
El incremento de los parmetros que minimiza la funcin de costos 6.71, utilizando el mtodo
de Gauss-Newton, se calcula mediante la siguiente expresin:

1
Xk = H(Xk )1 G(Xk ) 2 [A(Xk )]t A(Xk )
[A(Xk )]t f(Xk )

(6.77)

Xk+1 = Xk + Xk

(6.78)

El vector de parmetros del modelo en la iteracin k + 1 se calcula como:

Si en la iteracin k, el mdulo del vector Xk es menor que un cierto error especificado, el


problema converge al mnimo local ms cercano de la funcin de costos . Este mtodo presenta
ciertos problemas de convergencia, en particular cuando el peso de las segundas derivadas en la
matriz Hessiana es importante30. Para garantizar la convergencia del mtodo es recomendable
modificar la ecuacin 6.77 de la siguiente forma:
Xk+1 = Xk + Xk

(6.79)

Sustituyendo la ecuacin 6.79 en el vector de errores ponderados f(Xk+1 ) mediante la ecuacin


6.75 se puede obtener una funcin de costos para la iteracin k + 1 en funcin de las variables
de estado obtenidas en la iteracin k y el parmetro unidimensional :
(Xk+1 ) = (Xk + Xk ) = f(Xk + Xk )t f(Xk + Xk ) = ( )

(6.80)

Para obtener el nuevo vector de correccin Xk , se requiere determinar el valor del parmetro
que minimiza la funcin de costos.
Una vez obtenido el valor de las variables de estado que minimizan la funcin de costos en la
iteracin k + 1, se prosigue el clculo determinando una nueva direccin mediante la ecuacin
6.80 y un nuevo proceso de bsqueda del mnimo. Cuando el mdulo del vector de direccin
es inferior a la precisin requerida en los clculos, termina el proceso de minimizacin con la
mejor estimacin de los parmetros del modelo.
Uno de los inconvenientes que presenta el mtodo de Gauss-Newton modificado es la necesidad
de calcular un valor inicial de los parmetros. La funcin de costos puede tener mltiples
mnimos locales. La mejor solucin para el modelo es aquella que produce el menor de los
mnimos locales. Los valores iniciales de los parmetros pueden ser generados mediante una
30

Una aproximacin a la matriz Hessiana es: H At A.

231

Captulo VI La mquina de induccin

Figura 6.15 Diagrama de flujo del mtodo de minimizacin de Gauss-Newton


estimacin inicial que puede ser realizada mediante los ensayos de vaco y rotor bloqueado. De
cualquier forma, el mtodo de Gauss-Newton requiere arrancar con un valor inicial cercano a la
solucin para garantizar la convergencia a la solucin ptima. En la figura 6.15 se presenta el
diagrama de flujo del mtodo de minimizacin de Gauss-Newton.
Si se desea asegurar la convergencia del mtodo, es conveniente limitar la correccin mxima
Xk para que ninguno de los parmetros de la mquina definidos en el vector Xk pueda
aumentar o disminuir en ms de un 50 %. Esto puede reducir la velocidad del algoritmo, pero
asegura que los parmetros han de ser siempre positivos y evita divergencias debido a las no
linealidades del modelo.
El mtodo de Gauss-Newton es muy eficiente para la determinacin de los parmetros cuando
la funcin de costos se define por mnimos cuadrados. Otros mtodos de optimizacin no lineal
tambin pueden obtener soluciones con ms o menos dificultad. Como ejemplo se presenta listado 4 de un programa de estimacin de los parmetros de una mquina de induccin realizado en
el entorno de distribucin libre Scilab-3. Los parmetros del modelo de la mquina son previamente conocidos para permitir la comprobacin de esta tcnica. Con estos parmetros se evalan
las impedancias de entrada de la mquina para las condiciones de la prueba de vaco, carga y
rotor bloqueado mediante la funcin de costo que se presenta en el listado 4. Por el mtodo
convencional de los ensayos de vaco y rotor bloqueado se obtiene la estimacin inicial de los
parmetros x0 . Para encontrar el conjunto de parmetros de la mquina X que minimizan la fun-

232

VI.11 Condiciones de operacin


Parmetro
Re
X e
Rm
Xm
Rr
X r
(X)

Ensayos
0,0200
0,1200
48,000
3,3000
0,0276
0,1200
2,66977

Estimacin
0,0200
0,1006
49,8184
2,9994
0,0299
0,1493
6,2 106

Exacto
0,0200
0,1000
50,000
3,0000
0,0300
0,1500
3,5 108

Tabla 6.3 Comparacin entre el mtodo aproximado y la estimacin de parmetros


cin de costos se utiliza la rutina de optimizacin de funciones no lineales sin restricciones
optim.31 Para evitar la determinacin del gradiente G requerido por la rutina de optimizacin 32,
sta se invoca incluyendo el programa externo NDcost que determina la evaluacin numrica.
En el cuadro 6.3 se presenta una comparacin entre los resultados del clculo de los parmetros
de la mquina de induccin realizados mediante el mtodo de los ensayos de vaco y rotor
bloqueado con las estimacin paramtrica llevada a cabo mediante la minimizacin de la funcin
de costos. Aun cuando los resultados de la estimacin son ms cercanos a los valores exactos, el
clculo a partir de los ensayos, adems de ofrecer un punto de partida para el algoritmo, puede
ser utilizado directamente en muchas aplicaciones de la ingeniera elctrica.
VI.11

C ONDICIONES

DE OPERACIN

La mquina de induccin es un convertidor electromecnico de energa que puede operar como


motor, generador o freno. Estas tres condiciones o zonas de operacin se corresponden con rangos diferentes del deslizamiento. En la operacin como motor la mquina entrega par y potencia
en el eje mecnico, consumiendo potencia en el eje elctrico. En la condicin de generador ocurre la situacin inversa: se absorbe potencia y par del eje mecnico y se entrega potencia por
el eje elctrico. En la condicin de frenado ambos ejes introducen potencia al convertidor, la
cual es quemada en prdidas. A continuacin se presenta una descripcin ms detallada de estas
zonas de operacin de la mquina de induccin:
1. Motor: para que la potencia y el par en el eje sean positivos es necesario que la potencia
33
transferida a las resistencias Rsr y 1s
s Rr sean positivas :
1
1s
0 ;
0 0s1
s
s

(6.81)

En la zona de motorizacin, la carga es accionada por la mquina y se consume potencia


de la red.
31
32
33

La rutina optim del entorno Scilab puede manejar restricciones de frontera.


En este caso las no linealidades del modelo de impedancia de entrada de la mquina complican la evaluacin
analtica de la funcin gradiente.
La potencia transferida a Rsr determina el par elctrico y la transferida a 1s
s Rr define la potencia disponible en
el eje mecnico.

233

Captulo VI La mquina de induccin

Algoritmo 4 Estimacin de los parmetros de la mquina de induccin


//************************************************************
// Estima in de los parmetros de una mquina de indu in
// mediante la t ni a de los mnimos uadrados.
//************************************************************
//
// programa parmetros.
//
// Para este ejemplo se utiliz el ir uito equivalente para
// determinar la impedan ia de entrada para tres deslizamientos
// diferentes: va o(s=0), arga(s=0.03) y rotor bloqueado(s=1)
//
// Los parmetros del ir uito equivalente de esta mquina son:
//
Re = .02 p.u. Xe = .10 p.u.
//
Rm = 50. p.u. Xm = 3.0 p.u.
//
Xr = .15 p.u. Rr = .03 p.u.
//
// Los ensayos realizados dieron los siguientes resultados:
//
Zmedida(s=0)
= .199350+j3.0892 p.u.
//
Zmedida(s=0.03) = .833740+j.49141 p.u.
//
Zmedida(s=1)
= .047603+j.24296 p.u.
//
Re
= .02 p.u. (Medida dire ta)
//
// Utilizando el mtodo aproximado se onsiguen los siguientes
// valores de arranque.
//
Xeo = .12 p.u. Rmo = 48.0 p.u.
//
Xmo = 3.3 p.u. Xro =.12 p.u.
//
Rro = .0276 p.u.
//
// Estos valores se argan en el ve tor de arranque x0:
//
x0=[.12 48 3.3 .12 .0276';
//
// Finalmente se llama a la rutina optim que al ula los valores
// de los parmetros x que minimizan la fun in de osto.
//
[Psi,x,g = optim(list(ND ost, osto),x0);
//
// En el ve tor x se han argado los parmetros ptimos de la
// estima in. La solu in es:
//
Refin = 0.0
Xefin = x(1)
Rmfin = x(2)
Xmfin = x(3)
Rrfin = x(4)
Xrfin = x(5)
Psi

234

VI.11 Condiciones de operacin


Algoritmo 5 Funcin de costos para ser evaluada por la rutina optim
//************************************************************
//
fun tion Psi = osto(x)
//
//************************************************************
// Evalua in de la fun in de ostos por mnimos uadrados.
// Psi = Sumatoria(errores relativos)^2
//
// Deslizamientos orrespondientes a los ensayos de va o,
// arga y rotor bloqueado.
//
s = [1e-10 .03 1.';
//
Re = 0.02; // Medi in dire ta de la resisten ia estator
Xe = x(1); // Rea tan ia de dispersin del estator
Rm = x(2); // Resisten ia de magnetiza in
Xm = x(3); // Rea tan ia de magnetiza in
Rr = x(4); // Resisten ia del rotor referida al estator
Xr = x(5); // Rea tan ia dispersin rotor referida al estator
//
// Ve tor de las impedan ias de entrada medidas en los
// ensayos.
//
i= %i;j= %i;
Zmedida = [.1999350-3.0892*i
.833740-.49141*i
.047603-.24296*i';
//
// Evalua in de las impedan ias al uladas mediante la estima in
// de los parmetros del modelo.
//
Ze = Re+j*Xe;
// Impedan ia estator
Zm = (Rm*j*Xm)/(Rm+j*Xm);
// Impedan ia magnetiza in
Zth = Ze*Zm/(Ze+Zm)+j*Xr;
// Impedan ia de Thvenin
Ve = 1.00;
// Tensin del estator
Vth = Zm*Ve/(Zm+Ze);
// Tensin de Thevenin
Ir = Vth./(Zth+Rr./s);
// Corriente del rotor referida
Vm = Ir.*(Rr./s+j*Xr);
// Tensin rama magnetizante
Im = Vm./Zm;
// Corriente de magnetiza in
Ie = Im+Ir;
// Corriente del estator
Z al ulada=Ve./Ie;
// Impedan ia de entrada al ulada
//
// Cl ulo del error relativo entre las medidas y el modelo
//
err = (Zmedida-Z al ulada)./Zmedida;
//
// Cl ulo de la fun in de osto por mnimos uadrados
//
Psi = abs(err'*err);
//
endfun tion;

235

Captulo VI La mquina de induccin


2. Generador: la operacin como generador requiere que la mquina entregue potencia por
el estator. La energa entra por el eje mecnico, atraviesa el entrehierro y llega al estator.
En el circuito equivalente este fenmeno se obtiene cuando la resistencia de carga 1s
s Rr es
negativa. La potencia generada por esta resistencia proviene del accionamiento mecnico
externo. En este caso:
s0
(6.82)
Cuando el deslizamiento del rotor es negativo, la resistencia es negativa tambin. Un deslizamiento negativo implica que la velocidad del rotor es mayor que la velocidad sincrnica.
En estas condiciones el campo magntico rotatorio que se produce en el rotor adelanta al
campo magntico rotatorio del estator, el par elctrico se invierte de sentido y la potencia
fluye desde el rotor hacia el estator.
3. Freno: si la mquina gira en sentido contrario al del campo magntico rotatorio, el deslizamiento es mayor que uno:
s>1
(6.83)
Para esta condicin la resistencia de carga es negativa 1s
s Rr . Por otra parte, la potencia
Rr
transferida desde el estator al rotor que depende de s es positiva. En estas condiciones
la mquina consume potencia tanto de la fuente como del eje mecnico, y se disipa como
prdidas en las resistencias pasivas del circuito equivalente. En este caso la mquina utiliza potencia elctrica de la fuente para oponerse al sentido del movimiento. Estos puntos
de operacin pueden utilizarse para frenar un motor, consumiendo para este fin la energa cintica acumulada en la carga mecnica. Durante el funcionamiento como freno, la
mquina disipa internamente mucha energa y esto ocasiona un calentamiento importante,
por esta razn este tipo de operacin tan slo debe utilizarse durante cortos perodos de
tiempo. Para que la mquina de induccin opere en la condicin de freno, es necesario que
se invierta el sentido de giro del campo magntico rotatorio con respecto a la velocidad del
rotor. Esto se puede lograr invirtiendo la conexin de dos fases del estator, el sentido de
giro del campo se invierte y la mquina entra en la condicin de freno. El par elctrico que
produce la mquina tiene sentido contrario al movimiento del rotor y la carga mecnica
disminuye su velocidad. Cuando el rotor se detiene, se desconecta la mquina de la red y
culmina el proceso de frenado.
En la figura 6.16 se han representado las zonas de operacin correspondientes a la operacin
como generador, motor y freno respectivamente.

VI.12

C ARACTERSTICAS

NORMALIZADAS

El grfico par-deslizamiento de la mquina de induccin es una funcin que puede ser normalizada con respecto al par mximo y al deslizamiento correspondiente al par mximo. Esta
caracterstica normalizada tiene gran utilidad cuando se necesita determinar el comportamiento
de una mquina a la cual no se le conocen sus parmetros. Durante la etapa de diseo o especificacin de un accionamiento, este tipo de herramienta es de utilidad. La ecuacin 6.39 determina
el par elctrico en funcin del deslizamiento; el par mximo se obtiene sustituyendo en esta

236

VI.12 Caractersticas normalizadas

Figura 6.16 Zonas de operacin como generador, motor y freno de la mquina de induccin
expresin la condicin 6.42 para par mximo:
Temax =

3 Rr
2
e sTmax Vth

2
r
Rth + sTRmax
+ Xth2
3
e

2 + X2V2
Rth
th th
= 
=
q
2
2
2
2
Rth + Rth + Xth + Xth

3 2
e Vth

q


2 + X2
2 Rth + Rth
th

(6.84)

Dividiendo la caracterstica del par-deslizamiento 6.39 por el par mximo determinado en la


expresin 6.84, se obtiene:
q


2 + X2
R
2
R
+
th
th
th
Rr
Te
=
(6.85)


2
Tmax
s
Rth + Rsr + Xth2
Definiendo el factor de calidad Q34 de la mquina de induccin como el cociente entre la impedancia y la resistencia de Thvenin:
Xth
Q=
(6.86)
Rth

34

El factor de calidad es un parmetro utilizado frecuentemente en el diseo de filtros y est asociado con las
prdidas que tienen las inductancias y los condensadores.

237

Captulo VI La mquina de induccin

Figura 6.17 Par elctrico normalizado de la mquina de induccin


Introduciendo la definicin 6.86 y la condicin del par mximo 6.42 en la expresin del par
normalizado 6.85, se obtiene la siguiente relacin:
p
Te
1 + 1 + Q2

p
=
(6.87)
s
s
Tmax 1 + 1
+ Tmax
1 + Q2
2

sTmax

La expresin 6.87 determina el par elctrico de una mquina de induccin dado el par elctrico
mximo Tmax , el deslizamiento correspondiente al par mximo sTmax , el factor de calidad de
las bobinas Q y el deslizamiento correspondiente s. El factor de calidad de las bobinas Q es
un valor caracterstico de la mquina y vara en un rango estrecho, normalmente entre 3,0 y
10 aproximadamente. El deslizamiento correspondiente al par mximo sTmax tiene incidencia
directa sobre el rendimiento del punto nominal; cuanto menor es este deslizamiento, mayor es
el rendimiento. Sin embargo, una mquina con deslizamiento correspondiente al punto de par
mximo muy reducido, tambin produce un par de arranque pequeo. En la figura 6.17 se ha
representado la expresin 6.87 para varios valores tpicos del factor de calidad Q.
En algunas ocasiones la expresin 6.87 se particulariza considerando como aproximacin que el
factor de calidad Q de la mquina tiende a infinito. Esto es una buena aproximacin en mquinas
grandes, donde la resistencia de Thvenin Rth es muy pequea comparada con la reactancia de
dispersin Xth , que vara muy poco con el tamao o potencia de la mquina, cuando se expresa
en el sistema adimensional de unidades.35 La expresin que se obtiene cuando Q es:
Te
=
Tmax
35

Por unidad.

238

s
sTmax

2
s
+ Tmax
s

(6.88)

VI.13 Diagrama de crculo

Figura 6.18 Corriente normalizada del rotor de la mquina de induccin


Una expresin similar a la 6.87 se obtiene para la corriente del rotor cuando se normaliza por el
valor de la corriente y del deslizamiento correspondiente al par mximo:
v


p
u
2
u
2
1
+
1
+
Q
Ir
u
p

=t
(6.89)
sTmax 2
sTmax
IrT max
2
2 s + 1+ s
1+Q

Cuando el factor de calidad tiende a infinito Q , la expresin 6.89 tiende a:


s
Ir
2
=
2
s
IrT max
1 + Tmax

(6.90)

En la figura se muestra la corriente normalizada del rotor para varios valores tpicos del factor
de calidad Q.
VI.13

D IAGRAMA

DE CRCULO

El circuito equivalente de la mquina de induccin es una herramienta eficaz y eficiente para


la evaluacin de su comportamiento en cualquier punto de operacin. Hoy en da se dispone de
calculadoras y computadores de gran capacidad, potentes y rpidos que facilitan la aplicacin de
este modelo en el anlisis prctico y profesional de la mquina de induccin. Sin embargo, hace
algunos aos estas herramientas no existan o su costo no justificaba su aplicacin en clculos
rutinarios. Por este motivo se desarroll el mtodo del diagrama de crculo, como herramienta de
clculo geomtrico. Los lugares geomtricos reducen el nmero de operaciones aritmticas con
variables complejas y permiten visualizar en un solo grfico gran cantidad de informacin sobre
el comportamiento de la mquina de induccin en todo su rango operativo. En la actualidad
se han eliminado completamente las dificultades de clculo que existan en el pasado y podra

239

Captulo VI La mquina de induccin


parecer innecesario el uso del diagrama de crculo. Sin embargo, la posibilidad de visualizar en el
mismo grfico todas las corrientes posibles, as como sus correspondientes deslizamientos, pares
y potencias, ofrece a esta herramienta un respetable inters para incrementar la comprensin del
modelo de la mquina y las relaciones causa-efecto existentes entre las diferentes variables.
El diagrama de crculo permite el anlisis cuantitativo del comportamiento de la mquina de
induccin, interpretando racionalmente sus principales caractersticas y como stas se ven afectadas por variaciones en los parmetros o en la fuente de alimentacin.
El diagrama de crculo de la mquina de induccin es el lugar geomtrico del fasor corriente del
estator Ie , utilizando como parmetro el deslizamiento s. En estricta teora, el lugar geomtrico
de la corriente del estator no es un crculo exacto, aun cuando la diferencia con esta figura es muy
reducida en la prctica. Por otra parte, el lugar geomtrico de la corriente del rotor Ir representa
un crculo matemticamente preciso36 , cuando se representa en el plano este fasor para todos los
posibles deslizamientos de la mquina de induccin.
A partir del equivalente de Thvenin desarrollado en la figura 6.9, se obtiene la corriente del
rotor Ir como:
Ir =

Vth
Vth
Vth
Vth
=
=
sen (r (s)) r (s)
=
Zth + Rsr
Rth + Rsr + jXth |Z(s)| r (s) Xth

(6.91)

La expresin 6.91 se expresa en coordenadas cartesianas de la siguiente forma:


Ir = IrRe + j IrIm =


Vth 
sen (r (s)) cos (r (s)) jsen2 (r (s))
Xth

(6.92)

Multiplicando la corriente Ir por su conjugada se obtiene:


|Ir |2 = Ir2Re + Ir2Im =

Vth2
sen2 (r (s))
Xth2

(6.93)

De la parte imaginaria de la expresin 6.92 se deduce:


sen2 (r (s)) =

IrIm Xth
Vth

(6.94)

Sustituyendo este resultado en la ecuacin 6.93 y completando los cuadrados correspondientes,


se obtiene:




Vth 2
Vth 2
2
=
(6.95)
IrRe + IrIm +
2Xth
2Xth


Vth
y cuyo radio vale
La expresin 6.95 es la ecuacin de un crculo centrado en el punto 0, 2X
th
Vth
2Xth , tal como se puede observar en la figura

6.19. En el origen de coordenadas de este diagrama,


el deslizamiento de la mquina de induccin corresponde a la condicin de vaco s = 0. Para este

36

Considerando como vlidas las hiptesis simplificadoras incluidas en el modelo del circuito equivalente de la
mquina de induccin.

240

VI.13 Diagrama de crculo

Figura 6.19 Lugar geomtrico de la corriente del rotor


deslizamiento el modelo de carga resistiva que representa la potencia transferida al eje tiende a
infinito y la corriente que circula por el rotor es cero.
El punto diametralmente opuesto al de vaco en la figura 6.19 corresponde a la mxima corriente
del rotor, que se obtiene cuando la mquina opera en un deslizamiento para el cual la parte
resistiva de la impedancia es cero. En esta condicin la impedancia es igual a la reactancia de
Thvenin Xth :
Rr
Rr
(6.96)
Z(Irmax ) = Rth + + jXth = jXth sIr max =
s
Rth
En el deslizamiento correspondiente al rotor bloqueado s = 1, la reactancia de Thvenin es mayor que la suma de la resistencia de Thvenin y la resistencia de la carga. En este deslizamiento,
la corriente del rotor referida al estator es en magnitud muy parecida a la corriente mxima y su
ngulo tambin es cercano pero inferior a 90.
Para los deslizamientos positivos, el fasor corriente en la figura 6.19 debe estar en el cuarto
cuadrante del diagrama. En este cuadrante las potencias activas y reactivas consumidas por la
mquina son positivas. En el tercer cuadrante, la potencia reactiva es negativa, pero la potencia
activa es positiva. Todos los puntos de operacin del lugar geomtrico de la corriente del rotor

241

Captulo VI La mquina de induccin

Figura 6.20 Representacin de las potencias activas, reactivas y aparentes en el diagrama de


crculo
consumen potencia reactiva inductiva, debido a la necesidad de alimentar desde el estator las
fuerzas magnetomotrices de la mquina.
La potencia activa o reactiva en el diagrama de crculo se puede determinar trazando un segmento paralelo al eje real o imaginario respectivamente. Estos segmentos nacen en el corte con los
ejes coordenados ortogonales y finalizan en el punto de operacin deseado. Estos segmentos son
proporcionales a cada una de las potencias activa o reactiva, y la constante de proporcionalidad
que permite el clculo cuantitativo es el valor de la tensin de Thvenin. La potencia aparente,
por otra parte, es proporcional al mdulo de la corriente del rotor referido al estator. De acuerdo
con los puntos A, B y C definidos en la figura 6.20, las potencias aparente, activa y reactiva para
un punto cualquiera del crculo se obtienen de la siguiente forma:

(6.97)
SR = 3Vth Ir = 3Vth AB AB

PR = 3Vth Ir cosr = 3Vth BC BC


(6.98)

QR = 3Vth Ir senr = 3Vth AC AC


(6.99)
Las expresiones 6.97, 6.98 y 6.99 permiten calcular la potencia activa PR , reactiva QR o aparente
SR de cualquier punto de operacin de la mquina de induccin, midiendo la longitud de los
segmentos correspondientes AB, BC o AC.
El segmento BC de la figura 6.20 es proporcional a la potencia que entra a la mquina para el
deslizamiento del rotor bloqueado s = 1. En esta condicin de operacin, toda la potencia que

242

VI.13 Diagrama de crculo

Figura 6.21 Balance de potencias en el diagrama de crculo

atraviesa el entrehierro se disipa en la resistencia del rotor Rr y en la resistencia de Thvenin Rth .


En cualquier otro punto de operacin, el segmento es proporcional a la suma de las potencias
disipadas en la resistencia de Thvenin Rth , en la resistencia del rotor Rr y en la resistencia
equivalente de la carga 1s
s Rr . De este anlisis se pueden obtener las siguientes expresiones:
PRr = 3Ir2 Rr

(6.100)

Pth = 3Ir2 Rth

(6.101)

Pe je = 3Ir2Rr

1s
s

(6.102)

De las expresiones 6.100, 6.101 y 6.102 se pueden deducir las siguientes relaciones:
Rr
PRr
=
Pth
Rth
Pe je
Rr 1 s
=
Pth
Rth s
Pe je 1 s
=
PRr
s

(6.103)
(6.104)
(6.105)

Las relaciones anteriores indican que las potencias se encuentran entre s en la misma proporcin que las resistencias donde disipan. Estas prdidas pueden ser representadas en el diagrama
de crculo, para lo cual hay que determinar el punto correspondiente al deslizamiento infinito
s = . En este punto, toda la potencia se disipa en la resistencia de Thvenin. Con el rotor
bloqueado s = 1, la potencia se reparte entre la resistencia del rotor y la resistencia de Thvenin
en la proporcin de estas resistencias, segn se demuestra en la ecuacin 6.103.

243

Captulo VI La mquina de induccin


De la semejanza de los tringulos OBC y OEG en la figura 6.21 se establecen las siguientes
proporciones:
BD OD OA sen (r (sx ))
=
=

(6.106)
EG OG OE sen (r (s = 1))
OA = OJ sen (r (sx ))

OE = OJ sen (r (s = 1))

(6.107)
(6.108)

Sustituyendo las relaciones 6.107 y 6.108 en la proporcin 6.106, reemplazando cada segmento
que representa una corriente en el diagrama por su correspondiente valor y multiplicando el
numerador y el denominador por la resistencia de Thvenin ms la resistencia del rotor Rth + Rr ,
se obtiene el siguiente resultado:
BD
=
EG

OA
OE

2

Pth+Rr (sx )
Ir2(sx ) (Rth + Rr )
=
2
Ir (s = 1) (Rth + Rr ) Pth+Rr (s = 1)

(6.109)

Mediante la expresin 6.109 se demuestra que la proporcin entre los segmentos BD y EG del
diagrama de crculo es igual a la relacin entre las potencias de prdidas en los conductores de
la mquina en las dos condiciones de operacin indicadas. En la condicin del rotor bloqueado,
toda la potencia se consume en prdidas en la resistencia de Thvenin y en la resistencia del
rotor. En cualquier otro deslizamiento, el segmento BD es proporcional a estas mismas prdidas.
De igual forma se demuestra que el segmento CD es proporcional a las prdidas en la resistencia
de Thvenin. La proporcionalidad de los diferentes segmentos del diagrama de crculo para un
deslizamiento determinado es:
AD
CD
BC
BD
AB
AC

a la potencia de entrada Pe
a las prdidas en Rth
a las prdidas en Rr
a las prdidas totales Pptot.
a la potencia en el eje Pe je
a la potencia PRr y al par elctrico Te
s

La recta que pasa por el origen de coordenadas y por el punto s = 1 se denomina eje de la potencia mecnica, debido a que cualquier segmento paralelo al eje real con un extremo en un punto
de operacin localizado sobre el crculo y su otro extremo sobre este eje, es proporcional a la
potencia en el eje de la mquina. De igual forma, la recta que pasa por el origen de coordenadas
y por el punto correspondiente al deslizamiento s = se denomina eje del par elctrico.

Para determinar el deslizamiento de un punto del diagrama de crculo es posible utilizar diferentes metodologas. La forma ms simple puede ser despejar el deslizamiento de la expresin
6.105:
BC
BC
PRr (sx )
sx =
=
=
(6.110)
Pe je(sx ) + PRr (sx ) AB + BC AC
Este mtodo tiene dos inconvenientes: por una parte la dificultad prctica para medir deslizamientos cercanos a cero37 , y por otra el procedimiento es poco grfico debido a que requiere
37

En este caso el segmento es muy pequeo y la precisin de la medida es reducida.

244

VI.13 Diagrama de crculo


operaciones aritmticas para determinar el deslizamiento sx en cada punto de operacin38 . Un
mtodo prctico para resolver estos problemas consiste en trazar la recta del deslizamiento. Este
mtodo se fundamenta en trazar en el diagrama de crculo una recta paralela al eje del par o eje
del deslizamiento infinito s = . Esta recta se traza a una distancia arbitraria de este eje, pero
con la condicin de que el eje de la potencia mecnica s = 1 la corte en un punto dentro del
rea de trabajo. Esta construccin se muestra en la figura 6.22. La proporcin entre el segmento
AB y AJ determina el deslizamiento del punto de operacin considerado. Si al segmento se le
asigna un valor unitario, automticamente indica el valor del deslizamiento. Para demostrar esta
aseveracin se utiliza la semejanza existente entre los tringulos OCE y OAB de la figura
6.22. Estos tringulos son semejantes y por este motivo:
CE
OA
=
EO AB

(6.111)

Los tringulos ODE y OAJ tambin son semejantes y se puede establecer la siguiente
proporcin:
DE OA
=
(6.112)
EO
JA
Dividiendo la relacin de segmentos 6.112 por la 6.111 se obtiene el siguiente resultado:
PRr (sx )
DE AB
=
=
= sx
CE
JA Pe je (sx ) + PRr (sx )

(6.113)

La recta del deslizamiento se normaliza con respecto al segmento y posteriormente se calibra.


Si se desea conocer el deslizamiento de un punto cualquiera del diagrama, es suficiente con
trazar una recta que una el origen de coordenadas con el punto de inters sobre el crculo. La
interseccin de esta recta auxiliar sobre la recta del deslizamiento, previamente calibrada, indica
directamente el valor del deslizamiento sx del punto de operacin.
Los pares y las potencias mecnicas se obtienen a partir del diagrama, trazando rectas paralelas
al eje real que comienzan en el punto de operacin y culminan en los ejes del par elctrico
s = o de la potencia mecnica s = 1. Para determinar el punto del diagrama de crculo donde
se obtiene el par o la potencia mecnica mxima, es necesario encontrar las rectas tangentes al
crculo y paralelas a los ejes de par o potencia respectivamente. En la figura 6.23 se presenta el
procedimiento de clculo grfico para la determinacin de estos puntos.
El diagrama de crculo contiene toda la informacin referente a los modos de operacin como
motor, generador y freno. La idea fundamental que permite utilizar la informacin del diagrama
de crculo en todo el rango de deslizamiento, consiste en que adems de medir la longitud de los
segmentos, se debe interpretar su signo. Si un segmento representa potencia elctrica de entrada
y est por debajo del eje imaginario, esta potencia es negativa y la mquina entrega potencia
elctrica a la red. Si al determinar un deslizamiento, el punto aparece a la izquierda del origen,
es una indicacin de que la mquina opera con deslizamiento negativo. En la figura 6.20 se han
indicado las zonas de operacin en el diagrama de crculo.
38

Esto reduce las ventajas del diagrama de crculo como calculador geomtrico.

245

Captulo VI La mquina de induccin

Figura 6.22 Construccin de la recta del deslizamiento

Figura 6.23 Determinacin de los puntos de par mximo y potencia mxima con el diagrama
de crculo

246

VI.13 Diagrama de crculo

Figura 6.24 Diagrama de crculo de la corriente del estator y su modelo equivalente


El diagrama de crculo obtenido representa solamente el lugar geomtrico de las corrientes del
rotor. Esta corriente suministra una gran cantidad de informacin sobre la operacin de la mquina pero no es accesible para ser medida directamente. La mquina se alimenta por el estator,
y esta corriente es la que el usuario puede medir. Para obtener el lugar geomtrico de la corriente del estator Ie (s) es preciso sumar a la corriente del rotor Ir (s) la corriente de magnetizacin
Im (s). Este lugar geomtrico es un crculo aproximado, debido a que aun cuando la corriente de
magnetizacin depende del deslizamiento, es prcticamente constante para un rango muy amplio de deslizamientos. De esta forma para la construccin del diagrama de crculo aproximado
se puede utilizar un circuito equivalente en el cual la corriente del rotor se modela mediante
el equivalente de Thvenin que se ha venido utilizando, pero que en paralelo con la tensin de
Thvenin se aade la rama de magnetizacin. En la figura 6.24 se muestra este circuito y el
diagrama de crculo que se obtiene cuando se suman la corriente de magnetizacin y la corriente
del rotor. En esta figura el segmento FG representa las prdidas en el hierro de la mquina. Es
necesario recordar que los equivalentes de Thvenin no son conservativos en potencia porque
sta variable no es una caracterstica lineal del circuito. Por esta razn la resistencia de Thvenin
no incluye la informacin sobre las prdidas en el hierro. Este circuito tiene un comportamiento
similar al del modelo clsico en lo que respecta a las potencias.
Cuando se coloca la rama de magnetizacin delante de la resistencia y reactancia de dispersin
del estator se obtiene un circuito equivalente aproximado, semejante al circuito de Thvenin

247

Captulo VI La mquina de induccin

Figura 6.25 Circuito de Thvenin aproximado de la mquina de induccin

presentado en la figura 6.24. Este circuito se puede determinar de una forma ms simple y los
resultados obtenidos con l no difieren significativamente del modelo clsico de la mquina. El
circuito aproximado simplifica el clculo de la tensin e impedancia de Thvenin. En su lugar se
utiliza directamente la tensin de alimentacin, as como la resistencia y reactancia de dispersin
de la mquina. En la figura 6.25 se ha representado el circuito de Thvenin aproximado de la
mquina.
En la figura 6.26 se presenta una comparacin entre los lugares geomtricos de la corriente del
estator utilizando el circuito de Thvenin, el circuito de Thvenin aproximado y el modelo clsico de la mquina de induccin. Se debe recordar que en el modelo clsico la corriente del estator
no es un crculo exacto, pero se aproxima muy estrechamente para casi todos los deslizamientos.
En esta figura se observa que para deslizamientos cercanos al punto nominal, la correspondencia
entre los dos circuitos es prcticamente perfecta. Sin embargo, cuando los deslizamientos son
grandes, los errores se acrecientan. Esto se debe a que a valores de deslizamiento grandes, la
corriente resulta ser varias veces su valor nominal y produce cadas importantes en las reactancias de dispersin. Por esta razn, la tensin en la rama magnetizante es menor y la corriente de
magnetizacin tambin es ms pequea. El circuito equivalente reducido por Thvenin considera que la tensin Vth alimenta a la rama de magnetizacin para todos los deslizamientos y por
ello este modelo determina corrientes ligeramente mayores al compararse con el circuito clsico.
La diferencia entre estos diagramas no excede en la prctica la mitad de la corriente de vaco de
la mquina, debido a que la reactancias de dispersin del estator y rotor son aproximadamente
iguales. Con estas corrientes circulando por las bobinas del estator y rotor de la mquina, la
tensin en la rama de magnetizacin se deprime aproximadamente a la mitad y debido a esto la
corriente de magnetizacin se reduce en la misma proporcin.
La construccin del diagrama de crculo completo de la mquina de induccin se puede obtener
a partir del conocimiento de los parmetros del circuito equivalente o mediante ensayos sobre
la mquina en diferentes condiciones de operacin. Geomtrica y analticamente, un crculo
est definido por la posicin de su centro y la longitud de su radio. Tambin se puede trazar un
crculo a partir de tres puntos localizados sobre el crculo; en este caso es suficiente con trazar
las bisectrices entre dos de estos puntos y la interseccin de estas bisectrices es el centro del
crculo. El radio se obtiene midiendo la distancia desde el centro del crculo a uno de los tres
puntos iniciales. Otra posibilidad para trazar el diagrama consiste en conocer dos puntos del
crculo y una recta que pase a travs de su dimetro.

248

VI.13 Diagrama de crculo

Figura 6.26 Comparacin entre los diagramas de crculo utilizando el circuito Thvenin, el circuito de Thvenin aproximado y el circuito equivalente completo

249

Captulo VI La mquina de induccin


Para trazar el diagrama de crculo de la mquina de induccin a partir del circuito equivalente,
se puede utilizar el siguiente procedimiento:
1. Se traza un crculo de dimetro VXthth y se escoge una escala de corriente que permita que el
crculo pueda ser representado en las dimensiones del papel disponible.
2. Se calcula el factor de potencia de la corriente del rotor para la condicin de rotor bloqueado cos (r (s = 1)) y se traza con este ngulo el eje de la potencia mecnica. Es conveniente
medir este ngulo mediante relaciones triangulares y no con transportadores que producen
errores de medida importantes.
3. Dividiendo el segmento comprendido entre el punto del diagrama s = 1, paralelo al eje
real y que finaliza en el eje imaginario, en la proporcin existente entre la resistencia de
Thvenin Rth y la resistencia del rotor Rr , se traza el eje del par elctrico uniendo este
punto con el origen de coordenadas.
4. Se traza la recta del deslizamiento paralela al eje del par elctrico, de tal forma que intercepte al eje de la potencia mecnica dentro de la hoja de papel. Posteriormente se calibra
esta recta dividindola en tantas partes iguales como sea necesario.
5. Se suma al diagrama anterior la corriente de magnetizacin Im , determinando de esta
forma el lugar geomtrico de las corrientes del estator Ie .
6. Una vez realizados todos los pasos anteriores, se pueden calcular el par elctrico, la potencia de entrada, el rendimiento y la corriente para cada uno de los posibles puntos de
operacin de
la mquina. La escala de potencia se obtiene multiplicando la escala de
corriente por 3Vth ; la escala de par se determina dividiendo la escala de potencia entre
la velocidad sincrnica de la mquina.
El otro mtodo para trazar el diagrama de crculo, consiste en representar en el papel tres puntos
conocidos del diagrama. Estos tres puntos pueden ser cualquiera de los infinitos puntos posibles. Es muy frecuente disponer del punto de la condicin vaco39 s = 0, el punto de operacin
nominal40 s = sn y el punto de arranque o de rotor bloqueado 41 s = 1. En la prctica, si uno de
los puntos es el de la condicin de operacin en vaco, con slo otro punto se puede trazar el
crculo. Esto se debe al hecho de que el dimetro del crculo correspondiente al deslizamiento
de la condicin de vaco es paralelo al eje imaginario. Como se conoce este punto, con otro
punto cualquiera se puede determinar el centro del crculo. En la figura 6.27 se presenta una
construccin geomtrica con estas caractersticas.
VI.14

S UMARIO

1. La mquina de induccin es robusta y constructivamente simple, requiere un mantenimiento mnimo, puede operar en ambientes peligrosos y su confiabilidad es elevada. Estas
razones la han impuesto como el motor industrial ms utilizado en la actualidad.
39
40
41

Obtenido convencionalmente del ensayo de vaco.


Es un dato de placa y puede ser determinado tambin mediante ensayos de laboratorio.
Determinado mediante la prueba de rotor bloqueado o registrando las corrientes instantneas durante el arranque
de la mquina.

250

VI.14 Sumario

Figura 6.27 Construccin del diagrama de crculo a partir del punto de vaco y otro punto de
operacin
2. El modelo de la mquina de induccin trifsica en coordenadas primitivas requiere tres
ecuaciones para modelar las bobinas del estator, tres para las del rotor y una ecuacin para
el eje mecnico. El sistema de ecuaciones obtenido depende de la posicin instantnea
del rotor y en este contexto la solucin matemtica es un problema de difcil solucin y
comprensin.
3. Los vectores espaciales son una transformacin que permite representar en una forma
compacta las ecuaciones de la mquina de induccin debido a que proviene de la transformacin de componentes simtricas, la cual tiene la propiedad de diagonalizar matrices
simtricas y cclicas. En el dominio de los vectores espaciales, las seis ecuaciones de tensin que modelan las tensiones de las bobinas de la mquina de induccin en coordenadas
primitivas se convierten en dos ecuaciones complejas.
4. Al referir el sistema de referencia del rotor al estator, desaparece la dependencia de la
posicin angular de las ecuaciones de la mquina. En el nuevo sistema de coordenadas,
donde todas las variables estn referidas al estator, las ecuaciones diferenciales de la mquina tienen coeficientes constantes si la velocidad mecnica se considera constante. Este
modelo permite el anlisis transitorio, armnico y en rgimen permanente de la mquina
de induccin. Los desequilibrios pueden ser tratados pero es necesario incluir modelos de
secuencia negativa y cero en el anlisis.
5. La particularizacin a rgimen permanente de las ecuaciones diferenciales expresadas
en vectores espaciales, determina el modelo circuital denominado circuito equivalente

251

Captulo VI La mquina de induccin


de la mquina de induccin. El circuito equivalente T puede analizar el comportamiento
esttico del convertidor si se conocen los seis parmetros que lo constituyen (Re , Rr , Rm ,
X e , X r , Xm ) la tensin aplicada Ve y el deslizamiento de operacin sop .
6. Para determinar el deslizamiento de operacin sop es necesario resolver el equilibrio dinmico con el sistema mecnico. Este punto de equilibrio define el punto de operacin.
El punto de operacin que alcanza los lmites trmicos de la mquina se conoce como el
punto nominal. El lmite trmico es aquella temperatura de operacin en rgimen permanente que mantendra las propiedades fsico-qumicas42 de los materiales de la mquina
dentro de parmetros de degradacin adecuados para su buen funcionamiento, durante el
tiempo de vida media definido por el fabricante.
7. El sistema adimensional de unidades es una herramienta til en el anlisis de las mquinas elctricas, debido a que los parmetros del modelo en este sistema tienen pequeas
variaciones entre mquinas aun cuando stas sean de muy diferentes valores nominales.
Debido al rendimiento de la mquina de induccin es usual utilizar la potencia nominal
en el eje del motor como potencia base del sistema adimensional de unidades.
8. Los ensayos de vaco y rotor bloqueado permiten realizar un clculo aproximado de los
parmetros del modelo de la mquina de induccin, haciendo simplificaciones semejantes
a las que se utilizan en el tratamiento de los transformadores. Aun cuando estas consideraciones son menos aproximadas en la mquina de induccin que en los transformadores
por el entrehierro que produce mayor corriente de magnetizacin y mayores dispersiones,
desde el punto de vista prctico ofrecen un mtodo vlido desde el punto de vista de la
ingeniera para un gran espectro de aplicaciones. Sin embargo, cuando se requiere mayor
precisin en los resultados y anlisis es posible utilizar el mtodo de la estimacin paramtrica para ajustar los parmetros. Los mtodos de estimacin paramtrica se fundamentan
en la minimizacin de una funcin de costo establecida con los errores entre valores
medidos y valores calculados mediante el modelo.
9. La mquina de induccin tiene su principal mbito de aplicacin como motor, sin embargo
es posible utilizarla como generador o freno. La operacin como generador se establece
cuando la velocidad supera la velocidad sincrnica y el freno requiere que la velocidad
mecnica se oponga a la velocidad del campo magntico.
10. Cuando no se conocen los parmetros de la mquina es de utilidad emplear las caractersticas normalizadas de la mquina de induccin. El par y la corriente del rotor se
normalizan dividiendo estas funciones por el par y la corriente respectiva a la condicin
de deslizamiento que produce el par mximo. Se obtienen diferentes caractersticas en
funcin del factor de calidad Q de las bobinas. Resulta de utilidad considerar los casos
cuando el factor de calidad tiende a infinito, debido a que esta suposicin determina una
buena aproximacin para las mquinas convencionales.
11. El circuito equivalente de la mquina de induccin es un mtodo prctico para el anlisis
cuantitativo de su comportamiento. El diagrama de crculo constituye una herramienta
de indiscutible valor cualitativo que permite realizar interpretaciones causa-efecto con
42

Rigidez dielctrica de los materiales aislantes, corrosin, conductividad, desgaste mecnico de piezas, etc.

252

VI.15 Ejemplos resueltos


relativa sencillez. La posibilidad de presentar en un solo diagrama todo el comportamiento
de la mquina permite una mayor comprensin de sus capacidades y limitaciones.
VI.15

E JEMPLOS

RESUELTOS

Ejemplo 1: Clculo de parmetros utilizando impedancias medidas


En el listado 4 se presentaron tres impedancias de entrada de una mquina para tres diferentes
deslizamientos:
soper
0
0,03
1

Zentrada (soper )
0,199350 + j3,08920 pu
0,833740 + j0,49141 pu
0,047603 + j0,24296 pu

La resistencia del estator se midi directamente y el valor obtenido es 0, 02 pu. Utilizando el


mtodo aproximado determine el valor de los parmetros X e , X r , Xm , Rm y Rr .
Solucin:
De acuerdo con las consideraciones del mtodo aproximado el punto de vaco s = 0 determina
la resistencia y reactancia de magnetizacin:
Yent (s = 0) =

Rm

1
1
1

+
= 0,0208 j0,3224 pu
Zent
Rm jXm

1
1
= 48 pu ; Xm
= 3,1 pu
0,0208
0,3224

Para el deslizamiento de rotor bloqueado s = 1, es posible determinar las reactancias y resistencias serie:
Zent (s = 1) Re + Rr + j(X e + X r ) = 0,047603 + j0,24296 pu
Rr 0,047603 Re = 0,0276 pu
X e + X r 0,24296 pu ; X e X r = 0,1214 pu
Una alternativa para determinar la resistencia del rotor y las reactancias de dispersin consiste
en utilizar el deslizamiento s = 0,03. En este caso es conveniente determinar la corriente del
rotor Ir , restando a la corriente del estator Ie , la corriente de magnetizacin Im :
Ir = Ie Im

1
1

= 0,8694 j0,2023 pu
Zent (s = 0,03) Zent (s = 0)

Suponiendo que Vth Ve , Rth Re y Xth X e + X r :


253

Captulo VI La mquina de induccin

Rr
1
+ Re + j(X e + X r ) = 1,0912 + j0,2539 pu
s
Ir
Rr 0,0321 pu ; X e X r 0,1270 pu
Las dos soluciones obtenidas son similares y las diferencias se deben a las aproximaciones
empleadas en cada uno de los clculos. Se podran usar los valores calculados en una primera
iteracin para mejorar la evaluacin. Un ejemplo sera utilizar en los nuevos clculos la tensin
de Thvenin con los valores obtenidos en el paso previo. Tambin es posible considerar el efecto
de la reactancia de dispersin del estator en la medida de vaco. La estimacin paramtrica es
una alternativa ms prctica.
Ejemplo 2: Anlisis de la mquina de induccin cuando slo se conocen los datos de placa
Una mquina de induccin de dos pares de polos, conexin Y , posee los siguientes datos de placa:
Pn e je
5,0 kW

Vn
416V

cosn
0,8

n
nn
f
0,85 1.746 rpm 60 Hz

Todos los parmetros del circuito equivalente se encuentran dentro de los lmites tericos. Con
esta informacin, determine:
1. Parmetros del modelo de la mquina.
2. Par mximo y el deslizamiento correspondiente al par mximo.
3. Factor de potencia mximo y deslizamiento correspondiente a ese punto de operacin.
4. Los nuevos valores de placa si la mquina se alimenta con una tensin de 380V a 50 Hz.
Solucin:
1. Parmetros del modelo de la mquina.
Los datos de placa suministran informacin sobre el punto de operacin nominal; para
poder obtener los seis parmetros que determinan el comportamiento de la mquina de
induccin es necesario utilizar hiptesis adicionales. En este caso se puede asumir que la
corriente de vaco es un tercio de la corriente nominal y que las prdidas en el hierro son
despreciables:
Pn e je
Sn
In =
=
= 10,2 A Im = 3,4 A
3Vn
3Vn n cosn
Las bases del sistema adimensional de unidades son:
SB = Pn e je = 5 kW ; VB = Vn = 416V ;

254

VI.15 Ejemplos resueltos


SB
V2
IB =
= 6,9393 A ; ZB = B = 34,6112
SB
3VB
La reactancia de magnetizacin en por unidad es:
Xm =

1
Vth

= 2,04 pu
Im
0,48995

Como se desprecian las prdidas en el hierro, la resistencia de magnetizacin es infinita,


Rm . La corriente del rotor en el punto de operacin nominal puede calcularse como:
Ir = Ie Im = 1,4699 36,87 0,4899 90 = 1,2396 18,44
Rr
1
+ Re + j(X e + X r ) = 0,7653 + j0,2552
s
Ir
X e X r 0,1276 pu
Rr
Rr
+ Re
+ Rr 0,7653 pu Rr 0,0223 pu
sn
0,03
Una estimacin ms precisa se puede obtener recordando que en el punto nominal la caracterstica par-deslizamiento es lineal y depende inversamente de la resistencia del rotor:
Pne je
V2
= th sn
n
e Rr

2
2,04
0,03 0,97
2,04+0,1276
Tn =

Rr =

Vth2 sn n
=
e Pne je

11

= 0,0257 pu

Es posible asumir que la resistencia del estator es similar a la del rotor Re Rr 0,0257 pu.
2. Par mximo y deslizamiento correspondiente al par mximo.43
Tmax =

1 Vth2
(0,9411)2
= 1,7354 pu
=
2e Xth 2 1 0,2552

Rr
0,0257
= 0,10
sTmax = q
=p
2 + 0,25522
2
2
0,0257
Rth + Xth
3. Factor de potencia mximo y deslizamiento correspondiente a ese punto de operacin.
Un planteamiento que permite resolver esta pregunta consiste en analizar la geometra del
diagrama de crculo en el punto de factor de potencia mximo. En este punto, la corriente
del estator Ie debe ser tangente al crculo, tal como se muestra en la figura. De acuerdo
43

Observe que en la ecuacin del par mximo, expresada en por unidad, desaparece el coeficiente 3 al dividir por
la base correspondiente.

255

Captulo VI La mquina de induccin

Figura 6.28 Diagrama de crculo en el punto de operacin con factor de potencia mximo

con los datos del problema planteado, la corriente del estator es:
s
2 
2
 



Vth
Vth 2
1
Vth 2
1
Vth
2
=
In +
Ie =
In +

Ie +
2Xth
3
2Xth
3
2Xth
2Xth
Ie (min ) =

1,4699
0,9411
+
3
2 0,2552

2

0,9411

2 0,2552

2

= 0,7 pu

2Ie Xth 2 0,7 0,2552


=
= 0,3798 cos min = 0,9349
Vth
0,9411
Conocida la corriente del punto de factor de potencia mximo, el deslizamiento se calcula
resolviendo la ecuacin siguiente:
tan min =

1
1
Ir = Ie Im = Ie min In 90 = Ie cos min + j( In Ie sin min )
3
3
r
1
Vth

Ir = (Ie cos min )2 + ( In Ie sin min )2 = 0,6976 = q


3
(R + Rr )2 + X 2
th

s(min) = r

Rr

Vth2
Ir2

Xth2 Rth

=q

0,0257

0,94112
0,69762

th

= 0,0194

0,25522 0,02572

4. Los nuevos valores de placa si la mquina se alimenta con una tensin de 380V a 50 Hz.
En las nuevas condiciones de operacin, la corriente nominal se debe mantener aproximadamente constante, para producir prdidas semejantes al punto nominal especificado
en la placa. Las reactancias de la mquina cambian con la frecuencia, de tal forma que se

256

VI.15 Ejemplos resueltos


sn
0,040
0,041
0,042
0,043

|Zent (sn )|
0,6564
0,6440
0,6322
0,6209

Tabla 6.4 Determinacin del deslizamiento nominal para la nueva tensin y frecuencia

obtiene:
Ie =

380
Ve
= In = 1,4699 |Zent (sn )| = 416 = 0,6215 pu
|Zent (sn )|
1,4699

Zent (sn ) = Ze +

Zm Zr (s)
Zm + Zr (s)

2 50
0,1276 = 0,0257 + j0,1063 pu
2 60
2 50
2,04 = j1,7 pu
Zm = je Lm = j
2 60
Rr
0,0257
+ je L r =
+ j0,1063 pu
Zr (sn ) =
sn
sn
En la tabla 6.4 se presentan los valores del mdulo de la impedancia de entrada en funcin
del deslizamiento. El valor ms cercano a 0,6215 pu es el correspondiente al deslizamiento
0,043, el cual sera el deslizamiento nominal en las nuevas condiciones de operacin.
Ze = Re + je L e = 0,0257 + j

Una vez conocido el deslizamiento nominal, el resto de los valores nominales quedan
determinados de la forma siguiente:
nn = (1 sn )ns = (1 0,043) 1.800 = 1 722,6 rpm
Zent (sn ) = 0,5029 + j0,3642 = 0,620935,92 cos n = 0,8099
Pe (sn ) =

380
1,4699 0,8099 = 1,0874 pu
416

380
Vth
416 0,9411
r
=
= 1,305 pu
Ir (sn ) = r

2
2
0,0257
2
Rr
Rth + sn + Xth2
0,0257 + 0,043 + (2 0,1063)

Pe je (sn ) = Ir2 (sn ) Rr

1 0,043
1 sn
= 1,3052 0,0257
= 0,9744 pu
sn
0,043

(sn ) =

Pe je (sn ) 0,9744
=
= 0,896
Pe (sn )
1,0874

En este clculo no se han considerado las prdidas mecnicas ni las prdidas en el hierro
y por esta razn el rendimiento aumenta con respecto a los valores nominales iniciales.

257

Captulo VI La mquina de induccin


Ejemplo 3: Anlisis de la mquina de induccin a partir de dos puntos de operacin
Un motor de induccin conexin estrella de 5 kW, 240V, 60 Hz,opera en el punto nominal a una
velocidad de 1.653 rpm, con un factor de potencia de 0,803 inductivo y un rendimiento del
85,9 %. En el ensayo de vaco consume 4,542 A y 110W . La resistencia del estator se ha medido
directamente y su valor es 0,3456. Las prdidas mecnicas pueden ser despreciadas. Determine:
1. Los parmetros del circuito equivalente de esta mquina en por unidad de la base de
potencia de salida.
2. Los valores medidos por los instrumentos durante un ensayo a rotor bloqueado.
3. El deslizamiento a par mximo, el par mximo, el par y la corriente de arranque.
4. El deslizamiento nominal como generador y el rendimiento en esta condicin de operacin.
Solucin:
1. Determinacin de los parmetros de la mquina.
Del ensayo de vaco es posible determinar aproximadamente la resistencia y reactancia
de magnetizacin Rm y Xm :
Rm =

V2
V2
= 523,64 ; Xm =
= 30,56
Po
So
V2

B
La impedancia base es ZB = SBB =11,52 y la corriente base es IB = P3V
=12,028 A. La
B
resistencia y reactancia de magnetizacin en por unidad de la potencia en el eje y de la
tensin en bornes es:

Rm = 0,06783 pu ; Xm = 2,6527 pu
La resistencia del rotor se determina a partir del par nominal:
Tn =

PB
PB
= 28,832 Nm ; TB =
= 26,526 Nm ; Tn = 1,0869 pu
1.656
2 1.800
2 60
60

= 0,08) la relacin entre el par y el deslizamiento


En el punto nominal (sn = 1.8001.656
1.800
es aproximadamente lineal y de ella se se puede determinar la resistencia del rotor:
Vth2
V2
(0,96)2
sn Rr th sn =
0,08 = 0,06783 pu
s Rr
s Tn
1,0869
La resistencia del estator en por unidad de las bases seleccionadas es: Re =0,03pu. La
determinacin de la reactancia de dispersin puede determinarse con el equivalente de
Thvenin de la mquina y para esto se calcula la corriente por el rotor Ir = Ie Im . La
Tn

258

VI.15 Ejemplos resueltos

Figura 6.29 Parmetros del circuito equivalente en por unidad

corriente del estator en el punto nominal es:


5.000
cos1 (0,803) =
3240 0,859 0,803
= 17,5 36,582 A = 1,4549 36,582 pu

Ien =

La corriente de magnetizacin se puede determinar del ensayo de vaco y es aproximadamente Im = 4,542 86,66 A. La corriente rotrica en el punto nominal es:
Irn = Ien Im = 14,995 23,15 A = 1,2467 23,15 pu
Conocida la corriente por el rotor en el punto nominal, es posible determinar una aproximacin a la reactancia de Thvenin mediante la expresin siguiente:
Rth +

Rr
Vth
= 0,708 + j 0,3028 Xth = 0,3028
+ j Xth =
Sn
Irn

La reactancia de Thvenin es aproximadamente la suma de las dos reactancias de dispersin. Asumiendo que la dispersin del rotor es aproximadamente igual a la del rotor se
obtendra el siguiente resultado:
Xth
= 0,1514 pu
2
Los resultados obtenidos se muestran en la figura 6.29.
Xe Xr

2. Valores medidos en la prueba de rotor bloqueado.


La prueba de rotor bloqueado se realiza a corriente nominal In = 17,5 A = 1,4549 pu. Calculando la impedancia equivalente en bornes de la mquina para un deslizamiento s = 1
se obtiene:
Zeq (s = 1) = Ze +

Zr Zm
= 0,091065 + j 0,293050 pu
Zr + Zm

Vrb = |Zeq (s = 1)| In Vb = 107V

Prb = 3Vrb In cosrb = 962W

259

Captulo VI La mquina de induccin


3. Deslizamiento y par mximo, corriente y par de arranque.
El factor de calidad Q es cercano a 10; en este caso se puede aproximar la caracterstiT
ca del par con respecto al par mximo Tmax
correspondiente a Q de la figura 6.17:

Tmax = Tn

sn
sTmax

Ts=1 = Tmax
Is=1 =

sTmax = q

1
sTmax

Rr

= 0,2321

2 + X2
Rth
th

sTmax
sn

0,08
0,2321

= 1,0869
2
2
= 1,7587
s
+ Tmax
1

+ 0,2321
0,08
2
2

1
0,2321

= 1,7587 pu

+ 0,2321
1

= 0,7747 pu

1
V
Zm Zr = 0,091065 + j0,29305 = 3,2587 72,7 pu
Ze + Zm +Zr

4. Punto nominal como generador.


La condicin nominal de generacin se obtiene cuando con deslizamiento negativo s < 0,
se alcanza el mdulo de la corriente nominal del estator In =1,4549pu. La corriente del
estator se determina mediante la expresin:
V

|Ie | =
Zm Zr (s)
Ze + Zm +Zr (s)

A travs de una tabla es posible identificar rpidamente el deslizamiento que corresponde


con la corriente nominal de la mquina:
s
-0,105
-0,104
-0,103
-0,102

|Ie |
1,4815
1,4699
1,4584
1,4467

El deslizamiento nominal de acuerdo con la tabla sera sGn =-0,103, que corresponde a
una velocidad de 1.985 rpm. La corriente del estator para el deslizamiento nominal como
generador es:
Ie = 1,13573 j 0,91489 pu
La potencia de salida se determina mediante:
Ps = 1,0 |Ie| cose = 1,0 1,4584
260

1,13573
= 1,13573 pu
1,45839

VI.16 Ejercicios propuestos


La potencia de entrada se determina a partir de la corriente del rotor:
|Ir | = q

Vth

0,945
=p
= 1,3587 pu
(0,6313)2 + (0,292)2
(Rth + Rsr )2 + Xth2

Pe = |Ir |2 Rr

1 + 0,103
1s
= (1,3587)2 0,0678
= 1,3403 pu
s
0,103

Gn =

VI.16

E JERCICIOS

Ps 1,13573
=
= 0,8474
Pe
1,3403

PROPUESTOS

1. A una mquina de induccin se le realizaron los ensayos de vaco y rotor bloqueado cuyos
resultados se muestran en la tabla:
Ensayo
Vaco
Rotor bloqueado

P
3.602W
3.200W

V
416V
85V

I
30 A
100 A

n
3.598 rpm
0 rpm

La mquina se encuentra conectada en estrella y la resistencia por fase de las bobinas


del estator es de 53 m. Si la prueba de rotor bloqueado se realiz utilizando la mxima
corriente posible en rgimen permanente, determine:

a) El rendimiento en el punto nominal.


b) El par mximo.
c) El par de arranque.
d) El factor de potencia nominal.
e) La velocidad nominal.
f ) El diagrama de crculo completo de la mquina.

2. Una mquina de induccin posee los parmetros del circuito equivalente en por unidad de
las bases de tensin y potencia nominal de la mquina que se presentan en la figura 6.30.
La mquina posee dos polos y su tensin nominal lnea-neutro es de 240V . La potencia
mecnica nominal es de 10 kW , con una velocidad de 3.500 rpm. La mquina se encuentra
operando en un sistema de 208V al neutro, entregando una potencia mecnica en el eje de
9 kW,determine:

a) Si la condicin de operacin de la mquina en cuestin permite el funcionamiento


en rgimen permanente.
b) La caracterstica par-velocidad.
c) La curva de eficiencia con respecto a la potencia de salida.

261

Captulo VI La mquina de induccin

Figura 6.30 Parmetros del modelo de la mquina del problema 2


d) El diagrama de crculo completo.
3. Una mquina de induccin de rotor de jaula de ardilla, de 416V conexin en delta, 60 Hz
y 5 kW de potencia nominal, desarrolla en el punto nominal de operacin una velocidad
de 1.130 rpm, con una eficiencia del 84,25 % y un factor de potencia de 73,14 %. A esta
mquina se le ha realizado una prueba en vaco con tensin nominal, accionando el eje a
la velocidad sincrnica y en esta condicin consume 156W y 3,52 A,determine:

a) El par de arranque que desarrolla esta mquina.


b) La eficiencia y el deslizamiento en el punto de par mximo.
c) El punto nominal de operacin si la frecuencia se reduce a 50 Hz, manteniendo el
flujo constante en el entrehierro de la mquina.
d) El valor de los parmetros del modelo de la mquina de induccin en la base de la
potencia de salida de la mquina.

4. De un motor de induccin se conocen los siguientes parmetros definidos en las bases de


potencia en el eje y voltaje nominal de la mquina:
Re = 0,015 pu Rr = 0,035 pu Rm = 25 pu
X e = 0,2 pu X r = 0,2 pu Xm = 2,5 pu
Determine:

a) La corriente y el factor de potencia nominal de la mquina como motor y como


generador.
b) Los deslizamientos que corresponden al par y potencia mxima como motor y generador.
c) La tensin de alimentacin para que con el deslizamiento s = 2, la mquina se frene
con par mximo.

5. Un motor de 12 kW y 456V en delta, posee una eficiencia en el punto nominal de 85 %.


Las prdidas en el hierro son 5 % de la potencia mecnica nominal; el factor de potencia

262

VI.16 Ejercicios propuestos


nominal de la mquina es 0,85 y la corriente de arranque es cinco veces la corriente nominal. La velocidad nominal es 1.710 rpm, determine:

a) La corriente y el factor de potencia en vaco.


b) El par de arranque y el par mximo como motor.
c) El rendimiento para una velocidad de 1.600 rpm y 2.000 rpm.
d) El rendimiento del punto nominal de operacin, si la tensin y la frecuencia disminuyen a 380V y 50 Hz.
e) El diagrama fasorial completo para una velocidad de -1.900 rpm.

6. Una mquina de induccin de 416V y 60 Hz en la conexin delta, consume 100 A en el


punto de operacin nominal, con un factor de potencia de 0,85 inductivo. En estas condiciones la mquina entrega 50,8 kW en el eje mecnico, a una velocidad de 1.710 rpm.
La mquina consume un 2 % de prdidas en la base de la potencia aparente de entrada,
cuando el rotor gira a la velocidad de 1.800 rpm. La corriente de arranque es igual a seis
veces la corriente nominal y la resistencia del estator consume una potencia del 1 % en la
base de la potencia aparente de entrada. Determine:

a) Los parmetros del circuito equivalente.


b) El deslizamiento correspondiente de la mquina de induccin cuando se carga con
del par nominal.
c) El par mximo y el deslizamiento de la mquina que corresponde al punto nominal
si la misma se conecta en estrella.
d) Los nuevos datos de placa si esta mquina ha de ser utilizada en un sistema de 50 Hz.

7. Una mquina de induccin de 60 Hz conectada en delta, posee los siguientes datos de placa:
Pn = 100 kW

Vn = 416V

In = 170 A cos n =0,85 nn =1.750 rpm

Girando a la velocidad de 1.799 rpm, la mquina consume 44 A y 850W . La resistencia de cada una de las bobinas del estator a temperatura ambiente es de 42 m. Determine:

a) Los parmetros del modelo del convertidor en la base de la potencia nominal.


b) La corriente de arranque, el par de arranque, el par mximo y su deslizamiento
correspondiente.
c) El rendimiento, factor de potencia y deslizamiento cuando la mquina se encuentra
en el punto nominal como generador.

263

Captulo VI La mquina de induccin


d) El diagrama de crculo completo si la mquina se conecta en estrella en el mismo
sistema de tensiones.
8. A una mquina de induccin trifsica de 60 Hz, conexin delta, dos pares de polos y de
rotor bobinado, se le han realizado en el laboratorio de mquinas elctricas los siguientes
ensayos:
Ensayo
Vaco
Rotor bloqueado

Vmed
208V
35V

Imed
10 A
35 A

Pmed
540W
320W

Las prdidas hmicas del estator y rotor son similares. Las ranuras del estator y del rotor
son idnticas. Determine:

a) Parmetros del circuito equivalente en la base de la potencia nominal en el eje.


b) Deslizamiento, factor de potencia y rendimiento nominal de la mquina.
c) Rendimiento y factor de potencia en el punto nominal como generador.
d) Diagrama de crculo completo incluyendo la calibracin de la recta del deslizamiento.

264

Bibliografa

A LLER , J. M., Simple Matrix Method to Introduce Spatial Vector Transformations and Oriented Field Equations in Electric Machine Courses, ICEM96 Proceedings, vol. III, pp. 519-524,
Vigo, Spain, September, 1996.
B LASKE, F., The Principle of Field Orientation as Applied to New TRASKVEKTOR ClosedLoop Control System for Rotating-Field Machines, Siemens Review, vol. 34, pp. 217-220, May,
1972.
C ORTS , M., Curso moderno de mquinas elctricas rotativas, Editores Tcnicos Asociados,
Barcelona, 1994-1995.
C HAPMAN , S. J., Mquinas elctricas, McGraw-Hill, Mxico,1987.
F ITZGERALD , A. E., K INGSLEY, C. J R . & K USKO, A., Electric Machinery: The Processes,
Devices, and Systems of Electromechanical Energy Conversion, McGraw-Hill, Third Edition,
New York, 1971.
KOSTENKO, M. P. & PIOTROVSKI, L. M., Mquinas elctricas, vol. II, Editorial Mir, Second
edition, Mosc, 1979.
L ANGSDORF, A. S., Theory of Alternating Current Machinery, Tata McGraw-Hill, Second Edition, New York, 1974.
L EONHARD , W., Control of Electrical Drives, Springer-Verlag-Berlin, Heidelberg, 1985.
M AGINNINS , F. S. & S CHULTZ , N. R., Transient Performance of Induction Motors, AIEE
Transactions, vol. 64, pp. 651-656, 1944.
M C P HERSON , G. & L ARAMORE , R. D., An Introduction to Electrical Machines and Transformers, John Wiley & Sons, Singapur, 1990.
R ICHTER, R., Elektrische Maschinen, vol. IV, Verlag Birkhusen AG, Basel, 1954.
S ANZ , J., Mquinas elctricas, Prentice Hall, Madrid, 2002.

265

Captulo VI La mquina de induccin


S ERRANO , L., Fundamentos de mquinas elctricas rotativas, Universidad Politcnica de Valencia, 1989.
S ERRANO , L., The Modern Space-Phasor Theory, Part I: Its Coherent Formulation and its
Advantages for Transient Analysis of Converted-Fed AC Machines, European Transactions on
Electrical Power Engineering (ETEP), Vde-Verlag, vol. 3, N. 2, pp. 171-180, Mar./Apr. 1993,
S ERRANO , L., The Modern Space-Phasor Theory, Part II: Comparison with the Generalized
Machine Theory and the Space-Vector Theory, European Transactions on Electrical Power
Engineering (ETEP), Vde-Verlag, vol. 3, N. 3, pp. 213-219, May/June. 1993.
T IBERIO , F., Pruebas sobre mquinas elctricas, Editorial Vicens-Vives, Primera Edicin, Barcelona, 1965.
VAS , P., Vector Control of AC Machines, Oxford University Press, New York, 1990
VAS , P., Electrical Machines and Drives. A Space Vector Approach, Oxford University Press,
New York, 1992
W HITE , D. C. & WOODSON , H. H., Electromechanical Energy Conversion, John Wiley &
Sons, New York, 1959.
YAMAMURA , S., Spiral Vector Method and Symmetrical Component Method, Procc. Japan
Academy, vol. 67B, pp. 1-6, Japan,1994.

266

CAPTULO VII

Operacin de la mquina de induccin

En el captulo VI se obtuvieron varias herramientas que permiten la modelacin de la mquina


de induccin: el circuito equivalente, las caractersticas paramtricas y el diagrama de crculo.
La operacin de la mquina de induccin en diversos regmenes de operacin equilibrados,
desequilibrados, armnicos y transitorios requiere un anlisis detallado.
El rotor de las mquinas de induccin ha venido cambiando notablemente desde su diseo original, para adaptarse a necesidades operativas ms exigentes: alto rendimiento e intenso par de
arranque. Las corrientes de arranque deben ser limitadas mediante el uso de arrancadores para
reducir el impacto de este proceso en la red e incrementar la vida til del convertidor.
El accionamiento de la mquina de induccin con controladores de potencia y en redes elctrica contaminadas armnicamente hace necesario considerar en los modelos el impacto de estas
armnicas. Las distribuciones de los devanados producen armnicas espaciales en el campo
magntico rotatorio que tienen afectos sobre su comportamiento. Es necesario conocer la operacin desequilibrada porque reduce el par til e incrementa las corrientes, condiciones que ponen
en riesgo la mquina.
La operacin transitoria del convertidor es especialmente importante en la medida que los controladores electrnicos de potencia proporcionan la capacidad de operar estos equipos a velocidad variable y existen diversas estrategias como el control vectorial, el control por campo
orientado y el control directo de par que hacen uso intenso de la dinmica de esta mquina.

VII.1

A RRANQUE

DE MOTORES DE INDUCCIN

Cuando se utiliza una mquina de induccin para arrancar y accionar una carga mecnica a una
velocidad determinada, es posible que sucedan tres situaciones diferentes:

267

Captulo VII Operacin de la mquina de induccin

Figura 7.1 Condiciones de arranque para dos mquinas de induccin con diferente resistencia
en el rotor
1. El par elctrico de arranque que suministra la mquina puede ser inferior al par mecnico
que requiere la carga en reposo para comenzar a moverse. En esta situacin la mquina no
puede acelerar, el rotor est detenido o bloqueado. La corriente es varias veces la corriente
nominal y si no se pone remedio a esta situacin, la mquina corre un serio riesgo de
daarse por calentamiento excesivo.
2. El par elctrico es exactamente igual al par de la carga. Esta situacin tiene los mismos
problemas que el primer caso. Si los pares elctrico y mecnico estn equilibrados, no es
posible acelerar la mquina.
3. El par elctrico de arranque es mayor que el par mecnico de la carga. En estas condiciones, existe un par acelerante que permite incrementar la velocidad del rotor, hasta un punto
de la caracterstica par elctrico deslizamiento donde se equilibran los pares de la mquina
y de la carga. Si la velocidad alcanzada en este punto es cercana a la velocidad sincrnica,
la corriente disminuye a un nivel que permite la operacin en rgimen permanente. Cuando la mquina opera en este punto, un pequeo incremento de la carga mecnica reduce
la velocidad e incrementa el par elctrico de accionamiento, obtenindose un nuevo punto
de operacin.
En la figura 7.1 se observa que una mquina de induccin produce ms par de arranque en
la medida que la resistencia del rotor aumenta. Una mquina con alta resistencia en el rotor
tiene deslizamientos de operacin ms grandes. Las prdidas en el rotor aumentan durante la

268

VII.2 El rotor de jaula de ardilla

Figura 7.2 Rotor devanado con anillos deslizantes y escobillas


operacin en rgimen permanente cuando se utilizan resistencias grandes en estas bobinas, y
esto repercute desfavorablemente en el rendimiento de la mquina.
Es deseable que una mquina elctrica produzca un par de arranque elevado y un rendimiento
lo ms cercano posible a la unidad. Estas dos caractersticas de la mquina se contraponen
con respecto a la variacin de la resistencia del rotor. Por esta razn en algunas mquinas de
induccin se aade anillos deslizantes en el eje del rotor, cada uno de ellos se conecta a un
extremo de las fases del rotor. Estos anillos, permiten conectar elementos en el exterior de la
mquina con las bobinas del rotor, mediante contactos deslizantes1. Al tener acceso a las bobinas
del rotor, es posible incluir resistencia adicional con la finalidad de incrementar el par elctrico
de la mquina durante el arranque. Cuando la mquina est operando en rgimen permanente es
posible eliminar o cortocircuitar la resistencia adicional para incrementar el rendimiento.
El rotor bobinado con acceso mediante anillos deslizantes es capaz de regular el par de arranque, disminuir las corrientes durante el proceso de aceleracin y aumentar el rendimiento en el
punto de operacin, cuando se elimina la resistencia externa. Los principal inconveniente de esta
solucin residen en los incrementos en los costos de la mquina y en las prdidas mecnicas por
friccin con las escobillas.
VII.2

EL

ROTOR DE JAULA DE ARDILLA

La aplicacin industrial de mquinas de induccin con rotor devanado no es muy frecuente,


debido a que es posible una solucin mucho ms econmica y prctica. El campo producido
por las bobinas del estator genera fuerza electromotriz sobre cualquier conductor localizado
en el rotor. En lugar de construir un bobinado similar al del estator, se pueden colocar barras
1

Carbones o escobillas.

269

Captulo VII Operacin de la mquina de induccin

Figura 7.3 Corte de una mquina de induccin con rotor de jaula de ardilla. Cortesa de ABB
conductoras en la periferia del rotor. Sobre estas barras, paralelas al eje de la mquina, se inducen
fuerzas electromotrices por el campo magntico rotatorio originado en el estator. Si estas barras
estn cortocircuitadas en sus extremos mediante dos anillos conductores, circula corriente por
ellas y se obtiene un campo magntico rotatorio en el rotor. La interaccin entre los dos campos
magnticos rotatorios produce el par elctrico.
El rotor de jaula de ardilla es muy simple desde el punto de vista constructivo, adems es capaz
de soportar esfuerzos elctricos y mecnicos mucho mayores que el rotor devanado. En este rotor
no es posible incluir resistencia adicional en serie con los conductores. Sin embargo, durante la
construccin del rotor se puede variar el valor de la resistencia controlando el espesor de los
anillos que cortocircuitan las barras.
El rotor de jaula de ardilla se puede disear utilizando barras profundas o doble jaula, una externa
muy resistiva y otra profunda con menor resistencia. Estas modificaciones permiten utilizar el
efecto pelicular para producir una variacin de la resistencia equivalente del rotor durante el
arranque. Con el rotor en reposo, el deslizamiento es grande y la frecuencia de las corrientes
que circulan por l es proporcional al deslizamiento. Si las barras son profundas o el rotor es de
doble jaula, la corriente de alta frecuencia fluye por la periferia de la barra o por la jaula ms
externa y resistiva, incrementando sustancialmente la resistencia equivalente del rotor en esta
condicin. Cuando la mquina est operando cerca del punto nominal, los deslizamientos son
cercanos a cero y la frecuencia de las corrientes que circulan por el rotor tambin es reducida.
En este caso las corrientes se distribuyen uniformemente por las barras y consecuentemente la
resistencia equivalente disminuye. De esta forma es posible construir mquinas econmicas y
robustas, de alta eficiencia y pares de arranque elevados. Estas razones justifican por s solas, la
difusin industrial alcanzada por este tipo de accionamiento.
La mquina de induccin de doble jaula se puede modelar mediante un circuito equivalente
que considere los enlaces de flujo mutuos entre el estator y las barras del rotor, as como los

270

VII.2 El rotor de jaula de ardilla

(a) Rotor de doble jaula (p=2)

(b) Rotor de barra profunda (p=3)

Figura 7.4 Cortes transversales de rotores con jaula de ardilla

Figura 7.5 Circuito equivalente de la mquina de induccin con rotor de doble jaula
enlaces de dispersin de cada una de las jaulas. Cada uno de estos enlaces se representan en
el circuito equivalente por una reactancia. Las resistencias de cada una de las jaulas se modela
independientemente. En la figura 7.5 se muestra el circuito equivalente y los flujos en las barras.
En los motores de doble jaula, el par elctrico se calcula superponiendo los pares producidos
por cada una de las jaulas:


3 2 Rr1
2 Rr2
Te =
I
+ I2
e 1 s
s

(7.1)

En los rotores de jaula de ardilla con barras profundas el fenmeno es similar. Durante el arranque, el efecto pelicular reparte las corrientes no uniformemente en el interior de las barras del
rotor. Las corrientes circulan principalmente por la periferia de la barra con la finalidad de reducir la trayectoria de los enlaces de flujo 2 . Cuando la mquina alcanza el punto de operacin
permanente, el deslizamiento es muy pequeo, y las corrientes se distribuyen uniformemente en
2

Esta sera la condicin de menor energa.

271

Captulo VII Operacin de la mquina de induccin

Figura 7.6 Caracterstica de par elctrico-deslizamiento para un rotor de doble jaula

Figura 7.7 Circuito equivalente para un rotor de jaula de ardilla con barras profundas

272

VII.3 Corriente de arranque


las barras, con lo cual disminuye su resistencia. El modelo del rotor de doble jaula puede ser
empleado en el anlisis de este tipo de mquinas con suficiente precisin. El modelo de la mquina de induccin con rotor de barra profunda se puede mejorar sustancialmente, incluyendo
varias ramas adicionales en el circuito rotrico tal como se muestra en la figura 7.7.
La identificacin de los parmetros de las mquinas de induccin con rotor de doble jaula o rotor
de barras profundas es semejante al mtodo utilizado para la determinacin de parmetros en la
mquina de induccin convencional. Existen mayores complicaciones debido al mayor nmero
de ramas y elementos que es necesario identificar. Los ensayos sobre la mquina se realizan con
los lineamientos discutidos en la seccin VI.10. Los parmetros del rotor se pueden identificar
realizando ensayos de cortocircuito a diferentes frecuencias. Los ensayos de operacin en carga
tambin resultan convenientes para la estimacin de los parmetros. El algoritmo propuesto en
la figura 6.15, es capaz de realizar la estimacin paramtrica de estas dos modificaciones de la
mquina de induccin.

VII.3

C ORRIENTE

DE ARRANQUE

Un problema importante en la operacin de la mquina de induccin es la elevada corriente


que sta absorbe durante el proceso de arranque. La corriente de arranque de una mquina de
induccin se encuentra entre tres y seis veces la corriente nominal de la mquina, y en algunas
ocasiones an ms. La cada de tensin en los conductores de alimentacin y en el sistema puede
sobrepasar el mximo permitido. La tensin de alimentacin de la mquina no debe estar nunca
por debajo del cinco por ciento de su valor nominal 3.
Existen cargas mecnicas que a baja velocidad no ofrecen par resistente importante y este crece
paulatinamente con la velocidad. En este caso es posible utilizar sistemas de arranque de la
mquina de induccin a tensin reducida que contribuyen a disminuir la magnitud de la corriente
en la mquina durante el proceso de aceleracin. Un arranque a tensin reducida, incrementa el
tiempo de aceleracin de la mquina y su carga mecnica, pero las corrientes disminuyen en la
misma proporcin que la tensin aplicada.
Algunas mquinas de induccin que se utilizan en sistemas relativamente dbiles, como puede
ser el caso de los ascensores y elevadores residenciales, se disean con reactancias de dispersin
muy grandes4 , para reducir la corriente durante el proceso de arranque a d os o tres veces la
corriente nominal, disminuyendo de esta forma el impacto de los frecuentes arranques en el
perfil de tensin de la red elctrica.
Para reducir la corriente durante el proceso de aceleracin de la carga mecnica se han utilizado varios sistemas. Estos arrancadores difieren unos de otros en el mtodo de reduccin de
tensin. Algunos utilizan el cambio de conexiones de las bobinas de la mquina, otros utilizan
transformadores o autotransformadores y los ms modernos se disean mediante convertidores
electrnicos de potencia. Los arrancadores son costosos para ser aplicados a mquinas pequeas,
y el impacto de la corriente de arranque en estas mquinas no es importante ni para la mquina,
3
4

Hay que recordar que el par elctrico se reduce con el cuadrado de la tensin de alimentacin y la mquina puede
ser incapaz de acelerar la carga mecnica en condiciones de tensin reducida.
Esto es posible cerrando lo ms posible la cabeza del diente donde se colocan los conductores o las barras del
rotor de jaula de ardilla.

273

Captulo VII Operacin de la mquina de induccin


ni para la red. Es necesario recordar que una mquina pequea tiene una relacin superficievolumen muy grande y esto le permite disipar bien sus prdidas. A continuacin se detallan
algunos ejemplos de los arrancadores ms utilizados en la industria:
1. El arrancador estrella-delta:
El mtodo ms simple para reducir la tensin de una mquina consiste en conectarla inicialmente en estrella y cuando el deslizamiento es pequeo se cambia la conexin del
motor a delta. La tensin final sobre cada bobina de la mquina
debe ser su propia tensin nominal. Este mtodo de arranque reduce la tensin en 3 veces la tensin nominal
de la mquina y la corriente se reduce en esta misma proporcin. Los pares elctricos se
reducen a un tercio del par a tensin nominal. Este procedimiento es uno de los ms econmicos, pero es necesario disponer de un sistema adecuado de tensiones que permita la
conexin delta de la mquina durante el rgimen permanente. El cambio de conexin se
realiza cuando la mquina alcanza un deslizamiento cercano al de operacin en la conexin estrella. La orden de cambio puede ser dada por un temporizador si se conoce la
inercia de la carga o el tiempo de aceleracin con tensin reducida. Si el cambio de conexin se realiza antes de que las corrientes disminuyan, el arrancador pierde efectividad. El
tiempo total de arranque con este dispositivo es aproximadamente tres veces mayor que
el arranque en directo de la mquina, esto es importante en el momento de especificar las
protecciones del motor.

En la figura 7.8 (a) se presenta el grfico del par y la corriente en la mquina durante el
proceso de arranque estrella-delta. Con estos arrancadores es posible lograr que la corriente mxima no exceda el valor 3,0 pu, mientras que en un arranque directo, esta corriente
podra alcanzar 5,0 pu. En la figura 7.8 (b) se muestra el diagrama de un arrancador industrial estrella-delta. Se utilizan tres contactores y un temporizador que los activa. Adems
deben incluirse protecciones trmicas para desconectar la mquina en caso de sobrecarga.

2. El arrancador por autotransformador:


El arrancador estrella-delta es muy econmico, pero permite una sola posibilidad en la
reduccin de la tensin. Utilizando transformadores o autotransformadores, es posible
utilizar una reduccin arbitraria de la tensin. Tambin es posible arrancar la mquina en
varios pasos utilizando diferentes derivaciones del transformador. Este esquema de arranque es ms costoso.

3. Arranque por conexin de bobinas serie-paralelo:


En algunas mquinas, cada una de las bobinas del estator se dividen en dos partes, con
la intencin de utilizar diferentes tensiones de alimentacin, por ejemplo 208V 416V .
Si las bobinas de cada fase se conectan en serie, la mquina se puede conectar a un sistema de 416V . Si por el contrario las dos bobinas de cada fase se conectan en paralelo, el

274

VII.3 Corriente de arranque

(a) Caractersticas del par elctrico y la corriente del arrancador

(b) Diagrama esquemtico del arrancador

Figura 7.8 Arrancador estrella-delta

275

Captulo VII Operacin de la mquina de induccin

Figura 7.9 Arrancador suave por retardo de fase mediante tiristores


sistema de alimentacin debe ser de 208V . Existen esquemas similares al de la figura 7.8
para arrancar el motor de induccin en un sistema de 208 V con las bobinas de cada fase
conectadas en serie, y posteriormente reconectar estas bobinas en paralelo para alcanzar
el punto de operacin de rgimen permanente. Este esquema tiene un comportamiento
similar al del arrancador estrella-delta, con la salvedad de que las corrientes se reducen a
la mitad y el par elctrico a la cuarta parte durante la aceleracin.

4. Arrancadores suaves:
Mediante convertidores electrnicos de potencia, se puede realizar un arranque suave de
la mquina, incrementando la tensin en forma continua a medida que la carga mecnica
acelera. Este tipo de arrancador puede limitar la corriente de arranque y reducir considerablemente los esfuerzos mecnicos ocasionados por los arranques bruscos. Los principales
problemas de estos arrancadores son el costo y la inyeccin de corrientes armnicas a la
red. En la figura 7.9 se detalla un diagrama de un arrancador suave por tiristores.

VII.4

R GIMEN

DESEQUILIBRADO DE LAS MQUINAS DE INDUCCIN

Si una mquina de induccin se encuentra girando en rgimen permanente a la velocidad r , en


la misma direccin del campo magntico rotatorio, y en ese preciso instante se invierte la conexin de dos fases del estator, el campo rotante invierte el sentido de rotacin y el deslizamiento
de la mquina cambia bruscamente. La velocidad de la mquina durante este proceso prcticamente instantneo no vara apreciablemente debido a la inercia del sistema electromecnico.
Posteriormente el rotor de la mquina se frena, hasta detenerse y se acelera nuevamente en la
direccin del nuevo campo rotante, hasta alcanzar una nueva condicin de equilibrio.

276

VII.4 Rgimen desequilibrado de las mquinas de induccin

Figura 7.10 Par elctrico antes y despus de la inversin de fases

El deslizamiento de la mquina en el instante previo a la inversin de las fases es:


s1 =

e m
r
= 1
e
e

(7.2)

Cuando se invierte el sentido de rotacin del campo magntico rotatorio, pero an no ha variado
la velocidad, el deslizamiento es:
s2 =

r
e m
= 1+
e
e

(7.3)

Sumando las relaciones 7.2 y 7.3 se obtiene:


s1 + s2 = 2

(7.4)

El circuito equivalente de la mquina no cambia al invertir el campo magntico rotatorio, lo


nico que vara es el deslizamiento de la mquina en ese instante. La expresin 7.4 determina el
deslizamiento despus de la inversin de las fases, a partir del deslizamiento previo:
s2 = 2 s1

(7.5)

El par acelerante se calcula, segn se observa en la figura 7.10, por la diferencia entre el par
elctrico y el par mecnico. El par elctrico en la nueva condicin invierte su sentido por el
intercambio de las fases, el par neto acelerante es negativo y la mquina se frena perdiendo

277

Captulo VII Operacin de la mquina de induccin


velocidad:

d r
<0
(7.6)
dt
El proceso descrito anteriormente se puede representar mediante el circuito equivalente que
se muestra en la figura 7.11(b). Este circuito equivalente es idntico al circuito equivalente
analizado en el captulo 6, excepto por el deslizamiento.
Ta = Te Tm = J

Si las tensiones de alimentacin de una mquina de induccin no son balanceadas, las corrientes
que fluyen por las bobinas tampoco lo sern. Cuando el sistema de corrientes que circula por
las bobinas no es equilibrado, el campo magntico en el interior de la mquina no es rotatorio.
Si se mantienen las condiciones de linealidad en el modelo de la mquina, es posible sustituir
el sistema de tensiones trifsicas desequilibradas por tres sistemas equilibrados de secuencia
positiva, negativa y cero, haciendo uso de la transformacin de componentes simtricas. Cada
una de estas componentes se analiza independientemente y posteriormente se superponen las
tres componentes para determinar la solucin final.
Para descomponer un sistema trifsico en componentes simtricas se utiliza la siguiente transformacin conservativa en potencia:

1 1
1
1 1 1
V0
Va
Va
2
1
4
V+ = 1
(7.7)
1 e j 3 e j 3 Vb = 1 2 Vb
2
4
3
3 1 2
j
j
V
V
V
c
c
1 e 3 e 3
La transformacin inversa de componentes simtricas a magnitudes de fase es:

1 1
1
1 1 1
V0
Va
V0
2
4
1
Vb = 1
1 e j 3 e j 3 V+ = 1 2 V+
2
4
3
3 1 2
V
Vc
V
1 ej 3 ej 3

(7.8)

El modelo de secuencia positiva de la mquina de induccin se desarroll en el captulo VI5 . El


circuito equivalente de secuencia negativa difiere del modelo de secuencia positiva tan slo en el
deslizamiento. Cuando una mquina de induccin es alimentada mediante un sistema trifsico
equilibrado de secuencia negativa, el campo magntico rotatorio gira en sentido contrario a las
agujas del reloj, la velocidad del campo en estas condiciones es e .

Si se alimenta la mquina de induccin con un sistema de corrientes homopolares6 , los flujos


producidos por tres corrientes iguales se neutralizan dentro de la mquina, debido a que las
bobinas tienen sus ejes magnticos desfasados 120 unas de otras. El campo en el entrehierro es
nulo y por esta razn la mquina no est magnetizada. El nico flujo que producen las corrientes
de secuencia cero es el de dispersin de las bobinas. El modelo de secuencia cero tambin debe
incluir la resistencia del circuito estatrico.
Conocidas las tensiones de secuencia positiva, negativa y cero que se han aplicado a la mquina,
se calcula el par elctrico de secuencia positiva y de secuencia negativa. La secuencia cero
no contribuye al par elctrico, debido a que no produce campo magntico en el entrehierro. La
superposicin de los pares de secuencia positiva y negativa, que estn en oposicin, determina el
5
6

Queda representado por el circuito equivalente.


Esta palabra significa con la misma polaridad. Esto se refiere a la componente de secuencia cero.

278

VII.4 Rgimen desequilibrado de las mquinas de induccin

(a) Modelo de secuencia positiva

(b) Modelo de secuencia negativa

(c) Modelo de secuencia cero

Figura 7.11 Modelos de secuencia de la mquina de induccin

279

Captulo VII Operacin de la mquina de induccin

Figura 7.12 Apertura del fusible de la fase a de una mquina de induccin sin conexin de
neutro
par resultante en el eje de la mquina. El par elctrico disminuye cuando existen desequilibrios
en la fuente de alimentacin. La red de secuencia cero no produce par, pero incrementa las
corrientes, las prdidas y el calentamiento, reduciendo el rendimiento de la mquina. Por esta
razn es poco frecuente conectar el neutro de la mquina de induccin a la red.
Las componentes simtricas son una herramienta eficaz para el anlisis de los desequilibrios
ms frecuentes a que puede estar sometida la mquina de induccin7. Para resolver los desequilibrios es necesario definir las condiciones de contorno del problema en el dominio de las fases
y transformar estas relaciones al dominio de la secuencia. Las condiciones de contorno transformadas y los modelos de secuencia de la mquina de induccin, permiten establecer el sistema
de ecuaciones del cual se obtienen las tres tensiones de secuencia sobre la mquina. Con esta
informacin se puede calcular el par de secuencia positiva y el de secuencia negativa. Todo esto,
para una determinada velocidad o deslizamiento de la mquina. En algunos casos, el sistema
de ecuaciones se puede representar mediante un circuito elctrico, facilitando de esta forma la
solucin del problema. Algunos desequilibrios frecuentes en una mquina de induccin son:
1. Apertura del fusible de una fase, sin conexin del neutro:
En la figura se ha representado una mquina de induccin que se encontraba operando
en un punto estable, y repentinamente se abre el fusible de la fase a, quedando conectadas
las otras dos fases a la red. Esta mquina tiene una conexin sin neutro corrido.
Las tres condiciones de contorno de la mquina de induccin en las condiciones descritas
en la figura 7.12 son:
Ia = 0
(7.9)
Ib + Ic = 0
Vb Vc = V 120 V 240 =
7

3V 90 = j 3V

(7.10)
(7.11)

La aplicacin adecuada de esta herramienta requiere que el convertidor sea simtrico, los desequilibrios en el
interior de la mquina acoplan los modos de secuencia y vuelven imprctica la aplicacin de esta metodologa.

280

VII.4 Rgimen desequilibrado de las mquinas de induccin


Utilizando la expresin 7.7 para convertir las condiciones de contorno sobre las corrientes
de fase a condiciones de secuencia, se obtiene:

1 1 1
0
I0
Ia
I+ = 1 1 2 Ib = 1 2 Ib
(7.12)
3
3
2
2
I
I

La expresin 7.12 indica que la suma de las componentes simtricas de corriente para la
secuencia positiva y negativa es cero. Adems la componente de secuencia cero tambin
es nula debido a que la mquina no tiene el neutro conectado:
I0 = 0

(7.13)

I+ + I = 0

(7.14)

De la transformacin 7.8 y la condicin de contorno 7.9 se establece el siguiente resultado:


1 
Vb Vc = (V0 + 2V+ + V ) (V0 + V+ + 2V ) = j 3V
3

(7.15)

La ecuacin anterior determina una relacin definida entre las tensiones de secuencia positiva y negativa en la mquina:

(7.16)
V+ V = 3V
Adems deben incluirse las tres condiciones sobre las impedancias de secuencia de la
mquina:
V0 = Z0 I0
V+ = Z+ (s) I+
V = Z (s) I

(7.17)

Sustituyendo las expresiones 7.17 y 7.14 en 7.16 se obtiene el siguiente resultado:

3V = Z+ (s) I+ Z (s) I = [Z+ (s) + Z (s)] I+


(7.18)
En la figura 7.13 se representa el circuito equivalente en el dominio de la secuencia, identificado con el resultado obtenido en la expresin 7.18. Cuando la mquina se encuentra
operando a una velocidad r diferente de cero, los deslizamientos de secuencia positiva
s+ y de secuencia negativa s , son diferentes y por lo tanto las impedancias de secuencia tambin. Como la corriente de secuencia positiva circula por las dos impedancias, las
tensiones de secuencia sobre cada impedancia son distintas y se produce una diferencia
entre el par de secuencia positiva y el par de secuencia negativa. La mquina podr seguir operando si la carga no es demasiado grande. Si la mquina se encontraba detenida
previamente r = 0, no se produce par elctrico neto debido a que los deslizamientos de
secuencia positiva y negativa valen uno en esta condicin de velocidad, cada una de las
secuencias ofrece la misma impedancia a la fuente, y por tal motivo los pares de secuencia
tambin son iguales. En la figura 7.14 se presenta la caracterstica par-deslizamiento para

281

Captulo VII Operacin de la mquina de induccin

Figura 7.13 Circuito equivalente de la apertura del fusible de la fase a de una mquina de induccin sin neutro

una mquina de induccin con la fase a abierta. Se puede observar que no existe par de
arranque y que el par mximo es menor que en la operacin balanceada. Como la matriz
de componentes simtricas utilizada en este anlisis es hermitiana, la transformacin es
conservativa en potencia y no es necesario regresar nuevamente al dominio de las fases
para el clculo de potencias o pares. Si es necesario determinar las corrientes en las fases
hay que recurrir a la transformacin inversa de componentes simtricas definida en la relacin 7.8.

2. Apertura del fusible de la fase a con el neutro conectado:


En la figura 7.15 se presenta el diagrama de una mquina de induccin conectada con
neutro corrido, con apertura del fusible de la fase a. En este caso, la corriente por la fase
a es cero, pero la suma de las corrientes de las fases sanas no. Las tres condiciones de
contorno en el dominio de fase son:
Ia = 0
(7.19)
Vb = V 120

(7.20)

Vc = V 240

(7.21)

Sustituyendo las condiciones de contorno 7.20 y 7.21 en la transformacin 7.7 se obtiene


el siguiente resultado:

1 1 1
Va
Va + V + 2V
V0

V+ = 1 1 2 2V = 1
Va + 2V
(7.22)
3
3
2
2
V
V
1

V + V + V

282

VII.4 Rgimen desequilibrado de las mquinas de induccin

Figura 7.14 Caracterstica par-deslizamiento con el fusible de la fase a abierto y sin conexin
del neutro a la red

Figura 7.15 Apertura del fusible de la fase a con el neutro conectado

283

Captulo VII Operacin de la mquina de induccin

Figura 7.16 Circuito equivalente de la mquina de induccin con neutro conectado y fase a
abierta
En la expresin 7.22 se observa que las tensiones de secuencia negativa y cero son iguales, adems la diferencia entre la tensin de secuencia positiva y cualquiera de las otras
dos tensiones de secuencia, es independiente de Va . Por esta razn es posible expresar lo
siguiente:
V = V0
(7.23)

(7.24)
V+ V = V+ V0 = 3V
La transformacin 7.8 permite encontrar la relacin entre las corrientes de secuencia, a
partir de la condicin de contorno 7.19:
1
Ia = (I0 + I+ + I ) = 0 I0 + I+ + I = 0
3

(7.25)

A las expresiones 7.23, 7.24 y 7.25 es necesario aadir las relaciones entre las tensiones
de secuencia y las corrientes de secuencia 7.17. Con las expresiones 7.17 y 7.23 a 7.25 se
determina el sistema de ecuaciones siguiente:


Z+ (s) Z (s) 0
I+
3V
0
Z (s) Z0 I = 0
(7.26)
1
1
1
I0
0

El sistema de ecuaciones 7.26, puede ser resuelto para las tres corrientes. Una vez conocidas estas variables, se determinan de las expresiones 7.17, las tensiones de secuencia
positiva y negativa, con las cuales se pueden evaluar los pares y potencias para esta condicin de operacin. Las dos primeras ecuaciones del sistema 11.31, son ecuaciones de
mallas y la tercera es una ecuacin de nodos. Con estas ecuaciones se puede construir un
circuito equivalente tal como se muestra el la figura 7.16. Cuando el neutro est conectado, la mquina produce par de arranque con una fase desconectada. Las tensiones de
secuencia positiva y negativa son diferentes, debido a que la impedancia de secuencia cero queda conectada en paralelo con la impedancia de secuencia negativa. Recordando que

284

VII.4 Rgimen desequilibrado de las mquinas de induccin

Figura 7.17 Caracterstica par-deslizamiento con apertura de la fase a con neutro conectado

la impedancia de secuencia cero tiene un valor muy reducido, debido a que est formada
por la resistencia de la bobina estatrica y su reactancia de dispersin. El par de secuencia
positiva es mayor que el de secuencia negativa en este caso. En la figura 7.17 se presenta el
grfico de la caracterstica par-deslizamiento de una mquina de induccin en estas condiciones de operacin. Se ha indicado en la misma figura la caracterstica para la operacin
equilibrada.

3. Prdida de los fusibles de las fases b y c, con neutro conectado:


En la figura 7.18 se ha representado la condicin de operacin de un mquina de induccin con neutro corrido, en la cual se abren repentinamente las fases b y c.
Las condiciones de contorno en el dominio de las fases para este caso son:
Va = V 0 = V

(7.27)

Ib = Ic = 0

(7.28)

De la transformacin 7.8 y de la condicin de contorno 7.27, se obtiene:

1
Va = (V0 +V+ +V ) = V V0 +V+ +V = 3V
3

(7.29)

285

Captulo VII Operacin de la mquina de induccin

Figura 7.18 Apertura de las fases b y c con neutro conectado

Mediante la transformacin 7.10, y la condicin 7.28, se obtiene:


1 1 1
Ia
I0
1
I+ = 1 1 2 0 = 1 1 Ia
3 1 2
3 1
0
I

(7.30)

La expresin 7.30 indica que las tres corrientes de secuencia son iguales. Las relaciones
entre las tensiones de secuencia y las corrientes de secuencia estn determinadas por las
impedancias de secuencia 7.17. Mediante la ecuacin 7.29, el sistema 7.30 y las tres relaciones entre las tensiones y corrientes de secuencia 7.17 se establece el siguiente resultado
para este caso:


Z+ (s) + Z(s) + Z0 0
0
I+
3V

(7.31)
1
1 0 I = 0
1
0 1
I0
0
El sistema de ecuaciones 7.31, puede ser representado por un circuito equivalente donde
las tres impedancias
de secuencia se encuentran en serie, alimentadas por una fuente de
tensin de valor 3V . En este sistema la primera ecuacin representa la malla y las otras
dos los nodos del circuito. En la figura 7.19 se ha representado el circuito equivalente para
esta condicin de operacin. Es interesante destacar que es muy parecido al circuito de
la figura 7.13, haciendo la salvedad de que en este caso queda en serie con el circuito la
impedancia de secuencia cero. La conexin de la impedancia de secuencia cero en serie
con el circuito equivalente, reduce un poco las corrientes y tensiones sobre las redes de
secuencia. Esto limita aun ms el par de cada secuencia, pero la solucin se asemeja
mucho a la presentada en el grfico de la figura 7.14, correspondiente a la prdida del
fusible de la fase a de la mquina sin neutro conectado a la red. En estas condiciones,
tampoco existe par elctrico de arranque.

286

VII.4 Rgimen desequilibrado de las mquinas de induccin

Figura 7.19 Circuito equivalente de la operacin con las fases b y c abiertas y con neutro conectado

287

Captulo VII Operacin de la mquina de induccin


VII.5

A RMNICAS

TEMPORALES EN LA MQUINA DE INDUCCIN

En la modelacin de la mquina de induccin se ha considerado que las fuentes que alimentan


esta mquina pueden o no ser balanceadas, pero contienen una sola frecuencia, que se denomina
componente fundamental. En los procesos industriales se utilizan frecuentemente convertidores
electrnicos de potencia para la regulacin y el control. Estos equipos pueden introducir contenido armnico en las fuentes de alimentacin de las mquinas. Cuando estn presentes armnicas
temporales en la excitacin es necesario realizar consideraciones adicionales en el modelo para
poder realizar anlisis de la respuesta de la mquina.
La simetra de las ondas de excitacin inscriben el rango de las armnicas a los mltiplos impares de la componente de frecuencia industrial, o componente fundamental. Las armnicas
temporales ms frecuentes en los sistemas elctricos de potencia son las siguientes:
Armnica
frecuencia

1.
e

3.
3e

5.
5e

7.
7e

9.
9e

11.
11e

13.
13e

A continuacin se desarrolla el modelo de la mquina de induccin para cada una de las armnicas ms frecuentes. Si el convertidor es lineal, se puede utilizar el principio de superposicin
para determinar la respuesta completa de la mquina.
VII.5.1

Sistema de terceras armnicas 3e

En la figura 7.20 se presenta un sistema trifsico de tensiones, de primera y tercera armnica. Si


el sistema de primera armnica o fundamental es balanceado, las terceras armnicas se encuentran en fase. Un sistema de tres tensiones en fase se comporta exactamente igual que un sistema
de secuencia cero. Al estar en fase las tensiones, los flujos por las tres bobinas son iguales y se
anulan en el entrehierro de la mquina. El circuito de secuencia cero de la mquina de induccin modela adecuadamente este tipo de excitacin, si se tiene en cuenta que las reactancias se
incrementan en un factor de tres. La resistencia del modelo no se altera, siempre y cuando sea
posible despreciar el efecto pelicular en los conductores.
VII.5.2

Sistema de quintas armnicas 5e

Un sistema de tensiones trifsicas de quinta armnica tiene la siguiente estructura:

Va5 (et) = 2V5 sin 5(et) = 2V5 sin(5et)

(7.32)

2
4
) = 2V5 sin(5et )
(7.33)
3
3

4
2
Vc5 (et) = 2V5 sin 5(et ) = 2V5 sin(5et )
(7.34)
3
3
Estas tensiones corresponden a un sistema de secuencia negativa. Un sistema trifsico, sinusoidal
y balanceado de quintas armnicas se comporta como un sistema de secuencia negativa. Cuando
se alimentan las bobinas de una mquina trifsica con este sistema de tensiones, se produce
Vb5 (et) =

288

2V5 sin 5(et

VII.5 Armnicas temporales en la mquina de induccin

(a) sistema de terceras armnicas

(b) Modelo de terceras armnicas

Figura 7.20 Sistema de terceras armnicas temporales y modelo de la mquina

289

Captulo VII Operacin de la mquina de induccin

Figura 7.21 Circuito equivalente de la mquina de induccin excitada por quintas armnicas
un campo magntico rotatorio de secuencia negativa. El modelo de la mquina en este caso es
el circuito equivalente de la mquina en secuencia negativa, amplificando las reactancias por
cinco y calculando el deslizamiento correspondiente a la quinta armnica mediante la siguiente
relacin:
5e r 5e + r
r
6 s1
s5 =
=
= 1+
=
(7.35)
5e
5e
5e
5
El par elctrico producido por las quintas armnicas tiene la misma direccin que el campo
magntico rotatorio producido por estas componentes, es contrario a las agujas del reloj. En
la figura 7.21 se presenta el circuito equivalente de una mquina de induccin alimentada por
un sistema trifsico balanceado de tensiones de quinta armnica. Las resistencias del estator
y rotor se pueden mantener constantes, mientras que el efecto pelicular no sea importante. La
resistencia de magnetizacin para estas frecuencias se puede corregir para considerar el aumento
de prdidas en el hierro por incremento de la frecuencia de excitacin.

VII.5.3

Sistema de sptimas armnicas 7e

El sistema de tensiones trifsicas de sptimas armnicas tiene la siguiente estructura:

Va7 (et) = 2V7 sin 7(et) = 2V7 sin(7et)


(7.36)

2
2
Vb7 (et) = 2V7 sin 7(et ) = 2V7 sin(7et )
(7.37)
3
3

4
4
Vc7 (et) = 2V7 sin 7(et ) = 2V7 sin(7et )
(7.38)
3
3
Estas tensiones corresponden a un sistema de secuencia positiva. Un sistema trifsico balanceado de sptimas armnicas se comporta como un sistema de secuencia positiva. Cuando se
alimentan las bobinas de una mquina trifsica con este sistema de tensiones, se produce un
campo magntico rotatorio de secuencia positiva. El modelo de la mquina en este caso es el
circuito equivalente en secuencia positiva, amplificando siete veces las reactancias y calculando
el deslizamiento correspondiente a las sptimas armnicas mediante la siguiente relacin:
s7 =

290

7e r 6 + s1
=
7e
7

(7.39)

VII.5 Armnicas temporales en la mquina de induccin

Figura 7.22 Circuito equivalente de la mquina de induccin excitada por sptimas armnicas

Figura 7.23 Circuito equivalente para la armnica de orden h de secuencia positiva o negativa
El par elctrico producido por las sptimas armnicas tiene la misma direccin que el campo
magntico rotatorio producido por estas componentes, gira en el mismo sentido de las agujas del
reloj. En la figura 7.22 se presenta el circuito equivalente de una mquina de induccin alimentada por un sistema trifsico balanceado de tensiones de sptimas armnicas. Las resistencias
del estator y rotor se pueden mantener constantes, mientras que el efecto pelicular no sea importante. La resistencia de magnetizacin para estas frecuencias se puede corregir para considerar
el aumento de prdidas en el hierro por incremento de la frecuencia de excitacin.
VII.5.4

Sistema armnico de orden h he

Conocidos los modelos de primera, tercera, quinta y sptima armnica, es posible identificar la
generalizacin del modelo para cualquier armnica impar de orden h. El circuito equivalente
que modela el comportamiento de una mquina de induccin trifsica, excitada mediante una
fuente armnica de orden h se ha representado en la figura 7.23. Como en los casos anteriores,
las reactancias crecen proporcionalmente al orden de la armnica, las resistencias son constantes
mientras que pueda ser despreciado el efecto pelicular y el incremento de prdidas en el hierro.
Es importante destacar que aun cuando las prdidas en el hierro crecen con la frecuencia, la
densidad de flujo decrece segn la ley de Faraday, atenuando este incremento.
Para calcular el deslizamiento correspondiente a la armnica h, sh , es necesario determinar si
una armnica es de secuencia positiva, negativa o cero. Todas las armnicas mltiplos de tres
en un sistema trifsico son de secuencia cero, y como no producen campo magntico rotatorio,
no contribuyen a la produccin de par elctrico. El resto de las armnicas impares producen

291

Captulo VII Operacin de la mquina de induccin


pares elctricos positivos o negativos segn sea la secuencia del sistema armnico que genera el
campo magntico rotatorio.
Para que una armnica de orden h en un sistema trifsico sea de secuencia positiva, se debe
cumplir la siguiente relacin:
2
3k
2
h=
+ 2 k h = (2n 1) = 1 + 3k n = 1 +
3
3
2

(7.40)

Como h debe ser un nmero impar, n debe pertenecer a los nmero naturales, esto implica que
k debe ser mltiplo de dos:
k = 2m ; m N n = 1 + 3m h = 6m + 1 ; m N

(7.41)

Para que una armnica de orden h en un sistema trifsico sea de secuencia negativa, se debe
cumplir la relacin siguiente:
3(k + 1)
4
2
h=
+ 2 k h = (2n 1) = 2 + 3k n =
3
3
2

(7.42)

Como h debe ser un nmero impar, n debe pertenecer a los nmero naturales, y esto implica que
k + 1 debe ser mltiplo de dos:
k + 1 = 2m ; m N n = 3m h = 6m 1 ; m N, m 6= 0

(7.43)

En resumen:
Armnicas de secuencia positiva: h = 6m + 1, m = 0, 1, 2,
Armnicas de secuencia negativa: h = 6m 1, m = 1, 2, 3
El deslizamiento de una armnica de secuencia positiva se calcula mediante la siguiente monofsica:
sh+ =

h+ e r h+ e (1 s1 )e h+ 1 + s1
=
=
h+ e
h+ e
h+

(7.44)

y el deslizamiento para una armnica de secuencia negativa segn la expresin:


sh =

h e r h e + r h + 1 s1
=
=
h e
h e
h

(7.45)

Como aplicacin del modelo de armnicas temporales de la mquina de induccin se puede


evaluar la caracterstica par-deslizamiento de un convertidor alimentado mediante una fuente
trifsica balanceada de tensin peridica, no sinusoidal. Un de los casos ms frecuentes que
aparecen en la prctica se muestra en la figura 7.24, y corresponde a la salida de un inversor
trifsico sin modulacin de pulso.
Si se calcula la distribucin armnica de la fuente, mediante la descomposicin en series de Fourier, o con el algoritmo rpido de la transformada de Fourier 8 , se obtiene el espectro representado
en el ltimo grfico de la figura 7.24. Los valores de las diferentes componentes armnicas de
la tensin de alimentacin, obtenidos utilizando el algoritmo FFT, se resumen en la tabla 7.1.
8

FFT.

292

VII.5 Armnicas temporales en la mquina de induccin

Figura 7.24 Tensiones de alimentacin aplicadas a una mquina de induccin y su correspondiente espectro armnico

Frecuencia
Amplitud

1
0,955

5
0,192

7
0,138

11
0,089

13
0,076

17
0,059

19
0,054

Tabla 7.1 Amplitud de las tensiones armnicas del sistema de tensiones presentado en la figura
7.24

293

Captulo VII Operacin de la mquina de induccin

Figura 7.25 Distribucin armnica del par elctrico total


Mediante el espectro armnico se determinan las contribuciones al par elctrico de las principales armnicas, utilizando el circuitos equivalente ilustrado en la figura 7.23 correspondiente
al orden armnico considerado. Como en el caso considerado, las componentes armnicas son
reducidas con respecto a la componente fundamental, el par elctrico total es prcticamente
igual al par elctrico producido por la primera armnica de la fuente. En la figura 7.25 se han
representado las contribuciones al par elctrico de las armnicas consideradas, con respecto al
deslizamiento s. El par elctrico total TT se ha escalado por 100 para poder destacar las componentes armnicas. La reduccin de los pares armnicos en la mquina se debe principalmente a
dos factores: la reduccin de la tensin armnica que afecta cuadrticamente al par y el orden armnico que incrementa linealmente las reactancias, reduciendo an ms el par elctrico para esa
armnica de tensin. De este hecho, se deduce que la mquina de induccin se comporta como
un filtro pasabajo, atenuando los pares elctricos producidos por excitaciones de alta frecuencia.
Un comportamiento similar, pero no tan acentuado, ocurre con las corrientes.
VII.6

A RMNICAS

ESPACIALES EN LA MQUINA DE INDUCCIN

Los conductores en el interior de la mquina se encuentran repartidos dentro de las ranuras. La


distribucin de los conductores se realiza para producir fuerzas magnetomotrices sinusoidales
en el espacio. Como las ranuras se reparten uniformemente en la periferia, junto con la componente fundamental de la fuerza magnetomotriz, aparecen otras componentes. Los modelos de
la mquina desarrollados hasta el momento, consideran que la distribucin del campo es sinusoidal en el espacio. En esta seccin se analizan las causas y consecuencias de la presencia de
armnicas espaciales en el interior de la mquina.
El caso ms simple de distribucin de los conductores en una mquina se presenta en la figura
7.26. Se muestra un estator con dos ranuras, por una entran los conductores de la bobina y por la

294

VII.6 Armnicas espaciales en la mquina de induccin

Figura 7.26 Distribucin de la fuerza magnetomotriz en una mquina con dos ranuras

otra ranura salen los retornos. La fuerza magnetomotriz originada por esta mquina, se calcula
mediante la ley circuital de Ampre y se expresa de la siguiente forma:

Ni, si 0 <
(7.46)
F ( ) =
Ni, si < 2

La fuerza magnetomotriz 7.46 se puede expresar en series de Fourier mediante la siguiente


relacin:

4
sin(2k 1)
F ( ) = Ni
(7.47)
k=1 2k 1
Suponiendo que a la bobina de esta mquina se le inyecta corriente sinusoidal a una sola frecuencia:

4 2
sin(2k 1)
(7.48)
F ( ,t) =
NI sin(et)

2k 1
k=1
La expresin 7.48 determina la fuerza magnetomotriz en el espacio y en el tiempo, producida
por una bobina concentrada. Si la mquina posee m fases, espaciadas unas de otras un ngulo
de 2m y se inyecta a cada fase una corriente sinusoidal de igual magnitud y frecuencia, pero
desfasada temporalmente el mismo ngulo espacial de la bobina, la fuerza magnetomotriz para
la bobina genrica q es:



sin (2k 1)( 2 q)


2
4 2
m
(7.49)
NI sin(et q)
Fq ( ,t) =

m k=1
2k 1
295

Captulo VII Operacin de la mquina de induccin


Incluyendo el trmino sinusoidal temporal de la expresin anterior en la sumatoria y descomponiendo el producto de los senos en suma de cosenos, se obtiene el siguiente resultado:

 


2 2
4
1
Fq ( ,t) =
NI
cos et (2k 1) (1 k)q +

m
k=1 2k 1


4
(7.50)
cos et + (2k 1) kq
m
La expresin anterior indica que la fuerza magnetomotriz que produce cada bobina se descompone en infinitos campos magnticos rotatorios de secuencia positiva y de secuencia negativa.
La velocidad de fase de cada uno de estos campos es un subarmnico de la frecuencia de las
corrientes inyectadas en las bobinas.
La fuerza magnetomotriz resultante en el entrehierro se obtiene al sumar las contribuciones de
las m fases:

 

2 2 m1
4
1
cos et (2k 1) (1 k)q +
FT ( ,t) =
NI

m
q=0 k=1 2k 1


4
cos et + (2k 1) kq
(7.51)
m
2(1k)

Si m es un nmero entero, la superposicin de las m fuerzas magnetomotrices de secuencia


positiva se encuentran en fase para cada una de las armnicas espaciales. Por otra parte si 2k
m
es un nmero entero, entonces las fuerzas magnetomotrices de secuencia negativa son las que
estn en fase para cada uno de los rdenes armnicos. Cuando no se cumplen estas condiciones,
se superponen por cada armnica m cosenos igualmente desfasados unos de otro y la sumatoria
de estos trminos para las m bobinas es:

2 2
cos [et (2k 1)]
1k
FT2k1 ( ,t) =
mNI
; si 2
Z
(7.52)

2k 1
m

2k
2 2
cos [et + (2k 1)]
; si
Z
(7.53)
mNI
FT2k1 ( ,t) =

2k 1
m
2k
1k
FT2k1 ( ,t) = 0 ; si 2

/Z

/Z
(7.54)
m
m
La fuerza magnetomotriz armnica 7.52 produce un campo magntico rotatorio de secuencia
positiva cuya velocidad de fase es:
e
d
=
(7.55)
dt
2k 1
La fuerza magnetomotriz armnica 7.53 produce un campo magntico rotatorio de secuencia
negativa y su velocidad de fase es:
d
e
=
(7.56)
dt
2k 1
Las armnicas espaciales generan campos magnticos rotatorios que giran a velocidades que son
submltiplos de la velocidad sincrnica. Las armnicas espaciales se comportan, en el interior

296

VII.7 La mquina de induccin bifsica


de la mquina, como si sta tuviera el nmero de pares de polos correspondiente al orden de la
armnica. La amplitud de cada armnica depende de la distribucin de las bobinas en las ranuras
de la mquina. En una mquina cuyas bobinas se encuentran concentradas en un par de ranuras,
la amplitud de la fuerza magnetomotriz disminuye directamente con el orden de la armnica,
el par elctrico se calcula a partir del producto de las fuerzas magnetomotrices y por esta razn
disminuye con el cuadrado del orden de la armnica espacial correspondiente.
En las mquinas reales, las bobinas de cada fase se distribuyen en varias ranuras y el contenido
armnico espacial se puede reducir aun ms. Adems es posible eliminar algunas armnicas
mediante el acortamiento de paso9 . Este acortamiento reduce la contribucin de las fuerzas
electromotrices producidas por las bobinas. El factor de paso en una bobina con acortamiento
de paso para la primera armnica espacial es:
k p1 = cos

(7.57)

La armnica espacial 2k 1 se repite 2k 1 veces en el interior de la mquina. A cada perodo


2
espacial de repeticin del campo armnico le corresponde un ngulo mecnico igual a 2k1
.
Desde el punto de vista de las armnicas espaciales, el acortamiento de paso es un ngulo 2k 1
veces mayor. Por esta razn se tiene:
k p(2k 1) = cos

(2k 1)
2

(7.58)

Para eliminar las fuerzas magnetomotrices correspondientes a una armnica determinada, se


puede utilizar la relacin 7.58. Si se quiere eliminar la armnica 2k 1, se tiene:
k p(2k 1) = cos

(2k 1)

(2k 1)
=0
=
=
2
2
2
2k 1

(7.59)

En la figura 7.27 se muestra la caracterstica par elctrico-deslizamiento de una mquina de


induccin trifsica, con las bobinas de cada fase concentradas en un par de ranuras. Tambin se
han representado en el mismo grfico las contribuciones al par elctrico de la quinta y sptima
armnica espacial. En las mquinas reales, estos efectos se ven reducidos considerablemente
por la distribucin de los conductores en la periferia de la mquina.
VII.7

LA

MQUINA DE INDUCCIN BIFSICA

En el sector industrial, comercial y residencial se utiliza una gran variedad de mquinas de induccin monofsicas. Estas mquinas se encuentran en el rango de potencia comprendido entre
fracciones de kW y un mximo de 2 kW aproximadamente. Accionan electrodomsticos, bombas, ventiladores, extractores, compresores, cadenas de montaje, transporte de materias primas,
etc. Aun cuando se les denomina mquinas monofsicas, este nombre se refiere a la fuente de
alimentacin, porque en su estructura interna deben poseer al menos dos fases10 para que sea
posible producir par elctrico en cualquier deslizamiento.
9
10

El retorno de cada bobina se realiza en un ngulo inferior a 180.


Generalmente ortogonales para maximizar la produccin del par elctrico.

297

Captulo VII Operacin de la mquina de induccin

Figura 7.27 Efecto de las componentes armnicas espaciales en el par elctrico


Una mquina monofsica, produce un campo magntico pulsante cuando se excita su devanado
con corriente alterna. En una mquina bifsica, las fases se encuentran a 180 y el campo magntico que producen las corrientes 11 inyectadas en estas dos bobinas, tambin son pulsantes. El
caso trifsico no requiere explicaciones adicionales porque ha sido analizado con detalle a lo
largo del texto.
La mquina tetrafsica es muy peculiar e interesante, posee cuatro fases separadas espacialmente
90 unas de otras. Las corrientes balanceadas, inyectadas en una mquina tetrafsica, tambin
estn desfasadas 90 en el tiempo. Estos dos hechos implican que las bobinas a y c producen
flujos iguales, y con las bobinas b y d sucede exactamente lo mismo. En definitiva existen dos
grados de libertad, se genera campo en la direccin de las bobinas a y c, o en la direccin de
las bobinas b y d. La mquina bifsica convencional, es una mquina tetrafsica donde tanto las
fases a y c, como las fases b y d estn conectadas en serie.
La mquina bifsica desarrollada a partir de una mquina tetrafsica, funciona en rgimen equilibrado, exactamente igual que una mquina trifsica convencional. En la prctica es frecuente
operar estas mquinas con una fuente monofsica. Esto produce desequilibrios en la mquina
que deben ser analizados mediante las transformaciones de componentes simtricas. En esta
seccin se particularizar la teora de las componentes simtricas para el anlisis de la mquina
de induccin bifsica.
Un sistema lineal puede ser analizado mediante la formulacin de un sistema de ecuaciones lineales. Esta formulacin expresa las interrelaciones existentes entre las diferentes variables de
estado del sistema y generalmente estas variables pueden estar fuertemente interrelacionadas. Se
dice en esta situacin que el sistema de ecuaciones est acoplado. En los sistemas lineales algebraicos es relativamente sencillo invertir la matriz de acoplamientos, para calcular las variables
11

Bifsicas equilibradas: Ia = I, Ib = I.

298

VII.7 La mquina de induccin bifsica


de inters. En los sistemas de ecuaciones diferenciales lineales esta tcnica es ms compleja. El
anlisis modal mediante autovalores y autovectores, permite desacoplar un sistemas lineal de n
ecuaciones diferenciales de primer orden, en n sistemas completamente independientes.
En la modelacin de las mquinas de induccin aparecen siempre acoplamientos que pueden ser
expresados mediante matrices cclicas12 . La diagonalizacin de estas matrices mediante mtodos
modales es de gran utilidad prctica. El caso trifsico puede orientar sobre la generalizacin de
este mtodo al caso tetrafsico. Para diagonalizar una matriz cclica trifsica se deben obtener
en primer lugar los tres autovalores asociados:

b
c
a
a
b =0
det([C] [I]) = 0 det c
b
c
a
(a )2 + b2 + c2 3bc(a ) = 0

0 = a + b + c

(7.60)

4
3

(7.61)

2
3

(7.62)

1 = a + be j 3 + ce j
1 = a + be j 3 + ce j

La matriz de autovectores asociada con la matriz cclica es


componentes simtricas:

([C] [I]) [V ] ; [V ] = [V1 ] [V2 ] [V3 ]

1 1
1
1 1
4
2
1
1
[V ] = 1 e j 3 e j 3 = 1
4
2
3
3 1 2
1 ej 3 ej 3

la transformacin hermitiana de
[Vn ]

1
2
2 , = e j 3

(7.63)

Un procedimiento similar al realizado en 7.60 a 7.63 permite determinar la transformacin de


componentes simtricas tetrafsicas, que corresponde a la matriz de autovalores de la matriz
cclica tetrafsica:

b
c
d
a
d
a
b
c
=0
det([C] [I]) = 0 det
c
d
a
b
b
c
d
a

0 = a + b + c + d

1 = a + be j 2 + ce j2 2 + de j3 2 = (a c) + j(b d)
2 = a + be

12

j2 2

+ ce

j4 2

+ de

j6 2

= ab+cd

(7.64)
(7.65)
(7.66)

Tambin aparecen matrices completamente simtricas que pueden ser un caso particular de las cclicas. En el
sistema de ecuaciones 6.1 se pueden destacar estos tipos de matrices.

299

Captulo VII Operacin de la mquina de induccin

3 = a + be j3 2 + ce j6 2 + de j9 2 = (a c) + j(d b)

(7.67)

La matriz hermitiana de autovectores correspondiente es:


1 1

1 1 ej 2
[V ] =
j2
4 1 e 2
1 e j3 2

e j2 2

e j4 2

e j6 2

e j3 2

e j4 2

e j9 2

1 1
1
1
1 1 j 1 j

=
2 1 1 1 1
1 j 1 j

(7.68)

La primera y tercera fila de la matriz de autovalores 7.68 determinan las dos posibles componentes de secuencia cero de un sistema tetrafsico. La segunda fila define la componente de
secuencia positiva. La ltima fila de la expresin 7.68 determina la componente de secuencia
negativa en el sistema tetrafsico. Esta informacin se deduce aplicando la transformacin a un
vector de entrada de la secuencia adecuada. Por ejemplo, si se desea demostrar que la segunda
fila genera la secuencia positiva, se aplica la transformacin al vector de secuencia positiva:

0
1
x0
1 1
1
1
x1 1 1 j 1 j j 2

(7.69)
x2 = 2 1 1 1 1 1 = 0
0
j
x3
1 j 1 j

En las mquinas bifsicas de induccin la corriente de la fase a est en oposicin a la fase


c y la corriente de la fase b en oposicin a la de la fase d. Incluyendo estas condiciones de
contorno en la transformacin 7.69, y escogiendo los coeficientes apropiadamente para que la
nueva transformacin sea hermitiana, se obtiene:

x0
xa
1 1
1
1
0 0


x1 1 1 j 1 j xb 1 2 2 j xa

=
x2 2 1 1 1 1 xa 2 0 0 xb
x3
xb
1 j 1 j
2 2 j







1
1
x+
x1
1 j
xa
=
=
(7.70)
x
xb
2 x3
2 1 j





1
1 1
x+
xa
(7.71)
=
x
xb
2 j j
Las expresiones 7.70 y 7.71, se conocen como la transformacin directa e inversa de componentes simtricas bifsicas. En la figura 7.28 se presentan dos grupos de fasores, el de la izquierda
gira en secuencia positiva y el de la derecha en secuencia negativa.
Si a los devanados de la mquina bifsica de induccin se les aplican tensiones bifsicas desequilibradas, la expresin 7.70 permite descomponer en secuencia positiva y negativa las tensiones
de fase. Como la mquina bifsica equilibrada no se diferencia en su comportamiento de la
mquina de induccin trifsica equilibrada. Las redes de secuencia de la mquina bifsica son
iguales a las redes de secuencia positiva y negativa de una mquina trifsica. En la figura 7.29
se presenta una aplicacin de la mquina bifsica de induccin para el control de velocidad.
Mediante un transformador con relacin variable se puede ajustar la tensin de alimentacin en
una de las fases de la mquina. Este control permite ajustar el par y la velocidad del motor.

300

VII.7 La mquina de induccin bifsica

Figura 7.28 Fasores de secuencia positiva y negativa en un sistema bifsico equilibrado

Figura 7.29 Control de par elctrico mediante una mquina de induccin bifsica

301

Captulo VII Operacin de la mquina de induccin


Las tensiones de secuencia en la mquina de la figura 7.29 son:







1
1
1 j
V 0
1+k
V+
=
V
=
kV 90
V
2 1 j
2 1k

(7.72)

Variando la relacin de transformacin k se controla el par elctrico de la mquina. Cuando k =


1, el sistema est alimentado slo por secuencia positiva. A medida que k disminuye, aumenta
el par de secuencia negativa reduciendo el par efectivo. Si el valor de k invierte su polaridad,
la mquina girar en sentido contrario. Los deslizamientos de secuencia positiva y negativa se
calculan con la misma metodologa empleada en la mquina trifsica s+ + s = 2. El par se
evala superponiendo las contribuciones de secuencia positiva y negativa, obtenidas mediante
los circuitos equivalentes de la secuencia correspondiente.
En la pgina 285 se analiz el comportamiento de una mquina de induccin trifsica con las
fases b y c desconectadas. Esta mquina no produce par de arranque, pero es capaz de mantener
par elctrico en puntos de operacin cuyos deslizamientos estn prximos a cero. Este hecho
es utilizado para operar las mquinas de induccin alimentadas mediante redes monofsicas. El
arranque de estos motores implica consideraciones adicionales que sern discutidas posteriormente.
En la figura 7.30 se presenta el esquema de una mquina monofsica de induccin y la distribucin de la fuerza magnetomotriz producida por una corriente sinusoidal que circula por su
bobina. La forma de la fuerza magnetomotriz se mantiene constante, pero la magnitud vara
en el tiempo, proporcional a la corriente inyectada instantneamente. La fuerza magnetomotriz
producida por esta mquina es pulsante, se mantiene la forma en el espacio, pero vara su amplitud en el tiempo. Un campo pulsante se pueden descomponer en dos campos rotantes con
velocidades de fase contrarias:

Fa ( ,t) = Nia (et) cos = 2NI cos(et) cos

2NI
(7.73)
Fa ( ,t) =
[cos(et ) + cos(et + )]
2
En la figura 7.30 se muestra cmo la composicin de dos fuerzas magnetomotrices rotatorias, de
sentidos de giro contrarios e iguales en amplitud, producen una fuerza magnetomotriz pulsante.
La descomposicin de la fuerza magnetomotriz pulsante en dos fuerzas magnetomotrices rotantes iguales y contrapuestas en su sentido de giro, permite obtener un circuito equivalente para
la mquina de induccin monofsica. Cada una de las fuerzas magnetomotrices rotantes tiene la
mitad de amplitud de la fuerza magnetomotriz pulsante. Las fuerzas electromotrices generadas
por cada una de las fuerzas magnetomotrices rotantes, es la mitad de la fuerza electromotriz total
en la bobina de la mquina. Durante el arranque (s = 1), no existe par elctrico en la mquina
debido a que la componente de secuencia positiva es igual y contraria a la componente de secuencia negativa (Te = T+ T = 0). Las dos redes de secuencia en esta condicin son iguales,
y debe circular la misma corriente por cada una de ellas. Las redes de secuencia deben estar
conectadas en serie, para garantizar la igualdad de corriente y la superposicin de las fuerzas
electromotrices. Los parmetros de cada red de secuencia deben ser la mitad de los parmetros
de la bobina, para producir la mitad de la fuerza electromotriz de la mquina en cada secuencia,

302

VII.7 La mquina de induccin bifsica

Figura 7.30 Descomposicin de la fuerza magnetomotriz pulsante en dos rotantes

cuando el rotor est detenido. En la figura 7.31 se presenta el circuito equivalente propuesto para
la mquina monofsica de induccin.
Las componentes simtricas bifsicas permiten determinar el circuito equivalente de la mquina de induccin monofsica. Conectando una fuente monofsica a la fase a de una mquina
bifsica, se obtienen las siguientes condiciones de contorno:
Va = V 0 = V

(7.74)

Ib = 0

(7.75)

Transformando las condiciones de contorno 7.74 y 7.75 mediante la transformacin de componentes simtricas bifsicas 7.70 se obtienen las siguientes relaciones:

V+ +V = Z+ (s)I+ + Z (s)I = 2V
(7.76)
I+ = I

(7.77)

De las expresiones 7.76 y 7.77, aplicando la transformacin inversa de componentes simtricas


7.71, se obtiene:

Ia
2V = [Z+ (s) + Z(s)] I+ = [Z+ (s) + Z (s)]
2
Z+ (s) + Z(s)
V=
Ia
(7.78)
2
La expresin 7.78 coincide con el circuito equivalente presentado en la figura 7.31. De esta forma, el razonamiento intuitivo utilizado para obtener este circuito a partir de la descomposicin
de la fuerza magnetomotriz pulsante en fuerzas magnetomotrices rotantes queda demostrado.

303

Captulo VII Operacin de la mquina de induccin

Figura 7.31 Circuito equivalente de la mquina monofsica de induccin


Si el rotor no est detenido, las impedancias de secuencia positiva y negativa son diferentes.
Aparece una diferencia en las tensiones de secuencia y en los campos magnticos rotatorios
de secuencia positiva y negativa. La superposicin de dos campos magnticos rotatorios contrarotativos produce un campo pulsante. Si estos campos rotantes difieren en amplitud se obtiene un
campo magntico elptico. En el eje magntico de la bobina, el campo elptico obtiene una amplitud mxima y a 90 obtiene la amplitud mnima. En la figura 7.32 se muestra la forma del campo
magntico elptico producido durante la operacin de la mquina de induccin monofsica.
Cuando el rotor de la mquina est detenido, las dos fuerzas magnetomotrices son iguales, y
los pares se neutralizan. Si existe una velocidad en cualquiera de los dos sentidos, el campo
pulsante induce fuerza electromotriz en el rotor, esta fuerza electromotriz fuerza la circulacin
de corriente por este circuito, creando un campo magntico que refuerza una de las componentes
y debilita a la otra. En la figura 7.33 se presenta la caracterstica par-deslizamiento de este tipo
de mquinas.
Como la mquina de induccin monofsica no puede arrancar por s sola, es necesario utilizar
bobinas auxiliares para producir par elctrico durante el proceso de aceleracin de la mquina y
su carga. Mientras que el rotor de la mquina monofsica est detenido, el campo en el entrehierro es pulsante y no puede producir par. Es necesario un campo rotante, circular o elptico para
el arranque autnomo de la mquina. Durante la aceleracin de la carga mecnica se aade a la
mquina de induccin monofsica un bobina auxiliar en cuadratura con la bobina principal. En
estas condiciones el funcionamiento corresponde al de una mquina bifsica desequilibrada.
Aun cuando la mquina monofsica posee un devanado auxiliar en cuadratura con el devanado
principal, no se puede asociar directamente a una mquina bifsica. La bobina auxiliar se utiliza
en general, solamente durante el proceso de arranque y se disea con una seccin mucho menor
que la del devanado principal. Una vez que la mquina alcanza una velocidad comprendida

304

VII.7 La mquina de induccin bifsica

Figura 7.32 Campo magntico elptico de la mquina monofsica

Figura 7.33 Caracterstica par-deslizamiento de la mquina monofsica de induccin

305

Captulo VII Operacin de la mquina de induccin

Figura 7.34 Mquina monofsica de induccin con circuito auxiliar de arranque


entre un 70 y un 80 por ciento de la velocidad sincrnica, un interruptor centrfugo desconecta
la bobina auxiliar con la finalidad de aumentar el rendimiento de la mquina en el punto de
operacin.
Las diferencias fsicas entre la bobina auxiliar de arranque y el devanado principal de la mquina,
permiten obtener mediante una fuente de tensin monofsica, corrientes diferentes y desfasadas
en las dos bobinas. Al estar desfasadas estas corrientes, se produce un campo elptico y se
obtiene par elctrico. El desfase entre las corrientes se puede incrementar utilizando resistencias,
inductancias o condensadores en serie con la bobina auxiliar. Lo ms frecuente es conectar
condensadores en serie con la bobina auxiliar para producir el mximo desfase posible entre las
dos corrientes, incrementando de esta forma el par de arranque. En la figura 7.34 se presenta el
diagrama de esta solucin.
Las mquinas monofsicas comerciales, requieren condensadores muy grandes durante el arranque (50 200 F). Estos condensadores se disean especialmente para que sean econmicos,
a expensas de producir prdidas considerables cuando estn en operacin. Para incrementar el
rendimiento de la mquina, se desconecta el condensador de arranque y el devanado auxiliar
luego de la aceleracin de la carga. Al desconectar el condensador y la bobina auxiliar mediante
el interruptor centrfugo, la mquina contina su operacin como motor monofsico.
El anlisis de la mquina de induccin monofsica con devanado auxiliar y condensador de
arranque se realiza mediante la transformacin de componentes simtricas bifsicas, pero es
necesario realizar algunas consideraciones previas. Las componentes simtricas diagonalizan
sistemas acoplados, cclicos o simtricos13. Esto implica que la red debe ser simtrica, para
que los modos de secuencia resulten desacoplados. La mquina de induccin monofsica con
devanado auxiliar de arranque no es una mquina simtrica, sus bobinas se disean con diferente
nmero de vueltas. Las resistencias de las bobinas tambin pueden ser diferentes en ambas
bobinas. Para que el mtodo de las componentes simtricas desacople las redes de secuencia
13

Los sistemas simtricos son sistemas cclicos donde a 6= b = c = d.

306

VII.7 La mquina de induccin bifsica

Figura 7.35 Modelo equilibrado de la mquina monofsica con devanado auxiliar de arranque
es necesario equilibrar la mquina. Para este fin se supone que la mquina est constituida por
dos bobinas iguales separadas espacialmente 90, y que la diferencia en el nmero de vueltas
de la bobina auxiliar se obtiene mediante un transformador externo de relacin Naux a N p . Las
diferencias en resistencias o en impedancias conectadas en serie con el devanado auxiliar se
corrigen con la conexin externa de una impedancia Zx que realiza el balance.
Si la mquina se transforma a una mquina bifsica balanceada, sus bobinas tendrn el nmero
de vueltas N p del enrollado principal, y todos los desequilibrios se transfieren como conexiones
externas al convertidor. El nuevo esquema se resuelve mediante la descomposicin en componentes simtricas bifsicas. En la figura 7.35 se presenta el esquema del circuito propuesto, con
la mquina equilibrada y las conexiones externas necesarias para reproducir la situacin original
de la mquina bifsica desequilibrada.
Para mantener la fuerza magnetomotriz de la bobina auxiliar en la mquina bifsica equilibrada
se debe cumplir la siguiente relacin:

Faux = Naux Iaux = N p Iaux

Iaux Naux 1
=
=
Iaux
Np
a

(7.79)

La impedancia de entrada hacia el devanado auxiliar, vista desde la red, tiene que mantenerse
constante antes y despus de equilibrar la mquina:
Zarranque + Zaux =

Zx + Z p

a2

Zx = a2 (Zarranque + Zaux ) Z p

(7.80)

307

Captulo VII Operacin de la mquina de induccin


El circuito equivalente de la mquina monofsica con condensador de arranque se puede obtener
ahora utilizando las componentes simtricas bifsicas. Las tensiones de secuencia son:








1
1
V
1 j
Vp
1 j
V+
=
(7.81)
=

Vaux
V
aV Zx Iaux
2 1 j
2 1 j
Las corrientes de fase se expresan en funcin de las corrientes de secuencia, mediante la transformacin inversa:







1
1
Ip
1 1
I+
I+ + I
=
=
(7.82)

I
Iaux
2 j j
2 j(I I+ )

Sustituyendo el valor de Iaux obtenido en la expresin 7.82, en el sistema de ecuaciones 7.81 se


obtiene el siguiente resultado:
"
# 



(1 + ja)V Zx (I+ I )
1
Z+ I+
V+
2
(7.83)
=
=
Z
Z I
V
2 (1 ja)V x2 (I+ + I )
Agrupando trminos en la expresin 7.83 se obtiene el siguiente sistema de ecuaciones:
" 1+ ja # 


Zx
Zx
V
)

I
(Z
+
+
+
2
2
2
(7.84)
=
1
ja V
I
Z2x
(Z + Z2x )
2

El sistema de ecuaciones 7.84, representa el circuito equivalente de la figura 7.36. Donde Z+ y


Z son las impedancias de secuencia positiva y negativa de la mquina de induccin bifsica
equilibrada. Mediante el circuito equivalente de la mquina de induccin monofsica durante el
arranque, o con el sistema de ecuaciones 7.36, se pueden calcular las corrientes de secuencia y
las tensiones de secuencia. Como las fuentes de tensin aplicadas en las dos mallas del circuito equivalente son diferentes, las corrientes de secuencia tambin lo son, aun cuando el rotor
est detenido y las impedancias de secuencia sean iguales. Esto explica la aparicin de par de
arranque en la mquina monofsica de induccin con devanado auxiliar.
La impedancia Zx es responsable de la diferencia existente entre las corrientes de secuencia
positiva y negativa. Por esta razn el ajuste de esta impedancia permite variar el par elctrico
de arranque. Esta impedancia, incluye la impedancia adicional para incrementar el desfase entre
las corrientes de secuencia y un trmino que equilibra la mquina original, para que las redes de
secuencia queden desacopladas y sea vlida la expresin 7.83.
Al conectar un condensador para incrementar el par de arranque, la corriente en la bobina auxiliar adelanta a la corriente de la bobina principal. En un sistema bifsico de corrientes con
adelanto de la fase auxiliar con respecto a la principal, el campo magntico de secuencia negativa es mayor que el de secuencia positiva y la mquina gira en el sentido contrario al de las
agujas del reloj. Para invertir el sentido de giro, es suficiente con invertir la polaridad de una de
las dos bobinas.
En la figura 7.37 se muestran las caractersticas par elctrico-deslizamiento de una mquina
monofsica de induccin con el devanado auxiliar conectado y desconectado. En serie con el
devanado auxiliar se ha conectado un condensador que permite incrementar el par de arranque

308

VII.8 Anlisis transitorio

Figura 7.36 Circuito equivalente de la mquina monofsica de induccin con devanado auxiliar
de la mquina. Cuando la mquina alcanza un 70 por ciento de la velocidad de sincronismo,
se desconecta el devanado auxiliar y contina la operacin como motor monofsico. Algunas
mquinas se disean para mantener un condensador ms pequeo y con menos prdidas pero
que se mantiene conectado indefinidamente.
La mquina de induccin monofsica con devanado auxiliar es capaz de producir pares de arranque mayores que el par nominal. Esta importante ventaja es til en aplicaciones donde la cargas mecnicas en reposo requieren par de accionamiento muy grande. Los compresores son un
ejemplo de este tipo de carga, por esta razn es frecuente el uso de motores monofsicos con
devanados auxiliares en los equipos de refrigeracin comercial y residencial. Otro ejemplo de
aplicacin es en gras o pequeos elevadores. Si la potencia requerida por la carga mecnica
supera los 2 kW, el accionamiento monofsico no es conveniente por razones de eficiencia y es
ms aconsejable utilizar mquinas trifsicas.
VII.8

A NLISIS

TRANSITORIO

En la seccin VI.3 se obtuvo el modelo dinmico de la mquina de induccin expresado en el


sistema de coordenadas de los vectores espaciales referidos al sistema de referencia del estator 14 .
Esta representacin tiene las ventajas de ser independiente de la posicin angular 15 y reducir
la dimensin del sistema de ecuaciones diferenciales. Por otra parte, las variables de estado en
este modelo estn acopladas.
Un nivel de simplificacin y desacoplamiento mayor se obtiene en el modelo al proyectar los
diversos fasores espaciales con respecto a una referencia determinada. Estas proyecciones son
14
15

Sistema de ecuaciones 7.93 y figura 6.6.


Aun cuando se mantiene la dependencia con la velocidad angular r .

309

Captulo VII Operacin de la mquina de induccin

Figura 7.37 Comparacin de las caractersticas par-deslizamiento con el devanado auxiliar


conectado y desconectado
equivalentes a realizar una rotacin de los vectores espaciales a las coordenadas dq analizada
en el captulo 4. En la transformacin clsica a coordenadas dq el ngulo de rotacin se define
entre la referencia del estator 16 y la posicin del rotor. En general, se pueden seleccionar infinitas
referencias de rotacin completamente arbitrarias tales como la posicin del fasor espacial de
la corriente del estator, la corriente del rotor o la corrien te de magnetizacin y la seleccin de
cualquiera de estos patrones depende del anlisis o la aplicacin que se est realizando:

1. El vector espacial de la corriente del estator puede ser medido directamente.


2. El vector espacial de la corriente de magnetizacin est asociado directamente con el flujo
resultante en el entrehierro y con la produccin del par elctrico.
3. El vector espacial de la corriente del rotor tiene incidencia sobre el rendimiento de la
mquina y la transferencia de potencia al eje mecnico.
4. Posicin arbitraria , permite acelerar la integracin numrica de las variables de estado
del modelo cuando se sintoniza esta referencia con las fluctuaciones de las fuentes o de la
velocidad de rotacin.
5. El vector espacial de la corriente de magnetizacin modificada puede desacoplar las derivadas de los vectores espaciales de las corrientes del estator y rotor proporcionando un
modelo de la mquina de induccin donde se puede independizar la generacin del flujo y
16

Generalmente el eje magntico de la fase a.

310

VII.8 Anlisis transitorio

Figura 7.38 Par elctrico y velocidad angular de la mquina de induccin excitada con un sistema de tensiones trifsicas balanceadas
la produccin del par.17 Es una de las referencias ms utilizada en la literatura y se conoce
como modelo de campo orientado.
En la figura 7.38 se muestra el par elctrico y la velocidad angular producida por una mquina de
induccin excitada mediante un sistema de tensiones trifsicas balanceadas utilizando el modelo
7.93 definido en el sistema de coordenadas de las corrientes del estator. El cdigo fuente 6
desarrolla el modelo numrico de la mquina de induccin en este sistema de coordenadas.
La corriente de magnetizacin modificada que determina la referencia del modelo de campo
orientado se define como:
im ie +

Lr e
i = im (t) e j (t)
Mer r

(7.85)

El trmino MLerr refiere al sistema de referencia del estator todo el campo magntico producido
por las corrientes del rotor que atraviesa el entrehierro de la mquina. En la figura 7.39 se presenta un diagrama de los vectores espaciales correspondientes a las corrientes de la mquina. El
vector espacial de la corriente del estator se puede representar mediante dos componentes ortogonales, una paralela al fasor espacial de la corriente de magnetizacin im y la otra en cuadratura,
17

La difusin de este modelo se debe a la posibilidad de utilizar los esquemas de control de las mquinas de
corriente continua para regular la velocidad de las mquinas de induccin.

311

Captulo VII Operacin de la mquina de induccin

Algoritmo 6 Modelo de la mquina de induccin en vectores espaciales referidos al estator


//Modelo dinmi o de la mquina de indu in
//Programa prin ipal (S ilab 3.0)
global R L L_1 G Jm Tm Ve j VS Ler
j= %i;VS=sqrt(2/3)*[1 exp(j*2* %pi/3) exp(j*4* %pi/3);
Re=.02; Rr=0.06; Le=3.1; Lr= 3.1; Ler=3.0; Jm=250;
Ve=1;Tm=0;
R = [Re 0;0 Rr; L=[Le Ler;Ler Lr; G=[0 0;Ler Lr; L_1=inv(L);
x0=[0;0;0;0;0;0;t0=0;t=0:0.001*377:.5*377;
x=ode(x0,t0,t,maquina);
subplot(2,1,1)
plot(t/377,x(5,:))
Te=Ler*imag((x(1,:)+j*x(2,:)).*(x(3,:)-j*x(4,:)));
xgrid
subplot(2,1,2)
plot(t/377,Te)
xgrid
//Fin del programa prin ipal

//Cl ulo de derivadas del modelo de la mquina


fun tion px=maquina(t,x)
global R L L_1 G Jm Tm Ve j VS Ler
iae=x(1); ibe=x(2); iar=x(3); ibr=x(4); wm=x(5); theta=x(6);
ie=iae+j*ibe;
ir=iar+j*ibr;
ve=Ve*VS*[ os(t); os(t-2*%pi/3); os(t-4*%pi/3);
vr=0+j*0;
pii = L_1*([ve;vr-(R-j*wm*G)*[ie;ir);
pwm= (Ler*imag(ie*ir')-Tm)/Jm;
px(1)=real(pii(1));
px(2)=imag(pii(1));
px(3)=real(pii(2));
px(4)=imag(pii(2));
px(5)=pwm;
px(6)=wm;
endfun tion
//Fin del l ulo de las derivadas

312

VII.8 Anlisis transitorio

Figura 7.39 Vectores espaciales de las corrientes del modelo de la mquina de induccin
denominadas ide e iqe respectivamente. De acuerdo con la figura 7.39 se tiene:
ide (t) + jiqe(t) = ie e j (t) = (i e + ji e ) (cos j sin )


ide
iqe

i e
i e

cos sin
sin cos



i e
i e

(7.87)

cos sin
sin cos



ide
iqe

(7.88)

Donde:
ie = i e + ji e =

iae =

i e
i e

iae
ibe

(7.86)




2
e(ie ) ; ibe =
3

2
4
2
(iae + e j 3 ibe + e j 3 ice )
3

" q

1
2

" q

3
2

2
3
1

0
2
2

0
1
2

(7.89)

#

iae
ibe

(7.90)

#

i e
i e

(7.91)

2
2
e(ie e j 3 ) ; ice =
3

4
2
e(ie e j 3 )
3

(7.92)

Reemplazando la corriente ier de la definicin 7.85 de la corriente de magnetizacin modificada


im en el modelo de la mquina de induccin en coordenadas vectoriales referidas a las corrientes
del estator 7.93, se obtiene:

313

Captulo VII Operacin de la mquina de induccin

ve
ver

Re 0
0 Rr

 

ie
Le Mer
p Mer
+
+
Mer
Mer Lr
Lr (im ie )
Lr (im ie )



ie
0
0

j
Mer
Mer Lr
Lr (im ie )



ie

 
 
Mer
Mer m ie
(im ie )
Tm ( ) = J +
Lr

(7.93)

Reagrupando las variables de estado del sistema 7.93 se obtiene el modelo de la mquina de
induccin expresado en coordenadas de campo orientado:

ve
1 e
Mer vr

Re
T1r

0
1
Tr



ie
im

"

Le
0

0 0
0 1

2
Mer
Lr



# 

ie
p
+
im
1


2
Mer
Lr

ie
im

2
Mer
m {ie im } Tm ( ) = J +
Lr

Donde:
Tr =

Lr
Rr

(7.94)

(7.95)

La expresin del par elctrico en el modelo de la mquina de induccin en coordenadas de


campo orientado se simplifica si se incluye la definicin de la corriente de magnetizacin 7.85
y la transformacin de la corriente del estator a coordenadas dq 7.86:
Te =

n
o M2
2
M2
Mer
m {ie im } = er m ie im e j = er im iqe
Lr
Lr
Lr

(7.96)

La ecuacin de la tensin del rotor referida al sistema de referencia del estator en el modelo de
campo orientado es independiente de la derivada de las corrientes del estator. Por otra parte, es
frecuente que la tensin del rotor es cero ver = 0. Multiplicando por e j la ecuacin de tensin
del rotor y separando esta expresin en parte real e imaginaria se obtienen las dos ecuaciones
diferenciales escalares siguientes:
Tr pim + im = ide
(7.97)
Tr im ( ) = iqe
(7.98)
Las expresiones 7.96, 7.97 y 7.98 tienen un paralelismo con el modelo dinmico de la mquina
de corriente continua. La ecuacin 7.97 determina el comportamiento del campo18 de la mquina de induccin y se puede controlar ajustando la componente directa de la corriente del estator
18

Esta ecuacin es similar a la ecuacin del campo de una mquina de corriente continua L f pi f + R f i f = v f .

314

VII.8 Anlisis transitorio


ide . La componente cuadratura iqe por otra parte determina mediante la expresin 7.98 el deslizamiento ( ) existente entre la velocidad angular del campo y la velocidad angular del rotor19 .
El par elctrico 7.96 queda determinado por el producto de la magnitud de la corriente de campo
im y la componente cuadratura de la corriente del estator 20 . Una de las ventajas ms importantes
de este modelo reside en la posibilidad de regular el par y la velocidad de la mquina mediante
el control de las corrientes del estator. Con el uso de fuentes de corriente controladas, es posible
accionar la mquina a velocidad variable sin utilizar la ecuacin de las tensiones del estator.
El modelo escalar completo en coordenadas de campo orientado es:

o
n
2
2
2
i2qe
Mer
Mer
1 v (R + R Mer )i

i
+
+
i
pi
=
(L

)
+
R
m
qe
e
e
r
r
de
de
de
2
2

L
T
i
Ln
Lr m o
r
r m

2
2
2

ide iqe
M
Mer 1
M

(Re + Rr Ler2 )iqe Lerr m im vqe


piqe = m ide Tr im (Le Lr )
r
ide im
pi
=
m
Tr

p = m + Tiqei

o
n rm

pm = 1 Mer2 im iqe Tm (m )
J

(7.99)

Lr

El modelo de campo orientado 7.99, requiere que la corriente de magnetizacin sea diferente
de cero im 6= 0. Si este requisito no se cumple, se pierde la referencia , debido a que p
. En algunos casos es posible asumir que en las condiciones iniciales la corriente im tiene
un valor de remanencia que permita iniciar la integracin numrica, pero aun as si durante el
proceso en algn instante esta corriente se anula, el sistema de ecuaciones diferenciales pierde
la referencia y debe encontrarse algn modelo alterno que permita continuar la integracin.
El sistema de coordenadas referidas a una posicin angular arbitraria permite resolver este
problema y plantea una generalizacin del modelo que en muchos casos acelera el clculo de
las variables de estado.
En el modelo de referencia arbitraria se refieren todos los vectores espaciales del sistema 7.100
a una posicin angular , que gira a la velocidad . Para esto, se multiplican todos los vectores
espaciales por e j obtenindose el siguiente resultado:

ve
vr

 
  
ie
Le Mer
i
+
p e +

Mer Lr
ir
ir
 

Le
Mer
ie
+ j

( )Mer ( )Lr
ir

Re 0
0 Rr



n   o
Tm ( ) = J +
Mer m ie ir

(7.100)

El par elctrico calculado a partir de la integracin de las ecuaciones diferenciales, que modelan
el comportamiento de la mquina, presenta fuertes oscilaciones durante el arranque porque la
19
20

Esta ecuacin es comparable directamente con el modelo de la armadura de la mquina de corriente continua
Va Gm i f = Ra ia .
En la mquina de corriente continua el par queda determinado por el producto de la corriente de campo y la
corriente de armadura Te = Gi f ia .

315

Captulo VII Operacin de la mquina de induccin


fuente debe incrementar el flujo en el entrehierro para producir el par. Estas oscilaciones son
semejantes al fenmeno de energizacin de un transformador. La velocidad tambin es afectada
por las fuertes perturbaciones del par elctrico, pero en menor medida debido al retardo que
introduce la inercia.
VII.9

C ONTROL

DE VELOCIDAD

La mquina de induccin proporcion desde su invencin a finales del siglo XIX un mecanismo
conveniente para la conversin electromecnica de energa porque adems de ser simple, robusta y econmica, utiliza directamente fuentes de corriente alterna polifsicas. Las mquinas de
corriente continua requieren sistemas de rectificacin que si son controlables en tensin permiten la regulacin de velocidad. Las mquinas de induccin tienen una reducida controlabilidad
de la velocidad cuando operan en rgimen permanente alimentadas por una fuente de tensin
alterna de frecuencia industrial, pero cuando son alimentadas con tensin y frecuencia variable
es posible regular la velocidad mecnica de estos convertidores. Durante mucho tiempo esto fue
poco prctico porque para obtener sistemas de frecuencia variable se requeran varias mquinas operando conjuntamente21. Hoy en da los controladores electrnicos de potencia ofrecen
una gran variedad de fuentes reguladas que han permitido que la mquina de induccin sea una
alternativa importante para el accionamiento de cargas mecnicas a velocidad variable.
VII.9.1

Control tensin-frecuencia

Variando la frecuencia de las tensiones aplicadas a las bobinas del estator, cambia la velocidad
sincrnica de la mquina. La variacin de la frecuencia afecta proporcionalmente las reactancias
de magnetizacin y dispersin en el circuito equivalente, pero las resistencias se mantienen
aproximadamente constantes, considerando que el efecto pelicular es poco pronunciado22 . Para
que la densidad de flujo magntico se mantenga prcticamente constante, y en los lmites de
diseo de la mquina, es conveniente variar la amplitud de la tensin de alimentacin en la
misma proporcin que se vara la frecuencia. De esta forma, la magnitud del par elctrico es
similar a la que se obtiene con frecuencia nominal, pero la velocidad es variable.
En la figura 7.40 se presentan las caractersticas par elctrico-velocidad para una mquina de
induccin alimentada mediante cuatro frecuencias diferentes, manteniendo constante la relacin
entre la amplitud de la tensin y la frecuencia de alimentacin. Si se aumenta paulatinamente la
frecuencia es posible acelerar una carga mecnica pasando por los puntos 1, 2, 3 hasta alcanzar
el punto 4. Si la variacin de la frecuencia es lenta en comparacin con la inercia del conjunto
mquina-carga mecnica, la corriente se reduce con respecto a un arranque directo. Tambin
es posible mantener cualquier punto de operacin intermedio con el aumento o reduccin de la
velocidad. Operando a baja frecuencia, se incrementa el par de arranque, pero el par mximo de
21

22

El equipo denominado Ward-Leonard est configurado por cuatro mquinas: un motor de induccin que acciona
a una velocidad ms o menos constante a un generador de corriente continua, cuyo campo se ajusta para acelerar
a un motor de corriente continua a velocidad variable, que a su vez acciona a un generador sincrnico cuyo
devanado de campo regula la tensin generada, y su frecuencia queda determinada por el motor de corriente
continua.
Cuando una bobina se realiza con varias vueltas, el efecto pelicular tiende a reducirse si se compara con el
fenmeno que sucede en un conductor macizo.

316

VII.9 Control de velocidad

Figura 7.40 Control tensin-frecuencia constante en la mquina de induccin

la mquina es prcticamente constante, siempre y cuando las reactancias sean mucho mayores
que las resistencias del modelo.

Este controlador de velocidad requiere una fuente de tensin trifsica regulable en magnitud y
frecuencia. En el pasado esta fuente se poda obtener mediante una mquina sincrnica regulada
en velocidad y en su campo. Esta solucin trasladaba al eje mecnico de la mquina sincrnica
todo el problema de regulacin. Mediante interruptores electrnicos de alta velocidad es posible
construir fuentes de tensin alterna regulada en frecuencia y magnitud. Esta alternativa desarrollada durante la dcada de los treinta con las vlvulas de vapor de mercurio, evolucion en
la dcada de los ochenta con la aparicin de los tiristores y transistores de gran potencia. Los
inversores de tensin convierten fuentes de tensin o corriente continua en fuentes de tensin o
corriente alterna.

En la figura 7.41 se muestra el mecanismo de inversin en el caso ms simple, un inversor


monofsico. Alternadamente se conectan los interruptores 1, 2 y 3, 4. Esto conecta la mitad del
tiempo la carga entre positivo y negativo de la fuente de tensin Vcc , el resto del tiempo ocurre
lo contrario Vcc . El resultado final es una fuente de tensin alterna no sinusoidal, cuya frecuencia depende de la velocidad de operacin de los interruptores. La amplitud de esta fuente
es constante. Si la fuente de tensin continua Vcc es regulable, se puede obtener una fuente cuya
relacin tensin frecuencia sea constante. Mediante el inversor tambin es posible regular la amplitud de la tensin de salida. Para lograr esto durante algunos instantes en perodo de conexin
de un grupo de interruptores, se conectan los interruptores superiores o inferiores para aplicar
tensin cero sobre las bobinas V = 0, o incluso se conectan los interruptores contrarios en ese
perodo, lo que invierte la polaridad. Esta tcnica de control se denomina modulacin del ancho

317

Captulo VII Operacin de la mquina de induccin

Figura 7.41 Inversor monofsico y modulacin del ancho del pulso


del pulso, y existen varios mtodos que adems de regular el nivel de tensin, permiten reducir
el contenido armnico de la fuente23 .
La mquina de induccin convencional requiere una alimentacin polifsica balanceada para su
operacin. Esta fuente se obtiene mediante un puente inversor polifsico. Un puente inversor de
este tipo posee una rama con dos interruptores por cada fase. En un inversor es necesario conectar un diodo de descarga libre en paralelo con los semiconductores para permitir despus de la
desconexin de los interruptores estticos la circulacin de la corriente inductiva de la carga.
En un inversor trifsico son necesarios seis interruptores estticos, en tres ramas. Cada uno de
estos interruptores debe estar conectado durante la mitad del perodo total. Los interruptores de
la misma rama son complementarios, si uno est encendido, el otro debe estar apagado para evitar un corto circuito en la fuente. Las rdenes de encendido o apagado de los seis elementos se
encuentran desfasados en 23 . En la figura 7.42 se representa el esquema de un puente inversor
trifsico, la carga y la forma de onda de las tensiones aplicadas sobre ella. Los interruptores se
han numerado en el orden correlativo en que deben ser encendidos para producir un sistema trifsico balanceado de tensiones no sinusoidales de secuencia positiva. La amplitud de la tensin
es constante, pero su frecuencia depende del perodo de conexin de los interruptores.
Descomponiendo las formas de onda de la figura 7.42 en series de Fourier, se puede analizar
el comportamiento de la mquina de induccin sometida a este tipo de excitacin. Si la fuente
primitiva es de corriente alterna, la tensin de entrada al inversor puede ser variada mediante
un puente rectificador controlado. La fuente de corriente continua obtenida mediante bancos
de bateras o por rectificacin no controlada de sistemas de tensin alterna, se puede regular
mediante troceadores de tensin.24 La rapidez alcanzada por los dispositivos electrnicos de
potencia25 permite controlar el ancho del pulso de la onda e incluso su contenido armnico.
23

24
25

Una de las modulaciones ms utilizada en la prctica es la conocida como PWM (Pulse Width Modulation), la
cual ajusta el ancho de cada pulso siguiendo un patrn sinusoidal lo que permite una reduccin considerable de
las armnicas de baja frecuencia.
Denominados en ingls choppers.
Tiristores, Transistores de potencia, GTO, etc.

318

VII.9 Control de velocidad

(a) Puente inversor trifsico

(b) Tensiones aplicadas a la mquina por el puente inversor

Figura 7.42 Alimentacin de la mquina de induccin mediante un puente inversor trifsico

319

Captulo VII Operacin de la mquina de induccin

Figura 7.43 Variador de velocidad por control tensin-frecuencia constante


En la figura 7.43 se muestra el diagrama de un controlador de velocidad para un motor de
induccin que utiliza el mtodo tensin-frecuencia constante. El sistema realimenta la velocidad
o la posicin del eje mecnico y lo compara con una referencia determinada por el usuario o por
la aplicacin. El error obtenido de la comparacin entre las medidas y las referencias es utilizado
por el controlador para definir las seales de encendido y apagado de los interruptores. Este
controlador debe introducir un retardo en el proceso para reducir las corrientes de la mquina.
El controlador vara la amplitud de la tensin de la fuente de energa, que despus de ser filtrada
alimenta al inversor trifsico. El inversor aplica una tensin alterna no sinusoidal a la mquina
con una frecuencia y una tensin determinada por el controlador.
VII.9.2

Control por campo orientado

En la figura 7.44 se presenta el diagrama de bloques que corresponde al modelo de la mquina de


induccin en variables de campo orientado y la transformacin desde las coordenadas primitivas
a estas nuevas coordenadas. En este modelo, el par elctrico depende del producto de la corriente
de magnetizacin y de la corriente cuadratura del estator. Los sistemas de control por campo
orientado se fundamentan en la posibilidad de ajustar el valor de estas dos variables.
Tal como sucede en las mquinas de corriente continua, el campo tiene una constante de tiempo relativamente lenta. Para incrementar la velocidad de respuesta del sistema es conveniente
mantener la corriente de magnetizacin im en el mximo valor posible26 . De esta forma el par
se controla mediante la corriente cuadratura iqe . La corriente de magnetizacin se controla mediante el ajuste de la corriente directa ide . En rgimen permanente estas dos corrientes tienen el
mismo valor, hecho que se deduce inmediatamente de la ecuacin diferencial 7.97.
El principal problema de los controladores por campo orientado consiste en determinar el valor
de las corrientes o tensiones de alimentacin que producen los valores deseados de las variables
de campo orientado. La transformacin directa e inversa entre variables primitivas y variables
26

Normalmente el valor nominal, que por lo general est en el codo de saturacin del material ferromagntico.

320

VII.9 Control de velocidad

Figura 7.44 Modelo de la mquina de induccin en variables de campo orientado


de campo orientado dependen de la posicin instantnea (t) del vector espacial de la corriente
de magnetizacin im . Esto presenta un problema importante para este tipo de controladores,
por las dificultades que tiene la medicin o estimacin de este ngulo. La medicin requiere
sensores internos en la mquina27 . Para estimar la posicin del vector espacial de la corriente de
magnetizacin se requiere la integracin en tiempo real del sistema de ecuaciones diferenciales
que modelan la mquina. La primera solucin es costosa y difcil de implantar en la prctica, la
segunda alternativa depende de la velocidad del estimador, de la precisin del modelo y de la
variabilidad de los parmetros durante la operacin.
En la figura 7.45 se muestra el controlador de velocidad de un motor de induccin en coordenadas de campo orientado donde se utiliza un inversor controlado en corriente. El inversor inyecta
las corrientes en el estator de la mquina segn la referencia calculada previamente por el controlador. De la medicin directa de las corrientes por las bobinas y de la velocidad del rotor se
estiman los valores de las variables transformadas mediante un modelo semejante al ilustrado en
la figura 7.44. Estos valores permiten calcular el par elctrico y la corriente de magnetizacin.
Las estimaciones se comparan con los valores de par y velocidad deseados y el error obtenido se
utiliza para incrementar o disminuir las referencias de la corriente directa y cuadratura. Las referencias de corrientes, obtenidas a partir de las diferencias entre pares y velocidades, deseadas
y estimadas, se transforman al sistema de coordenadas y , para lo cual es necesario utilizar
la estimacin de la posicin del vector espacial de la corriente de magnetizacin. Las corrientes
de referencia en coordenadas y se transforman nuevamente en variables primitivas a, b y
c. Las corrientes de referencia se aplican como entrada al inversor controlado en corriente. El
inversor sigue muy de cerca a las referencias de corriente en coordenadas primitivas e inyecta a
las bobinas de la mquina estas corrientes.
Es conveniente mantener la referencia de la corriente de campo en el mayor valor posible para incrementar la velocidad de respuesta del sistema. Cuando la mquina excede la velocidad
sincrnica, se recomienda debilitar el campo para no exceder el lmite de la potencia nominal.
El inversor controlado por corriente es un convertidor electrnico que mide las corrientes por cada fase del puente y las compara con las referencias. Cuando la diferencia entre el valor medido
de la corriente en una fase y su referencia exceden un cierto valor de histresis, se conecta uno
27

Bobinas exploradoras o pastillas de efecto Hall que detecten la intensidad del campo magntico en el entrehierro.

321

Captulo VII Operacin de la mquina de induccin

Figura 7.45 Controlador de velocidad en coordenadas de campo orientado


de los interruptores de la rama del puente que corrige el error. Si la corriente es menor que la referencia, se conecta la fase correspondiente a la barra positiva del puente mediante el interruptor
esttico. Si la corriente es mayor que la referencia se conecta la fase a la barra negativa. Para las
diferencias comprendidas dentro del rango de la histresis no se alteran las condiciones previas
de conectividad de los interruptores. En este control del inversor es necesario un cierto retardo
en la variacin de las corrientes para evitar que la frecuencia de operacin de los interruptores
estticos sea muy elevada. El retardo depende de la relacin entre las inductancias y resistencias
de la mquina. Aumentar la histresis del control, disminuye la frecuencia de interrupcin, pero
reduce la precisin en el seguimiento de la referencia.
El controlador de velocidad de la figura 7.45 tambin puede ser realizado mediante puentes
controlados por tensin, pero en este caso es necesario aadir al modelo de estimacin de variables, la ecuacin de tensin del estator y calcular las referencias de tensin que producen las
condiciones del par y velocidad deseadas.
El principal problema del estimador de variables internas de la mquina es la variabilidad de
los parmetros con la temperatura, la frecuencia y la saturacin. En particular el modelo del
estimador ilustrado en la figura 7.44 es muy sensible al valor de la constante de tiempo del rotor
Tr , debido a que influye directamente en la magnitud y direccin instantnea de la corriente de
magnetizacin. Los errores en la estimacin del verdadero ngulo producen errores en la transformacin de coordenadas primitivas a coordenadas de campo orientado, y esta transformacin
es la que permite desacoplar el par elctrico en dos componentes independientes. Los variadores
de velocidad modernos incluyen esquemas de control adaptativo que determinan y corrigen en
lnea el valor de la constante de tiempo del rotor utilizado por el estimador de variables.
VII.9.3

Control directo de par

El control directo del par28 es una herramienta empleada frecuentemente en el control de velocidad de la mquina de induccin. El inters fundamental de esta tcnica reside en la posibilidad
de controlar el flujo y el par sin utilizar modelos de la mquina. Los mtodos de control fundamentados en modelos de las mquinas para estimar el par o los enlaces de flujo, introducen
28

DTC por sus siglas en ingls (Direct Torque Control).

322

VII.9 Control de velocidad


error Te
eT > 0
eT > 0
eT < 0
eT < 0

error e
e > 0
e < 0
e > 0
e < 0

Z1
V2
V3
V6
V5

Z2
V3
V4
V1
V6

Z3
V4
V5
V2
V1

Z4
V5
V6
V3
V2

Z5
V6
V1
V4
V3

Z6
V1
V2
V5
V4

Tabla 7.2 Seleccin del vector espacial de la tensin que realiza la correccin ms rpida del
error de enlace de flujo y par elctrico para cada una de las seis zonas espaciales
errores como consecuencia de las hiptesis simplificadoras que se usan29 para su deduccin o
por la variacin de los parmetros en la operacin 30 . El control directo del par se fundamenta en
la posibilidad de obtener el valor del par elctrico utilizando variables que pueden ser medidas
directamente en el estator de la mquina.31 El par elctrico se obtiene del producto vectorial del
vector espacial32 del enlace de flujo del estator y el vector espacial de la corriente del estator:
Te = e ie

(7.101)

Para determinar el enlace de flujo e en la expresin 7.101 se integra la fuerza electromotriz que
aparece en las bobinas del estator:

e =

Z t
0

ee dt =

Z t
0

(ve Re ie ) dt = |e (t)| e je (t)

(7.102)

El mdulo del enlace de flujo |e (t)| y su correspondiente ngulo e (t) se pueden controlar
mediante la seleccin apropiada de la tensin ve . En un inversor o en un rectificador activo
existen solamente siete posibles vectores espaciales de la tensin tal como se muestra en la figura
7.46. Seleccionar apropiadamente el vector espacial de la tensin ve que produce la variacin
deseada del enlace de flujo y del par elctrico para un estado dado de estas variables genera la
tabla 7.2. En una de las seis zonas donde puede estar el flujo se escoge el vector espacial que
incrementa ms su variacin, con lo cual este vector se acelera o frena para regular el par.
En la figura se presenta el par elctrico y el vector espacial del enlace de campo de una mquina
de induccin cuya velocidad ha sido regulada mediante un control directo de par. En comparacin con el control por campo orientado, este mtodo introduce fluctuaciones muy rpidas del
par para ajustar su valor medio de acuerdo con la referencia, pero a cambio es prcticamente
independiente de la variacin de los parmetros de la mquina33 .
29
30
31
32

Saturacin, prdidas, ranuras, desequilibrios, etc.


Los cambios en la temperatura del convertidor o la saturacin del material ferromagntico pueden ser la causa
de esta variacin.
Tensiones
q y corrientes.
4
2

x = 2 (x + x e j 3 + x e j 3 ).
3

33

Con excepcin de la resistencia del estator que puede ser ajustada peridicamente, y cuyas fluctuaciones introducen errores poco significativos para el controlador de velocidad.

323

Captulo VII Operacin de la mquina de induccin

Figura 7.46 Vectores espaciales de la tensin de salida de un inversor activo

(a) Par elctrico

(b) Vector espacial del flujo del estator

Figura 7.47 Par elctrico y enlace de flujo de un motor de induccin accionado mediante un
controlador directo de par.

324

VII.10 Sumario
VII.10

S UMARIO

1. El proceso de arranque de la mquina de induccin tiene requerimientos importantes tanto


para el sistema elctrico como para el propio convertidor. Cuando se realiza a plena tensin
las corrientes pueden alcanzar magnitudes superiores a cinco veces los valores nominales.
Para limitar las cadas de tensin en los alimentadores y el calentamiento de la mquina
se utilizan diversos arrancadores tales como el estrella-delta, por autotransformador o el
arranca suave, cuyo fundamento se basa en reducir la tensin aplicada a las bobinas. El
inconveniente de estos mecanismos de arranque es la reduccin del par de accionamiento
proporcionalmente al cuadrado de la tensin aplicada.
2. La mquina de induccin con rotor de jaula de ardilla puede incrementar el par elctrico
durante el arranque cuando se disea con barras profundas. El efecto pelicular distribuye
no uniformemente las corrientes en estas barras dependiendo de la frecuencia del campo
con que son cortadas. Cuando la velocidad del rotor es cero, la frecuencia es mxima y
las barras presentan una alta resistencia equivalente debido al efecto pelicular. Durante la
operacin en rgimen permanente, el deslizamiento y la frecuencia de corte de las barras
por el campo son mucho menores y el efecto pelicular se hace despreciable, reduciendo
la resistencia equivalente del rotor. Las mquinas de induccin de rotor bobinado pueden
aadir resistencias externas en el rotor durante el arranque para incrementar el par, que si
se elimina durante la operacin en rgimen permanente mejora el rendimiento.
3. Las componentes simtricas son una herramienta eficaz para el anlisis desequilibrado de
la mquina de induccin. Permiten descomponer un modelo acoplado en varios modelos
desacoplados. Las condiciones de contorno en fase se deben convertir al dominio de la
secuencia, conformando junto con los modelos de secuencia un sistema cuya solucin es
ms simple por no tener acoplamientos entre secuencias. Las componentes simtricas pueden ser aplicadas solamente sobre mquinas totalmente simtricas, pero sus conexiones
externas pueden tener cualquier clase de desequilibrio. En algunas ocasiones es posible
determinar un circuito equivalente para ciertas conexiones de desequilibrio, lo que simplifica la solucin del sistema de ecuaciones. Las componentes simtricas tienen gran
utilidad cuando son aplicadas para desacoplar los modelos transitorios de la mquina de
induccin.
4. El modelo de la mquina de induccin puede ser extendido para permitir el anlisis de
armnicas temporales en la fuente de alimentacin o por las armnicas espaciales debidas
a la distribucin de los devanados. En estos casos se utiliza el principio de superposicin, y
en cada armnica espacial o temporal se debe analizar su secuencia correspondiente, de tal
manera que la superposicin del par se realice con el signo adecuado. Las reactancias del
modelo de una armnica temporal se amplifican en este orden mientras que las resistencias
se consideran prcticamente constantes. En las subarmnicas espaciales ocurre un efecto
similar pero en este caso hay un decrecimiento de la frecuencia y de la reactancia. Para
la determinacin del deslizamiento de cada armnica o subarmnica se debe tener en
cuenta su correspondiente secuencia. Cuando las armnicas temporales incrementan las
corrientes para un determinado deslizamiento, su efecto sobre el par elctrico es menos
importante de lo que cabra pensar, debido a la dependencia cuadrtica del par con la

325

Captulo VII Operacin de la mquina de induccin


tensin de alimentacin. La distribucin de los conductores en las mquinas industriales
de induccin, reduce considerablemente el efecto de las armnicas espaciales.
5. Generalizando las componentes simtricas polifsicas es posible encontrar una transformacin que desacople las mquinas tetrafsicas. Esta geometra permite la representacin
de la mquina bifsica de induccin mediante dos secuencias. Los desequilibrios internos
de la mquina bifsica de induccin pueden ser transformados en desequilibrios externos
aplicados a una mquina equilibrada. Las mquinas bifsicas tienen un mbito de aplicacin muy extenso para aquellos accionamientos residenciales, comerciales e industriales
de baja potencia (< 2 kW ). En la prctica se utiliza un arranque bifsico mediante un condensador en serie que desfasa la corriente por la bobina auxiliar y cuando se ha alcanzado
entre un 70~75 por ciento de la velocidad nominal, se desconecta este circuito mediante
un contactor centrfugo que gira con el eje del motor. Una vez que el circuito auxiliar es
desconectado la mquina se mantiene operando monofsicamente.
6. Las componentes simtricas y los vectores espaciales permiten representar eficazmente
la mquina de induccin en rgimen transitorio. Estos modelos determinan fenmenos
no previstos por el modelo clsico de la mquina como las oscilaciones del par elctrico
durante un arranque a plena tensin y frecuencia. Los modelos transitorios son necesarios
para analizar el comportamiento de los controladores de velocidad. Estos modelos pueden
ser utilizados para estimar el valor instantneo de variables internas de la mquina como
el enlace de flujo o el par elctrico.
7. Con el desarrollo de la electrnica de potencia ha sido posible ampliar el rango de aplicacin de la mquina de induccin al control de velocidad. La disponibilidad de fuentes
de tensin y frecuencia variable permite la operacin de estos convertidores a cualquier
velocidad, con bajas corrientes y altos pares de accionamiento. Los controles de velocidad por regulacin tensin-frecuencia, campo orientado y control directo de par son tres
soluciones utilizadas frecuentemente.
VII.11

EJEMPLOS

RESUELTOS

Ejemplo 1: Mquina de induccin con rotor de barra profunda


Una mquina de induccin de rotor de jaula de ardilla con barras profundas posee los siguientes
valores de sus parmetros en el sistema adimensional de medidas:
Re
0,02

X e
0,1

Rm
100

Xm
4,0

X12
0,06

R1
0,08

X2
0,06

R2
0,02

Determinar34 :
1. El deslizamiento del punto nominal, si se asume que los parmetros se han especificado
en la base de la potencia nominal en el eje.
34

Es recomendable crear un programa en una aplicacin de alto nivel del tipo Scilab, Matlab u Octave para
obtener los resultados de este ejemplo.

326

VII.11 Ejemplos resueltos


2. Las corrientes del estator en las siguientes condiciones:
a) Arranque (s = 1; m = 0)
b) Punto nominal (s = sn ; m = mn )
c) Vaco (s = 0; m = e = 1)
d) Par mximo
3. La estimacin de los parmetros de dicho convertidor hecha sobre la base de las cuatro
corrientes determinadas en el punto anterior.
4. El par elctrico y el rendimiento producido por el convertidor deben estar en funcin del
deslizamiento.
Solucin:
1. El deslizamiento del punto nominal, si se asume que los parmetros se han especificado
en la base de la potencia nominal en el eje:
Cuando la mquina alcanza el deslizamiento nominal sn , el eje entrega a la carga la potencia nominal 1, 0. De esta forma se tiene:
Pe jen =

i
1 sn h
Rr1 |Ir1 (sn )|2 + Rr2 |Ir2 (sn )|2 = 1, 0
sn

Donde:


Ie (sn )
Ir2 (sn )

"

Rr1 s1n
Zth + Rr1 s1n
Rr1 s1n
(Rr1 + Rr2 ) s1n + jX2

#1 

(7.103)

Vth
0

Ir1 (sn ) = Ie (sn ) Ir2(sn )


Vth = Ve

Ze Zm
Zm
; Zth =
+ jX12
Ze + Zm
Ze + Zm

Ze = Re + jXe ; Zm =

jXm Rm
Rm + jXm

En el cdigo 7 se presenta un programa ejecutable mediante la aplicacin Scilab 3 que


permite determinar el deslizamiento nominal a partir de la expresin 7.103. El resultado
obtenido es:
sn = 0,01886

2. Las corrientes del estator en las siguientes condiciones:


Con el mismo algoritmo desarrollado en el cdigo 7, es posible determinar las corrientes siguientes:

327

Captulo VII Operacin de la mquina de induccin

Algoritmo 7 Determinacin del deslizamiento nominal para una mquina de induccin con rotor
de jaula profunda cuyos datos se expresan en el sistema adimensional de unidades
j= %i;i=j;pi= %pi;error_max=1e-3;
Re=0.02; Rm=100; Rr1=0.08; Rr2=0.02;
Xe=0.10; Xm=4; X12=0.06; X2=0.06;
Ve=1;Pneje=1;
Ze =Re+j*Xe;
Zm=j*Xm*Rm/(Rm+j*Xm);
Zth=Ze*Zm/(Ze+Zm)+j*X12;
Vth=Ve*Zm/(Ze+Zm);
s=.0001:1/1000:1;
n=size(s);
sn=0; Pn=0;
for i=1:n(2);

//Defini in de onstantes
//Parmetros resistivos
//Parmetros indu tivos
//Tensin y poten ia nominal
//Impedan ia del estator
//Impedan ia de magnetiza in
//Impedan ia de Thvenin
//Tensin de Thvenin
//Barrido del deslizamiento
//Tamao del ve tor deslizamiento
//Ini ializa in de variables
//Barrido del deslizamiento

Ier=inv([Zth+Rr1/s(i),-Rr1/s(i);-Rr1/s(i),((Rr1+Rr2)/s(i))+j*X2)*[Vth;0;
Zr =((Rr2/s(i)+j*X2).*(Rr1/s(i)))/((Rr2/s(i)+j*X2)+(Rr1/s(i)));
Vm = Ier(1)*Zr;
//Cl ulo de la tensin de magnetiza in
Ie(i)=Ier(1)+Vm/Zm;
//Alma ena la orriente del estator
Ir1=abs(Ier(1)-Ier(2));
//Corriente por Rr1
Ir2=abs(Ier(2));
//Corriente por Rr2
Te(i)=(1/s(i))*(Rr1*Ir1^2+Rr2*Ir2^2);
//Cl ulo del par el tri o
Peje(i)=((1-s(i))/s(i))*(Rr1*Ir1^2+Rr2*Ir2^2);//Cl ulo de la poten ia
Pent(i)=real(Ve*Ie(i));
//Poten ia de entrada
efi ien ia(i)=Peje(i)/Pent(i);
//Efi ien ia
if abs((Peje(i)-Pneje)/Pneje)<=error_max & s(i)<=.1 then
sn=s(i);Pn=Peje(i);i_n=i;

//Alma ena el punto nominal

end,
if abs(s(i)-0.08)<=error_max then
sTmax=s(i);Tmax=Te(i);i_Tmax=i;IeTmax=Ie(i);//Alma ena punto de par mximo
end,
end
//Resultados
sn Pn In=Ie(i_n)
sTmax IeTmax
Ie_va =Ie(1) Ie_arr=Ie(n(2))
subplot(2,1,1)
plot(s,Te)
subplot(2,1,2)
plot(s(1:200),efi ien ia(1:200))

328

// Punto nominal
// Par mximo
// Corrientes de va o y rotor bloqueado
// Grfi o del par
// Grfi o del rendimiento

VII.11 Ejemplos resueltos


Parmetro
Re
X e
Rm
Xm
X 12
Rr1
X 2
Rr1
(X)

Inicial
0,0200
0,12
92
3,6
0,05
0,09
0,05
0,03
0,2007

Estimacin
0,0200
0,0999802
100,00115
4,0000196
0,0600199
0,0799995
0,0600002
0,0200001
1,84 1013

Exacto
0,02
0,1
100
4
0,06
0,08
0,06
0,02
7,39 1014

Tabla 7.3 Parmetros obtenidos mediante el programa 8


a) Arranque (s = 1; m = 0) : Ie1 = 5,2915 74,7 p.u.

b) Punto nominal (s = sn ; m = mn ): Ien = 5,2915 74,7 p.u.


c) Vaco (s = 0; m = e = 1): Ie0 = 1,1821 24 p.u.

d) Par mximo (s = 0,0809): ITmax = 1,4784 44,3 p.u.


3. La estimacin de los parmetros de dicho convertidor hecha sobre la base las cuatro
corrientes determinadas en el punto anterior.
Para este fin se puede adaptar el algoritmo ?? presentado en el captulo 6 para resolver
la estimacin de los parmetros de mquinas con rotor de jaula de barra profunda. El algoritmo 8 es una adaptacin que permite resolver este tipo de problemas. En la tabla 7.3
se muestran los resultados obtenidos:

4. El par elctrico y el rendimiento producido por el convertidor en funcin del deslizamiento.


En la figura 7.48 se presentan los grficos del par elctrico y rendimiento en funcin
del deslizamiento para la mquina analizada.

Ejemplo 2: Conexin desequilibrada de la mquina de induccin


Una mquina de induccin est alimentada mediante una fuente sinusoidal de frecuencia industrial en la fase a. La fase b tiene conectada un condensador de valor conocido C. La fase c se
encuentra en circuito abierto. Determine la expresin del par elctrico en funcin del deslizamiento.

329

Captulo VII Operacin de la mquina de induccin

Algoritmo 8 Estimacin de parmetros de la mquina de induccin con rotor de barra profunda


// Los parmetros del ir uito equivalente de esta mquina son:
// Re = .02 p.u. Xe = .10 p.u. Rm = 100. p.u. Xm = 4.0 p.u.
// X12= .06 p.u. Rr1= .08 p.u. X2 = .06 p.u. Rr2= .02
// Ve tor de arranque x0: [Xe,Rm,Xm,X12,Rr1,X2,Rr2
x0=[.12 92 3.6 .05 .09 .05 .03'
//
//Llamada a la rutina optim que al ula los valores
// de los parmetros x que minimizan la fun in de osto.
//
[Psi,x,g = optim(list(ND ost, osto),x0);
//
// En el ve tor x se han argado los parmetros ptimos de la
// estima in. La solu in es:
//
Refin = 0.02
// Medi in dire ta de la resisten ia estator
Xefin = x(1)
// Rea tan ia de dispersin del estator
Rmfin = x(2)
// Resisten ia de magnetiza in
Xmfin = x(3)
// Rea tan ia de magnetiza in
X12fin= x(4)
// Rea tan ia fuga estator rotor
Rr1fin= x(5)
// Resisten ia de la barra superfi ial
X2fin = x(6)
// Rea tan ia de fuga de la barra profunda
Rr2fin= x(7)
// Resisten ia de la barra profunda
Psi
// Costo final

Figura 7.48 Par elctrico y rendimiento con respecto al deslizamiento de la mquina de induccin con rotor de barras profundas

330

VII.11 Ejemplos resueltos

Algoritmo 9 Funcin de costo a ser minimizada


//************************************************************
fun tion Psi = osto(x)
//************************************************************
// Evalua in de la fun in de ostos por mnimos uadrados.
// Psi = Sumatoria(errores relativos)^2
// Deslizamientos orrespondientes a los ensayos de va o,
// arga, par mximo y rotor bloqueado.
//
Re = 0.02; // Medi in dire ta de la resisten ia estator
Xe = x(1); // Rea tan ia de dispersin del estator
Rm = x(2); // Resisten ia de magnetiza in
Xm = x(3); // Rea tan ia de magnetiza in
X12= x(4); // Rea tan ia fuga estator rotor
Rr1= x(5); // Resisten ia de la barra superfi ial
X2 = x(6); // Rea tan ia de fuga de la barra profunda
Rr2= x(7); // Resisten ia de la barra profunda
//
// Ve tor de las impedan ias de entrada medidas en los ensayos.
//
j= %i; // Defini in de j omo nmero omplejo
// Los parmetros exa tos del ir uito equivalente de esta mquina son:
// Re = .02 p.u. Xe = .10 p.u.
// Rm = 100. p.u. Xm = 4.0 p.u.
// X12= .06 p.u. Rr1= .08 p.u.
// x2 = .006 p.u. Rr2= .02 p.u.
//
// Los ensayos realizados dieron los siguientes resultados:
s
=[0.0001 0.01886 0.0809 1';
Zmedida = [.2773721+j*4.0832533;.7997782+j*0.360365
.2122956+j*0.2080942;.0522325+j*0.1893435;
//
// Evalua in de las impedan ias al uladas mediante la estima in
// de los parmetros del modelo.
//
Ze = Re+j*Xe;
// Impedan ia estator
Zm = (Rm*j*Xm)/(Rm+j*Xm);
// Impedan ia magnetiza in
Zth= Ze*Zm/(Ze+Zm)+j*X12;
// Impedan ia de Thvenin
Ve = 1.00;
// Tensin del estator
Vth= Zm*Ve/(Zm+Ze);
// Tensin de Thvenin
for i=1:4
Ier=inv([Zth+Rr1/s(i),-Rr1/s(i);-Rr1/s(i),(((Rr1+Rr2)/s(i))+j*X2))*[Vth;0;
Ir(i)=Ier(1);
// Alma ena la orriente del rotor
end
Zr=((Rr2./s+j*X2).*(Rr1./s))./((Rr2./s+j*X2)+(Rr1./s));
Vm = Ir.*Zr;
// Tensin rama magnetizante
Im = Vm/Zm;
// Corriente de magnetiza in
Ie = Im+Ir;
// Corriente del estator
Z al ulada=Ve./Ie;
// Impedan ia de entrada al ulada
// Cl ulo del error relativo entre las medidas y el modelo
err = (Zmedida-Z al ulada)./Zmedida;
endfun tion;

331

Captulo VII Operacin de la mquina de induccin


Solucin:
Para determinar el par elctrico en funcin del deslizamiento, es necesario convertir las condiciones de contorno en las fases al dominio de la secuencia. En un sistema trifsico deben obtenerse
tres condiciones de contorno independiente, que en este caso pueden expresarse como:
Va = V ; Vb = j

Ib
; Ic = 0
C

(7.104)

Utilizando la transformacin de componentes simtricas 7.7 se obtienen las condiciones de contorno en el dominio de la secuencia:
1
Va = V = (V0 +V+ +V )
3
1
j
Vb = (V0 + 2V+ + V ) =
(I0 + 2 I+ + I )
3
3C
1
Ic = 0 = (I0 + I+ + 2 I )
3

(7.105)
(7.106)
(7.107)

En las expresiones 7.105, 7.106 y 7.107 se relacionan las tres tensiones de secuencia con las
tres corrientes de secuencia. Para completar el sistema de ecuaciones que permita determinar
el comportamiento de la mquina de induccin sometida a estas condiciones desequilibradas es
necesario aadir las tres expresiones que relacionan las tensiones de secuencia con las corrientes
de secuencia en una mquina de induccin trifsica equilibrada:
V+ = Z+ (s) I+ ; V = Z (s) I ; V0 = Z0 I0
De esta forma se obtiene el siguiente sistema de ecuaciones:

1
1
1
V0 (s)
3V
j
j

1+ j
2
V+ (s) = 0

CZ0 (1 + CZ+ (s) ) (1 + CZ (s) )


1
1
1
V (s)
0
Z
Z
Z
0

(7.108)

(7.109)

Del sistema de ecuaciones 7.109 se obtienen las tres tensiones de secuencia en funcin del deslizamiento. Conocidas V+ (s) y V (s), se determina el par elctrico que produce cada secuencia 35 :
Te = Te+ Te =
Donde:
Vth+ (s) =
35

2 (s) Rr
Vth+
s

2 (s) Rr
Vth
2s

 


Rr 2
(Rth + Rsr )2 + Xth2
(Rth + 2s
) + Xth2
Zm
Zm
V+ (s) ; Vth (s) =
V (s)
Zm + Ze
Zm + Ze

Es necesario recordar en este punto que la secuencia cero no produce par elctrico.

332

(7.110)

VII.11 Ejemplos resueltos


Ejemplo 3: Armnicas temporales de una mquina de induccin pentafsica
Determine la direccin en la que las armnicas temporales de una mquina pentafsica de induccin producen par elctrico.
Solucin:

1. P RIMERA ARMNICA: Las primeras armnicas de las corrientes de una mquina pentafsica tienen la estructura siguiente:

2I1 cos( t)
ia (t) =

2
2I1 cos( t )
ib (t) =
5

4
2I1 cos( t )
ic (t) =
(7.111)
5

6
id (t) =
2I1 cos( t )
5

8
2I1 cos( t )
ie (t) =
5
La fuerza magnetomotriz producida por las corrientes 7.111 es:

25NI1
2k
2k
)cos(
)=
cos( t )
FR1 (t, ) = 2NI1 cos( t
5
5
2
k=a
que es un campo magntico rotatorio de secuencia positiva
a = 0, b = 1, , e = 4.

d
dt

= . En esta expresin

2. T ERCERA ARMNICA: En este caso se obtiene la expresin siguiente para la fuerza magnetomotriz de tercera armnica:
FR3 (t, ) =

2NI3 cos3( t
k=a

2k
2k
)cos(
)=0
5
5

que es un campo magntico rotatorio de secuencia cero.

3. Q UINTA ARMNICA: En este caso se obtiene la expresin siguiente para la fuerza magnetomotriz de quinta armnica:
F (t, ) =

2NI5 cos5( t
k=a

2k
2k
)cos(
)=0
5
5

que es un campo magntico rotatorio de secuencia cero.

333

Captulo VII Operacin de la mquina de induccin


4. S PTIMA ARMNICA: En este caso se obtiene la expresin siguiente para la fuerza magnetomotriz de sptima armnica:
FR7 (t, ) =

2NI7 cos7( t
k=a

2k
2k
)cos(
)=0
5
5

que es un campo magntico rotatorio de secuencia cero.


5. N OVENA ARMNICA : En este caso se obtiene la expresin siguiente para la fuerza magnetomotriz de sptima armnica:

25NI9
2k
2k
F (t, ) = 2NI9 cos9( t
)cos(
)=
cos(9 t + )
5
5
2
k=a
que es un campo magntico rotatorio de secuencia negativa de velocidad

d
dt

= 9 .

6. U NDCIMA ARMNICA: En este caso se obtiene la expresin siguiente para la fuerza


magnetomotriz de sptima armnica:

25NI11
2k
2k
)cos(
)=
cos(11 t )
F (t, ) = 2NI11 cos11( t
5
5
2
k=a
que es un campo magntico rotatorio de secuencia positiva de velocidad

d
dt

= 11 .

En resumen, la secuencia de las armnicas temporales para una mquina de induccin pentafsica equilibrada es la siguiente:
1.
+

3.
0

5.
0

7.
0

9.

11.
+

13.
0

15.
0

17.
0

19.

21.

Ejemplo 4: Seleccin del condensador de arranque de una mquina de induccin


monofsica
Una mquina de induccin monofsica posee un devanado auxiliar construido con la mitad
de vueltas del principal y con un conductor de igual seccin, para permitir el arranque por
condensador. Los datos de esta mquina son los siguientes:
Pn e je
500W

Vn
110V

n
65 %

nn
1.700 rpm

cos n
0,80

f
60 Hz

Determine el valor del condensador que produce el mximo par de arranque36 .


36

Puede despreciar el efecto de las prdidas y de la rama de magnetizacin.

334

VII.11 Ejemplos resueltos


Solucin:
En primer lugar es necesario determinar los parmetros del circuito equivalente de esta mquina.
En el punto nominal de operacin acta solamente el devanado principal y el circuito equivalente, ilustrado en la figura 7.31. Despreciar la rama de magnetizacin, simplifica los clculos
necesarios para la determinacin paramtrica. En este caso:
Ia =
=

Vn

Re + 21 Rr

1
sn

i
=
1
+ 2s
+
j(X
+
X
)

r
n

Pn e je
arc cos(0,80) = 8,741 36,87 A
Vn cos n n

(7.112)

De la expresin 7.112, obtenemos las siguientes relaciones:


Re + 9,257Rr = 10,06
Xe + Xr 2Xe = 7,55
En el punto nominal se entrega el par nominal en el eje:


Pn e je
500
1
1
2 Rr
Te n =
= 2 1.700 = 2,8086 Nm = Ie n

n
2s sn 2 sn
60
Rr =

2 Te n s
h
i = 0,7925
1
|Ie n |2 s1n 2s
n

Re = 10,06 9,257 0,7925 = 2,7238


Xe Xr 3,775

El devanado auxiliar tiene la mitad de vueltas del enrollado principal y la seccin del conductor
es la misma, por tanto su resistencia debe ser la mitad de la resistencia del estator. La reactancia
depende del cuadrado del nmero de vueltas y por tal motivo la reactancia de esta bobina es la
cuarta parte de la reactancia del devanado principal. En resumen:
Rep
2,7238

X ep
3,775

La relacin de transformacin a =
con la expresin 7.80 como:

Rr
0,7925
Np
Naux

X r
3,775

Raux
1,3619

Xaux
0,94375

= 2, permite determinar la impedancia Zx , de acuerdo

j
+ Zaux ) Z p =
C
= 4( jXc + 1,3619 + j0,94375) 2,7238 j3,775 =
= 2,7238 j4Xc

Zx = a2 (

335

Captulo VII Operacin de la mquina de induccin


Las impedancias de secuencia positiva y negativa en el arranque son iguales y su valor es:
Z+ = Z = Re + Rr + j(X e + X r ) = 3,5163 + j7,55
Introduciendo estos valores en el sistema de ecuaciones 7.84 se pueden obtener las corrientes
de secuencia positiva I+ y negativa I para un determinado valor del condensador C, o de la
reactancia Xc . El par de arranque se obtiene de la siguiente forma:
Te (s = 1) =

i
Rr h 2
|I+ | |I |2
s

A continuacin se presentan los resultados obtenidos al hacer un barrido de la reactancia capacitiva Xc:
Xc
Te

1,8
2,06 Nm

2,0
2,17 Nm

2,2
2,22 Nm

2,4
2,20 Nm

2,6
2,12 Nm

La reactancia que produce el mximo par en el arranque es aproximadamente 2,2 , por esta
razn la mquina debera utilizar un condensador de 1,2 mF.

Ejemplo 5: Vector espacial de las tensiones lnea-lnea


Utilizando la transformacin a vectores espaciales determine:

1. El vector espacial de las tensiones del estator de una mquina de induccin a partir de las
tensiones lnea a lnea.
2. La potencia activa y reactiva instantnea.

Solucin:

1. El vector espacial de las tensiones del estator de una mquina de induccin a partir de las
tensiones lnea a lnea:
El vector espacial de las tensiones del estator se obtiene directamente a partir de las tensiones lnea a neutro mediante la expresin 6.7:
r
2
ve =
(va + vb + 2 vc )
3
336

VII.12 Ejercicios propuestos


Si se aplica esta transformacin a las tensiones lnea a lnea se obtiene el siguiente resultado:
r
2
LL
ve
=
(vab + vbc + 2 vca )
3
r

2
(va + vb + 2 vc ) (vb + vc + 2 va ) =
=
"r3
#
r
2
2
=
(va + vb + 2 vc ) 2
( vb + 2 vc + 3 va ) =
3
3


= ve 2 ve = (1 2 )ve
1
ve =
vLL =
1 2 e

2e j
3

2
6

(vab + vbc + 2 vca )

2. La potencia activa y reactiva instantnea:


Para obtener la potencia activa y reactiva instantnea mediante los vectores espaciales
se realiza el producto del vector espacial de la tensin espacial por el conjugado del vector
espacial de la corriente:
r
r
2
2

2
s = ve i e =
(va + vb + vc )
(ia + 2 ib + ic ) =
3
3

2
(va ia + vb ib + vc ic ) + (va ic + vb ia + vc ib ) + 2 (vc ia + vb ic + va ib ) =
3


2
1
=
(va ia + vb ib + vc ic ) (va ic + vb ia + vc ib + vc ia + vb ic + va ib ) +
3
2
"
#
2
3
+ j
(va ic + vb ia + vc ib vc ia vb ic va ib ) =
3 2


1
1
1
2
(va ia + vb ib + vc ic va (ic + ib ) vb (ia + ic ) vc (ia + ib ) +
3
2
2
2
"
#
2
3
+ j
(vab ic + vbc ia + vca ib ) =
3 2

1
= (va ia + vb ib + vc ic ) + j (vab ic + vbc ia + vca ib ) = p + jq
3

VII.12

EJERCICIOS

PROPUESTOS

1. Un motor trifsico de induccin, cuyos parmetros se encuentran dentro de los lmites


tericos, se conecta como se muestra en la figura 7.49. Determine:

337

Captulo VII Operacin de la mquina de induccin

Figura 7.49 Conexin monofsica de una mquina de induccin trifsica


Re = 0,01 pu
X12 = 0,15 pu

X e = 0,2 pu
Rr1 = 0,15 pu

Xm = 3 pu
Rr2 = 0,06 pu

Rm = 30 pu Vn = 416V
X2 = 0,3 pu In = 97 A

Tabla 7.4 Parmetros del motor de induccin del ejercicio 3


a) El valor de Xc para que la mquina arranque con par mximo.
b) El deslizamiento nominal de la mquina.
c) El rendimiento del punto nominal.

2. Conocidos todos los parmetros de una mquina de induccin con rotor de doble jaula,
realice un algoritmo que permita calcular:

a) El par mximo de la mquina.


b) El deslizamiento correspondiente a una potencia especfica en el eje.

3. De un motor de induccin de doble jaula se conocen los parmetros y valores nominales


que se expresan en la tabla 7.4. Calcule:

a) El par de arranque con una tensin de la fuente de 400 V. b.


b) La potencia nominal en el eje del motor y el rendimiento en este punto de operacin.

4. Un motor de induccin trifsico balanceado con sus valores dentro de los lmites tericos
para su circuito equivalente, se alimenta con tensin y frecuencia nominal en su fase a,
mientras que en cada una de las fases b y c, se conecta una reactancia capacitiva de valor
0,2 pu. Determine:

a) El par y la corriente de arranque de la mquina en estas condiciones.

338

VII.12 Ejercicios propuestos


b) El rendimiento en el punto nominal de operacin.

5. Una mquina monofsica de induccin con arranque por condensador de 1,5 HP, 208V, 60 Hz,factor
de potencia 0,77, rendimiento nominal 65 % y velocidad nominal 1.700 rpm, tiene un devanado auxiliar con un tercio de vueltas del principal, pero su resistencia es idntica a la
del principal. Determine:
a) El condensador necesario en el circuito auxiliar para que la mquina arranque con
un campo magntico rotatorio.
b) La corriente y el par de arranque conectando el devanado auxiliar sin el condensador
de arranque.

6. Una mquina monofsica de induccin con arranque por condensador de 1 HP, 120V ,
60 Hz, factor de potencia 0,8, rendimiento nominal 60 % y velocidad nominal 1.650 rpm,
posee una reactancia de dispersin de 0,4 pu.37 El devanado auxiliar tiene la mitad de
vueltas del principal, pero su resistencia es el doble. El condensador de arranque es de
30 F. Determine:
a) La corriente y el par de arranque con el condensador conectado.
b) La corriente y el par de arranque conectando el devanado auxiliar sin el condensador
de arranque.
7. Una mquina de induccin monofsica de 41 HP, 120V , 3,7 A, factor de potencia nominal
0,7 y velocidad nominal 1.650 rpm, tiene en su devanado auxiliar un tercio del nmero de
vueltas de la bobina principal y su resistencia es el doble. Durante el proceso de arranque
se utiliza un condensador de 5 F, pero cuando la mquina alcanza el 75 % de la velocidad nominal, el circuito auxiliar se desconecta. La rama de magnetizacin puede ser
despreciada. Determine:
a) La corriente y el par de arranque.
b) El punto de operacin si la mquina se encuentra operando a 1.750 rpm.

8. Calcule el valor de la capacitancia del condensador que es necesario conectar en serie con
una de las bobinas, para producir un campo magntico rotatorio durante el arranque de
una mquina bifsica equilibrada, alimentada por una fuente monofsica. Se conoce la
impedancia de entrada de la mquina de induccin en la condicin de rotor bloqueado.
9. Realice un programa que le permita integrar las ecuaciones de campo orientado que representan el comportamiento transitorio de la mquina de induccin. Simule el arranque en
37

La rama de magnetizacin puede ser despreciada en este problema.

339

Captulo VII Operacin de la mquina de induccin


vaco y con plena carga. Represente grficamente, para las dos condiciones de arranque,
las siguientes variables en funcin del tiempo38 :
a) El par instantneo.
b) La magnitud de la corriente de magnetizacin.
c) La velocidad del vector de corriente de magnetizacin.
d) La velocidad del rotor.
e) Las corriente directa y la corriente cuadratura.
f ) La corriente en la fase a de la mquina.
g) El lugar geomtrico de la corriente del estator desde el arranque hasta el rgimen
permanente.

10. Demuestre que independientemente del nmero de fases de una mquina de induccin,
siempre es posible su modelacin en rgimen permanente y transitorio con las ecuaciones
de campo orientado.
11. Demuestre mediante el modelo de campo orientado de la mquina de induccin en condiciones de rgimen permanente, que el par mximo se obtiene cuando son iguales la
componente directa y la cuadratura de la corriente del estator, representada en el sistema
de referencia rotrico.
12. Determine el vector espacial de las tensiones del estator a partir de las tensiones lnea a
lnea que alimentan la mquina de induccin39 .
13. Determine el valor de la potencia activa y reactiva instantnea a partir de las definiciones
de los vectores espaciales de tensin y corriente 40 .

38
39
40

Sugerencia: puede utilizar cualquier herramienta de clculo tal como Matlab, Scilab, Octave, Fortran, C++,
Simulink, Basic o equivalentes.
Sugerencia: aplique la transformacin de vectores espaciales a las tensiones de lnea a lnea.
Sugerencia: recuerde que la potencia aparente se determina multiplicando la tensin por el conjugado de la
corriente.

340

Bibliografa

A LLER , J. M., Simple Matrix Method to Introduce Spatial Vector Transformations and Oriented Field Equations in Electric Machine Courses, ICEM96 Proceedings, vol. III, pp. 519-524,
September, Vigo, Spain, 1996.
B LASKE, F., The Principle of Field Orientation as Applied to New TRASKVEKTOR ClosedLoop Control System for Rotating-Field Machines, Siemens Review, vol. 34, pp. 217-220, May
1972.
C LARKE, E., Circuit Analysis of A-C Power Systems: Symmetrical and Related Components,
Volume I, General Electric Series, Wiley, New York, 1950.
C ORTS , M., Curso moderno de mquinas elctricas rotativas, Editores Tcnicos Asociados,
Barcelona, 1994-1995.
C HAPMAN , S. J., Mquinas elctricas, McGraw-Hill, Mxico, 1987.
F ITZGERALD , A. E., K INGSLEY, C. J R . & K USKO, A., Electric Machinery: The Processes,
Devices, and Systems of Electromechanical Energy Conversion, McGraw-Hill, Third Edition,
New York, 1971.
F ORTESCUE , C. L., Method of Symmetrical Coordinates Applied to the Solution of Polyphase
Networks, AIEE Transactions, vol. 37, 1918, pp. 1027-1140.
KOSTENKO, M. P. & PIOTROVSKI, L. M., Mquinas elctricas, vol. II, Editorial Mir, Second
edition, Mosc, 1979.
K RON , G., The Application of Tensors to the Analysis of Rotating Electrical Machinery,
General Electric Review, Schenectady, New York 1942.
L ANGSDORF, A. S., Theory of Alternating Current Machinery, Tata McGraw-Hill, Second Edition, New York, 1974.
L EONHARD , W., Control of Electrical Drives, Springer-Verlag, Berlin, Heidelberg, 1985.

341

Captulo VII Operacin de la mquina de induccin


M AGINNINS , F. S. & S CHULTZ , N. R., Transient Performance of Induction Motors, AIEE
Transactions, vol. 64, pp. 651-656, 1944.
M C P HERSON , G. & L ARAMORE , R. D., An Introduction to Electrical Machines and Transformers, John Wiley & Sons, Singapur, 1990.
R ICHTER , R., Elektrische Maschinen, vol. IV, Verlag Birkhusen AG, Basel, 1954.
S ANZ , J., Mquinas elctricas, Prentice Hall, Madrid, 2002.
S ERRANO , L., Fundamentos de mquinas elctricas rotativas, Universidad Politcnica de Valencia, 1989.
S ERRANO , L., The Modern Space-Phasor Theory, Part I: Its Coherent Formulation and its
Advantages for Transient Analysis of Converted-Fed AC Machines, European Transactions on
Electrical Power Engineering, ETEP, Vde-Verlag, vol. 3, Mar./Apr. 1993, N. 2, pp. 171-180,
1.993.
S ERRANO , L., The Modern Space-Phasor Theory, Part II: Comparison with the Generalized
Machine Theory and the Space-Vector Theory, European Transactions on Electrical Power
Engineering, ETEP, Vde-Verlag, vol. 3, May/June. N. 3, pp. 213-219, 1993.
T HALER , G. & W ILCOX , M., Mquinas elctricas, Editorial Limusa, Mxico, 1979.
T IBERIO , F., Pruebas sobre Mquinas elctricas, Editorial Vicens-Vives, Primera Edicin, Barcelona, 1965.
VAS , P., Vector Control of AC Machines, Oxford University Press, New York, 1990.
VAS , P., Electrical Machines and Drives. A Space Vector Approach, Oxford University Press,
New York, 1992.
V EINOTT, C. G., Motores elctricos de potencia fraccionaria y subfraccionaria, Marcombo,
Barcelona, 1978.
W HITE , D. C. & WOODSON , H. H., Electromechanical Energy Conversion, John Wiley &
Sons, New York, 1959.

342

CAPTULO VIII

La mquina sincrnica

Las mquinas de corriente continua y de induccin tienen un amplio rango de aplicaciones industriales tales como traccin, bombeo, control y otros 1 . Sin embargo, la operacin del sistema
elctrico de potencia requiere la conversin de grandes cantidades de energa primaria 2 , en energa y potencia elctrica. La energa elctrica puede ser transportada y convertida en otras formas
de energa en forma limpia y econmica. La mquina sincrnica es hoy por hoy el convertidor
ms utilizado para realizar esta tarea.
Dependiendo del sistema mecnico de accionamiento3 , las mquinas sincrnicas pueden construirse de rotor liso, cuando deban operar en altas velocidades4, o con rotor de polos salientes,
cuando son accionadas a menor velocidad. En la figura 8.1 se observan tres salas de mquinas,
dos de plantas de generacin hidroelctrica y una trmica 5 .
Aun cuando un gran porcentaje de mquinas sincrnicas se emplean como generadores en las
plantas de produccin de energa elctrica debido fundamentalmente al alto rendimiento que
es posible alcanzar con estos convertidores 6 y a la posibilidad de controlar la tensin en numerosas ocasiones se emplea a nivel industrial como elemento motriz. Como otros convertidores
electromecnicos, la mquina sincrnica es completamente reversible y da a da aumenta el
nmero de aplicaciones donde puede utilizarse con grandes ventajas, en especial cuando se controla mediante fuentes electrnicas de frecuencia y tensin variable. El principal inconveniente
para su uso como motor es que no desarrolla par de arranque, pero si se incluye en el rotor de
la mquina un devanado auxiliar de jaula de ardilla, es posible obtener par de aceleracin como
1
2
3
4
5
6

Condiciones de motorizacin o traccin de carga mecnica.


Petrleo, gas natural, agua, carbn, uranio, viento, oleaje, luz.
Tipo de turbina hidrulica, trmica, elica, etc.
3.000 rpm a 50Hz 3.600 rpm a 60Hz.
Guri y Tacoa en Venezuela y la planta nuclear Diablo Canyon en California.
Las mquinas de induccin no pueden producir par sin prdidas en el rotor, a diferencia de las mquinas sincrnicas, donde este requisito desaparece.

343

Captulo VIII La mquina sincrnica

(a) Guri en Venezuela

(b) Diablo Canyon en California

(c) Guri, estator en construccin

(d) Tacoa en Venezuela

(e) Guri, casa de mquinas

(f) Macagua, sala de mquinas

Figura 8.1 Plantas hidroelctricas y trmicas. Fotos cortesa de EDELCA, EDC y PG&E

344

VIII.1 Descripcin de la mquina sincrnica

(a) Estator de la mquina sincrnica

(b) Rotor de polos salientes

Figura 8.2 Partes de las mquinas sincrnicas


motor de induccin hasta una velocidad cercana a la de sincronismo y excitar en el momento
apropiado la bobina del campo, con la finalidad de sincronizar la mquina a la red mediante
los pares transitorios adicionales que se obtienen durante este proceso. Si la fuente de alimentacin puede reducir la frecuencia angular de las tensiones o corrientes de armadura a valores
muy bajos, la mquina es capaz de sincronizarse a esa red y posteriormente ser acelerada al
mismo tiempo que se incrementa paulatinamente la frecuencia de la fuente. Como la construccin de fuentes de gran potencia controladas en frecuencia es hoy da factible mediante puentes
inversores con interruptores estticos, es posible que en el futuro esta mquina incremente notablemente su importancia como accionamiento industrial e incluso desplace a las mquinas de
corriente continua.
VIII.1

D ESCRIPCIN

DE LA MQUINA SINCRNICA

La mquina sincrnica es un convertidor electromecnico de energa con una pieza giratoria denominada rotor o campo, cuya bobina se excita mediante la inyeccin de una corriente continua,
y una pieza fija denominada estator o armadura, por cuyas bobinas circula corriente alterna. Las
corrientes alternas que circulan por los enrollados del estator producen un campo magntico
rotatorio que gira en el entrehierro de la mquina con la frecuencia angular de las corrientes de
armadura. El rotor debe girar a la misma velocidad del campo magntico rotatorio producido
en el estator para que el par elctrico medio pueda ser diferente de cero. Si las velocidades angulares del campo magntico rotatorio y del rotor de la mquina sincrnica son diferentes, el
par elctrico medio es nulo. Por esta razn a esta mquina se la denomina sincrnica; el rotor
gira mecnicamente a la misma frecuencia del campo magntico rotatorio del estator durante la
operacin en rgimen permanente. En la figura 8.2 (a) y (b), se observa el estator y rotor de una
mquina sincrnica de polos salientes.
Durante la operacin de la mquina sincrnica en rgimen permanente, la velocidad mecnica
del rotor es igual a la velocidad angular del campo magntico rotatorio producido por el estator.
En estas condiciones, sobre los conductores o bobinas del campo no se induce fuerza electromotriz. Para producir fuerza magnetomotriz en el rotor es necesario inyectar corriente en esta

345

Captulo VIII La mquina sincrnica


bobina mediante una fuente externa. De esta forma se obtienen dos campo magnticos rotatorios
que giran a la misma velocidad, uno producido por el estator y otro por el rotor. Estos campos
interactan produciendo par elctrico medio y se realiza el proceso de conversin electromecnica de energa. De acuerdo con la expresin 4.44, la condicin necesaria, pero no suficiente,
para que el par medio de la mquina sea diferente de cero es:

e = p m

(8.1)

Donde:
p

es el nmero de pares de polos de la mquina sincrnica.

La bobina del rotor o campo de la mquina sincrnica se alimenta mediante la inyeccin de


corriente continua, como se mencion anteriormente, con la finalidad de producir un campo
magntico de magnitud constante, semejante al de un imn permanente, pero de una intensidad
mucho mayor. Debido a que el rotor de la mquina gira en rgimen permanente a la velocidad
sincrnica, el campo magntico constante producido en este sistema se comporta, desde el punto
de vista del estator, como un campo magntico rotatorio. En la figura 8.3 se ha representado el
esquema bsico de una mquina sincrnica trifsica de polos salientes.
Para evaluar la magnitud del par en una mquina sincrnica se puede recordar la expresin 4.55:
Te = k Fr Fe sin

(8.2)

Donde:
k

es una constante de proporcionalidad que depende de la geometra de


la mquina y de la disposicin fsica de las bobinas

Fe

es la amplitud de la distribucin sinusoidal de la fuerza magnetomotriz


del estator

Fr

es la amplitud de la distribucin sinusoidal de la fuerza magnetomotriz


del rotor

es el ngulo entre las amplitudes de las dos fuerzas magnetomotrices,


conocido generalmente como ngulo de carga

Las fuerzas magnetomotrices del estator Fe y del rotor Fr tienen una amplitud constante y para
que en la expresin 8.2 el par medio resulte constante, es necesario que el ngulo entre las
dos fuerzas magnetomotrices sea constante en el tiempo durante la operacin en rgimen permanente. Para lograr esto, las dos fuerzas magnetomotrices deben girar a la misma velocidad
angular.
Cuando la mquina sincrnica se encuentra desequilibrada, el campo magntico rotatorio producido por las bobinas del estator es elptico. Este campo se puede descomponer a su vez en
dos campos magnticos rotatorios circulares de sentidos contrarrotativos. Para que sea posible
la produccin de par elctrico medio en estas condiciones, es necesario que la velocidad del rotor est sincronizada con uno de los dos campos magnticos contrarrotativos. El campo que est

346

VIII.1 Descripcin de la mquina sincrnica

(a) Modelo elemental demostrativo

(b) Esquema bsico

Figura 8.3 Esquema bsico de una mquina sincrnica trifsica de polos salientes

347

Captulo VIII La mquina sincrnica


fuera de sincronismo y gira en el sentido contrario del rotor, produce par elctrico transitorio,
pero su valor medio es cero.
Si se cortocircuita la bobina de campo en el rotor de la mquina sincrnica, es posible en ciertos
casos acelerar el rotor como si fuera un motor de induccin con rotor devanado. En el campo se
inducen fuerzas electromotrices con la frecuencia del deslizamiento cuando el campo magntico
rotatorio del estator corta a los conductores del campo. La fuerza electromotriz inducida en el
rotor fuerza la circulacin de corrientes por este devanado. Aun cuando el par elctrico puede ser
muy reducido, en algunas ocasiones este mtodo puede ser utilizado para arrancar en la mquina
sincrnica sin cargas mecnicas acopladas.
VIII.2

M ODELO

DE LA MQUINA SINCRNICA

Analizando el comportamiento de los ejes elctricos de la mquina sincrnica en el sistema de


coordenadas correspondiente a las bobinas reales o fsicas, se satisface el siguiente sistema de
ecuaciones:
 d 

 


vabc, f = Rabc, f iabc, f +
abc, f
(8.3)
dt

En los sistemas lineales, la relacin entre las corrientes que circulan por las bobinas y los enlaces
de flujo que las enlazan vienen dados por la relacin:

 


(8.4)
abc, f ( , i) = Labc, f ( ) iabc, f

Sustituyendo esta relacin en la expresin 8.3 se obtiene el resultado siguiente:

vabc, f

d 
 


 d d 

iabc, f + Labc, f
iabc, f +
Labc, f iabc, f =
dt dt  

 dt

 



= Rabc, f iabc, f + Labc, f p iabc, f + abc, f iabc, f

Rabc, f

(8.5)

El sistema de ecuaciones diferenciales 8.5 representa el comportamiento dinmico de las bobinas


de la mquina sincrnica en coordenadas primitivas. Este sistema se expresa en forma cannica
como:

 
1 
 



 
vabc, f Rabc, f + abc, f
p iabc, f = Labc, f
(8.6)
iabc, f


La matriz de inductancia Labc, f depende de la posicin relativa del rotor con respecto al
estator, por esta razn la matriz de transicin de estado tambin depende de la posicin angular
del rotor. Si la velocidad de la mquina es constante, la posicin angular del rotor es:

= 0 + m t

(8.7)

La solucin del sistema 8.6 puede obtenerse mediante mtodos numricos de integracin, utilizando algoritmos tales como Euler, Runge-Kutta o Adams, entre muchos otros. El principal
inconveniente que se presenta es la necesidad de evaluar e invertir la matriz de inductancias
de la mquina en cada paso de integracin, debido a la dependencia de esta matriz con la posicin angular del rotor. Los computadores personales actuales son capaces de resolver este

348

VIII.2 Modelo de la mquina sincrnica


problema, aun cuando en el pasado estos clculos representaban grandes dificultades. Por este
motivo durante varias dcadas se desarrollaron transformaciones de coordenadas que simplifican el problema, aceleran notablemente los clculos y permiten interpretar ms fcilmente el
comportamiento dinmico y esttico de la mquina sincrnica.
Durante los perodos transitorios, la velocidad angular de la mquina cambia y la posicin angular del rotor es una nueva variable de estado que debe ser evaluada para determinar su dependencia temporal. En este caso es necesario incorporar una ecuacin adicional al sistema 8.6 para
representar el comportamiento dinmico del eje mecnico de la mquina:


t 
1
iabc, f abc, f iabc, f Tm = J +
2

(8.8)

La ecuacin 8.8 representa el balance del par elctrico y mecnico en el eje del rotor. El par
acelerante es igual al par elctrico del convertidor, menos el par resistente opuesto por la carga
y por las prdidas mecnicas. La ecuacin diferencial de segundo orden 8.8 puede expresarse
mediante dos ecuaciones diferenciales de primer orden:

 
(
t 


1 1

m = J 2 iabc, f abc, f iabc, f Tm


(8.9)
= m
Donde:
J

es el momento de inercia del rotor

Tm

es el par mecnico resistente

es el coeficiente de friccin dinmica

El sistema de seis ecuaciones diferenciales formado por las cuatro ecuaciones del sistema 8.6
y las dos ecuaciones mecnicas representadas por la expresin 8.9, definen el comportamiento
dinmico y transitorio completo de la mquina sincrnica de la figura 8.3. Este sistema de ecuaciones diferenciales es no lineal y los coeficientes son variables en el tiempo, por este motivo hay
que recurrir a tcnicas numricas para evaluar el comportamiento de la mquina o simplificar el
problema mediante la tcnica de transformacin de coordenadas.
En la matriz de inductancia de la mquina sincrnica, se encuentra toda la informacin necesaria
para determinar su comportamiento. En la matriz de inductancias se resume la informacin sobre
la disposicin geomtrica de las bobinas, sus acoplamientos, nmero de vueltas y reluctancias
de los diferentes caminos magnticos. Una vez conocida la matriz de inductancias, se puede
evaluar la matriz de par calculando la derivada parcial de esta matriz con respecto a la posicin
angular del rotor. La matriz de inductancias de la mquina sincrnica esquematizada en la figura
8.3 posee la siguiente estructura:




[Lee ( )] [Ler ( )]
(8.10)
Labc, f ( ) =
[Lre ( )]
Lf

Laa ( ) Mab ( ) Mac ( )


M
(

)
a
f
 
t

[Lee ( )] = Mba ( ) Lbb ( ) Mbc ( ) ; Le f ( ) = L f e ( ) = Mb f ( )
Mca ( ) Mcb ( ) Mcc ( )
Mc f ( )

349

Captulo VIII La mquina sincrnica


Donde:
e

es el subndice referido a las bobinas del estator

es el subndice referido a las bobinas del campo

a, b, c

son los subndices de las tres bobinas fsicas del estator

Para evaluar cada una de las inductancias definidas en la expresin 8.10, es preciso utilizar las
expresiones 3.10 y 3.11, desarrolladas en el captulo 3.
Cada una de las inductancias de la mquina sincrnica se puede representar como una funcin
del ngulo , que es peridica porque se repite nuevamente cada vez que el rotor realiza un giro
completo. Esta propiedad permite representar estas funciones mediante expansiones en series de
Fourier, con el ngulo como variable. Si la pieza polar se disea convenientemente7, es posible
representar las inductancias de la mquina con un nmero reducido de los trminos de la serie.
La expresin de la matriz de inductancias ms simple consiste en considerar trminos dependientes hasta en 2 , para las inductancias estator-estator, y trminos en para las inductancias
estator-rotor.
La inductancia del rotor L f es independiente de la posicin del rotor, debido a que el estator
de la mquina es aproximadamente liso8. El resto de las inductancias propias y mutuas depende
de la posicin angular , si el rotor de la mquina es de polos salientes. Las permeanzas de los
caminos magnticos de las bobinas del estator y de los acoplamientos estator-rotor son dependientes de la posicin angular . Cuando la pieza polar del rotor se encuentra alineada con una
de las bobinas del estator, el camino magntico posee la mxima permeanza. Si la pieza polar se
encuentra en cuadratura con la bobina, el entrehierro es muy grande y disminuye la permeanza.
La variacin de la permeanza depende del ngulo 2 porque una bobina alineada con el polo
norte del rotor tiene el mismo camino magntico cuando el alineamiento ocurre con el polo sur.
Estas inductancias se pueden representar aproximadamente mediante las siguientes funciones:
Laa ( ) = L1e + M2e cos 2 +
2
)+
3
4
Lcc ( ) = L1e + M2e cos 2( ) +
3

Mab ( ) = Mba ( ) = M1e M2e cos 2( + ) +


6

Mac ( ) = Mca ( ) = M1e M2e cos 2( ) +


6

Mbc ( ) = Mcb ( ) = M1e M2e cos 2( ) +


2
Lbb ( ) = L1e + M2e cos 2(

(8.11)
(8.12)
(8.13)
(8.14)
(8.15)
(8.16)

Donde9 :
7
8
9

Variando el entrehierro para producir una densidad de campo magntico distribuido sinusoidalmente.
Para esta consideracin es necesario despreciar el efecto de las ranuras del estator.
En este caso la aproximacin se debe a que la dispersin de la bobina no est siendo considerada. La dispersin
puede ser considerada en el modelo como si estuviese conectada externamente a los bornes de la mquina.

350

VIII.3 Transformacin a vectores espaciales

3
3
Ld (L1e + M2e ) ; Lq (L1e M2e ) ; Ld f
2
2
Ld Lq
Ld + Lq
; M2e =
L1e =
3
3
L1e
M1e
2

3
Me f
2

(8.17)
(8.18)
(8.19)

En lo que se refiere a los acoplamientos mutuos estator-rotor, la funcionalidad de las inductancias es diferente porque al girar el rotor 180, la bobina del campo invierte su polaridad. Las
inductancias del estator varan entre un valor mximo y un mnimo, siempre positivo respecto a
la posicin angular del rotor. Sin embargo, los acoplamientos mutuos estator-rotor varan entre
un valor mximo positivo hasta un valor mximo negativo, que en valor absoluto son idnticos cuando el rotor de la mquina gira 180. Las inductancias mutuas entre el estator y el rotor
pueden ser aproximadas mediante las siguientes funciones:
Ma f ( ) = M f a ( ) = Me f cos +

(8.20)

2
)+
(8.21)
3
4
Mc f ( ) = M f c( ) = Me f cos( ) +
(8.22)
3
Si el rotor de la mquina sincrnica es liso, todas las inductancias del estator son
 independientes
de la posicin del rotor. En esta situacin la matriz de inductancias Labc, f ( ) se expresa de la
siguiente forma:

L1e
M1e
M1e
Me f cos


M1e
L1e
M1e
Me f cos( 23 )
(8.23)
Labc, f ( ) =

M1e
M1e
L1e
Me f cos( 43 )
Me f cos Me f cos( 23 ) Me f cos( 43 )
Lf
Mb f ( ) = M f b ( ) = Me f cos(

An para el caso de una mquina sincrnica de rotor liso, la solucin del sistema de ecuaciones
diferenciales que determina el comportamiento de la mquina sincrnica requiere el uso de
mtodos numricos, debido a la dependencia de las inductancias mutuas entre el estator y el
campo, con la posicin del rotor. El modelo de la mquina sincrnica de rotor liso o de
polos salientes se puede obtener mediante transformaciones del sistema de coordenadas. La
transformacin a vectores espaciales y a coordenadas dq0, introducidas en los captulos V y VI,
permiten simplificar notablemente estos modelos.
VIII.3

TRANSFORMACIN

A VECTORES ESPACIALES

Para aplicar la transformacin de vectores espaciales a las ecuaciones 8.5 y 8.8 que representan
el comportamiento de la mquina sincrnica en coordenadas primitivas, es conveniente expresar
por separado las ecuaciones del estator y del rotor:

 
[ve ] = [Re ] [ie ] + p [Lee ] [ie ] + Le f i f
(8.24)
351

Captulo VIII La mquina sincrnica


vf = Rf if + p





L f e [ie ] + L f i f

(8.25)

Aplicando esta transformacin de vectores espaciales a la expresin 8.24, se obtienen el siguiente resultado:
(
)
r
3
3
ve = Re ie + p (L1e + M1e ) ie + M2e e j2 ie +
Me f e j i f
(8.26)
2
2
Donde:
r

L1e M1e

M1e L1e

M1e M1e


2
(8.27)
ve =
va + vb + 2 vc
3
r

2
1 2 [Re ] [ie ] = Re ie
(8.28)
3
r

2
1 2 [Lee ] [ie ] =
3
r

2
1 2
=
3

cos 2( + 6 ) cos 2( 6 )
cos 2
M1e
M1e + M2e cos 2( + 6 ) cos 2( 23 ) cos 2( 2 ) [ie ] =

L1e
cos 2( 6 ) cos 2( 2 ) cos 2( 43 )

3
(L1e + M1e ) ie + M2e e j2 ie =
2
1
1
= (Ld + Lq ) ie + (Ld Lq )e j2 ie
2
2
r
r



2
3
1 2 Le f i f =
Me f e j i f = Ld f e j i f
3
2

(8.29)
(8.30)

Reemplazando las definiciones de los vectores espaciales en la ecuacin 8.25 se obtiene:


 j



e ie + e j ie
v f = R f i f + p Ld f
+Lf if
(8.31)
2
El par elctrico es:
 1

t 
 
1
Te = iabc, f abc, f iabc, f = [ie ]t [ee ] [ie ] + [ie ]t e f i f =
2
2

ej 3
e j 3
1
e j 3
1
ej 3

M2e t
4
4

j2
j2

j
j

j
j

j
j
[ie ] e e 3 e 3
=j
e
e
e
[ie ] +
e 3 e 3

j 3
j 83
j 3
j 83

e
e
e
e
e
e
352

VIII.4 Transformacin a coordenadas rotricas

if =

Me f
2

2
[ie ]t e j e j 3 e j e j 3

2
4
4

e j 3
ej 3
r n
n
o
o
M
3
3 j
e
f
= M2e (e j ie )2 (e j ie )2 + j
e ie e j ie i f =
4j
2
2
n
o
n
o
1
j
2
j
= (Ld Lq )m (e ie ) + Ld f m e ie i f
2
+ j

(8.32)

Las expresiones 8.26, 8.31 y 8.32 modelan la mquina sincrnica utilizando vectores espaciales.
La principal ventaja de esta transformacin consiste en la reduccin de las tres ecuaciones del
estator a una sola en variable compleja. Por otra parte, aun cuando la dependencia angular en
se mantiene en este sistema de coordenadas, las correspondientes expresiones han sido simplificadas convenientemente al utilizar los trminos e j . En la expresin 8.32 correspondiente al
par elctrico pueden observarse dos componentes: el par de reluctancia y el par producido entre
las fuerzas magnetomotrices del estator y del campo.

VIII.4

TRANSFORMACIN

A COORDENADAS ROTRICAS

Para eliminar la dependencia en existente en el modelo de la mquina sincrnica en vectores


espaciales, es posible referir las variables del estator al sistema de referencia del rotor, el cual se
encuentra exactamente en la posicin con respecto al sistema solidario con el estator. Por esta
razn es posible multiplicar la ecuacin del estator por e j para referir estas ecuaciones a un
sistema de coordenadas sincronizado con el eje del campo. Este nuevo sistema de coordenadas
es conocido como ejes d y q. El eje directo d apunta en la misma direccin que el eje del campo
f . El eje cuadratura q se encuentra a 90 de adelanto con respecto al eje d. De esta forma se
pueden introducir las siguientes definiciones:
j
vdq
e = vd + jvq = ve e

(8.33)

j
idq
e = id + jiq = ie e

(8.34)

Derivando la expresin 8.34 se obtiene la relacin siguiente:


e j pie = pid + jpiq + j idq
e

(8.35)

Al multiplicar la ecuacin 8.26 por el trmino de rotacin e j se obtiene:




1
1
j2
j
j
j
j
(Ld + Lq ) ie + (Ld Lq )e ie + Ld f e i f
+e
p
e v e = Re i e e
2
2
 1




1
dq
dq
dq
dq
dq

i
+

i
+ Ld f pi f + j i f
(L
+
L
)
pi
+
j
(L

L
)
pi
+
j
vdq
=
R
i
+
q
q
e e
d
d
e
e
e
e
e
2
2
(8.36)
353

Captulo VIII La mquina sincrnica


Descomponiendo la expresin 8.36 en parte real y parte imaginaria resulta:

vd = Re id + p Ld id + Ld f i f Lq iq = Re id + pd q


vq = Re iq + p Lq iq + Ld id + Ld f i f = Re iq + pq + d

(8.37)
(8.38)

Realizando transformaciones semejantes en la ecuacin 8.31 se obtiene el resultado siguiente:




Ld f h dq  dq  i
+Lf if =
i + ie
vf = Rf if + p
2 e

v f = R f i f + p L f i f + Ld f id = R f i f + p f
(8.39)
Finalmente, transformando las variables espaciales de la expresin 8.32 correspondiente al par
elctrico, se obtiene:
n
o
n
o
1
2
j
)
+
L
m
e
Te = (Ld Lq )m (idq
i
e if =
df
e
2

(8.40)
= Ld Lq id iq + Ld f iq i f = d iq q id = edq idq
e

El sistema de ecuaciones diferenciales que determina el comportamiento dinmico de la mquina sincrnica se puede expresar de la siguiente forma:

vd = Re id + pd q

v = R i + p +
q
e q
q
d
(8.41)

v
=
R
i
+
p

f
f
f
f

dq
dq
J = e ie Tm ( )
Donde:

d = Ld id + Ld f i f
q = Lq iq
f = L f i f + Ld f id
edq = d + jq
VIII.5

TRANSFORMACIN

DE

PARK

En la mquina sincrnica, el campo magntico rotatorio producido por las fuerzas magnetomotrices de los devanados estatricos, gira a la velocidad sincrnica e . El rotor de la mquina
tambin gira a la velocidad sincrnica r = e . Por esta razn es conveniente referir las ecuaciones diferenciales que definen el comportamiento de la mquina a un sistema de coordenadas
solidario con el rotor. De acuerdo con estos lineamientos se definen los siguientes ejes magnticos:
Eje d :

354

Gira con respecto al estator a la velocidad del rotor y en todo momento se encuentra
colineal con el eje magntico del campo.

VIII.5 Transformacin de Park


Eje q :

Rota con respecto al estator a la velocidad del rotor y en todo momento se encuentra
en cuadratura con el eje magntico del campo.

Eje 0 :

Fijo en el estator y se encuentra desacoplado magnticamente del resto de los ejes


de la mquina.

Eje f :

Solidario con el sistema rotrico y colineal con el eje magntico de la bobina de


campo.

Aun cuando los ejes d y q giran a igual velocidad que el rotor, ambos representan variables del
estator. El eje 0 es necesario para permitir que la transformacin de coordenadas sea bidireccional, es decir, se pueda transformar de variables primitivas a variables dq0 y viceversa. El eje 0
tiene una estrecha relacin con las variables de secuencia cero de la transformacin de componentes simtricas. En la prctica el eje 0 permite representar flujos de dispersin que no estn
acoplados con otras bobina de la mquina. En la figura 8.3(b) se ha representado el sistema de
coordenadas dq0 f .
La matriz de transformacin de coordenadas dq0 f a coordenadas primitivas se define mediante la relacin:




(8.42)
iabc, f = [A] idq0, f

Si la transformacin anterior se escoge de tal forma que la matriz [A] sea hermitiana10 , la transformacin es conservativa en potencia. Cuando la matriz es hermitiana y real, se obtiene:






(8.43)
idq0, f = [A]1 iabc, f = [A]t iabc, f

La matriz de transformacin [A] se puede obtener multiplicando la transformacin de coordenadas primitivas a coordenadas ortogonales 0,11 por la transformacin de coordenadas 0 a
coordenadas dq0:12

1
1
0
2
i
ia

3
1 i
ib = 2 21
(8.44)
2

2
3
ic
i0
21 23 1
2

i
id
cos sin 0
i = sin cos 0 iq
(8.45)
0
0
1
i0
i0

r
1
cos
sin
id
ia
2


2
2
1
ib = 2
(8.46)
cos 3  sin 3  2 iq
3
4
4
1
i
ic

0
sin 3
cos 3
2

10
11
12

Inversa de la matriz de transformacin [A] igual a su traspuesta conjugada.


Transformacin de Clark.
Rotacin en introducida en el captulo 4.

355

Captulo VIII La mquina sincrnica


La matriz de la expresin 8.46 se conoce como transformacin de Park. La transformacin de
coordenadas primitivas abc, f a coordenadas dq0, f es:



2
4

cos
0
cos

cos

3 
3 
id
i
a
r
2
sin 43
0
iq
sin sin 3
ib

= 2 1

1
1
(8.47)

0
ic
i0
3
2
2
2
q

3
if
if
0
0
0
2
La transformacin de Park utilizada es hermitiana y por tanto es invariante en potencia:

t  
  
t 

[A] idq0, f =
p(t) = vabc, f iabc, f = [A] vdq0, f

t 
 
t


= vdq0, f [A]t [A] idq0, f = vdq0, f idq0, f = p(t)

Aplicando la transformacin 8.47 al sistema de ecuaciones 8.5 se obtiene:



 

 
 




vdq0, f = Rdq0, f idq0, f + Ldq0, f p idq0, f + Gdq0, f idq0, f

(8.48)

(8.49)

Donde:



Rdq0, f = [A]t Rabc, f [A]




Ldq0, f = [A]t Labc, f [A]

 



 d
 


Gdq0, f = dq0, f + Hdq0, f = [A]t abc, f [A] + [A]t Rabc, f
[A]
d

(8.50)
(8.51)
(8.52)

Por otra parte, la ecuacin dinmica del movimiento se puede expresar de la siguiente forma:
t 


1
J + = idq0, f dq0, f idq0, f Tm
2

(8.53)

Evaluando explcitamente las expresiones 8.50 a 8.52 y sustituyendo el resultado en las ecuaciones diferenciales 8.49 y 8.53 se obtiene:

0
Ld f p
id
vd
Re + Ld p Lq

vq

Re + Lq p
0
Ld f
iq

= Ld
i0
v0

0
0
R0 + L0 p
0
Ld f p
0
0
Rf +Lf p
if
vf

(8.54)
J p = Ld Lq id iq + Ld f iq i f Tm
El modelo de la mquina sincrnica obtenido a partir de la transformacin de vectores espaciales
referidos a las coordenadas del rotor 8.41 coincide con el modelo 8.54, obtenido aplicando la
transformacin de Park 8.51. La transformacin a vectores espaciales 8.27 y la transformacin
de Clark 8.44 estn ntimamente relacionadas. Lo mismo sucede entre la rotacin 8.45 y referir
las variables espaciales del estator al sistema de coordenadas del rotor multiplicndolas por el
trmino e j .

356

VIII.5 Transformacin de Park

Figura 8.4 Modelo en coordenadas dq0 f de la mquina sincrnica


En un sistema trifsico sin neutro no circula corriente de secuencia cero, pero cuando las tres
corrientes de fase encuentran un camino de retorno, es necesario considerar esta componente.
La componente de secuencia cero representa la circulacin de corrientes iguales y en fase por
las bobinas de la mquina. Estas corrientes no producen magnetizacin debido a que la suma de
las fuerzas magnetomotrices de las tres bobinas es cero. Sin embargo, los flujos de dispersin
s poseen componente de secuencia cero. En el modelo de la mquina no existe acoplamiento
magntico de esta secuencia con el resto de las bobinas. Esta componente no puede producir par
elctrico, pero influye en las prdidas de la mquina y en las fuerzas electromotrices sobre las
bobinas. En la expresin 8.54 no aparecen fuerzas electromotrices de generacin sobre la bobina
de campo. Esto se debe a que el sistema de coordenadas dq0 es solidario al eje f del campo. Los
flujos de las bobinas d y q no cruzan tangencialmente a los conductores del campo. Sin embargo,
en este eje pueden aparecer fuerzas electromotrices por transformacin, debido a que el flujo de
la bobina del eje directo atraviesa el devanado de campo. Por el contrario, el eje cuadratura no
puede producir ningn efecto sobre el campo debido a que se encuentra permanentemente en
una posicin ortogonal.
La mquina sincrnica puede ser representada mediante un modelo fsico en coordenadas dq0
f , similar al obtenido en el captulo 4 para la mquina generalizada. En la figura se presenta
el modelo en coordenadas dq0 f que satisface las ecuaciones 8.54. En la mquina real, las
corrientes id e iq no circulan por ningn devanado fsico, para determinar las corrientes reales es
necesario aplicar la transformacin inversa de coordenadas dq0 f a coordenadas primitivas.
Cada pareja de escobillas separa las capas de corriente de las bobinas equivalentes. La fuerza
electromotriz de todos los conductores que forman cada una de las bobina se obtiene en bornes
de las escobillas. Cuando por un par de escobillas se inyecta una corriente, sta circula entrando

357

Captulo VIII La mquina sincrnica


a los conductores a la derecha del eje que define la posicin de estas escobillas y saliendo en los
conductores a la izquierda. Esta configuracin produce una fuerza magnetomotriz orientada en
el eje de las escobillas, tal como se muestra en la figura 8.4.
Las fuerzas electromotrices de generacin que aparecen sobre los conductores se recolectan
en los circuitos que se encuentran en cuadratura con el flujo que las produce. El campo y la
bobina del eje d producen generacin sobre la bobina del eje q, y la bobina del eje q produce
generacin sobre el eje d, pero sobre la bobina de campo no se produce generacin porque este
devanado no es cortado por el flujo de los dems ejes. En el sistema de referencia utilizado, las
fuerzas electromotrices de generacin aparecen adelantadas 90 con respecto a los flujos que las
producen. Si en las bobinas primitivas se inyecta un sistema balanceado de corrientes trifsicas,
se obtienen las siguientes corrientes en el sistema de coordenadas dq0:


r

cos
cos 23  cos 43 
cos( t + )
id
2
iq = 2
sin 43
sin sin 3
2Ie cos( t + 23 ) =
3
1
1
1

i0
cos( t + 43 )
2
2
2

cos( t )
id

iq = 3Ie sin( t )
0
i0

(8.55)

Si la posicin angular del rotor se sincroniza 13 con la variacin angular de las corrientes en
la expresin 8.55, las corrientes en las coordenadas dq0 son independientes del tiempo. En esta condicin, los trminos que dependen de las derivadas de las corrientes se anulan. En este
sistema de coordenadas, corrientes constantes en el tiempo producen fuerzas magnetomotrices
constantes en las bobinas dq0 transformadas. Como la transformacin est sincronizada con la
velocidad angular de las corrientes durante el rgimen permanente, el campo magntico producido por las bobinas d y q gira con la misma velocidad y como resultado se obtiene el mismo
campo magntico rotatorio de la mquina sincrnica en coordenadas primitivas, excitada mediante un sistema trifsico balanceado de corrientes.
El par electromagntico de la mquina est determinado por la interaccin entre fuerzas magnetomotrices no alineadas. Por una parte la fuerza magnetomotriz del campo produce par al
interactuar con el flujo de la bobina que representa al eje q. La fuerza magnetomotriz del eje
d produce par en su interaccin con la fuerza magnetomotriz del enrollado cuadratura. Exactamente igual pero con sentido contrario, la fuerza magnetomotriz del eje q produce par con la
fuerza magnetomotriz del eje d. Si la reluctancia de los caminos magnticos d y q son iguales,
estos dos pares se neutralizan. Cuando la reluctancia del eje d es menor que la del eje q, el par
que produce la fuerza magnetomotriz del eje d sobre el eje q es mayor que en la direccin contraria y se produce un par neto resultante debido a la variacin de reluctancia entre los dos ejes.
Desde otro punto de vista se puede interpretar que la pieza polar intenta alinearse con la fuerza
electromotriz resultante en la mquina. Si la mquina posee un rotor cilndrico, este par es nulo.
En la ecuacin 8.40, el par elctrico se divide en dos componentes: la primera es proporcional
al producto de la corriente de campo i f por la corriente de la bobina cuadratura iq y la segunda
13

(t) = t + 0 .

358

VIII.6 Rgimen permanente


depende del producto de las corrientes id e iq . Esta ltima componente se anula si la inductancia
Ld es igual a la inductancia Lq . La inductancia Ld est definida por la permeanza del eje directo,
mientras que Lq se define por la permeanza del eje cuadratura.
En la figura 8.4 se han representado las fuerzas magnetomotrices en coordenadas dq0. Se observa que sobre la pieza polar aparecer un par elctrico que intentar alinear el rotor con la fuerza
magnetomotriz total. Cuando se analizan las fuerzas electromotrices de generacin en el sistema
de ecuaciones 8.54 se observan dos trminos similares: el primero depende de la inductancia Lq ,
que es proporcional a la permeanza del camino cuadratura y determina la generacin sobre el eje
directo; el segundo trmino depende de Ld , es proporcional a la permeanza del camino directo y
determina parte de la generacin sobre el eje cuadratura.
VIII.6

R GIMEN

PERMANENTE

Para analizar el comportamiento de la mquina sincrnica en rgimen permanente es necesario


excitar los circuitos de armadura con un sistema equilibrado y simtrico de corrientes. Adems,
en estas condiciones el rotor de la mquina debe girar a la velocidad sincrnica. La posicin
relativa del rotor con respecto al sistema de referencia solidario al estator es:

= t + 0

(8.56)

Sustituyendo la expresin 8.56, en la transformacin a coordenadas dq0 definida mediante la


relacin 8.55, se obtiene el siguiente resultado:

(8.57)
id = 3Ie cos(0 ) ; iq = 3Ie sin(0 ) ; id = 0
Las corrientes de rgimen permanente en coordenadas primitivas, transformadas al sistema de
coordenadas dq0, son independientes del tiempo. El argumento de las funciones trigonomtricas
(0 ) proyecta la fuerza magnetomotriz producida por el sistema balanceado de corrientes
primitivas, segn las direcciones de los nuevos ejes coordenados. En la figura 8.4 se representa
el efecto de la transformacin para un sistema en rgimen permanente y equilibrado. Como las
corrientes id , iq e i0 son independientes del tiempo, los trminos de transformacin son nulos
en el nuevo sistema de coordenadas y en estas condiciones, las ecuaciones del modelo 8.54 se
reducen a:
vd = Re id Lq iq = Re id Xq iq
(8.58)

VIII.7

vq = Re iq + Ld id + Ld f i f = Re iq + Xd id + e f

(8.59)

vf = Rf if

(8.60)

Te = (Ld Lq )id iq + Ld f iq i f

(8.61)

D IAGRAMA

FASORIAL

Mediante la transformacin inversa de Park 8.46 se puede obtener la tensin de la fase a:


r
1
2
(vd cos vq sin + v0 )
(8.62)
va (t) =
3
2
359

Captulo VIII La mquina sincrnica


La tensin v0 es nula debido a que no existe corriente de secuencia cero en el sistema trifsico
balanceado14 . Por otra parte, la transformacin de coordenadas gira a velocidad sincrnica segn
se describe en la expresin 8.56. En estas condiciones se determina la tensin en bornes de la
fase a de la mquina como:
r
r
i


2 
2 h
va (t) =
vd cos( t + 0 ) vq sin( t + 0 ) =
e vd + jvq e j( t+0 ) =
3
3
h
i
h
i

= e 2 Vd + jVq e j( t+0 ) = e 2Ve e j( t+0 )
(8.63)

De acuerdo con esta expresin, el fasor que representa el valor efectivo de la tensin en la fase
a del estator de la mquina sincrnica en rgimen permanente es:
vq
vd
Ve = Vd + Vq = Vd + jVq = + j
3
3

(8.64)

Con un razonamiento similar para las corrientes en rgimen permanente se obtiene la siguiente
expresin:
iq
id
Ie = Id + Iq = Id + jIq = + j
(8.65)
3
3
Reemplazando las expresiones 8.64 y 8.65 en las ecuaciones 8.58 y 8.59, se obtienen las siguientes relaciones fasorales:
Vd = Re Id + jXq Iq
(8.66)
1
Vq = Re Iq + jXd Id + j e f = Re Iq + jXd Id + E f
3

Ve = Vd + Vq = Re Id + Iq + jXd Id + jXq Iq + E f
Ve = Re Ie + jXd Id + jXq Iq + E f

(8.67)

(8.68)

En las expresiones 8.66 a 8.68, los fasores con subndice d estn orientados segn la direccin
del eje directo, y los fasores con subndice q apuntan en la direccin del eje cuadratura. El fasor
E f se orienta en la direccin del eje q debido a que representa la fuerza electromotriz producida
por la corriente del campo i f sobre el eje q. En la ecuacin 8.67 se observa que el fasor E f se
obtiene multiplicando por j15 la fuerza electromotriz e f producida por el campo y dividiendo
este resultado por el factor 13 . Todas las magnitudes de los fasores de las expresiones anteriores
se han definido en trminos de valores efectivos,
por esta razn no aparece en la definicin de

cada uno de los trminos el coeficiente 2. En la ecuacin fasorial 8.68 aparecen los trminos
jXd Id y jXq Iq , los cuales aun cuando aparentan ser cadas de tensin reactivas, en realidad representan fuerzas electromotrices de generacin. Es necesario recordar que el operador imaginario
j produce una rotacin de 90. Como el fasor Xd Id est dirigido segn el eje directo, el fasor
jXd Id se orienta segn la direccin del eje cuadratura. En otras palabras, el flujo producido por
la bobina del eje directo de la mquina, corta a los conductores fijos del estator e induce fuerza
electromotriz de generacin en el eje cuadratura. De forma semejante el trmino Xq Iq representa
un fasor con direccin cuadratura, jXq Iq rota 90 y el fasor resultante apunta en la direccin nega14
15

En el sistema trifsico balanceado se tiene: v0 = va + vb + vc = 0.


Direccin del eje cuadratura.

360

VIII.7 Diagrama fasorial

Figura 8.5 Diagrama fasorial de la mquina sincrnica de polos salientes en convencin motor

tiva del eje directo. En la figura 8.5 se representa el diagrama fasorial de la mquina sincrnica
en rgimen permanente.

Si el rotor de la mquina sincrnica es liso, las reactancias directa y cuadratura son iguales, en
este caso se define una sola reactancia denominada reactancia sincrnica Xs . Para la mquina
sincrnica de rotor liso la ecuacin fasorial 8.68 se simplifica cuando se agrupan los trminos
de generacin:
Ve = (Re + jXs ) Ie + E f
(8.69)
Las relaciones anteriores estn escritas en la convencin motor. En otras palabras, las corrientes
que circulan por las bobinas de la mquina entran por su punto de polaridad relativa. En la
convencin motor una potencia positiva indica que la mquina consume potencia elctrica. Si
la potencia es negativa, la mquina genera potencia elctrica. Las mquinas sincrnicas son
empleadas con mucha frecuencia como generadores y es ventajoso en estos casos utilizar la
convencin generador en lugar de la convencin motor para describir su comportamiento. En la
convencin generador las corrientes de armadura salen por el punto de polaridad de cada bobina.
En ambas convenciones, la direccin de referencia de la corriente de campo se define entrando
por el punto de polaridad relativa, porque este eje elctrico es pasivo y en general consume
potencia elctrica. El cambio de convencin se realiza invirtiendo el sentido de circulacin de
las corrientes de los ejes directo y cuadratura; para este fin se cambia el signo de las corrientes
Ie , Id e Iq , en las ecuaciones 8.66, 8.67 y 8.68. La fuerza electromotriz que produce el campo
no cambia de signo en la nueva convencin, debido a que la corriente de campo i f mantiene la
misma referencia en las dos convenciones. De esta forma, la ecuacin de la mquina sincrnica
de polos salientes en rgimen permanente y en convencin generador se puede expresar como:
E f = Ve + Re Ie + jXd Id + jXq Iq

(8.70)

361

Captulo VIII La mquina sincrnica

Figura 8.6 Diagrama fasorial de la mquina sincrnica de polos salientes en la convencin generador
En la figura 8.6 el tringulo ABC es semejante al tringulo DEF, por esta razn se puede
establecer la siguiente relacin:
jXq Iq
Vz
EF DF
=

=
Vz = jXq Ie
Ie
Iq
AC
AB

(8.71)

La tensin Vz , aun cuando no posee una interpretacin fsica concreta, es una herramienta muy
til en la construccin del diagrama fasorial de la mquina sincrnica de polos salientes. Cuando
se suma fasorialmente la tensin de armadura en bornes de la mquina Ve , la cada resistiva Re Ie
en el circuito de armadura y el fasor Vz , el fasor resultante est orientado en la direccin del eje
cuadratura tal como se observa en la figura 8.6. Conociendo la posicin del eje cuadratura de la
mquina, es posible proyectar la corriente Ie en sus dos componentes, Id e Iq . Conocido el fasor
Id se determina la fuerza electromotriz producida por el campo, sumando el trmino j(Xd
Xq )Id al extremo del fasor que representa la tensin Vz en el diagrama fasorial. Expresando
matemticamente el planteamiento anterior, se tiene:
AE = D = Ve + Re Ie + jXq Ie


Xq Ie cos e Re Ie sin e
= arctan
Ve + Re Ie cos e + Xq Ie sin e
D=

q
(Ve + Re Ie cos e + Xq Ie sin e )2 + (Xq Ie cos e Re Ie sin e )2
|Id | = Id = |Ie | sin(e + )

E f = AE + j(Xd Xq )Id = D + j(Xd Xq )Id


E f = D + (Xd Xq )Ie sin(e + )

(8.72)
(8.73)
(8.74)
(8.75)
(8.76)
(8.77)

Mediante las expresiones anteriores se determina el diagrama fasorial de la mquina sincrnica


de polos salientes, conocida la resistencia del estator Re , las reactancias directa Xd y cuadratura
Xq , la tensin de armadura Ve , la corriente de armadura Ie y el ngulo del factor de potencia en
el punto de operacin e .

362

VIII.8 Potencia y par elctrico


VIII.8

P OTENCIA

Y PAR ELCTRICO

Para calcular del par elctrico se pueden utilizar las expresiones 8.40 o 8.61. Sin embargo, las
variables independientes de esta ecuacin son ficticias y por esta razn es conveniente expresar
el par y la potencia elctrica mediante variables asociadas con el diagrama fasorial. Las mquinas sincrnicas tienen rendimientos muy altos, particularmente cuando son de gran potencia.
En una mquina sincrnica tpica, la potencia mecnica en el eje es prcticamente igual a la
potencia elctrica en bornes de la mquina. Empleando esta aproximacin es posible desarrollar expresiones del par y de la potencia elctrica dependientes de variables mesurables en la
prctica. Con estas condiciones se tiene:
Pm = Tm m Pe = Te e

(8.78)

La potencia elctrica se determina de la siguiente forma:


Pe (t) = va ia + vb ib + vc ic = vd id + vq iq + v0 i0

(8.79)

En rgimen permanente equilibrado, las corrientes y las tensiones en coordenadas transformadas


son independientes del tiempo. La corriente y la tensin de secuencia cero son nulas. La potencia
elctrica se calcula como:


(8.80)
Pe (t) = vd id + vq iq = 3Vd 3Id + 3Vq 3Iq = 3(Vd Id +Vq Iq )
Despreciando la cada de tensin en la resistencia Re en el diagrama fasorial representado en la
figura 8.6, se deducen las siguientes relaciones:
Ve cos + Xd Id = E f Id =
Ve sin = XqIq Iq =

E f Ve cos
Xd

(8.81)

Ve sin
Xq

(8.82)

Vd = Ve sin

(8.83)

Vq = Ve cos

(8.84)

Reemplazando las ecuaciones 8.81 a 8.84 en la expresin 8.80 se obtiene el siguiente resultado:
Pe = 3

E f Ve
Xd Xq 2
sin + 3
V sin 2
Xd
2Xd Xq e

(8.85)

El segundo trmino de la expresin anterior depende de la diferencia entre las reactancias del
eje directo y cuadratura. En otras palabras, depende de la variacin de reluctancia del circuito
magntico. El primer trmino depende de la fuerza electromotriz E f producida por la corriente
de campo. En una mquina de rotor liso, ste es el nico trmino de la potencia elctrica que
interviene en el proceso de conversin de energa. El par elctrico se calcula dividiendo la expresin 8.85 por la velocidad angular sincrnica mecnica m = pe , donde p es el nmero de pares
de polos de la mquina. El ngulo se denomina ngulo de carga de la mquina y representa la
diferencia de fase entre la fuerza electromotriz producida por el flujo del campo y la tensin de

363

Captulo VIII La mquina sincrnica

(a) Potencia activa

(b) Potencia reactiva

Figura 8.7 Potencia elctrica de la mquina sincrnica de polos salientes


armadura. El ngulo de carga define el estado o punto de operacin de la mquina, es anlogo a
la variable deslizamiento en el caso de la mquina de induccin. En la figura 8.7(a) se presenta
el grfico potencia elctrica con respecto al ngulo de carga para una mquina sincrnica tpica,
indicando las dos componentes de la potencia elctrica y la potencia elctrica total.
La potencia aparente en el estator de la mquina sincrnica se calcula de la siguiente forma:
Se = 3Ve Ie = 3(Vd + jVq )(Id jIq ) =


= 3 (Vd Id +Vq Iq ) + j(Vq Id Vd Iq ) = Pe + jQe

(8.86)

La ecuacin anterior determina la potencia activa y reactiva de la mquina sincrnica. La potencia reactiva expresada en funcin de las variables del diagrama fasorial se obtiene reemplazando
en la expresin 8.85, las relaciones 8.81 a 8.84:
Qe = 3(Vq Id Vd Iq ) = 3

E f Ve
V2
cos 3 e (Xq cos2 + Xd sin2 )
Xd
Xd Xq

(8.87)

En la figura 8.7(b) se representa la potencia reactiva en funcin del ngulo de carga para una
mquina sincrnica tpica de polos salientes.
El punto de operacin de la mquina sincrnica queda definido al conocer el valor del ngulo
de carga . En la figura 8.8 se observa que a medida que aumenta la potencia entregada por la
mquina al sistema elctrico, se incrementa el valor del ngulo de carga. Sin embargo, la caracterstica potencia elctrica en funcin del ngulo de carga tiene un valor de potencia mxima
que puede entregar la mquina. Si por el sistema mecnico se entrega una potencia mayor, no
es posible realizar la conversin de toda la potencia y el exceso o diferencia acelerar el rotor.
Si el rotor de la mquina se acelera, el ngulo de carga aumentar continuamente y la mquina perder el sincronismo con el sistema elctrico de potencia. Cuando ocurre este fenmeno
es necesario desconectar la mquina sincrnica de la red para evitar las fuertes oscilaciones de
potencia y la aceleracin de la mquina, que es capaz de alcanzar el nivel de embalamiento
del rotor.
Para determinar el ngulo de carga correspondiente a la mxima potencia que puede entregar la
mquina, se deriva la expresin 8.85 con respecto a este ngulo. En el valor max la derivada de

364

VIII.8 Potencia y par elctrico

Figura 8.8 Variacin de la potencia elctrica con el ngulo de carga y punto de mxima potencia

la potencia con respecto al ngulo de carga es nula:


Xd Xq 2
Pe E f Ve
V cos 2
cos +
=

Xd
Xd Xq e

(8.88)

E f Ve
Xd Xq 2
Pe
cos max +
V cos 2max = 0
(max ) =

Xd
Xd Xq e

(8.89)

Recordando la identidad trigonomtrica cos 2 2 cos2 1, se puede expresar la ecuacin


8.89 como una ecuacin cuadrtica:
2

E f Ve
Xd Xq 2 2
Xd Xq 2
Ve cos max +
V =0
cos max
Xd Xq
Xd
Xd Xq e

(8.90)

Simplificando la expresin anterior se puede obtener:


cos2 max +

1 Xq E f
1
cos max = 0
2 Xd Xq Ve
2

(8.91)

Cuya solucin es:


s

max = arc cos

Xq2 E 2f
16(Xd Xq )2Ve2

Xq E f
1

+
2 4(Xd Xq )Ve

(8.92)

365

Captulo VIII La mquina sincrnica


Para las mquinas sincrnicas de rotor liso, las reactancias directa y cuadratura son iguales, y en
este caso se obtiene a partir de la expresin 8.89:

max = arc cos(0) =

VIII.9

C ONVENCIONES

E f Ve

Pe max =
2
Xs

(8.93)

DE LA MQUINA SINCRNICA

En la seccin 8.7 se desarroll el diagrama fasorial de la mquina sincrnica para las convenciones motor y generador respectivamente. Estas dos convenciones se diferencian tan slo en
la referencia de circulacin de las corrientes de armadura. En la convencin generador, un valor positivo de la potencia implica flujo de energa desde el sistema mecnico hacia el sistema
elctrico de potencia. En la convencin motor, los valores positivos representan absorcin de potencia de la red y su entrega por el eje mecnico. En ambos casos la potencia se calcula mediante
la misma expresin fasorial:
S V I = V I (cos + j sin )

(8.94)

Donde:

= V I es el ngulo de atraso de la corriente con respecto a la tensin.


La expresin 8.94 define como referencia la potencia reactiva inductiva. Si la corriente est
retrasada fasorialmente con respecto a la tensin, al ser conjugada, el trmino imaginario es
positivo.16 De esta forma quedan completamente definidos, con respecto a la potencia activa
y reactiva, los cuatro cuadrantes en los que puede localizarse la corriente de armadura de la
mquina sincrnica en las dos convenciones ms utilizadas.17 En la figura 8.9(a) y (b) se resumen
los aspectos y definiciones ms resaltantes de estas dos convenciones.
En la convencin generador, un generador inductivo entrega potencia reactiva a la red y un
generador capacitivo la consume. Un motor inductivo en convencin motor consume potencia
reactiva inductiva. La convencin define el sentido de referencia del intercambio. Siempre se
utiliza el concepto de potencia reactiva inductiva, para mantener la coherencia de la definicin
8.94. En la figura 8.9 (c) y (d) se presentan los diagramas fasoriales de una mquina sincrnica
de polos salientes, en convencin generador y en convencin motor, cuando se encuentra generando hacia la red potencia activa y reactiva inductiva. En estos dos diagramas se observa que
aun cuando la corriente se invierte de sentido entre ambas convenciones, las cadas de tensin
producidas por la corriente de armadura tambin cambian y los diagramas fasoriales en las dos
convenciones reproducen los mismos resultados.
16

17

Aun cuando la necesidad de conjugar uno de los dos fasores es para determinar el ngulo entre ellos por diferencia
de sus respectivas fases, si se conjuga la corriente, la referencia es la potencia reactiva inductiva y si se conjuga
la tensin, la referencia es la potencia reactiva capacitiva.
Motor o generador inductivo.

366

VIII.9 Convenciones de la mquina sincrnica

(a) Convencin generador

(b) Convencin motor

(c) Convencin generador

(d) Convencin motor

Figura 8.9 Convenciones motor y generador inductivos de la mquina sincrnica

367

Captulo VIII La mquina sincrnica


VIII.10

VALORES

NOMINALES DE LA MQUINA SINCRNICA

Tanto los transformadores, las mquinas de corriente continua, las mquinas de induccin como
las mquinas sincrnicas, poseen un conjunto de valores nominales o datos de placa que determinan sus lmites de operacin en rgimen permanente. Los valores nominales estn definidos
por los lmites trmicos del convertidor. Si estos lmites de temperatura son excedidos durante
perodos prolongados de tiempo, se produce un rpido envejecimiento del aislamiento de la mquina y un deterioro prematuro de la misma. Los datos de placa de las mquinas sincrnicas son
los siguientes:
1. La corriente nominal de armadura:
Est determinada por las prdidas Joule de las bobinas del estator y por las caractersticas
trmicas de la mquina. Las caractersticas fsicas y qumicas de los materiales utilizados
en el aislamiento de los devanados de armadura, definen la mxima temperatura de operacin que permiten estos materiales sin que se produzca una degradacin prematura de
los mismos. El sistema de intercambio de calor juega un papel importante porque a medida que aumenta la capacidad de extraccin de calor, es posible incrementar las prdidas
sin aumentar la temperatura mxima de las bobinas. Durante la etapa de diseo, y posteriormente en el banco de pruebas, el fabricante define el valor mximo de la corriente de
armadura, que no excede la temperatura mxima del aislamiento con el sistema de refrigeracin utilizado por la mquina. Durante la operacin de la mquina este valor puede ser
sobrepasado por un cierto tiempo, aun cuando exceder la corriente nominal de armadura
durante periodos prolongados de tiempo o en rgimen permanente, reduce la vida til de
la mquina.
2. La tensin nominal de armadura:
La tensin nominal est determinada por las prdidas en el material magntico de la mquina. Las prdidas por histresis y por corrientes parsitas dependen de la densidad de
flujo mximo en el hierro. La tensin aplicada a las bobinas es aproximadamente igual
a la fuerza electromotriz en el devanado cuando se desprecian las prdidas resistivas. La
fuerza electromotriz en las bobinas es igual a la derivada con respecto al tiempo de los enlaces de flujo. A partir de la ley de Faraday, para excitaciones sinusoidales de las bobinas
de la mquina se obtiene la siguiente expresin:

Ve = Ee = 2 f Ne max = 4,44 f Ne A Bmax


(8.95)
Como en la expresin 8.95, la frecuencia f , el nmero de vueltas de la bobina Ne y el rea
A de la seccin transversal del enrollado son constantes, la densidad de flujo mximo depende directamente de la tensin de armadura. Por esta razn el fabricante define un valor
nominal de la tensin de armadura que permite utilizar una densidad de flujo cercana al
codo de saturacin del material magntico, donde las prdidas del hierro son relativamente
reducidas. En la prctica las densidades de flujo de diseo se encuentran entre 1,0 y 1,4 Wm2b
para los materiales ferromagnticos de grano no orientado utilizados habitualmente en la

368

VIII.10 Valores nominales de la mquina sincrnica

(a) Factor de potencia unitario

(b) Condensador sincrnico

Figura 8.10 Operacin de la mquina con potencia aparente nominal con cos unitario y puramente inductivo
construccin de las mquinas elctricas rotativas.
3. La potencia aparente nominal:
La potencia aparente nominal resume en un valor de mrito la corriente y tensin nominal de armadura, aun cuando no representa en s mismo un lmite trmico. En el sistema
trifsico balanceado, la potencia aparente nominal es:

(8.96)
Sn = 3Vnln Inl = 3Vnll Inl
La mquina sincrnica puede operar con potencia aparente nominal en infinitos puntos,
con diferentes ngulos de desfasaje entre la tensin y la corriente de armadura. En la figura
8.10 se representan los diagramas fasoriales de la mquina para dos factores de potencia
notables: unitario y cero inductivo. En el segundo caso se observa que es necesaria una
fuerza electromotriz mayor.
4. El factor de potencia nominal:
En la figura 8.11 se observa que para mantener un factor de potencia inductivo es necesaria
una mayor fuerza electromotriz producida por el flujo del campo de la mquina sincrnica.
La fuerza electromotriz depende directamente de la corriente i f que circula por la bobina
del campo. Si bien esta bobina maneja una pequea fraccin de la potencia aparente nominal de armadura, las prdidas resistivas del conductor producen calentamiento local en la
misma. Por esta razn es preciso imponer un valor de corriente de campo que garantice

369

Captulo VIII La mquina sincrnica

Figura 8.11 Mquina sincrnica de polos salientes en el punto nominal de operacin


el funcionamiento en rgimen permanente de la mquina sincrnica, sin envejecer prematuramente el aislamiento de esta bobina. Como la corriente de campo est limitada a un
valor mximo en rgimen permanente, tambin queda limitada la mxima fuerza electromotriz producida por la corriente de campo. A medida que se reduce el factor de potencia
de la mquina durante la operacin a potencia aparente nominal, se incrementa la fuerza
electromotriz necesaria para mantener el punto de operacin. El factor de potencia para
el cual la fuerza electromotriz producida por el campo es mxima se define como factor
de potencia nominal. El fabricante, en lugar de expresar el valor mximo de la corriente
de campo, indica en los datos de placa el valor del factor de potencia nominal, que es el
mnimo factor de potencia con el cual la mquina puede operar con tensin y corriente
nominal sin exceder la temperatura mxima de las bobinas del campo.
5. La corriente de campo nominal:
La corriente de campo nominal es aquella que produce la tensin nominal en la armadura de la mquina sincrnica cuando sta se encuentra en vaco y girando a la velocidad
nominal. Esta corriente es inferior a la corriente mxima del campo definida por el factor
de potencia nominal en las condiciones nominales de operacin.
6. La velocidad nominal:
El rotor de la mquina sincrnica gira en rgimen permanente a una velocidad mecnica exactamente igual a la velocidad angular del campo magntico rotatorio del estator.

370

VIII.11 Lugares geomtricos


Esta velocidad depende de la frecuencia de la red elctrica y del nmero de pares de polos
p de la mquina. La velocidad nominal de la mquina sincrnica es:

n = sin =

VIII.11

LUGARES

2 f
p

(8.97)

GEOMTRICOS

Las posibles condiciones de operacin de la mquina sincrnica circunscriben las diferentes


variables que definen su comportamiento, en ciertos rangos y figuras geomtricas, cuando se
representan grficamente. Un determinado punto de operacin queda definido mediante un diagrama fasorial, pero la variacin del factor de potencia genera un lugar geomtrico para la fuerza
electromotriz producida por el campo. El anlisis de estos diagramas permite evaluar las diferentes posibilidades y limitaciones en la operacin de la mquina sincrnica. Algunos diagramas
o lugares geomtricos son suministrados por el fabricante de la mquina debido a su utilidad
e importancia. En otras ocasiones es necesario construirlos a partir de la informacin disponible. Todos los lugares geomtricos analizados en esta seccin mantienen constante la tensin de
armadura.
En las mquinas sincrnicas grandes, la potencia mecnica en el eje es aproximadamente igual
a la potencia elctrica. Por esta razn, si la potencia mecnica se mantiene constante, la potencia
elctrica es independiente de la intensidad de la corriente del campo. El lugar geomtrico de
la corriente de armadura cuando la mquina opera a potencia elctrica y tensin de armadura
constante es una lnea recta paralela al eje imaginario, tal como se puede observar en la figura
8.12. Si para cada uno de los puntos del lugar geomtrico de la corriente de armadura a potencia
constante se realiza el diagrama fasorial de la mquina sincrnica de rotor liso, el extremo del
fasor que representa la fuerza electromotriz producida por el campo es una lnea recta paralela
al eje real.
En la figura 8.13 se observa que, durante la operacin a potencia elctrica constante, cuando la
proyeccin de la fuerza electromotriz producida por el campo sobre la tensin de armadura es
mayor que sta, la mquina entrega potencia reactiva inductiva a la red elctrica. Si la proyeccin
de la fuerza electromotriz sobre la tensin de armadura es menor que la tensin de armadura,
la mquina consume potencia reactiva inductiva de la red elctrica. Cuando la proyeccin de
la fuerza electromotriz producida por el campo iguala a la tensin de armadura, la mquina se
encuentra operando con factor de potencia unitario y no consume, ni produce, potencia reactiva
inductiva. En la mquina sincrnica de polos salientes la situacin es mucho ms compleja, pero
proyecciones grandes de la fuerza electromotriz sobre la tensin tienden a inyectar reactivos a
la red, y en el caso contrario, consumen reactivos del sistema elctrico de potencia.
Cuando la tensin y el mdulo de la corriente de armadura se mantienen constantes y se permite
la variacin del ngulo del factor de potencia, los lugares geomtricos de la fuerza electromotriz
que produce el campo y de la potencia aparente de la mquina sincrnica de rotor liso son crculos. El crculo de la potencia aparente est centrado en el origen de coordenadas del diagrama
fasorial, y el centro del crculo correspondiente al lugar geomtrico de la fuerza electromotriz
producida por la corriente de campo con corriente de armadura nominal, se encuentra en el extremo del fasor que representa la tensin de armadura. La fuerza electromotriz que produce el

371

Captulo VIII La mquina sincrnica

Figura 8.12 Lugar geomtrico de la fuerza electromotriz producida por el campo, a potencia
constante, para una mquina sincrnica de rotor liso

Figura 8.13 Zona inductiva y capacitiva del lugar geomtrico de la fuerza electromotriz a potencia constante

372

VIII.12 Circuito equivalente de la mquina sincrnica


campo de la mquina sincrnica se encuentra limitada por la mxima corriente de campo; este
valor se obtiene en la condicin de operacin nominal de la mquina. Por esta razn existe un
crculo que representa la fuerza electromotriz mxima que intercepta al lugar geomtrico de la
fuerza electromotriz a corriente nominal de armadura. En la figura 8.14 se representa el crculo
correspondiente a la corriente nominal de armadura, el lugar geomtrico asociado con la fuerza
electromotriz y el crculo correspondiente a la fuerza electromotriz mxima. La zona de operacin posible, en rgimen permanente, est definida por el rea de la interseccin de los lugares
geomtricos de la fuerza electromotriz mxima y de la fuerza electromotriz a corriente nominal.
Fuera de la interseccin se excede la corriente nominal de armadura, la corriente mxima de
campo o ambas. Como la fuerza electromotriz mxima est limitada por la corriente de campo, esto se traduce en una limitacin adicional sobre la corriente de armadura. En una mquina
sincrnica de rotor liso se cumple la siguiente relacin:


Ve E f max
Ef
Ve

E f = Ve + jXs Ie Ie = j j
|Ie | j
(8.98)

Xs
Xs
Xs
En ocasiones, al diagrama se le incorporan varios lmites adicionales. Por una parte los anlisis
de estabilidad del sistema de potencia definen unos lmites mximos a los ngulos de carga
de cada una de las mquinas sincrnicas de la red; esto se traduce en una limitacin sobre
las fuerzas electromotrices y corrientes de la mquina. Tambin es posible que la turbina de
accionamiento o la carga mecnica estn limitadas a la potencia del punto nominal de operacin.
Esto tambin define una limitacin sobre las fuerzas electromotrices y corrientes, tal como se
discuti anteriormente al analizar los lugares geomtricos a potencia constante.

En algunas ocasiones es necesario definir tambin la fuerza electromotriz mnima E f min , calculada como aquella que produce la potencia elctrica nominal con el ngulo de carga correspondiente a la mxima potencia que puede entregar la mquina en rgimen permanente. Esto se
representa mediante un crculo centrado en el origen de coordenadas del diagrama fasorial. En
la mquina de rotor liso, la fuerza electromotriz mnima es:
Pe = Pn =

E f min Ve

Pn Xs
sin E f min =
Xs
2
Ve

(8.99)

En la mquina sincrnica de polos salientes, los lugares geomtricos son ms complejos. La


figura 8.15 presenta el lugar geomtrico de la fuerza electromotriz de una mquina sincrnica con polos salientes, el crculo correspondiente a la potencia aparente nominal y el crculo
correspondiente a la fuerza electromotriz mxima.
El grfico de la figura 8.22 se obtiene mediante el programa MATLAB presentado en el cdigo
fuente 10.
VIII.12

C IRCUITO

EQUIVALENTE DE LA MQUINA SINCRNICA

A partir del modelo 8.54 que define el comportamiento dinmico de las corrientes de la mquina sincrnica en convencin motor, se puede modelar la mquina mediante cuatro circuitos
elctricos acoplados por trminos de generacin y transformacin, mediante transformadores y

373

Captulo VIII La mquina sincrnica

Figura 8.14 Lugares geomtricos de la corriente y de la fuerza electromotriz a potencia aparente


constante para la mquina sincrnica de rotor liso

Algoritmo 10 Lugares geomtricos de la mquina sincrnica de polos salientes


Van = 1.; Ien = 1.; San = 1.;
CosFin = 0.8;
Xd = 1.2; Xq = 0.8;
Re = 0.0;
Fi = 0:-.05:-2*pi;
Ie = Ien*( os(Fi)+j*sin(Fi));
D = Van + (Re+j*Xq).*Ie;
delta = atan2(imag(D),real(D));

% Tensin, orriente y poten ia nominal


% Fa tor de poten ia nominal
% Rea tan ia del eje dire to y uadratura
% Resisten ia de las bobinas de armadura
% Ve tor de posibles fp de opera in
% Ve tor de orrientes de armadura
% Ve tor de fasores D para ada orriente
% Ve tor de ngulos de arga para ada Ie
% Cl ulo de la orriente Id
Id = Ien*sin(delta-Fi).*(sin(delta)-j* os(delta));
Ef = D + j*(Xd-Xq).*Id;
% Cl ulo de la fuerza ele tromotriz
Fin = a os(CosFin);
% ngulo del fa tor de poten ia nominal
Iex = Ien*( os(Fin)-j*sin(Fin));
% Fasor de la orriente nominal
Dx = Van + (Re+j*Xq)*Iex;
% Fasor D para la ondi in nominal
deltax = atan (imag(Dx)/real(Dx));
% ngulo de arga nominal
% Corriente dire ta nominal
Idx = Ien*sin(deltax-Fin).*(sin(deltax)-j* os(deltax));
Efx = Dx + j*(Xd-Xq).*Idx;
% Fasor de la fuerza ele tromotriz mxima
Efmax = abs(Efx)*( os(Fi)+j*sin(Fi)); % Magnitud de la fuerza ele tromotriz mxima
axis('square');
% Defini in de ejes uadrados para el grfi o
plot (Ef),hold on;
% Lugar geomtri o de la fem (S= te)
plot (Ie);
% Lugar geomtri o de la pot. aparente onst.
plot(Efmax), hold off;
% Lugar geomtri o de la fem mxima

374

VIII.12 Circuito equivalente de la mquina sincrnica

Figura 8.15 Lugares geomtricos de una mquina sincrnica de polos salientes

375

Captulo VIII La mquina sincrnica


fuentes de tensin dependientes de corriente. En la figura 8.16 (a) se presenta el circuito equivalente de la mquina sincrnica de polos salientes. Durante la operacin equilibrada en rgimen
permanente, las corrientes id , iq e i f son constantes en el tiempo y la corriente i0 es nula. De
esta forma, el circuito desacoplado correspondiente a la secuencia cero no tiene influencia y las
inductancias del resto de los circuitos no producen cada de tensin. La corriente del campo se
puede calcular evaluando el cociente entre la tensin aplicada al campo y la resistencia de esta
bobina. Los dos circuitos restantes, correspondientes al eje directo y cuadratura, estn configurados tan slo mediante resistencias y fuentes de tensin dependientes de corrientes que circulan
por otros circuitos. Asociando con el eje real la polaridad positiva de la corriente y tensin del
circuito correspondiente al eje directo, y con el eje imaginario la del circuito cuadratura, se representa en la figura 8.16 (b) el circuito equivalente fasorial de la mquina sincrnica de polos
salientes en rgimen permanente equilibrado.
Si la mquina sincrnica es de rotor liso, las reactancias del eje directo y del eje cuadratura son
iguales y se denomina entonces reactancia sincrnica Xs . Para mquinas sincrnicas de rotor liso
de la figura 8.16 (b) y de las expresiones 8.33 y 8.34, se deduce la siguiente relacin:
Ve = Vd + jVq = (Re + jXs )(Id + jXq ) + jE f = (Re + jXs )Ie + E f

(8.100)

En la figura 8.16 (c) se presenta el circuito equivalente en convencin motor, de la mquina


sincrnica de rotor liso en rgimen permanente equilibrado. El circuito equivalente de la mquina sincrnica de rotor liso permite una interpretacin simple de las ecuaciones. Esta mquina
se comporta como una fuente equivalente de Thvenin cuya tensin de circuito abierto es la
fuerza electromotriz que produce la corriente de campo sobre la armadura y la impedancia de
Thvenin est formada por la resistencia de las bobinas de armadura y por la reactancia sincrnica. La cada reactiva modela la desmagnetizacin ocasionada por la circulacin de la corriente
de armadura.

VIII.13

C URVAS

EN V

Estas curvas estn formadas por una familia de grficos que representan la relacin entre la
magnitud de la corriente del estator Ie y la corriente de campo o corriente de excitacin i f ,
utilizando como parmetro diferentes valores de la potencia elctrica. Estos grficos se realizan
manteniendo la tensin de armadura en un valor constante, generalmente en su valor nominal.
Mediante el diagrama fasorial de la mquina sincrnica se pueden obtener directamente las
curvas en V de la mquina sincrnica.
En la figura 8.17 se ha representado una familia de curvas en V para una mquina sincrnica de
polos salientes. Es interesante destacar la linealidad de la curva cuando la potencia elctrica es
cero. En este caso todas las cadas de tensin y fuerzas electromotrices coinciden exactamente
con el eje cuadratura, obtenindose una relacin lineal entre la fuerza electromotriz y la corriente de armadura. La zona a la derecha del grfico corresponde a inyeccin de reactivos desde
la mquina hacia la red y a la izquierda de la caracterstica de factor de potencia unitario, se
consume potencia reactiva inductiva desde la red elctrica.
Las curvas en V fueron utilizadas en el pasado con la finalidad de evitar los laboriosos clculos fasoriales. Estas curvas permiten una rpida visualizacin de los lmites operativos de la

376

VIII.13 Curvas en V

(a) Modelo transitorio de polos salientes

(b) Modelo permanente de polos salientes

(c) Modelo permanente de rotor liso

Figura 8.16 Circuitos equivalentes de la mquina sincrnica en convencin motor

377

Captulo VIII La mquina sincrnica

Figura 8.17 Curvas en V para una mquina sincrnica de polos salientes


mquina. Sin embargo, actualmente no es necesario el uso de esta herramienta debido a las facilidades de clculo disponibles. El cdigo fuente 11 realizado en el entorno Scilab reproduce
el grfico de la figura 8.17. El algoritmo utiliza la rutina f solve18 incorporada en el entorno de
programacin, para determinar el valor del ngulo de carga en cada punto de operacin. Este
cdigo emplea varios mtodos empleados habitualmente en el anlisis de mquinas sincrnicas
en rgimen permanente.
VIII.14

M EDICIN

DE LAS REACTANCIAS PERMANENTES

Existen diversos mtodos para medir las reactancias de rgimen permanente de la mquina sincrnica de rotor liso y de polos salientes. Los mtodos para determinar la reactancia sincrnica
de las mquinas de rotor liso y la reactancia del eje directo de la mquina de polos salientes son
similares. En estos ensayos se acciona la mquina a la velocidad sincrnica, supliendo solamente las prdidas mecnicas. En estas condiciones la mquina entrega o consume potencia reactiva
inductiva dependiendo de la fuerza electromotriz que produce el campo. Como la corriente de
armadura se encuentra 90 retrasada o adelantada con respecto a la tensin de armadura, la cada
en la reactancia Xq es paralela a esta tensin. El eje cuadratura est ubicado en la direccin del
fasor de la tensin de armadura. La corriente del eje directo es igual a la corriente de armadura
y la cada de tensin que produce esta componente de la corriente tambin est dirigida segn el
eje cuadratura. En la figura 8.10 (b) se muestra esta situacin.
Del diagrama fasorial de la figura 8.10 (b) se deduce la siguiente expresin:
E f = Ve + Xd Ie , si cos = 0 Xd =
18

E f Ve
Ie

Las rutinas f solve de Scilab, Octave o Matlab determinan los ceros de funciones no lineales.

378

(8.101)

VIII.14 Medicin de las reactancias permanentes

Algoritmo 11 Clculo de las curvas en V de la mquina sincrnica


//Fun in de la poten ia el tri a
fun tion p = Pdelta(delta)
global 1 Ef Ven xd xq Pe;
p = Ef( 1)*Ven*sin(delta)/xd+Ven^2*(xd-xq)*sin(2*delta)/(2*xd*xq)-Pe;
endfun tion
//
global 1 Ef Ven xd xq Pe;
Ven = 1.;Ien = 1.; Sen = 1.; j= %i;
CosFin = 0.8;
xd = 0.9; xq = 0.6;
Re = 0.0;
Fin = a os(CosFin);
Iex = Ien*( os(Fin)-j*sin(Fin));
Dx = Van +(Re+j*xq)*Iex;
deltax=atan(imag(Dx)/real(Dx));

// Tensin, orriente y pot. nom.


// Fa tor de poten ia nominal
// Rea tan ias del eje d y q
// Resisten ia de armadura
// ngulo del fp nominal
// Fasor de la orriente nominal
// Fasor D para la ondi in nominal
// ngulo de arga nominal
// Corriente dir. nom.
Idx =Ien*sin(deltax-Fin).*(sin(deltax)-j* os(deltax));
Efx = Dx + j*(xd-xq).*Idx;
// Fasor de la fem mxima
Efmax=abs(Efx)*( os(Fin)+j*sin(Fin));
// Magnitud de la fem mxima
Ef=0.1:.01:2.0;
// Rango de fem
kk=0;
for Pe=0:.2:1.2;
// Rango de poten ias el tri as
kk=kk+1
for l=1:length(Ef);
delta(l)=0;
// Ini ia in del ngulo de arga
end
for 1=1:1:length(Ef);
aaa=Pe*xd/(Ef( 1)*Van);
if abs(aaa) < 1.0,
delta 1=asin(aaa);
// Estima in ini ial del ngulo de arga
delta 1=fsolve(delta 1(1),Pdelta);
// Solu in de la e ua in no lineal
// Cl ulo de la poten ia rea tiva
Q=Ef( 1)*Ven.* os(delta 1)/xd-Ven^2*(xq* os(delta 1).^2+...
...+xd*sin(delta 1).^2)/(xd*xq);
S=sqrt(Pe*Pe+Q.*Q);
// Cl ulo de la poten ia aparente
Ie( 1,kk)=S/Van;
//Cl ulo de la orriente de armadura
else
Ie( 1,kk)=0;
// Corriente de armadura ( asos no fa tibles)
end
end
end
// Grfi o de las urvas en V
axis([0 2 0 1.5);
// Defini in de los ejes del grfi o
plot(Ef,Ie(:,1),Ef,Ie(:,2),Ef,Ie(:,3),Ef,Ie(:,4),...
...
Ef,Ie(:,5),Ef,Ie(:,6),Ef,Ie(:,7),);
plot(abs(Efmax),0.0,'x')
// Fuerza ele tromotriz mxima

379

Captulo VIII La mquina sincrnica


Una alternativa consiste en anular la fuerza electromotriz producida por el campo reduciendo
paulatinamente la corriente de campo a cero; en ese momento, a partir de la expresin 8.101 se
obtiene la reactancia del eje directo de la mquina como el cociente entre la tensin y la corriente
de armadura. Otro de los mtodos se fundamenta tambin en la expresin 8.101, pero determina
la fuerza electromotriz E f desconectando la armadura y midiendo la tensin en bornes, la cual
coincide exactamente con la fuerza electromotriz de la mquina en vaco. Este mtodo se puede
simplificar an ms si la mquina se encuentra previamente en cortocircuito girando a velocidad
sincrnica, con una corriente de campo tal que fuerce la circulacin de la corriente nominal por
la armadura. Si en este preciso instante se desconecta la armadura de la mquina, la tensin en
bornes es igual a la fuerza electromotriz interna durante el cortocircuito y el cociente con la
corriente nominal de armadura determina la reactancia del eje directo de la mquina.
La discusin anterior es vlida tambin para las mquinas sincrnicas de rotor liso. La reactancia del eje directo es igual a la reactancia del eje cuadratura, y si se aplican los ensayos
utilizados para la determinacin de la reactancia del eje directo, se puede obtener con el mismo
procedimiento el valor de la reactancia sincrnica de una mquina de rotor liso.
La reactancia del eje cuadratura se puede determinar a partir de la medicin de un punto de
operacin determinado, si se conocen todas las variables y parmetros de la mquina. Conocida
la potencia, tensin de armadura, fuerza electromotriz, ngulo de carga y reactancia del eje
directo, es posible obtener el valor de la reactancia cuadratura de la mquina a partir de la
expresin 8.85:
Xd
Xq = 2(X P E V sin )
(8.102)
f a
d e
+
1
V 2 sin 2
e

Otro mtodo que permite determinar la reactancia del eje cuadratura de la mquina consiste en
medir la variacin de reluctancia cuando se gira el rotor 90 elctricos. La reluctancia mnima
corresponde a la reactancia del eje directo y la reluctancia mxima al eje cuadratura. Como las
dos bobinas poseen el mismo nmero de vueltas, se puede determinar la reactancia cuadratura a
partir de esta informacin y del valor de la reactancia del eje directo.
Las reactancias del eje directo y cuadratura de la mquina sincrnica representan trminos de
generacin, estn asociadas con los flujos de la armadura de la mquina y por tanto se asemejan
ms a reactancias de magnetizacin que a reactancias de dispersin de una bobina. En la prctica
las reactancias sincrnicas de las mquinas de rotor liso y las reactancias del eje directo de las
mquinas de polos salientes se encuentran en un rango que oscila entre 0, 8 y 1, 2 pu de las bases
propias del convertidor. Las reactancias del eje cuadratura estn comprendidas normalmente
entre 0, 5 y 0, 7 pu aproximadamente. En algunas mquinas especiales, tal como es el caso del los
motores sincrnicos de reluctancia, la reactancia del eje cuadratura es mayor que la reactancia
del eje directo.
VIII.15

A NLISIS

DE LA MQUINA SINCRNICA CONSIDERANDO LA

SATURACIN

Cuando el material magntico de la mquina se satura, la relacin entre la corriente del campo
i f y el enlace de flujo que esta corriente produce f , no es lineal. La caracterstica de la fuerza
electromotriz E f producida por el flujo del campo tampoco es lineal con respecto a la corriente

380

VIII.15 Anlisis de la mquina sincrnica considerando la saturacin


de campo. La funcin E f (i f ) se conoce como caracterstica de excitacin en vaco e indica
la variacin de las permeanzas del material magntico con el nivel de fuerza magnetomotriz
aplicada al campo de la mquina. Las mquinas sincrnicas operan normalmente en puntos
ubicados por encima del codo de saturacin y es necesario analizar su comportamiento en estas
condiciones. El nivel de saturacin de una mquina afecta las permeanzas del circuito magntico
y por tanto sus correspondientes reactancias. La caracterstica de magnetizacin o curva de vaco
suministra la informacin necesaria para corregir las reactancias de la mquina en cada punto
de operacin.
Las reactancias de la mquina sincrnica representan trminos de generacin, pero asociada
en serie con cada una existe una pequea inductancia de dispersin de la bobina fsica. Las
reactancias de dispersin estn definidas por caminos magnticos que se cierran en el aire y
por esta razn no estn sometidas a procesos de saturacin. Por este motivo, es conveniente
separar en dos partes cada una de las reactancias de la mquina sincrnica, una que representa el
camino de fuga y la otra que representa el camino principal o de magnetizacin. La reactancia
de fuga no sufre los efectos de la saturacin del material ferromagntico, pero la reactancia de
magnetizacin s es afectada y por ello es conveniente expresar las reactancias de la mquina
sincrnica de polos salientes de la siguiente forma:
Xd = Xmd + X d
Xq = Xmq + X q

(8.103)

Separando las reactancias de fuga y magnetizacin indicadas en la expresin 8.103, se puede


definir una nueva fuerza electromotriz detrs de las reactancias de fuga de la mquina que indican el nivel de saturacin al que se encuentran sometidas las reactancias de magnetizacin de la
mquina. Asumiendo que las reactancias de fuga de los dos ejes son prcticamente iguales, la
fuerza electromotriz Ee se evala mediante la expresin siguiente:
Ee = Ve + jX Ie

(8.104)

En la figura 8.18 se presenta una caracterstica de magnetizacin o curva en vaco tpica, donde
se han indicado las asntotas que permiten linealizar.
Para analizar la saturacin de la mquina sincrnica es necesario descomponer las fuerzas magnetomotrices producidas por cada eje. La fuerza magnetomotriz de armadura est orientada segn la direccin de la corriente de armadura y se puede descomponer en dos componentes, una
en la direccin del eje directo asociada con la corriente del eje directo y otra segn la direccin
del eje cuadratura asociada con la corriente del eje cuadratura. La fuerza electromotriz del campo est dirigida segn el eje cuadratura y debe ser producida por una fuerza magnetomotriz que
est adelantada 90 con respecto a ella. De esta forma el diagrama de fuerzas magnetomotrices
de la mquina sincrnica es similar al que se observa en la figura 8.19.
Las fuerzas magnetomotrices orientadas en la direccin del eje cuadratura actan sobre reluctancias grandes, debido a que en esta zona el entrehierro de la mquina es considerable. Por este
motivo, las reactancias definidas por los flujos producidos por estas fuerzas magnetomotrices
no estn afectados por los fenmenos de saturacin. En el eje directo la situacin es diferente,
los flujos del eje directo no son proporcionales a las fuerzas magnetomotrices que los producen si no que dependen del nivel o grado de saturacin alcanzado por la mquina en su punto

381

Captulo VIII La mquina sincrnica

Figura 8.18 Caracterstica de magnetizacin en vaco de la mquina sincrnica

382

VIII.15 Anlisis de la mquina sincrnica considerando la saturacin

Figura 8.19 Diagrama fasorial de fuerzas magnetomotrices y enlaces de flujo de la mquina


sincrnica
de operacin. En la figura 8.19 se presenta un diagrama fasorial donde se muestran las fuerzas
electromotrices y flujos en los diferentes ejes de la mquina sincrnica de polos salientes.
Las fuerzas magnetomotrices de los ejes directo y cuadratura no se pueden sumar porque estn
aplicadas sobre caminos magnticos diferentes, con reluctancias diferentes. Una pequea fuerza magnetomotriz resultante sobre el eje directo produce un flujo de gran magnitud debido a
que este eje posee una gran permeanza. Por el contrario, la fuerza magnetomotriz segn el eje
cuadratura produce un dbil flujo en esta direccin debido a la gran reluctancia de este eje. La
superposicin de los flujos resultantes en cada uno de los ejes coordenados produce el flujo resultante total T ilustrado en la figura 8.19. Este flujo induce la fuerza electromotriz Ee que est
retrasada 90 con respecto a este fasor. De las dos componentes del flujo, solamente la componente resultante del eje directo d se ve afectada por la saturacin del material magntico. Esta
componente del flujo produce la fuerza electromotriz Eeq , retrasada tambin 90 con respecto
al flujo.
La fuerza magnetomotriz Eeq es proporcional al flujo resultante del eje directo d y este valor
puede ser utilizado como indicador del grado de saturacin del eje directo de la mquina. Las
permeanzas de los caminos magnticos del eje directo y del campo son iguales, por lo tanto es
posible utilizar la caracterstica de magnetizacin para corregir la reactancia del eje directo de
la mquina sincrnica en un punto de operacin determinado. La reactancia del eje cuadratura

383

Captulo VIII La mquina sincrnica


no se satura porque su permeanza es muy reducida. Este hecho permite identificar la posicin
del eje cuadratura y determinar de esta forma la magnitud del fasor Eeq . Si esta componente de
la fuerza electromotriz fuese generada mediante el flujo producido por la corriente de campo,
se necesitara un determinado valor de corriente de campo en la mquina no saturada y una
magnitud mayor en la mquina saturada. La proporcin entre estas dos corrientes es dependiente directamente de la variacin de permeanza existente entre el caso lineal y el saturado. La
reactancia de magnetizacin del eje directo se puede expresar como:
Xmd = Ld = Ne2md = Ne2d f

(8.105)

La fuerza electromotriz producida en la armadura por la bobina de campo es:


1
1
E f = Ld f i f = Ne N f d f i f
3
3

(8.106)

En las dos expresiones anteriores se observa que las permeanzas de los caminos magnticos
son iguales, por esta razn se utiliza la caracterstica de magnetizacin o curva de vaco para
evaluar la permeanza del camino magntico. En la figura 8.19 se observa que para producir una
determina fuerza electromotriz Eq es necesaria la corriente i f 0 para una mquina lineal e i f sat
para la mquina saturada. Mediante la expresin 8.106 se deduce la siguiente relacin entre la
permeanza saturada y lineal de la mquina para el camino magntico que une al campo con la
bobina del eje directo:
1
1
Eeq = Ne N f d f lin i f 0 = Ne N f d f sat i f sat
3
3

(8.107)

De la expresin 8.107 se puede deducir el siguiente resultado:

d f lin i fsat
=s
=
d f sat
if0

(8.108)

Donde:
s

se define como grado de saturacin de la mquina sincrnica para el


punto de operacin dado.

Reemplazando la expresin obtenida en 8.104 en la 8.101 se obtiene la reactancia de magnetizacin saturada del eje directo:
Xmd sat = Ne2d f sat = Ne2

d f lin Xmd lin


=
s
s

(8.109)

La expresin anterior indica que la reactancia de magnetizacin saturada Xmd sat es menor que
la reactancia lineal Xmd lin , debido a que el grado de saturacin s es siempre mayor o igual a la
unidad segn la definicin 8.108. La reactancia saturada total del eje directo se calcula aadiendo
a la reactancia saturada de magnetizacin del eje directo, la reactancia de fuga de la mquina.
Esta ltima reactancia es independiente del grado de saturacin de la mquina:
Xd sat = Xmdsat + X =

384

Xmd lin
Xd lin X
1
s1
+ X =
+ X = Xd lin +
X
s
s
s
s

(8.110)

VIII.15 Anlisis de la mquina sincrnica considerando la saturacin


Conocidas las reactancias de la mquina en un punto de operacin determinado, se puede obtener
el diagrama fasorial completo. Conocida la magnitud de la fuerza electromotriz producida por
el campo, es posible calcular la corriente de campo necesaria a partir de la expresin 8.106:

d f lin
1
1
1 Ld f lin
E f sat = Ne N f d f sat i f = Ne N f
if =
if
s
s
3
3
3
Por lo tanto:

if0
Eeq
1
E f sat = Ld f lin
if =
if
i f sat
i f sat
3

(8.111)

(8.112)

La ecuacin 8.112 indica que para el punto de operacin dado, la caracterstica que determina la
fuerza electromotriz producida por el campo en la condicin de carga es una recta que pasa por
el origen y por el punto de interseccin entre la fuerza electromotriz Eeq y la corriente i f sat , tal
como se ilustra en la figura 8.18.
Si se conoce la magnitud de la tensin y corriente de armadura, el ngulo del factor de potencia,
las reactancias lineales de la mquina y la caracterstica de magnetizacin en vaco, se puede
obtener el diagrama fasorial completo y la corriente de magnetizacin necesaria para mantener
el punto de operacin. En primer lugar se procede a ubicar la posicin del eje cuadratura, multiplicando la corriente de armadura por la reactancia cuadratura y sumando este trmino a la
tensin de armadura.
El segundo paso consiste en calcular el fasor Ee sumando a la tensin de armadura la cada en
la reactancia de fuga. Posteriormente se proyecta ortogonalmente este fasor con respecto al eje
cuadratura para obtener la magnitud de Eeq . Con esta fuerza electromotriz se entra en la curva de
vaco y se calcula el grado de saturacin correspondiente al punto de operacin de la mquina. El
grado de saturacin s, determinado mediante la caracterstica de vaco, se utiliza para corregir la
reactancia del eje directo mediante la expresin 8.110, y con esta reactancia se calcula la fuerza
electromotriz producida por el campo.
Para determinar la corriente de campo necesaria para mantener el punto de operacin se puede
utilizar la ecuacin 8.112. Si la mquina cambia su punto de operacin, hay que recalcular el
nuevo grado de saturacin y evaluar la reactancia del eje directo y la corriente del campo. Las
figuras 8.18 y 8.19 muestran el procedimiento descrito anteriormente.
El anlisis de la saturacin de la mquina sincrnica de rotor liso presenta un inconveniente
prctico. En la mquina de polos salientes, la reactancia del eje cuadratura no se satura debido
a que en este eje la reluctancia es muy grande. En los rotores lisos, este hecho es diferente y
la reactancia de eje cuadratura tambin se satura. Si esta reactancia est saturada y el grado de
saturacin depende del punto de operacin de la mquina, no es posible evaluar directamente el
valor de esta reactancia sin determinar previamente la posicin del eje cuadratura. Como esta
posicin est indeterminada, se utiliza un proceso iterativo para localizar el eje. En principio se
puede considerar que la mquina no est saturada en el eje cuadratura y calcular con esta aproximacin la proyeccin de la fuerza electromotriz Ee sobre el eje directo. Con esta proyeccin se
determina el grado de saturacin del eje cuadratura mediante el procedimiento descrito anteriormente para el eje directo. Esta primera aproximacin al grado de saturacin se usa para corregir
la reactancia del eje cuadratura y recalcular la posicin de este eje. Con la nueva posicin se
repiten todos los pasos anteriores hasta que el grado de saturacin en la iteracin anterior y la
actual converjan en un valor de error inferior al que se ha especificado previamente. A partir de

385

Captulo VIII La mquina sincrnica

Figura 8.20 Caracterstica de saturacin en carga (tringulo de Potier)


este punto, el diagrama fasorial se determina de igual forma que en el caso de la mquina sincrnica de polos salientes, debido a que la posicin de los ejes de la mquina queda completamente
definida. En presencia de saturacin, las reactancias del eje directo y cuadratura de la mquina
sincrnica de rotor liso son diferentes.
En el pasado se empleaba con frecuencia la curva de saturacin en carga para analizar los fenmenos de saturacin de la mquina sincrnica. Esta caracterstica representa la tensin en
bornes de la mquina en funcin de la corriente de campo y se determina para las condiciones
de mayor cada de tensin en las reactancias, es decir con la corriente de armadura nominal y
carga puramente inductiva. En este caso las cadas en las reactancias estn en fase con la tensin
de armadura. La caracterstica de saturacin en carga se obtiene a partir de la caracterstica de
excitacin en vaco, restando la cada en la reactancia de dispersin e incrementando la corriente
de campo lo suficiente para compensar la desmagnetizacin ocasionada por la corriente de armadura. Manteniendo constantes la corriente de armadura y el factor de potencia, se mantienen
constantes la cada en la reactancia de dispersin y la fuerza electromotriz necesaria para magnetizar la mquina. De esta forma se obtiene el tringulo de Potier, que determina la caracterstica
de saturacin en carga a partir de la caracterstica de excitacin en vaco. En la figura 8.20 se
muestra este diagrama y el tringulo de Potier correspondiente.

VIII.16

LA

MQUINA SINCRNICA EN EL SISTEMA ELCTRICO

La mquina sincrnica se puede utilizar como generador, tanto para alimentar cargas aisladas
como para entregar potencia a un sistema elctrico de potencia. Para incrementar la cantidad de
potencia es necesario aumentar el flujo de vapor, agua o gas que est circulando por la turbina
de accionamiento. Al aumentar la potencia de accionamiento de un generador que alimenta a
una carga aislada, las masas rotantes del sistema se aceleran, con lo cual crecen tanto la frecuencia como la fuerza electromotriz. Estas nuevas condiciones de operacin deben ser corregidas
mediante un controlador de velocidad y tensin que mantenga dentro de los lmites tolerables a

386

VIII.16 La mquina sincrnica en el sistema elctrico

Figura 8.21 Generador sincrnico alimentando una carga aislada


estas variables. Este controlador debe realizar sus funciones ante otros tipos de perturbaciones
como pueden ser variaciones de la carga elctrica del sistema. El controlador de la mquina
sincrnica debe actuar sobre el sistema de accionamiento mecnico para regular la velocidad y
sobre la excitatriz o campo para regular la tensin en bornes ante variaciones de la carga. Con estas dos acciones de control es posible garantizar un suministro de tensin y frecuencia constante
a una carga aislada, independientemente del nivel de demanda de potencia elctrica al que est
siendo sometido el generador. En la figura 8.21 se ha representado el diagrama de un generador
sincrnico que alimenta a una carga aislada y es controlado en velocidad y tensin.
Cuando la mquina sincrnica inyecta potencia a un sistema elctrico de potencia, no es indispensable regular la tensin y velocidad porque estas funciones son realizadas por el acoplamiento mquina-sistema. Al aumentar el caudal de fluido energtico que circula por la mquina motriz, se incrementa el par de accionamiento y la potencia mecnica inyectada por el eje mecnico.
La velocidad mecnica permanece prcticamente constante durante este proceso, incrementndose la potencia elctrica entregada a la red para mantener la condicin de equilibrio existente.
Aun cuando la velocidad permanece prcticamente constante, el ngulo de carga aumenta para
permitir una mayor entrega de potencia al sistema elctrico. Esta situacin se puede mantener
mientras que la mquina no alcance el punto crtico correspondiente a la potencia elctrica mxima. Si por el eje mecnico del rotor se entrega an ms potencia que la mxima potencia
elctrica que puede generar la mquina sincrnica, el ngulo de carga contina incrementndose
por encima del valor crtico, esto ocasiona una reduccin de la potencia elctrica generada por la
mquina y se produce una aceleracin mantenida del rotor que hace perder el sincronismo con
la red.
Durante el funcionamiento sincronizado de la mquina con el sistema, se puede controlar el nivel
de potencia reactiva entregada o consumida por la mquina sincrnica ajustando su corriente de
campo. Tambin es posible controlar mediante la corriente de campo el nivel de tensin en
bornes de la mquina. Cuando se controla el nivel de reactivos de la mquina sincrnica, se
dispone de una barra PQ en lo que se refiere al flujo de carga del sistema. Si se controla la
tensin en bornes del generador, el comportamiento de la barra es del tipo PV . En ambos casos
el sistema de control de la planta mantiene la potencia dentro de unos lmites cercanos a una
referencia y el controlador de la excitacin mantiene los reactivos o la tensin de referencia.

387

Captulo VIII La mquina sincrnica

Figura 8.22 Operacin de la mquina sincrnica acoplada al sistema elctrico de potencia


Una de las mquinas del sistema elctrico de potencia se utiliza para controlar la velocidad
sincrnica, se realimentan en frecuencia y se define una referencia de frecuencia y tensin. En
los estudios de flujo de carga, a esta mquina se le denomina barra de referencia o barra slack.
En la figura 8.22 se presenta un diagrama simplificado que muestra el comportamiento de la
mquina sincrnica en equilibrio con un sistema elctrico de potencia.
Para conectar una mquina sincrnica a la red, se hace girar el rotor a la velocidad sincrnica
mediante la mquina motriz. Como la mquina se encuentra desconectada de la red, es necesario
menos del 1 % de la potencia nominal mecnica para suplir las prdidas durante la operacin en
vaco. El siguiente paso consiste en alimentar la excitatriz de la mquina y ajustar el nivel de
tensin de armadura al nivel de tensin de la red elctrica. Una vez verificado que las tensiones
de la mquina y del sistema poseen la misma magnitud, fase y secuencia, se cierra el interruptor
que conecta la mquina con el sistema. Si la sincronizacin ha sido realizada exitosamente, la
corriente que circula por la conexin es prcticamente despreciable. Si alguna de las condiciones
necesarias para una sincronizacin adecuada no es satisfecha, las corrientes de armadura pueden
ser tan grandes como para activar las protecciones de la planta de generacin. La deteccin de
las condiciones de sincronizacin se realizan mediante un equipo denominado sincronizador. El
sincronizador mide las tensiones, frecuencias y diferencias de fase entre la mquina y la red.
Los sincronizadores modernos utilizan microprocesadores para realizar las operaciones de supervisin y mando de los controladores que ajustan la velocidad de la mquina, la tensin de
armadura y la fase relativa. Una vez que el sincronizador automtico detecta las condiciones favorables, enva una orden de cierre al interruptor principal, logrando de esta forma una conexin
rpida y segura de la mquina sincrnica a la red elctrica de potencia.
Una vez que la mquina ha sido sincronizada al sistema elctrico, es suficiente con aumentar
la potencia mecnica de accionamiento para entregar potencia elctrica a la red. Incrementando
la corriente de campo se aumenta el nivel de reactivos entregados por la mquina. Como las
corrientes de armadura de la mquina consumen potencia reactiva inductiva al circular por sus
reactancias, es preciso incrementar la corriente del campo si se desea entregar potencia reactiva
neta desde la mquina hacia la red. Si posteriormente a la sincronizacin aumenta la potencia de
accionamiento, la mquina entregar a la red una potencia elctrica equivalente. Sin embargo, si
la corriente de excitacin no se incrementa, el aumento de la corriente de armadura para permitir
la inyeccin de potencia activa a la red, repercutir en un consumo de reactivos desde la red
hacia la mquina. En ciertos casos esta situacin es conveniente, debido al exceso de potencia

388

VIII.17 Sumario
reactiva que producen algunas lneas largas de alta tensin. En cualquier caso la excitacin
permite controlar esta inyeccin.
Una de las ventajas que se obtiene cuando se accionan cargas mecnicas grandes mediante
motores sincrnicos aparte del elevado rendimiento que es posible obtener consiste en la
posibilidad de controlar el consumo de reactivos y los niveles de tensin en las barras de la
planta industrial. En la prctica esto puede reducir considerablemente la facturacin de energa
y los cargos por bajo factor de potencia.
VIII.17

SUMARIO

1. Las mquinas sincrnicas son los convertidores electromecnicos ms utilizadas en las


centrales de generacin, debido al gran rendimiento que pueden alcanzar y a la posibilidad de controlar el flujo de reactivos. En algunas aplicaciones industriales pueden ser
utilizadas como motores y con el desarrollo de la electrnica de potencia estn reemplazando a la mquina de continua, al ser comercializadas como mquinas de corriente
continua sin escobillas.
2. Los grandes alternadores de las plantas trmicas suelen emplear mquinas sincrnicas de
rotor liso debido a los altos requerimientos de velocidad de estas centrales. Las plantas
de generacin hidrulicas utilizan generalmente mquinas sincrnicas de polos salientes,
las cuales permiten incrementar la conversin de potencia por el efecto de la reluctancia
variable, es decir reducir la velocidad por el incremento del nmero de polos.
3. El modelo de la mquina sincrnica en coordenadas primitivas depende de la posicin
angular del rotor. La transformacin a vectores espaciales permite reducir las tres ecuaciones correspondientes a las bobinas del estator a una sola ecuacin cuyas variables son
complejas. La transformacin de estas variables espaciales a la referencia rotrica elimina la dependencia del ngulo del modelo y permite que las variables espaciales en
rgimen permanente sean constantes. La transformacin de Park reproduce el modelo en
vectores espaciales referidos al rotor de la mquina sincrnica e introduce un eje adicional
ortogonal que permite analizar los posibles desequilibrios.
4. En rgimen permanente las ecuaciones de la mquina sincrnica pueden ser representadas
mediante el diagrama fasorial o con un circuito equivalente. El diagrama fasorial es una
herramienta muy til porque permite analizar cuantitativamente y con gran simplicidad
el comportamiento de estos convertidores electromecnicos en condiciones de equilibrio
dinmico. Cuando se conocen la tensin, la corriente, el factor de potencia y los parmetros de la mquina, con este diagrama se puede determinar el ngulo de carga, la fuerza
electromotriz del campo y la corriente de campo.
5. Cuando se especifica el flujo de potencias activas o reactivas de la mquina sincrnica es
necesario emplear las expresiones de par o potencia elctrica en funcin de los parmetros, del ngulo de carga, de la fuerza electromotriz del campo y la tensin del sistema
elctrico de potencia para determinar la condicin de operacin. Estas expresiones son
una herramienta muy importante para analizar el comportamiento de la mquina cuando
se conoce la tensin de la red, la corriente de campo, el flujo de potencia y los parmetros.

389

Captulo VIII La mquina sincrnica


6. Las ecuaciones de la mquina sincrnica se pueden expresar tanto en convencin motor
como generador; en ambos casos es habitual utilizar la convencin de potencia reactiva
inductiva. La diferencia de ambas convenciones reside en la direccin seleccionada de
las corrientes del estator. En el caso del motor se supone que las corrientes entran desde
el sistema de potencia hacia la mquina, mientras que en la convencin generador las
corrientes de la armadura se dirigen desde la mquina hacia la red. En ambas convenciones
se considera que la corriente del campo entra hacia el punto de polaridad de la bobina.
7. La mquina sincrnica define como punto nominal varias magnitudes que determinan
un punto de operacin en rgimen permanente que elevan la temperatura hasta el punto
donde el fabricante puede garantizar el cumplimiento de la vida media del equipo. Tensin, corriente, potencia, velocidad, frecuencia y factor de potencia nominal son los datos
principales. Los puntos nominales establecen lmites trmicos y operativos que pueden
ser representados como lugares geomtricos que definen zonas donde la operacin de la
mquina es factible.
8. La caracterstica de vaco de la mquina sincrnica permite representar el efecto de la
saturacin del circuito magntico. Para un punto de operacin determinado se obtiene el
grado de saturacin comparando la corriente de campo requerida para mantener el enlace
de flujo en condiciones de linealidad y de saturacin. Con el grado de saturacin obtenido
se corrigen las reactancias del eje directo y se prosigue con el anlisis utilizando bien sea
el diagrama fasorial o las ecuaciones de potencia.
9. Los controles de potencia activa y reactiva que son posibles con las mquinas sincrnicas
definen tres tipos de barras diferentes en los estudios de flujo de carga: PQ, PV y slack.
Cuando la turbina controla la potencia activa y la corriente de campo la potencia reactiva,
la barra se define como PQ. Si en lugar de controlar el flujo de reactivos se controla la
tensin de la barra se tiene una PV y cuando se entrega la potencia necesaria para mantener
la frecuencia de la red estamos en presencia de una barra slack. La seleccin apropiada de
estas barras en los puntos de generacin determinan el flujo de potencia activa y reactiva
por la red.
VIII.18

EJEMPLOS

RESUELTOS

Ejemplo 1: Anlisis de la mquina sin saturacin


Una mquina sincrnica de polos salientes posee los siguientes datos de placa:
Sn
Vn
10 kVA 230V

cos n
0,7

ifn
5A

i f max
f
8,33 A 60 Hz

Como condensador sincrnico la mquina entrega a la red 8.319 kVAR cuando es excitada con
la corriente de campo mxima. Determine:
1. Las reactancias Xd y Xq de la mquina.
2. Los reactivos que aporta la mquina al sistema cuando la mquina inyecta a la red 5 kW ,
con las corrientes de campo nominal y mxima.

390

VIII.18 Ejemplos resueltos


3. La corriente de campo necesaria para mantener a 240 V, la potencia activa nominal como
motor, con el factor de potencia nominal y con factor de potencia unitario.
Solucin:
1. Las reactancias Xd y Xq de la mquina:
La reactancia del eje directo Xd se puede determinar directamente a partir de los datos
de operacin como condensador sincrnico:
i

f max
E f max Ven
1,666 1
i f n Ven
=
Xd =
=
= 0,8 pu
0 jQcond
Ie
0,8319
Ven

La determinacin de la reactancia cuadratura se puede obtener mediante la operacin en


el punto nominal. Debido a la naturaleza no lineal de las ecuaciones involucradas es ms
simple realizar una tabla donde se calcule el valor de la fuerza electromotriz del campo
E f en funcin del posible valor de esta reactancia. El resultado buscado se obtiene cuando
para un determinado valor de Xq coincide E f con E f max para las condiciones correspondientes al punto nominal. El algoritmo propuesto requiere los siguientes clculos:
p
2 P2
Pen Qen Pen Sen
en
Ien =
=
= 1 45,57
Ven
Ven
Dn = Ven + jXq Ien
sin n =

m(Dn )
n = arcsin(sin n )
|Dn |

cos n = 0,7 n = arc cos(cos n ) = 45,57


Id = |Ien | sin(n n )

E f = |Dn | + (Xd Xq ) Id
En la siguiente tabla se presentan las fuerzas electromotrices obtenidas para un barrido de
la reactancia cuadratura entre 0,4 y 0,8 pu:
Xq
Ef

0,4
1,655

0,5
1,662

0,6
1,666

0,7
1,668

0,8
1,669

En la tabla anterior se observa que para un valor de la reactancia Xq = 0,6 pu, la fuerza electromotriz del campo coincide con la generada por la corriente de campo mxima
i f max = 1,666 pu correspondiente al valor esperado para la operacin en el punto nominal.
2. Los reactivos que aporta la mquina al sistema cuando la mquina inyecta a la red 5 kW ,
con las corrientes de campo nominal y mxima:

391

Captulo VIII La mquina sincrnica

Para calcular estos reactivos se puede utilizar la expresin 8.85:19


Pe =

E f Ve
V2 1
1
sin + e ( ) sin 2
Xd
2 Xq Xd

Cuando por el campo circula la corriente nominal i f n :


0,5 = 1,25 sin + 0,2083 sin 2 (i f n ) = 0,3085 rad = 17,67
Si el campo est excitado con la corriente mxima i f max :
0,5 = 2,0825 sin + 0,2083 sin 2 (i f max ) = 0,2022 rad = 11,58
Con los ngulos de carga se pueden obtener las potencias reactivas correspondientes utilizando la expresin 8.87:
Qe =

E f Ve
V2
cos e (Xq cos2 + Xd sin2 )
Xd
Xd Xq

Qe (i f n) = 1,25 cos 17,67 2,0818 (0,6 cos2 17,67 + 0, 8 sin2 17,67) = 0, 0974 pu

Qe (i f max ) = 2,0825 cos 11,58 2,0818 (0,6 cos2 11,58 + 0,8 sin2 11,58) = 0, 7727 pu
La comprobacin de estas soluciones puede obtenerse realizando los correspondientes
diagramas fasoriales, tal como se muestra en las figuras siguientes:

3. La corriente de campo necesaria para mantener a 240 V, la potencia activa nominal como
motor, con factor de potencia nominal y con factor de potencia unitario:
19

En esta expresin el trmino 3 desaparece cuando se utiliza una base trifsica de potencia en el sistema adimensional de unidades.

392

VIII.18 Ejemplos resueltos


En este caso, el diagrama fasorial es la herramienta ms apropiada para determinar las
corrientes de campo adecuadas para mantener los puntos de operacin sealados. La tensin que es necesario mantener en la barra es 1,0435 pu y las corrientes necesarias para
mantener la potencia activa nominal con factor de potencia nominal y unitario son:
Ie (Pn , cos n ) =

Pn jPn tan n 0,7 j0,71


=
= 0,9583 134,43 pu
Ve
1,0435

Ie (Pn , cos = 1) =

Pn j0 0,7 j0
=
= 0,6708180 pu
Ve
1,0435

Para determinar las corrientes de campo hay que calcular las fuerzas electromotrices:
D(cos n ) = Ve + jXq Ie (Pn, cos n ) = 1,4106 j0,4025 pu = 1,4669 15,93
D(cos = 1) = Ve + jXq Ie (Pn, cos = 1) = 1,0000 j0,4025 pu = 1,0780 21,92
Las corrientes Id para cada una de estas condiciones de operacin son:
Id (cos n ) = |Ie (Pn, cos n )| sin( n ) = 0,9583 sin(15,93 + 134,43) = 0,8422 pu
Id (cos = 1) = |Ie (Pn , cos = 1)| sin( ) = 0,6708 sin(21,09 + 180) = 0,2414 pu
Las fuerzas electromotrices son:
E f (cos n ) = |D(cos n )| + (Xd Xq ) Id (cos n ) = 1,6353 pu
E f (cos = 1) = |D(cos = 1)| + (Xd Xq ) Id (cos = 1) = 1,1263 pu
Las corrientes de campo requeridas para mantener los puntos de operacin solicitados son:
i f (Pn , cos n ) = 1,6353 pu = 8,1765 A
i f (Pn , cos = 1) = 1,1263 pu = 5,6314 A
En las figuras siguientes se presentan los diagramas fasoriales de los dos puntos de operacin calculados:

393

Captulo VIII La mquina sincrnica

Ejemplo 2: Anlisis de la mquina con saturacin


De una mquina sincrnica de polos salientes de 100 MVA, 10 kV, 60 Hz,6 pares de polos, i f n =
200 A, cos n = 0,85, se han determinado que las reactancias lineales de eje directo y cuadratura
son 1,0 pu y 0,7 pu respectivamente y la reactancia de dispersin es 0,2 pu. La fuerza electromotriz en vaco es lineal hasta el valor de la corriente de campo nominal, y la pendiente de esta
funcin decae a la cuarta parte para corrientes superiores al valor nominal. Determine:
1. La corriente de campo mxima y el grado de saturacin del punto nominal.
2. El factor de potencia y el ngulo de carga cuando la mquina consume de la red 50 MW
con la corriente mxima en el campo.
Solucin:
1. La corriente de campo mxima y el grado de saturacin del punto nominal:
En la siguiente figura se muestra el grfico de la caracterstica de vaco correspondiente a la mquina en cuestin:

394

VIII.18 Ejemplos resueltos

De acuerdo con los datos del problema la caracterstica de vaco en por unidad se puede expresar de la siguiente forma:

i , si i f 1
(8.113)
E f (i f ) = 1f
3
4 i f + 4 , si i f > 1
Como la reactancia del eje cuadratura no se satura debido a que su entrehierro es mucho
ms grande que el correspondiente al eje directo, es posible obtener la posicin de estos
ejes directamente:
D = Ve + jXq Ie = 1 + j0, 7 1 arc cos(0,85) = 1,492523,5
Tambin la posicin de la fuerza electromotriz detrs de la reactancia de dispersin Ee es
necesaria para la determinacin del valor de Eq que define el grado de saturacin a travs
de la caracterstica de vaco de la mquina:
Ee = Ve + jX Ie = 1 + j0, 2 1 arc cos(0,85) = 1, 11848,74
La fuerza electromotriz Eq se determina de la siguiente forma:
Eq = |Ee | cos(D Ee ) = 1,1184 cos(23,5 8,74) = 1,0815
Utilizando las expresiones 8.113 que han linealizado la caracterstica de vaco es posible
obtener el grado de saturacin correspondiente al punto nominal:
3
1
i f sat + = Eq = 1,0815 i f sat = 1,3260
4
4
i f lin = Eq = 1,0815
s=

i f sat
1,3260
= 1,2261
=
i f lin 1,0815

Conocido el grado de saturacin de la mquina en el punto nominal de operacin es posible corregir la reactancia del eje directo:
1
1
s1
0,2261
Xd sat = Xd +
X =
1+
0,2 = 0,8525
s
s
1,2261
1,2261
Para obtener la fuerza electromotriz del campo se siguen los siguientes pasos:
Idn = Ien sin(n n ) = 1 sin(23,5 + 31,79) = 0,8220
E f max = Dn + (Xd sat Xq ) Idn = 1,4925 + (0,8525 0,7) 0,8220 = 1,6179

395

Captulo VIII La mquina sincrnica

La corriente de campo mxima se determina a partir de la caracterstica linealizada del


campo para el punto nominal:
1
E f max = i f max i f max = sE f max = 1,2261 1,6179 = 1,9837 = 396,7 A
s
2. El factor de potencia y el ngulo de carga cuando la mquina consume de la red 50 MW
con la corriente mxima en el campo:
En este caso es necesario utilizar la ecuacin de potencia activa en funcin del ngulo
de carga, pero se desconoce la reactancia saturada del eje directo y la fuerza electromotriz
correspondiente a la corriente de campo mxima en esta condicin de saturacin. Una
tcnica que permite resolver este problema consiste en asumir una condicin inicial de saturacin s;20 con este dato se ajusta el valor de la reactancia saturada del eje directo Xd sat
y se calcula la fuerza electromotriz E f sat correspondiente a la corriente de campo mxima
i f max . De la expresin de la potencia elctrica Pe ( ) se obtiene el ngulo de carga y con
ste la potencia reactiva Qe ( ). Conocida la potencia activa y reactiva, se determina la
corriente de armadura Ie . Esta corriente permite obtener los vectores D y Ee de los cuales
se calcula la fuerza electromotriz Eq producida por el flujo resultante del eje directo. La
fuerza electromotriz Eq se utiliza para corregir el valor del grado de saturacin asumido inicialmente. Con el nuevo grado de saturacin se reinicia todo el clculo anterior y
se repite hasta que dicha variable converja en un valor de error establecido previamente.
Asumiendo que la mquina inicialmente no se encuentra saturada en el punto de operacin especificado se tendra:
Valores iniciales:
1
s = 1 ; E f = i f = 1,9837 ; Pe = 0,5 ; Xd = 1,0
s
Clculo del ngulo de carga:
0,5 = 1,9873 sin + 0,2143 sin2 = 12,01
Clculo de la potencia reactiva:
Qe = 1,9873 cos(12,01) 1,4286 (0,7 cos2 (12,01) +1,0 sin2 (12,01)) = 0,9217 pu
Clculo de la corriente de armadura:
Ie = 0,5 j0,9217 = 1,0486 118,5
20

Puede ser el mismo grado de saturacin del punto nominal o asumir una condicin de operacin lineal s = 1
como valor de arranque del proceso iterativo.

396

VIII.18 Ejemplos resueltos

Determinacin del grado de saturacin:


D =Ve + jXq Ie = 1,6820 12,01 ; Ee = Ve + jX Ie = 1,1886 4,83
Eq = 1,1886 cos(12,01 + 4,83) = 1,1793

1
3
i f sat + = Eq = 1,1793 i f sat = 1,7172 ; i f lin = Eq = 1,1793
4
4
i f sat
1,7172
s=
= 1,4318
=
i f lin 1,1793
Como el grado de saturacin es diferente al asumido inicialmente, es necesario realizar el
siguiente proceso iterativo:
Primera iteracin (s = 1,4318):

= 14,99 Qe = 0,4385
Ie = 0,6650 138,75
D =1,3530 14,99 ; Ee = 1,0923 5,25
Eq = 1,0923 cos(14,99 + 5,25) = 1,0766
s=

i f sat
1,3064
= 1,2135
=
i f lin 1,0766

Segunda iteracin (s = 1,2135):

= 13,38 Qe = 0,6728
Ie = 0,8383 126,62
D =1,5120 13,38 ; Ee = 1,1390 5,03
Eq = 1,1390 cos(13,38 + 5,03) = 1,1269
s=

i f sat
1,5077
= 1,3379
=
i f lin 1,1269

Tercera iteracin (s = 1,3379):

= 14,27 Qe = 0,5369
Ie = 0,7337 132,96
D =1,4197 14,27 ; Ee = 1,1119 5,16
Eq = 1,1119 cos(14,27 + 5,16) = 1,0979
s=

i f sat
1,3915
= 1,2674
=
i f lin 1,0979

397

Captulo VIII La mquina sincrnica


Cuarta iteracin (s = 1,2674):

= 13,76 Qe = 0,6131
Ie = 0,7911 129,19
D =1,4714 13,76 ; Ee = 1,1271 5,09
Eq = 1,1271 cos(13,76 + 5,09) = 1,1142
s=

i f sat
1,4568
= 1,3070
=
i f lin 1,1142

Quinta iteracin (s = 1,3070):

= 14,05 Qe = 0,5701
Ie = 0,7583 131,25
D =1,4422 14,05 ; Ee = 1,1185 5,13
Eq = 1,1185 cos(14,05 + 5,13) = 1,1050
s=

i f sat
1,4149
=
= 1,2850
i f lin 1,1050

Sexta iteracin (s = 1,2850):

= 13,88 Qe = 0,5939
Ie = 0,7763 130,09
D =1,4584 13,87 ; Ee = 1,1185 5,13
Eq = 1,1232 cos(13,89 + 5,11) = 1,1100
s=

i f sat
1,4402
=
= 1,2974
i f lin 1,1100

Sptima iteracin (s = 1,2974):

= 13,98 Qe = 0,5805
Ie = 0,7662 130,73
D =1,4493 13,98 ; Ee = 1,1206 5,12
Eq = 1,1206 cos(13,98 + 5,12) = 1,1072
s=

i f sat
1,4289
= 1,2906
=
i f lin 1,1072

Entre los dos ltimos valores obtenidos del grado de saturacin, el error relativo es inferior al 0,53 % con lo cual queda determinado el factor de potencia del punto de operacin
como 0,6478 y el ngulo de carga 13,92. En la siguiente tabla se resumen los principales
resultados de clculo iterativo:

398

VIII.19 Ejercicios propuestos

iteracin
0
1
2
3
4
5
6
7
8

12,01
14,99
13,38
14,27
13,76
14,05
13,88
13,98
13,92

s
1,0000
1,4318
1,2135
1,3379
1,2674
1,3070
1,2850
1,2974
1,2906

Qe ( )
0,9217
0,4385
0,6728
0,5369
0,6131
0,5701
0,5939
0,5805
0,5878

e
118,5
138,75
126,62
132,96
129,19
131,25
130,09
130,73
130,38

|Ie |
1,0486
0,6650
0,8383
0,7337
0,7911
0,7583
0,7763
0,7762
0,7717

Eq
1,1793
1,0766
1,1269
1,0979
1,1142
1,1050
1,1100
1,1072
-

Si se utiliza como grado de saturacin el promedio de los valores obtenidos en iteraciones


consecutivas, el proceso de convergencia puede ser acelerado. Esto se debe fundamentalmente a que el sistema oscila amortiguadamente sobre el valor de la solucin, tal como se
observa en el siguiente grfico:
s

Oscilaciones en el proceso iterativo

1.40
1.30
1.20
1.10
1.00

VIII.19

EJERCICIOS

Iter.

PROPUESTOS

1. Una mquina sincrnica de polos salientes de 800 MVA, 13,8 kV,factor de potencia nominal 0,87 y 3,0 kA de corriente nominal de campo, tiene impedancias de 1,0 pu y 0,7 pu
en los ejes directo y cuadratura respectivamente. Si el comportamiento de la mquina es
completamente lineal, determine:

a) La mxima potencia reactiva que esta mquina puede entregar a una barra infinita
con 1,05 pu de tensin cuando genera 650 MW .
b) La corriente de excitacin necesaria para motorizar 500 MW , con factor de potencia
unitario y 0,97 de la tensin nominal.

399

Captulo VIII La mquina sincrnica


2. Cuando una mquina sincrnica de polos salientes de 100 MVA, 10 kV, 60 Hz,6 pares de
polos, i f n = 200 A, cos n = 0,85 est operando en el punto nominal y se desconecta de
la red se obtiene 17,39 kV de tensin lnea-lnea. Cuando entrega 56,5 MW a una bomba
alimentada de un sistema de 10,5 kV , con un factor de potencia 0,707 inductivo, requiere
342 A en la bobina de campo. Determine:

a) Las reactancias del eje directo y cuadratura de esta mquina.


b) La corriente de campo necesaria para generar 90 MW , con una tensin de 9,5 kV y
factor de potencia mnimo y mximo.
c) La potencia reactiva consumida o generada por la mquina si consume 70 MW a
tensin nominal, cuando se inyecta en el campo la corriente nominal y la corriente
mxima.

3. Una mquina sincrnica de polos salientes 60 Hz posee los siguientes datos de placa:
Sn
Vn
150 MVA 23 kV

cos n
ifn
i f max
0,8192 500 A 794 A

Con la corriente de campo mxima y sin potencia en el eje mecnico, por el estator de
la mquina circulan 2,772 kA. Determine:
a) Las reactancias Xd y Xq de la mquina y compruebe mediante el diagrama fasorial
que el punto nominal coincide con los datos de placa.
b) Repita la determinacin de reactancias Xd y Xq del punto anterior utilizando las ecuaciones de potencia.
c) La corriente de campo necesaria para mantener a 24 kV la potencia activa nominal
como motor, con el factor de potencia nominal y con factor de potencia unitario.
d) El diagrama fasorial completo21 cuando la mquina consume de la red 120 MW con
la corriente de campo mxima.

4. Determine las expresiones de la potencia activa y reactiva en funcin del ngulo de carga,
para una mquina sincrnica de polos salientes, incluyendo el efecto de la resistencia de
armadura.
5. De una mquina sincrnica de polos salientes se conocen los siguientes datos de placa:
Sn = 80 MVA Vn = 10 kV
X = 0,15 pu Xd = 1,05 pu

cos n = 0,8 ind. i f n = 550 A


Xq = 0,85 pu
f = 60 Hz

La caracterstica de vaco se puede aproximar mediante las siguientes rectas: la zona lineal definida para corrientes inferiores a la nominal y la pendiente de la zona saturada es
21

Esquemtico pero con la indicacin de los valores exactos calculados analticamente.

400

VIII.19 Ejercicios propuestos


la mitad de la pendiente lineal.

a) La mquina entrega 48 MW a la red con un factor de potencia 0,64 inductivo. Determine en esta condicin de operacin la corriente de campo necesaria para mantener
el punto. Es posible mantener esta condicin de operacin?
b) Si se mantiene la corriente de campo mxima y la potencia activa de la pregunta
anterior, determine el factor de potencia y la corriente de operacin de la mquina.

6. De una mquina sincrnica de polos salientes se conocen los siguientes datos de placa:
Sn = 10 kVA Vn = 416V
X = 0,2 pu Xq = 0,7 pu

cos n = 0,8 ind. i f n = 3 A


f = 60 Hz
i f max = 6 A

La caracterstica de vaco se muestra en la figura:

a) Determine la reactancia saturada del eje directo en las condiciones nominales de


operacin.
b) Determine el ngulo de carga y el factor de potencia de la mquina cuando se excita
con la corriente nominal de campo y mantiene la potencia activa en condicin motor.
c) Calcule el punto de operacin si la mquina genera 8 kW y consume 3 kVA reactivos.
d) Determine si la mquina puede motorizar una carga de 8 kW y entregar 5 kVAR a la
red en rgimen permanente.

7. De una mquina sincrnica de polos salientes se conocen los siguientes datos de placa:
Sn = 5 kVA
Vn = 416V
Xd = 0,5 pu n =1.800 rpm

cos n = 0,85 ind. i f n = 5 A X = 5,5


f = 60 Hz
i f max = 9 A
p=2
401

Captulo VIII La mquina sincrnica


La caracterstica de vaco se puede expresar en el sistema adimensional de unidades como:



3 + e0,5i f
i f
Ef0 = 1e
+
if
10
a) Determine las reactancias del eje directo y del eje cuadratura de la mquina en cuestin.
b) Calcule la potencia reactiva entregada o consumida por el convertidor si motoriza un
molino de 4 kW , con la corriente de excitacin nominal.
c) Calcule la corriente de campo mnima que permite la operacin de la mquina en
rgimen permanente.
d) Recalcule los puntos a, b y c considerando que la mquina no se satura.

8. Una mquina sincrnica de polos salientes de 100 MVA de potencia nominal y 10 kV de


tensin de lnea a lnea, tiene un factor de potencia nominal de 0,85 inductivo. La corriente
nominal de campo es de 100 A y la corriente de campo mxima es de 154 A. Inyectando
corrientes de secuencia cero en la armadura de la mquina se determin que la reactancia de fuga es de 0,1 . En el ensayo de cortocircuito se obtuvieron 8.248 A circulando
por la armadura cuando se aplicaba corriente nominal en el campo. De un ensayo de
deslizamiento se pudo determinar que la reactancia del eje cuadratura de la mquina era
aproximadamente un 70 % de la reactancia no saturada del eje directo. La caracterstica
de vaco se puede representar experimentalmente de la siguiente forma:
E f 0 = 1,876(1 e0,7615i f ) pu
Donde la fuerza electromotriz E f 0 est en por unidad de la tensin nominal de armadura y
la corriente de campo i f en por unidad de la corriente nominal de campo. Con todos estos
antecedentes, determine:

a) La reactancia del eje directo no saturada y el grado de saturacin del punto nominal.
b) La corriente de campo necesaria para generar 50 MW y consumir 25 MVAR de la red,
cuando la tensin es un 4 % mayor que su valor nominal.
c) La mxima potencia reactiva que puede entregar la mquina como condensador sincrnico cuando la tensin en bornes de la mquina est en su valor nominal.
d) Los reactivos generados por la mquina si motoriza una carga de 80 MW con una
corriente de 120 A en el campo.

9. Una mquina sincrnica de polos salientes posee los siguientes datos de placa:
Sn
Vn
10 kVA 230V

cos n
0,8

ifn
5A

i f max
9,147 A

Operando como condensador sincrnico, la mquina entrega a la red 5 kVAR cuando es

402

VIII.19 Ejercicios propuestos


excitada con una corriente de campo de 8,718 A. La reactancia de fuga se determina a
partir de una prueba de secuencia cero y tiene un valor de 0,7935 . La caracterstica de
excitacin de la mquina en vaco se puede ajustar mediante la siguiente expresin:

E f = 3,3014 1 e0,3029i f pu
En estas condiciones, determine:

a) Las reactancias no saturadas Xd y Xq de la mquina.


b) Los reactivos que entrega la mquina al sistema cuando la mquina motoriza un
molino de piedras de 4 kW , con la corriente de campo mxima y mnima.
c) La corriente de campo necesaria para que circule corriente nominal en la armadura
cuando la mquina est en cortocircuito.
d) La potencia activa que entrega esta mquina al sistema cuando recibe 2 kVAR de la
red, si opera con tensin corriente de armadura nominal.

10. Una mquina sincrnica de polos salientes posee los siguientes datos de placa:
Sn
Vn
10 MVA 10 kV

cos n
0,85

ifn
1,5 kA

Operando como inductor sincrnico, la mquina consume de la red 5 MVAR cuando es


excitada con una corriente de campo de 771 A. La prueba de deslizamiento arroja una
relacin de 1,2857 entre las reactancias no saturadas de eje directo y cuadratura. La reactancia de fuga se determina a partir de una prueba de secuencia cero y tiene un valor de
0,2 pu. La caracterstica de excitacin de la mquina en vaco se puede ajustar mediante
la siguiente expresin:

E f = 3,3014 1 e0,3029i f pu
En estas condiciones, determine:

a) El grado de saturacin, el ngulo de carga y la corriente del campo en el punto


nominal.
b) La corriente de campo mnima a potencia activa nominal y tensin nominal.
c) Los reactivos que la mquina entrega al sistema cuando se alimenta el campo con la
corriente nominal, si la mquina consume 6 MW de la red y la tensin es de 9,5 kV .

11. Una mquina sincrnica de polos salientes 60 Hz posee los siguientes datos de placa:
Sn
Vn
400 MVA 20 kV

cos n
0,8

ifn
500 A

403

Captulo VIII La mquina sincrnica

La caracterstica de vaco de la mquina se puede expresar en cantidades fsicas como:




1,3862103 i f
E f = 40 1 e
kV

El ensayo de secuencia cero determin que la reactancia de dispersin es de 0,1 . En


la prueba de cortocircuito con corriente de campo nominal circul por la armadura una
corriente de 12 kA. La prueba de deslizamiento determin que la relacin entre la reactancia directa y cuadratura es de 160 %. Determine:

a) La corriente mxima de campo, el grado de saturacin y el ngulo de carga en el


punto nominal.
b) La corriente de armadura cuando la mquina opera con la potencia activa nominal
como motor y la corriente nominal en el circuito de campo.
c) Las mximas potencias reactivas que puede entregar o absorber la mquina operando
en las condiciones de condensador o inductor sincrnico.

12. Desarrolle un procedimiento que permita considerar la saturacin del material magntico
en las mquinas de rotor liso. Recuerde que en este caso ambos ejes pueden saturarse a
diferencia de las mquinas de polos salientes, que no pueden saturarse en el eje cuadratura. Una vez desarrollado este procedimiento, determine:

a) La corriente de campo requerida para mantener una potencia activa y reactiva determinada en una barra infinita.
b) La potencia reactiva inyectada a una barra infinita cuando se acciona la mquina de
rotor liso a una potencia mecnica y la corriente de campo a valores constantes.

404

Bibliografa

A DKINS, B. & H ARLEY, R. G., The General Theory of Alternating Current Machines, Chapman and Hall, London, 1975.
C ONCORDIA , C., Synchronous Machines: Theory and Performance, General Electric Company,
New York, 1951.
E LGERD , O. I., Electric Energy System Theory: An Introduction, McGraw-Hill, New York,
1971.
KOSTENKO, M. P. & PIOTROVSKI, L. M., Mquinas elctricas, vol. II, Editorial Mir, Second
edition, Mosc, 1979.
L ANGSDORF, A. S., Theory of Alternating Current Machinery, Tata McGraw-Hill, Second Edition, New York, 1974.
M C P HERSON , G. & L ARAMORE , R. D., An Introduction to Electrical Machines and Transformers, John Wiley & Sons, Singapur, 1990.
PARK , R. H., Two Reaction Theory of Synchronous machines, Part. 1, AIEE Transactions,
vol. 48, pp. 716-730, 1929.
PARK , R. H., Two Reaction Theory of Synchronous machines, Part. 2, AIEE Transactions,
Vol. 52, pp. 352-355, 1933.

405

Captulo VIII La mquina sincrnica

406

CAPTULO IX

Rgimen transitorio de la mquina sincrnica

Durante los cambios en la condicin de operacin de una mquina sincrnica existe energa
atrapada tanto en los campos magnticos como en la masa rotante. El convertidor debe adaptar esta energa a las nuevas condiciones desde el punto de operacin inicial. El cambio desde
un estado energtico a otro se denomina transitorio de la mquina. El anlisis de los procesos
transitorios de la mquina sincrnica se puede dividir en dos tipos principales: Los transitorios electromagnticos y los transitorios electromecnicos. Aun cuando el proceso transitorio
es continuo, la existencia de constantes de tiempo muy diferenciadas permite realizar estas divisiones. Las constantes de tiempo mecnicas son generalmente muy lentas en mquinas de
grandes dimensiones, cuando se comparan con las constantes de tiempo electromagnticas. El
anlisis de transitorio electromagntico estudia el comportamiento de los flujos, enlaces de flujo,
fuerzas electromotrices y tensiones de la mquina durante perturbaciones rpidas, considerando que la velocidad mecnica es prcticamente constante en todo el proceso. En los estudios
transitorios electromecnicos se determina el comportamiento dinmico del sistema mecnico,
utilizando valores medios de la potencia. Estos procesos estn estrechamente interrelacionados,
la separacin es un tanto artificial, pero tiene el propsito de simplificar el anlisis y la solucin del problema. El desarrollo actual de las herramientas de cmputo posibilita la solucin
global del problema. Esto es de gran utilidad cuando el transitorio tiene constantes de tiempo
electromagnticas y electromecnicas del mismo orden de magnitud.
Mediante el anlisis transitorio electromagntico se evalan las solicitudes fsicas que las diferentes perturbaciones pueden ocasionar sobre la mquina. Con los resultados obtenidos en estos
estudios se ajustan los reguladores de la excitatriz y el sistema de proteccin. El anlisis transitorio electromecnico determina los lmites de estabilidad dinmica de las diferentes mquinas
acopladas a la red elctrica de potencia. Los estudios de estabilidad se utilizan para planificar
la expansin de la red y con la finalidad de ajustar los reguladores de velocidad de la mquina
motriz.

407

Captulo IX Rgimen transitorio de la mquina sincrnica


Para analizar los transitorios de las mquinas sincrnicas se utilizan las ecuaciones diferenciales
en coordenadas dq0 f desarrolladas en el captulo 8. Las condiciones impuestas en las fases
se transforman a este sistema de coordenadas, se resuelven las ecuaciones diferenciales y finalmente se antitransforman estas soluciones para determinar el comportamiento de la mquina en
el sistema de coordenadas primitivas.
El sistema de ecuaciones 8.54 define el comportamiento electromecnico y electromagntico
completo de una mquina sincrnica de polos salientes en coordenadas dq0 f . En estas ecuaciones, las corrientes [i] y la velocidad mecnica del eje m , constituyen las variables de estado
del sistema. Las tensiones [v] y el par mecnico Tm , representan las variables de control del proceso. Por otra parte, el sistema de ecuaciones diferenciales 8.41 expresa el modelo de la mquina
sincrnica en funcin de los enlaces de flujo de los ejes dq0 f .
Para resolver las ecuaciones diferenciales que modelan la mquina sincrnica es necesario transformar las condiciones de contorno de las variables de control y las condiciones iniciales de las
variables de estado al sistema de coordenadas dq0 f . Las condiciones de contorno pueden ser:
cortocircuitos, cambios sbitos en la carga o en el par de accionamiento, apertura de interruptores, o fallas del sistema elctrico de potencia.
Las constantes de tiempo del sistema mecnico son generalmente mucho mayores que las constantes de tiempo del sistema electromecnico. Por esta razn se puede aproximar durante el
anlisis de los transitorios electromagnticos, que la velocidad de la mquina permanece prcticamente constante. Con esta aproximacin, el sistema de ecuaciones diferenciales 8.54 es lineal
y puede ser resuelto analticamente. El transitorio mecnico se resuelve mediante la ecuacin
dinmica1 , evaluando en forma ms o menos aproximada la potencia elctrica media durante
el proceso dinmico electromecnico. Una vez que se ha evaluado la velocidad para un preciso
instante, se determina un nuevo transitorio electromagntico. Desacoplando estos dos procesos,
es posible realizar una integracin rpida de las ecuaciones diferenciales.

IX.1

T RANSITORIOS

ELECTROMAGNTICOS

Las ecuaciones diferenciales de la mquina sincrnica son lineales si se considera que la velocidad del rotor es constante. Los sistemas de ecuaciones diferenciales lineales de primer orden
con coeficientes constantes se resuelven mediante tcnicas analticas bien conocidas: autovalores-autovectores o mediante la transformada de Laplace. La tcnica de autovalores-autovectores
obtiene respuestas temporales directas. La transformada de Laplace, determina la respuesta en
el dominio de la frecuencia, posteriormente es necesario antitransformar para obtener las soluciones temporales.

Ecuacin de Newton.

408

IX.1 Transitorios electromagnticos


IX.1.1

Solucin mediante autovalores-autovectores

El sistema de ecuaciones 8.54 se puede descomponer de la siguiente forma:


id
vd
Ld 0 0 Ld f
Re 0 0 0
vq

= 0 Re 0 0 iq + 0 Lq 0
p
v0
0 0 R0 0 i 0 0
0 L0 0
0 0 0 Rf
if
vf
Ld f 0 0 L f

0 Lq 0 0
id
Ld

0
0 0
iq
+
0

i0
0
0 0
if
0
0
0 0

id
iq
+
i0
if
(9.1)

El sistema de ecuaciones diferenciales 9.1, se puede expresar en forma compacta como:


[v] = [[R] + [G]] [i] + [L] p [i]

(9.2)

Despejando de la expresin 9.2 el vector de derivadas de las variables de estado p [i], se obtiene:
p [i] = [L]1 {[v] [[R] + [G]] [i]}

(9.3)

Los autovalores de la matriz caracterstica de la ecuacin anterior determinan los modos naturales de la respuesta homognea de la mquina sincrnica en coordenadas dq0 f . Para obtener
la matriz caracterstica es necesario invertir la matriz de inductancias [L]:

1
1
0 0

Ld f
Ld 0 0 Ld f
Ld
1
0

0 0
0 Lq 0
0

1
Lq

(9.4)
[L] =
=

0
0 L0 0
0 0 L
0
0

1
1
Ld f 0 0 L f
0 0

Ld f

Donde:

Ld Ld

L2d f
Lf

; Lf Lf

L2d f

Lf

; Ld f Ld f

Ld

Ld L f
Ld f

(9.5)

La matriz caracterstica [A] es:

Re

Ld
Ld
Lq

1
[A] = [L] [[R] + [G]] =
0

Re

Ld f

Lq

Ld
Re
Lq

Lq

Ld f

0
0
R0
L0

Rf

Ld f
Ld f
Lq

0
Rf

Lf

(9.6)

Los autovalores i de la matriz caracterstica 9.6 se obtienen al resolver la siguiente ecuacin


algebraica:
det [[A] i [I]] = 0
(9.7)
409

Captulo IX Rgimen transitorio de la mquina sincrnica


Como el eje 0 se encuentra desacoplado del resto del sistema, es posible reducir en uno el grado
del polinomio caracterstico definido por la expresin 9.7. Las resistencias de las bobinas son
muy pequeas al compararse con las reactancias de la mquina. Despreciando el efecto de las
resistencias del estator y rotor de la mquina, se obtiene de las expresiones 9.6 y 9.7 el polinomio
caracterstico del sistema:



R0
+
3 + 2 = 0
(9.8)
L0
El autovalor correspondiente al eje 0 es:

0 =
y los otros tres autovalores son:

R0
L0

(9.9)

1 = 0 ; 2,3 = j

(9.10)

Al despreciar la resistencia de las bobinas de la mquina sincrnica, se obtienen dos autovalores


complejos conjugados 2 y 3 , cuya magnitud es igual a la velocidad sincrnica de la mquina.
Estos dos autovalores se deben a las realimentaciones de fuerza electromotriz de generacin
existentes entre el eje directo y el eje cuadratura del convertidor. El autovalor en el origen 1 se
debe fundamentalmente a la bobina de campo. Si no se desprecia la resistencia de las bobinas,
el polo del origen y los autovalores complejos conjugados se desplazan ligeramente hacia el
semiplano izquierdo y se reduce la frecuencia natural de oscilacin.
Para obtener la solucin homognea del sistema de ecuaciones diferenciales 9.3 es necesario
determinar la matriz de autovectores correspondiente a los cuatro autovalores determinados del
polinomio caracterstico 9.8. Cada uno de los autovalores se calcula obteniendo las soluciones
no triviales del siguiente sistema de ecuaciones:
[[A] i [I]] [Vi ] = [0]
La matriz de autovectores queda formada de la siguiente forma:

Ld f
Ld f

0 Ld
Ld



Ld f
[V ] = [V0 ] [V1 ] [V2 ] [V3 ] = 0
0
j Lq

1
0
0
0
1
1

La solucin homognea del sistema de ecuaciones diferenciales es:

Ld f
Ld f
Ld f

idh (t)
Ld

Ld
Ld

iqh (t)
Ld f
Ld f
= ki [Vi ] eit =
[ih (t)] =

j
0
0
j
i0h (t)
Lq
Lq

i=0
1
0
0
0
i f h (t)
0
1
1
1
410

(9.11)

Ld f

Ld

Ld f
j Lq

0
1

k0 e0t

k1 e1t

k2 e2t

k3 e3t

(9.12)

(9.13)

IX.1 Transitorios electromagnticos


Al superponer la solucin homognea y la solucin particular, correspondiente a la condicin
final de rgimen permanente, se determinan los coeficientes ki , que satisfacen las condiciones
iniciales de las variables de estado:
[i(t = 0)] = [ih (0)] + [i p(0)] = [V ] [k] + [i p (0)] [k] = [V ]1 [[i(0)] [i p (0)]]

(9.14)

La solucin particular del problema est forzada por las variables de control, tensiones en bornes de las bobinas de la mquina y par mecnico en el eje del rotor. En el captulo 8 se analiz
el comportamiento de la mquina sincrnica en rgimen permanente, la tcnica del diagrama
fasorial permite evaluar las condiciones forzantes de las fuentes en las coordenadas primitivas
y transformadas. Los fasores deben ser transformados a valores instantneos para obtener las
soluciones particulares en el dominio del tiempo. Representar magnitudes trifsicas sinusoidales mediante fasores y proyectarlos segn los ejes d y q, equivale a realizar la transformacin
de Park.

IX.1.2

Solucin mediante la transformada de Laplace

Aplicando la transformada de Laplace al sistema de ecuaciones diferenciales 9.1, se obtiene el


siguiente sistema de ecuaciones algebraicas:

Re + Ld s Lq
Vd (s)
0
Ld f s
d (0+)
Id (s)
+
Vq (s) Ld

Re + Lq s
0
Ld f
=

Iq (s) q (0+) (9.15)


V0 (s)
I0 (s) 0 (0 )
0
0
R0 + L0 s
0
V f (s)
I f (s)
Ld f s
0
0
Rf +Lf s
f (0+ )

Donde:


d (0+ )
Ld 0 0 Ld f
q (0+ ) 0 Lq 0
0


0 (0+ ) = 0
0 L0 0
f (0+ )
Ld f 0 0 L f

id (0+)
iq (0+ )

i0 (0+ )
i f (0+)

(9.16)

En forma compacta, el sistema anterior se expresa como:





[Z(s)] [I(s)] = [V (s)] + Ldq0 f idq0 f (0+ )

(9.17)

Donde [Z(s)] es la matriz de impedancia operacional de la mquina sincrnica.


El determinante de esta matriz define los polos de la respuesta transitoria. Se observa que cada
uno de los miembros de las ecuaciones 9.15 y 9.17 tienen dimensiones fsicas de flujo magntico
[W b]. La transformada de Laplace convierte un balance de tensiones en el dominio del tiempo, en
un equilibrio de flujos en el dominio de la frecuencia. El determinante de la matriz de impedancia
operacional de la mquina sincrnica [Z(s)], en coordenadas dq0 f es:
D = (R0 + L0 s)

(Re + Ld s)(Re + Lq s)(R f + L f s) +

2 L2d f Lq s L2d f s2 (Re + Lq s) + 2 Ld Lq (R f + L f s)

(9.18)

411

Captulo IX Rgimen transitorio de la mquina sincrnica

Figura 9.1 Cortocircuito brusco de la mquina sincrnica


Cuando se desprecian las resistencias del estator y del campo, la expresin 9.18 se simplifica a:

D = Ld Lq L0 L f s (s2 + 2 ) (s +

R0
)
L0

(9.19)

Los polos de este polinomio son:


s0 =

R0
; s1 = 0 ; s2,3 = j
L0

(9.20)

que corresponden exactamente con los autovalores de la matriz caracterstica [A], obtenidos en
la seccin IX.1.1.
Multiplicando la expresin 9.17 por la inversa de la matriz operacional se determinan las corrientes transformadas. Una vez definidas las fuentes independientes y las condiciones iniciales, se
obtiene la solucin del problema. Resolviendo las ecuaciones diferenciales mediante la tcnica
de autovalores y autovectores es necesario obtener una solucin particular y ajustar los coeficientes indeterminados con las condiciones iniciales. Aplicando la transformada de Laplace,
la solucin completa se obtiene directamente debido a que las funciones de transferencia contienen toda la informacin necesaria. Antitransformando cada una de las funciones, se obtiene
directamente la respuesta temporal del problema.

IX.2

C ORTOCIRCUITO

BRUSCO DE LA MQUINA SINCRNICA

Uno de los anlisis transitorios de mayor inters en el estudio de la mquina sincrnica es el


cortocircuito trifsico brusco y franco en bornes de las bobinas de armadura. En la figura 9.1 se
muestra un diagrama esquemtico de las condiciones previas y posteriores al cortocircuito.
Las corrientes de fase son cero durante el tiempo anterior al cortocircuito. Aplicando la transformacin de Park 8.42 a estas condiciones, se obtiene que las corrientes en coordenadas dq0
en la condicin previa al cortocircuito tambin son nulas. La corriente del campo antes del cor-

412

IX.2 Cortocircuito brusco de la mquina sincrnica


tocircuito es:
i f (0 ) =

vf
Rf

(9.21)

Las condiciones iniciales en el instante de tiempo inmediatamente posterior al cierre del interruptor (t = 0+ ), se determinan considerando que los enlaces de flujo en las bobinas de la
mquina se conservan entre el instante inmediatamente anterior y posterior al cortocircuito. De
esta forma, las variables transformadas satisfacen la siguiente condicin:

d (0+ )
d (0)
q (0+ )
q (0)

=
(9.22)
0 (0+ )
0 (0)
f (0+ )
f (0 )

id (0)
Ld 0 0 Ld f
id (0+ )
Ld 0 0 Ld f

+

0 Lq 0
0
0
iq (0) (9.23)

iq (0+ ) = 0 Lq 0
0
0
0 L0 0 i0 (0 )
0 L0 0 i0 (0 )
i f (0)
Ld f 0 0 L f
i f (0+ )
Ld f 0 0 L f

y por tanto:

id (0)
id (0+ )
+
iq (0 ) iq (0)

i0 (0+ ) = i0 (0)
i f (0 )
i f (0+ )

0
0
=

(9.24)

vf
Rf

Durante la condicin de cortocircuito, las tensiones en bornes de la armadura de la mquina son


cero, y las tensiones en coordenadas dq0 tambin son nulas. La tensin en el devanado de campo
permanece constante en el valor v f . La solucin particular se obtiene al considerar el comportamiento en rgimen permanente de la mquina en un tiempo muy largo posterior al cortocircuito.
Al despreciar la resistencia del estator, el fasor que representa la corriente de cortocircuito en
rgimen permanente, se retrasa 90 con respecto a la direccin del eje cuadratura. La corriente de armadura est orientada segn la direccin del eje directo, con lo cual se obtiene de las
ecuaciones fasoriales de la mquina sincrnica en la convencin motor el siguiente resultado:
Ve = E f + jXd Id + jXq Iq = E f + jXd Id = 0
id =

Ld f v f
; iq = 0
Ld R f

(9.25)

De la expresin 9.14 se determinan los coeficientes ki de la solucin homognea del problema:

Ld f
Ld f
Ld f

Ld f v f

Ld R f
0
1
k0
L

d
Ld
Ld
0
L v 1

k1
df f 2

Ld f
Ld f
0

=
(9.26)
=
0
0
j Lq j Lq 0
vf
k2
v

Ld R f 0

R
f
f
1
1
0
0
0
k3
Rf
0
2
0
1
1
1
413

Captulo IX Rgimen transitorio de la mquina sincrnica


Las corrientes instantneas se obtienen superponiendo la solucin homognea 9.13 y la solucin
particular 9.25:
Ld f v f
(cos t 1)
Ld R f
Ld f v f
sin t
iq (t) =
Lq R f
"
#
Ld f
vf
i f (t) = 1 + (1 cos t)
Rf
Ld f
id (t) =

(9.27)
(9.28)
(9.29)

i0 (t) = 0

(9.30)

Para obtener las corrientes en coordenadas primitivas, se realiza la transformacin inversa de


Park; la corriente instantnea de la fase a queda:
r
r

1
2
2
id cos( t + 0 ) iq sin( t + 0 ) =
(id cos iq sin + i0 ) =
ia (t) =
3
3
2
"
#
r
1
1 1
1 1
1
1
2 vf
Ld f cos( t + 0 ) + ( ) cos(2 t + 0 ) + ( + ) cos 0 (9.31)
=
3 Rf
2 Ld Lq
2 Ld Lq
Ld
La expresin 9.31 puede ser mejor interpretada si se multiplica y divide por la velocidad sincrnica , y se incorpora la definicin de la fuerza electromotriz que produce el campo E f :
#
"

1 1
1 1
1
1
1
ia (t) = 2E f cos( t + 0 ) + ( ) cos(2 t + 0 ) + ( + ) cos 0 (9.32)
2 Xd Xq
2 Xd Xq
Xd
Utilizando el mtodo de la transformada de Laplace se obtienen exactamente los mismos resultados. En este caso es necesario determinar las corrientes transformadas de la expresin 9.15:



(9.33)
[I(s)] = [Z(s)]1 ( Ldq0 f i(0+ ) + [V(s)])

Cuando se desprecian las resistencias de los devanados, la matriz de impedancia inversa es:
1
Ld s Lq 0 Ld f s
Ld
Lq s
0 Ld f
=
[Z(s)]1 =
0
0
L0 s
0
Ld f s
0
0
Lf s

414

Ld (s2 + 2 )
s
Lq (s2 + 2 )

Ld (s2 + 2 )
L (s2+ 2 )
q

Ld f s

Ld L f (s2 + 2 )

Ld f

Ld L f (s2 + 2 )

Ld f

Ld L f s

1
L0 s

Lfs

(9.34)

IX.3 Interpretacin fsica de las inductancias transitorias


Como las tres fuentes de tensin de las fases son nulas durante la aplicacin del cortocircuito,
en el dominio de la frecuencia se obtienen las siguientes corrientes transformadas:
Ld f 2
vf

2
2
s(s + )Ld R f
Ld f
vf
Iq (s) = 2
2
(s + )Lq R f
I0 (s) = 0
#
"
2
s
Ld v f
I f (s) =
+
s2 + 2 s(s2 + 2 ) Ld R f
Id (s) =

(9.35)
(9.36)
(9.37)
(9.38)

Transformando las expresiones anteriores al dominio del tiempo se obtienen las mismas corrientes instantneas 9.27 a 9.30, calculadas mediante el mtodo de los autovalores y autovectores.
Las corrientes por la armadura y por el campo dependen de la velocidad angular mecnica de la
mquina, porque durante el transitorio electromagntico la velocidad permanece prcticamente
constante. Las oscilaciones de las corrientes son automantenidas y no amortiguadas debido a
que se han despreciado las resistencias de las bobinas. El ngulo 0 determina la posicin del
rotor con respecto al eje magntico de la fase a del estator en el instante de tiempo inicial del
cortocircuito. La expresin 9.32 se utiliza frecuentemente en el ajuste de las protecciones contra
cortocircuito de las mquinas sincrnicas.

IX.3

I NTERPRETACIN

FSICA DE LAS INDUCTANCIAS TRANSITORIAS

En la expresin 9.5 se han definido las inductancias Ld , L f y Ld f para simplificar el anlisis

de los transitorios electromagnticos de la mquina sincrnica. Las inductancias Ld y L f tienen


una interpretacin fsica concreta y representan parmetros equivalentes vistos desde el eje d y
f respectivamente. En la figura 9.2 se representa la configuracin esquemtica de las bobinas
del eje directo y del campo. Estas bobinas se encuentran perfectamente alineadas por la definicin de la transformacin y constituyen un circuito acoplado magnticamente, semejante a un
transformador de dos devanados, d y f .
El modelo matemtico que define el comportamiento del circuito magntico representado en la
figura 9.2 es el siguiente:


 

vd
id
Ld p Ld f p
=
(9.39)
vf
Ld f p L f p
iq
Cuando el circuito de campo se excita mediante una fuente de tensin, se puede referir este
circuito al eje directo mediante la reduccin de Krn:
vd = (Ld

L2d f
Lf

)pid +

Ld f
Ld f

v f = Ld pid +
vf
Lf
Lf

(9.40)

415

Captulo IX Rgimen transitorio de la mquina sincrnica

Figura 9.2 Acoplamiento magntico entre las bobinas del eje directo y del campo

Alimentando el eje directo con una fuente de tensin y reflejando este devanado sobre el campo
de la mquina mediante la reduccin de Krn, se obtiene:
v f = (L f

L2d f
Ld

)pi f +

Ld f
Ld f

vd = L f pi f +
vd
Ld
Ld

(9.41)

El reflejo de la bobina del campo sobre el eje directo y el reflejo de la bobina del eje directo

sobre el campo define las inductancias transitorias Ld y L f , respectivamente. El acoplamiento


entre estos dos circuitos implica una reduccin efectiva de la inductancia equivalente que se
observa desde cada una de las bobinas cuando la otra se excita mediante una fuente de tensin.
L
La tensin de excitacin aparece reflejada en la bobina del eje directo en la proporcin Ldff que es
d
prcticamente igual2 a la relacin de vueltas entre las dos bobinas N
N f . En el circuito de campo,
la situacin es similar y aparece el reflejo de la tensin del eje directo vd a travs de la relacin
N
de transformacin del nmero de vueltas Ndf .

Si un circuito acoplado magnticamente con la bobina se encuentra abierto, o se excita mediante


una fuente de corriente, la inductancia propia de la bobina permanece constante. Como el eje
cuadratura no est acoplado magnticamente con ninguna otra bobina, la inductancia propia de
este eje es independiente de los fenmenos transitorios en el resto de las bobinas. En estos casos,
las constantes de tiempo de las bobinas estn determinadas por el cociente entre la inductancia
propia y la resistencia de cada enrollado. Cuando la mquina se conecta a fuentes de tensin,
en la armadura o en el campo, las constantes de tiempo se reducen debido al acoplamiento
magntico existente entre estas bobinas.

Despreciando los caminos de dispersin del campo.

416

IX.4 Tensin de armadura en circuito abierto


IX.4

T ENSIN

DE ARMADURA EN CIRCUITO ABIERTO

Cuando la armadura de la mquina sincrnica se encuentra en circuito abierto, aparecen en


estas bobinas fuerzas electromotrices, pero no existe acoplamiento electromagntico entre estos
circuitos y el campo. La ecuacin del circuito de campo es:
vf = Rf if +Lf

di f
dt

(9.42)

Esta ecuacin diferencial define la constante de tiempo del circuito de campo cuando los devanados de armadura se encuentran en circuito abierto:

f0 =

Lf
Rf

(9.43)

En la prctica, la constante de tiempo del campo con las bobinas del estator en circuito abierto,
est comprendida entre 1 y 20 segundos debido a que la resistencia R f de este enrollado se
disea lo ms pequea posible para reducir las prdidas hmicas.
Las fuerzas electromotrices que aparecen sobre las bobinas de los ejes directo y cuadratura, con
las bobinas de armadura en circuito abierto son:
di f
dt
= Ld f i f

vd = Ld f

(9.44)

vq

(9.45)

Determinando la solucin de la ecuacin diferencial 9.42 y sustituyendo esta corriente en las


expresiones anteriores, se obtienen los siguientes resultados:
vf
t
(1 e f 0 )
Rf
Ld f v f ft 0
vd (t) =
e
f0 Rf
Ld f v f
t
(1 e f 0 )
vq (t) =
f0 Rf
i f (t) =

(9.46)
(9.47)
(9.48)

La fuerza electromotriz en la fase a de la mquina se consigue mediante la transformacin


inversa de Park 8.46:
r

2
va (t) =
vd (t) cos( t + 0 ) vq (t) sin( t + 0 )
(9.49)
3

Al comparar las expresiones 9.44 y 9.45 se observa que la fuerza electromotriz inducida sobre
la bobina del eje cuadratura vq , es mucho mayor que la inducida sobre el eje directo vd , debido
a que generalmente la velocidad sincrnica de la mquina es mucho mayor que el inverso de
la constante de tiempo f 0 . Despreciando la fuerza electromotriz del eje directo se obtiene la

417

Captulo IX Rgimen transitorio de la mquina sincrnica

Figura 9.3 Tensin de armadura en circuito abierto ante una energizacin del campo
siguiente tensin sobre la bobina de la mquina:
r
vf
2
t
Ld f (1 e f 0 ) sin( t + 0 )
va (t) =
3
Rf

(9.50)

En la figura 9.3 se presenta un grfico de la fuerza electromotriz en bornes de la mquina, durante


la energizacin del campo, cuando la armadura se encuentra en circuito abierto.

IX.5

S ISTEMA

ADIMENSIONAL DE UNIDADES

Al igual que en las otras mquinas elctricas y en el sistema elctrico de potencia es conveniente
utilizar el sistema adimensional de unidades, tambin conocido como sistema en por unidad,
para cuantificar las diferentes variables que definen el comportamiento de la mquina sincrnica.
Como sucede en los transformadores, la mquina sincrnica posee un rendimiento elevado en
el rango de operacin industrial y es conveniente definir la potencia elctrica aparente de la
mquina como potencia base del sistema adimensional de unidades. En muchas ocasiones es
aconsejable utilizar como base de potencia la potencia aparente monofsica de la armadura de
la mquina, especialmente cuando se definen las bases del rotor, con la finalidad de centrar las
magnitudes en por unidad de las variables y parmetros asociados con el rotor. Estas variables
pueden obtener valores extremos, porque el circuito de campo consume una potencia que es
varios rdenes de magnitud inferior a la de armadura.
Adems de fijar las bases de potencia, tensin y corriente en los convertidores electromecnicos
es necesario definir las bases de par, velocidad, tiempo, impedancia e inductancia. El par base
se define como el cociente entre la potencia base y la velocidad base de la mquina:
TB =

418

SB
B

(9.51)

IX.5 Sistema adimensional de unidades


La velocidad base de la mquina en general se escoge igual a la velocidad sincrnica:

B = s =

e
f
= 2
p
p

(9.52)

Donde p es el nmero de pares de polos del convertidor y f es la frecuencia de las corrientes


de armadura. Al definir la velocidad base de la mquina, el tiempo base y el ngulo base se
encuentran relacionados; al escoger uno, el otro queda automticamente definido:

B =

B
B 1 rad
tB =
=
tB
B
s

(9.53)

Para convertir las inductancias al sistema adimensional de unidades es necesario definir la impedancia e inductancia base del circuito, de acuerdo con las expresiones 9.52 y 9.53 se tiene:
V

B
V2
ZB
I
= B = B
LB =
B B B SB

(9.54)

Con las bases definidas anteriormente, la inductancia e impedancia adimensional es:


L pu =

L
Z
B L
=
= Z pu
=
LB B LB ZB

(9.55)

Las inductancias propias de las bobinas de la mquina sincrnica estn constituidas por una
componente de magnetizacin y otra de dispersin:
Ld = Lmd + L d = Lmd (1 + d )
Lq = Lmq + L q = Lmq (1 + q )
L f = Lm f + L f = Lm f (1 + f )

(9.56)
(9.57)
(9.58)

L f
L q
L d
; q =
; f =
Lmd
Lmd
Lm f

(9.59)

Donde:

d =

Las inductancias de magnetizacin del eje directo y del campo, estn relacionadas con la inductancia mutua entre el eje directo y el campo porque poseen el mismo camino magntico. Los
valores fsicos de estas inductancias se distinguen porque cada una de ellas est definida por
diferentes nmeros de vueltas:

Donde:

Lmd = Nd2md ; Lm f = N 2f m f ; Ld f = Nd N f d f

(9.60)

md = m f = d f

(9.61)

Si los nmeros de vueltas de las bobinas del campo y del eje directo son iguales, las tres inductancias de la expresin 9.60 tienen el mismo valor. Definiendo las bases del sistema adimensional
de unidades, es posible obtener esta simplificacin. Cuando se define la potencia base de la mquina para el circuito de armadura y la tensin base de cada bobina como su tensin nominal, en
el estator la corriente bases es igual a la corriente nominal. En el rotor esta situacin es diferente,

419

Captulo IX Rgimen transitorio de la mquina sincrnica


la tensin nominal de la bobina de campo y la potencia aparente de armadura no producen una
corriente base del mismo orden de magnitud que la corriente nominal del campo. Los valores en
por unidad son extremadamente grandes o pequeos, y se pierden las ventajas del sistema adimensional de unidades. Una solucin posible para este problema consiste en definir los enlaces
de flujo base de cada bobina:

Bmd = Lmd IB = Ld f IBF


BFm = Ld f IB = Lm f IBF

(9.62)
(9.63)

Multiplicando las expresiones anteriores se obtiene la siguiente relacin entre las corrientes base
en la armadura y el campo:
s
Nd
I
Lmd
BF
2
=
=
(9.64)
Lmd IB2 = Lm f IBF

IB
Lm f
Nf

Utilizando la base monofsica de potencia aparente de armadura en ambos circuitos, se obtiene


la relacin entre las bases de tensin de armadura y campo:
SB = VB IB = VBF IBF

Nf
VBF
IB
=
=
VB
IBF
Nd

(9.65)

La expresin 9.65 define un sistema de tensiones y corrientes base en armadura y campo. Este sistema de bases simplifica notablemente la matriz de inductancias propias y mutuas de la
mquina sincrnica cuando sta se expresa en por unidad de las bases de enlace de flujo. Las
impedancias base propias y mutuas, en el sistema adimensional de unidades son:
ZB =

ZBF
ZBDF =

VB VB2
=
IB
SB

N 2f VB
VBF
= 2
=
=
IBF
Nd IB

(9.66)
Nf
Nd

2

ZB

N f VB N f
VB
VBF
=
=
= ZBFD
ZB =
IBF
Nd IB
Nd
IB

(9.67)
(9.68)

Donde:

1
VB = Vnln y SB = Sn1 = Sn
3
Las inductancias adimensionales de la mquina sincrnica, expresadas en el sistema de bases de
enlaces de flujo y potencia aparente monofsica son:
Lmd (pu) =

Lm f (pu) =

420

Nd2d f
B 2
=
N d f
LB
ZB d
N 2f d f
LBF

B 2
N d f
ZB d

(9.69)

(9.70)

IX.5 Sistema adimensional de unidades

Tabla 9.1 Rango tpico de los valores de las inductancias de la mquina sincrnica de polos
salientes
Inductancia
Rango en pu
Ld f = Lmd = Lm f
0,7 1,1
Lmq
0,5 0,7
L d L q = d Ld f
(0,1 0,2) Ld f
L f = f Ld f
(0,2 0,3) Ld f
Ld = (1 + d )Ld f
(1,1 1,2) Ld f
L f = (1 + f )Ld f
(1,2 1,3) Ld f
Lq = (1 + q )Lmq
(1,1 1,2) Lmq

Ld = Ld

Lf = Lf

Ld f (pu) =

L2d f
Lf
L2d f
Ld

(0,27 0,43)Ld f
(0,29 0,47)Ld f

Nd N f d f
B 2
=
N d f
LBDF
ZB d

(9.71)

En el sistema adimensional que utiliza las bases de enlace de flujo y potencia aparente monofsica, las tres inductancias calculadas en las expresiones 9.69, 9.70 y 9.71 son iguales. El valor
en por unidad de estas inductancias depende de la permeabilidad del camino magntico mutuo estator-rotor d f . Valores tpicos adimensionales de las inductancias propias, mutuas y de
dispersin en las mquinas sincrnicas convencionales se presentan en la tabla 9.1.
Utilizando valores numricos medios de las inductancias estimadas anteriormente es posible
evaluar cuantitativamente la corriente instantnea de la fase a, para todo tiempo posterior al
cortocircuito brusco de la mquina sincrnica tal como se obtuvo en la ecuacin 9.32. Si la
fuerza electromotriz E f es 1,0 en por unidad, debido a que en la condicin previa al cortocircuito
la mquina se encontraba en vaco a tensin nominal, la corriente instantnea resulta ser:
ia (t) = 4,29 cos( t + 0 ) + 1,18 cos(2 t + 0 ) + 3,10 cos 0

(9.72)

La corriente de cortocircuito posee una componente de frecuencia fundamental cuyo valor efectivo es superior a 3,0 en por unidad de la corriente nominal. Evaluando la corriente de cortocircuito mediante el diagrama fasorial de la mquina sincrnica, se obtiene que el valor efectivo
de la corriente es aproximadamente 0,95 en por unidad. Un cortocircuito mantenido durante un
tiempo suficiente largo como para alcanzar el rgimen permanente, producir el decaimiento
de las corrientes instantneas hasta alcanzar la solucin obtenida mediante el diagrama fasorial.
Durante los primeros instantes de tiempo, el acoplamiento magntico entre el campo y el eje
directo reflejan una reactancia transitoria del eje directo mucho ms pequea que la reactancia
de rgimen permanente y por esta razn se incrementa el nivel de cortocircuito de la mquina
sincrnica. En la figura 9.4 se representa la corriente de la expresin 9.72, cuando el cortocircuito se inicia en el momento en el cual el eje magntico del campo coincide con el eje magntico
de la fase a del estator (0 = 0).

421

Captulo IX Rgimen transitorio de la mquina sincrnica

Figura 9.4 Cortocircuito brusco de la mquina sincrnica (0 = 0), sin resistencia en los devanados
IX.6

A NLISIS

TRANSITORIO CON RESISTENCIAS

En las secciones anteriores se han despreciado las resistencias de las bobinas para simplificar la
solucin analtica del problema. Adems se considera en todo el desarrollo que las tensiones en
las bobinas son las variables independientes, y las corrientes son las variables de estado del sistema. Si el campo se excita mediante una fuente de corriente, el problema tiene un planteamiento
y solucin diferente. Las ecuaciones dinmicas de la mquina sincrnica se pueden representar
de la siguiente forma:


 

[ie ]
[Zee ] [Zer ]
[ve ]
=
=
[ir ]
[Zre ] [Zrr ]
[vr ]
 
 

 

Ld f p
Re + Ld p Lq
id
vd
iq
Ld
Re +Lq p
= vq  =
(9.73)
 Ld f 
 
Ld f p 0
Rf +Lf p
vf
if

Como las condiciones forzantes son hbridas; tensiones del estator y corrientes en el rotor, la
expresin 9.73 se puede descomponer y reordenar de la forma siguiente:
[ve ] = [Zee ] [ie ] + [Zer ] [ir ]
[vr ] = [Zre ] [ie ] + [Zrr ] [ir ]

(9.74)
(9.75)

Despejando el vector [ir ] de la expresin 9.75, y reemplazando el resultado en la 9.74 se obtiene:


[ir ] = [Zrr ]1 [Zre ] [ie ] + [Zrr ]1 [vr ]
h
i
[ve ] = [Zee ] [Zer ] [Zrr ]1 [Zre ] [ie ] + [Zer ] [Zrr ]1 [vr ]
422

(9.76)
(9.77)

IX.6 Anlisis transitorio con resistencias


Estas dos ecuaciones se pueden expresar as:
i
h
i h




1
1
[Z
]

[Z
]
[Z
]
[Z
]
[Z
]
[Z
]
ee
er
rr
re
er
rr
[i
]
[ve ]
e

h
i
=
[vr ]
[ir ]
[Zrr ]1 [Zre ]
[Zrr ]1

(9.78)

Realizando las operaciones matriciales indicadas en el sistema de ecuaciones anterior se obtiene:



L2d f p
Ld f p

Re + Ld R f +L f p p Lq

R f +L f p
vd

id


2
vq =
Ld f
(9.79)
Ld Ld f p
iq
R
+
L
p
e
q

R f +L f p
R f +L f p
if

vf
Ld f p
1
R f +L
0
R f +L f p
fp
De la expresin 9.79 se puede obtener despus de realizar la transformacin de Laplace la impedancia operacional propia del eje directo en el dominio de la frecuencia (p s):
!
L2d f s
s
(9.80)
Zdd (s) = Re + Ld
Rf +Lf s

Si se aplica un escaln de corriente en el eje directo de la mquina con el devanado de campo en


cortocircuito se obtiene la siguiente tensin en la bobina del eje directo:
!
L2d f s
Re
I
I
(9.81)
+ Ld
Vd (s) = Zdd (s) =
s
s
Rf +Lf s

La tensin en el instante inmediatamente posterior al cortocircuito (t = 0+ ), se consigue aplicando el teorema del valor inicial:
!
2 s2
L

d
f
I sLd I
(9.82)
vd (0+ ) = lm s Vd (s) = lm Re + sLd
s
s
Rf +Lf s
En el instante inicial, el eje directo se comporta como un circuito abierto con una impedancia
infinita. Si se intercambia en la ecuacin 9.80, el operador de Laplace s, por el operador de
rgimen permanente j , se obtiene la respuesta en frecuencia para la impedancia del eje directo:
!
!
2 L2d f L f
R f L2d f 2
+ j Ld 2
(9.83)
Zdd ( j ) = Re + 2
R f + 2 L2f
R f + 2 L2f
Cuando la frecuencia j es reducida pero diferente de cero, la bobina tiende a comportarse
de acuerdo con sus parmetros de rgimen permanente, Re y Ld . A medida que aumenta la
frecuencia, el acoplamiento entre el campo y el eje directo incrementa la resistencia equivalente
de la bobina y reduce la inductancia. En el lmite, cuando la frecuencia tiende a infinito, la
resistencia del rotor queda reflejada en el eje directo segn la relacin cuadrtica del nmero de

423

Captulo IX Rgimen transitorio de la mquina sincrnica

Figura 9.5 Lugar geomtrico de la resistencia e inductancia propia del eje directo al variar la
frecuencia

vueltas, y la inductancia de la bobina tiende al valor transitorio Ld :


Zdd ( 0) = Re + j Ld
N2

Zdd ( ) = Re + d2 R f + j Ld
Nf

(9.84)
(9.85)

En la figura 9.5 se representa el lugar geomtrico de la impedancia propia de la bobina del eje
directo con la frecuencia como parmetro. En este diagrama se puede observar que la mquina
sincrnica vara durante el transitorio su impedancia de entrada. En los primeros instantes, la reactancia transitoria se manifiesta plenamente y a medida que transcurre el tiempo la impedancia
se estabiliza en el valor de rgimen permanente.
En el listado 12 se reproduce un procedimiento de clculo en el entorno de programacin SCILAB que permite evaluar las corrientes en coordenadas primitivas y transformadas durante un
cortocircuito brusco de la mquina sincrnica, considerando las atenuaciones debidas a las resistencias de las bobinas. En la figura 9.6 (a) se muestra en un grfico los resultados obtenidos para
las variables en el sistema de coordenadas dq f , mientras que en la figura 9.6 (b) se representan
las corrientes en coordenadas primitivas a y f .

424

IX.6 Anlisis transitorio con resistencias

Algoritmo 12 Clculo de las corrientes de la mquina sincrnica de polos saliente durante un


cortocircuito brusco
// ANALISIS TRANSITORIO DE MAQUINAS SINCRONICAS
// p[i=-[L^-1*([R+w*[G)+[L^-1*[v
// [A =-[L^-1*([R+w*[G)
w = 1.0;
// velo idad sin rni a
re = 0.01;rf = 0.02;
// resisten ia del estator y del ampo
Ld = 1.0;Lq = 0.6;Lf = 1.0;Ldf= 0.8;
// indu tan ia del eje d, q, f y mutua df
R = diag([re re rf,0);
// matriz de resisten ias
L = [Ld 0 Ldf;0 Lq 0;Ldf 0 Lf;
// matriz de indu tan ias
G = w*[0,-Lq,0;Ld,0,Ldf;0,0,0;
// matriz de genera in
Lin = inv(L);
// inversa de L
A =-Lin*(R+G);Ain = inv(A);
// matriz de transi in de estado y su inversa
[V,g = eig(A);
// autove tores y autovalores de A
io = [0;0;1;
// ondi iones ini iales prefalla
t = 0:.1:160;
// tiempo de la solu in en pu
e1=exp(g(1,1)*t);
// de aimiento del 1er autovalor
e2=exp(g(2,2)*t);
// de aimiento del 2do autovalor
e3=exp(g(3,3)*t);
// de aimiento del 3er autovalor
ip =-Ain*Lin*[0;0;rf*1;
// solu in parti ular p[i=0
ih = io-ip;
// orrientes homogneas en t=0+
k=inv(V)*ih;
// oefi ientes homogneos
ke = diag(k,0)*[e1;e2;e3; i = V*ke;
// solu in homognea
for m=1:length(t);
// solu in total
i(1,m)=i(1,m)+ip(1);
// id
i(2,m)=i(2,m)+ip(2);
// iq
i(3,m)=i(3,m)+ip(3);
// if
end;
s f(0)
plot(t,i(1,:),t,i(2,:),t,i(3,:))
// grfi o de las orrientes id,iq e if
// transforma in inversa de Park
s f(1)
ia=(i(1,:).* os(w*t)-i(2,:).*sin(w*t)); // grfi o de las orrientes ia e if
plot(t,ia,t,i(3,:))

425

Captulo IX Rgimen transitorio de la mquina sincrnica

(a) Corrientes directa, cuadratura y de campo

(b) Corrientes de armadura y de campo

Figura 9.6 Corrientes de cortocircuito de la mquina sincrnica

426

IX.7 Constantes de tiempo en circuitos acoplados magnticamente


IX.7

C ONSTANTES

DE TIEMPO EN CIRCUITOS ACOPLADOS MAGNTICAMENTE

En la mquina sincrnica, el eje directo y la bobina del campo se encuentran acoplados magnticamente tal como se muestra en la figura 9.2. Las ecuaciones que representan el comportamiento
dinmico de los ejes directo y cuadratura son:

 


vd
Rd + Ld p
Ld f p
id
=
(9.86)
vf
Ld f p
Rf +Lf p
if
En el dominio de la transformada de Laplace se obtiene:

 

 


Vd (s)
Rd + Ld s
Ld f s
Id (s)
Ld Ld f
id (0+ )
=

V f (s)
Ld f s
Rf +Lf s
I f (s)
Ld f L f
i f (0+ )

(9.87)

Si en el instante inicial se cortocircuitan simultneamente ambos terminales del transformador,


en presencia de flujo atrapado en las bobinas, las corrientes en el dominio de la frecuencia
resultan ser:
"
#



Ld (R f + L f s) L2d f s
Ld f (R f + L f s) L f Ld f s
1
Id (s)
id (0+ )
=
(9.88)
I f (s)
i f (0+)
D Ld f Ld s + Ld f (Rd + Ld s) L2d f s + L f (Rd + Ld s)
Donde:
D = (Rd + Ld s)(R f + L f s) L2d f s2

(9.89)

Considerando que en la bobina d del transformador no circulaba corriente en el instante inicial,


la corriente por este circuito despus del cortocircuito es:
Id (s) =

Ld f R f i f (0+ )
(Rd + Ld s)(R f + L f s) L2d f s2

(9.90)

Las constantes de tiempo del circuito estn determinadas por el denominador de la ecuacin
9.90. Este denominador se puede expresar de la siguiente forma:


1
1
2
2
D = Ld f (1 + d )(1 + f )(
(9.91)
+ s) s
+ s)(
d0
f0
Donde:

d0 =

Lf
L f
Ld
L d
; f0 =
; d =
; f =
Re
Rf
Ld f
Ld f

(9.92)

Cuando el acoplamiento es perfecto, los coeficientes de dispersin d y q son cero, en este caso
el denominador 9.91 se reduce a:


1
1
1
1
1
1
2
)s +
= L2d f (
)(s +
)
(9.93)
D = Ld f (
+
+
d0 f 0
d0 f 0
d0 f 0
d0 + f 0
Si el acoplamiento es perfecto, el sistema tiene una sola constante de tiempo que es igual a la
suma de las constantes de tiempo de cada una de las bobinas. En cambio, cuando el acoplamiento
es muy dbil, los coeficientes de dispersin son muy grandes y el denominador 9.90 se reduce

427

Captulo IX Rgimen transitorio de la mquina sincrnica


aproximadamente a:
D = L2d f (1 + d )(1 + f )(s +

1
1
)(s +
)
d0
f0

(9.94)

Las constantes de tiempo son idnticas en este caso a cada una de las constantes de tiempo de
los circuitos propios en vaco. Cuando el circuito magntico est fuertemente desacoplado, las
bobinas actan independientemente una de la otra.
En un circuito magntico ideal, el acoplamiento es perfecto. Aun cuando fsicamente esto no es
posible, en la prctica el acoplamiento puede llegar a ser prcticamente perfecto. Los enlaces de
flujo no pueden cambiar en un tiempo cero sin un consumo infinito de potencia, por esta razn
se cumple siempre en cualquier caso:

 


 

 
d (0 )
d (0+ )
Ld Ld f
id (0 )
Ld Ld f
id (0+)
=
(9.95)
=
=
f (0 )
f (0+ )
Ld f L f
i f (0 )
Ld f L f
i f (0+)
En la condicin ideal de acoplamiento perfecto, el determinante de la matriz de inductancias es
cero:
(9.96)
Ld L f L2d f = L2d f (1 + d )(1 + f ) L2d f 0, si d q 0
Una de las dos ecuaciones del sistema 9.95 es linealmente dependiente de la otra. En presencia de acoplamiento perfecto, las corrientes entre el instante t = 0 y t = 0+ no se mantienen
necesariamente constantes. En cambio, los enlaces de flujo s tienen que permanecer constantes
entre estos dos instantes. Por esta razn, siempre se cumple que:
Ld id (0 ) + Ld f i f (0) = Ld id (0+) + Ld f i f (0+ )

(9.97)

Con cualquier otro acoplamiento, se mantienen constantes las corrientes entre los instantes anterior y posterior al cortocircuito:
id (0) = id (0+)
i f (0) = i f (0+ )

IX.8

A NLISIS

(9.98)
(9.99)

TRANSITORIO APROXIMADO

Los trminos de transformacin en el modelo en coordenadas dq f de la mquina sincrnica son


despreciables en comparacin con los trminos de generacin3 . En el campo todas las fuerzas
electromotrices son de transformacin, y por esta razn no es posible despreciar ningn trmino en la ecuacin correspondiente a esta bobina. Las ecuaciones de la mquina sincrnica, al
despreciar los trminos de transformacin asociados con el eje directo y cuadratura, se pueden
representar de la siguiente forma:

vd
id
Re Xq
0
vq = Xd
Ld f iq
Re
(9.100)
Ld f p 0 R f + L f p
vf
if
3

Recuerde que en el sistema de coordenadas dq f las corrientes y tensiones de rgimen permanente son constantes.

428

IX.8 Anlisis transitorio aproximado

Figura 9.7 Diagrama fasorial aproximado de la fuerza electromotriz transitoria

Durante los primeros instantes del perodo transitorio, el enlace de flujo de la bobina de campo f se mantiene prcticamente constante. Las corrientes id e i f deben variar para mantener
constante este enlace de flujo. Si se considera que el enlace de flujo se mantiene mediante una
corriente equivalente que circula por la bobina de campo, se puede evaluar la fuerza electromotriz que este enlace produce sobre el eje cuadratura:

e f = Ld f i f equi = Ld f
= Ld f i f +

L2d f
Lf

L f i f + Ld f id
f
= Ld f
=
Lf
Lf

id = e f + (Xd Xd )id

(9.101)

La fuerza electromotriz transitoria e f permanece constante mientras que no vare el enlace de


flujo del campo f . La ecuacin 9.101 determina el valor de esta fuerza electromotriz. En con
vencin generador y en representacin fasorial, la fuerza electromotriz transitoria E f es:

E f = E f j(Xd Xd )Id = Ve + jXd Id + jXq Iq

(9.102)

La expresin 9.102 determina la fuerza electromotriz transitoria, que permanece prcticamente


constante mientras no decae el enlace de flujo del campo. Para evaluar la fuerza electromotriz
transitoria es necesario determinar las corrientes Id e Iq de rgimen permanente, antes de que
ocurra la perturbacin. Con las corrientes de rgimen permanente se construye el diagrama

fasorial utilizando como parmetro la reactancia transitoria del eje directo Xd , en lugar de la
reactancia de rgimen permanente Xd . En el eje cuadratura no existe acoplamiento magntico
con la bobina de campo, y la reactancia de rgimen permanente de este eje contina siendo Xq .
En la figura 9.7 se muestra el diagrama fasorial del procedimiento descrito.
El diagrama fasorial de la figura 9.7 representa la fuerza electromotriz transitoria que se mantiene constante durante los primeros instantes posteriores a la perturbacin, evaluada a partir de
las condiciones de operacin previas. El decaimiento de estas condiciones est determinado por

429

Captulo IX Rgimen transitorio de la mquina sincrnica


la constante de tiempo transitoria del campo:

f =

Lf
Rf

Lf

L2d f
Ld

= d0

Rf

L2d f

(9.103)

R f Ld

Si la condicin de operacin una vez ocurrida la perturbacin es una variacin sbita de la


tensin de armadura, tal como sucede en el caso del cortocircuito brusco, se puede recalcular
el diagrama fasorial transitorio aproximado utilizando la nueva tensin de armadura, la fuerza
electromotriz transitoria, la reactancia transitoria del eje directo y la reactancia del eje cuadratura. Las corrientes resultantes de este anlisis son aproximadamente las que se obtienen en los
primeros instantes del transitorio para la componente de frecuencia fundamental de la solucin.
Durante el perodo de validez de esta aproximacin, la potencia elctrica media suministrada
por la mquina a la red se determina aproximadamente mediante la siguiente expresin:
#
"

Ve E f
Ve2 1
1
(9.104)
Pe = sin +
sin 2

2 Xq Xd
Xd
Utilizando este mtodo para evaluar las corrientes de cortocircuito franco de la mquina sincrnica, se determina en primer lugar la fuerza electromotriz transitoria con las condiciones de
operacin previas a la perturbacin:

E f = Ve + jXq Ie + j(Xd Xd )Id

(9.105)

Posteriormente se resuelve el diagrama fasorial con las nuevas condiciones impuestas por la
perturbacin:

E f = Ve + jXd Id + jXq Iq = Ve + jXq Ie + j(Xd Xq )Iq


(9.106)

Durante el cortocircuito, la tensin de armadura Ve es cero. La corriente se encuentra retrasada

90 con respecto a la fuerza electromotriz E f , cuando se considera que la impedancia interna


de la mquina es completamente inductiva, al despreciar las resistencias de los devanados. La
corriente transitoria de cortocircuito se calcula de la siguiente forma:

E f = Ve + jXq Id + j(Xd Xq )Iq = jXd Id Id =

IX.9

P EQUEAS

Ef

jXd

(9.107)

OSCILACIONES DE LA MQUINA SINCRNICA

En los anlisis transitorios electromagnticos realizados en las secciones precedentes se considera que la velocidad mecnica del rotor de la mquina permanece prcticamente constante durante
todo el tiempo que dura el proceso. Esta aproximacin se justifica por la diferencia existente entre las constantes de tiempo del circuito electromagntico y del sistema mecnico. Cuando se
considera que la velocidad de la mquina permanece constante, se simplifica notablemente el
problema porque el conjunto de ecuaciones diferenciales que definen su comportamiento es lineal y puede ser resuelto mediante tcnicas analticas. Sin embargo, la velocidad de la mquina
depende del equilibrio existente entre el par elctrico y el par mecnico. Este equilibrio se rompe

430

IX.9 Pequeas oscilaciones de la mquina sincrnica


frecuentemente durante la operacin normal de la mquina, por variaciones del par mecnico de
accionamiento, perturbaciones de la red o cambios en la excitatriz de la mquina. Muchas de
estas perturbaciones o variaciones son de pequea magnitud y aun cuando el modelo que representa el comportamiento de la mquina es no lineal y de gran complejidad, es posible simplificar
el problema, linealizando las ecuaciones en el entorno de un determinado punto de operacin. De
esta forma se analizan las oscilaciones naturales de la mquina sincrnica sometida a pequeas
perturbaciones de sus condiciones de operacin.
La ecuacin diferencial que representa matemticamente la dinmica del sistema mecnico es
la siguiente:
d2
d
d m
J 2 +
(9.108)
=J
+ m = Tm Te
dt
dt
dt
Donde:
J

es la constante de inercia de las masas rotantes, y

es el coeficiente de friccin.

Multiplicando la expresin anterior por la velocidad mecnica y dividiendo por la potencia aparente base de la mquina, se obtiene el balance de potencias en el sistema adimensional de
unidades:
J m d m m
m = Pm (pu) Pe (pu)
(9.109)
+
SB dt
SB
Multiplicando y dividiendo el primer miembro de la ecuacin 9.109 por la velocidad base B ,
se obtiene la siguiente expresin:
J m B d m (pu) m B
m (pu) = Pm (pu) Pe(pu)
+
SB
dt
SB

(9.110)

La velocidad mecnica del rotor permanece en todo momento cercana a la velocidad sincrnica
m B , que es la base de velocidad del sistema adimensional de unidades utilizado. Por esta
razn, la ecuacin 9.110 se puede expresar en forma aproximada como:
J B2 d m (pu) B2
+
m (pu) = Pm (pu) Pe (pu)
SB
dt
SB

(9.111)

Definiendo la constante de inercia H en el sistema adimensional de unidades como el cociente


entre la energa cintica Wk acumulada en la masa rotante de la mquina girando a la velocidad
sincrnica B y la potencia base SB :
1
J 2
Wk (B )
H=
= 2 B
SB
SB

(9.112)

la ecuacin 9.111 se expresar como:


2H

d m (pu)
+ Pp (pu)m (pu) = Pm (pu) Pe(pu)
dt

(9.113)

Donde:

431

Captulo IX Rgimen transitorio de la mquina sincrnica


Pp(pu)

representa las prdidas mecnicas a la velocidad sincrnica en por unidad de la potencia base.

El ngulo de carga est definido por las posiciones respectivas del eje magntico de la pieza
polar y de la amplitud del campo magntico rotatorio producido por las bobinas del estator. El
primero gira a la velocidad mecnica y el segundo a la velocidad sincrnica. La variacin del
ngulo de carga est determinado por la diferencia entre estas dos velocidades:


d
m s
= B [m (pu) 1]
= m s = B

(9.114)
dt
B B

m (pu) =

1 d
+1
B dt

(9.115)

1 d2
d m
=
dt
B dt 2

(9.116)

Reemplazando las expresiones 9.115 y 9.116 en la ecuacin 9.113, se obtiene:


2H d 2 Pp d
+
= Pm Pe Pp
B dt 2 B dt

(9.117)

En rgimen permanente la velocidad es constante, por tanto las derivadas primera y segunda del
ngulo de carga con respecto al tiempo son nulas, con lo que la expresin 9.117 es:
Pm Pe Pp = 0

(9.118)

La expresin 9.118 determina el punto de equilibrio en rgimen permanente y la ecuacin diferencial 9.117 representa la dinmica del sistema mecnico. Esta ecuacin diferencial no es lineal
debido a que la potencia elctrica entregada por la mquina sincrnica tiene una dependencia no
lineal con el ngulo de carga. Sin embargo, cuando la variacin del ngulo de carga es pequea,
se puede linealizar la ecuacin diferencial en el entorno del punto de equilibrio, obtenindose
las siguientes relaciones:
= 0 +
(9.119)
d 0 d
d
d
=
+
=
dt
dt
dt
dt


2
2
d
d d
d
=
=
2
dt
dt dt
dt 2

(9.120)
(9.121)

La ecuacin de la potencia elctrica, que linealiza en el entorno del punto de equilibrio es:
"
#

Ve E f
1
Ve2 1

Pe =
sin 2(0 + )
sin(0 + ) +
2 Xq Xd
Xd

432

"

Ve E f

Xd

sin 0 +

Ve2
2

"

1
1
sin 20 +
Xq Xd

"

Ve E f

Xd

cos 0 +Ve2

"

1
1
cos 2
Xq Xd

IX.9 Pequeas oscilaciones de la mquina sincrnica


Pe (0 ) +

Pe (0 )
Pe (0 + ) Pe ( )

(9.122)

La expresin anterior aproxima la potencia elctrica mediante una expansin en series de Taylor
centrada en el punto de equilibrio de potencias y truncada a partir de los trminos de segundo
orden. El primer trmino de la ecuacin 9.122 representa la potencia elctrica en el punto de
equilibrio, y el segundo trmino se define como el incremento de la potencia elctrica con el
ngulo de carga en el entorno del punto de equilibrio, conocido como potencia sincronizante de
la mquina sincrnica.
Reemplazando la expresiones 9.118, 9.120, 9.121 y 9.122 en la ecuacin 9.117, se obtiene el
siguiente resultado:
2H d 2 Pp d Pe (0 )
+
+
= Pm Pe Pp = 0
B dt 2
B dt

(9.123)

Esta expresin es una ecuacin diferencial lineal y homognea de segundo grado. Aplicando la
transformada de Laplace a la ecuacin anterior, se obtiene:


2H d (0+ ) Pp
Pe (0 )
2H 2 Pp
(9.124)
s +
s+
+
(0+ )
(s) =
B
B

B
dt
B
Considerando que la variacin del incremento del ngulo con respecto al tiempo es despreciable en el instante inmediatamente posterior a la perturbacin, se obtiene la siguiente ecuacin
algebraica:
Pp
+
2H (0 )
(s) =
(9.125)
Pp
Pe (0 )
2H 2
s
+
s
+
B
B

Los polos del polinomio del denominador de la expresin anterior determinan las frecuencias
naturales y los respectivos amortiguamientos de las oscilaciones de la mquina sincrnica sometida a pequeas perturbaciones:
q
Pp Pp2 8H B Pe (0 )
s1,2 =
(9.126)
4H
Como las prdidas mecnicas son muy pequeas, pueden ser despreciadas en esta ltima expresin, obtenindose de esta forma la frecuencia natural de oscilacin de la mquina sincrnica en
el entorno de un determinado punto de equilibrio:
s
B Pe (0 )
= jnat
(9.127)
s1,2 = j
2H
Debido a que las prdidas de la mquina son despreciables, la mquina sincrnica oscila alrededor del punto de equilibrio permanentemente o con un decaimiento exponencial muy lento, tal
como ocurre en las oscilaciones de un pndulo fsico alrededor de su punto de equilibrio. Para
estabilizar el punto de operacin de la mquina es necesario incrementar el amortiguamiento
del sistema, evitando aumentar las prdidas mecnicas en el punto de operacin. Esto se puede
lograr incluyendo un enrollado amortiguador en el rotor de la mquina sincrnica semejante al

433

Captulo IX Rgimen transitorio de la mquina sincrnica

(a) Lugar de las races

(b) Respuesta temporal

Figura 9.8 Lugar de las races y respuesta temporal de las pequeas oscilaciones
devanado de jaula de ardilla de la mquina de induccin. Cuando la mquina opera a la velocidad
sincrnica, no circula corriente por el devanado amortiguador. Si vara la velocidad mecnica de
la mquina, el deslizamiento entre el campo y el rotor, fuerza la circulacin de corrientes por
las barras del devanado amortiguador y se produce un par elctrico contrario a la direccin de la
diferencia de velocidades.
Si las perturbaciones en el ngulo de carga son pequeas, el deslizamiento tambin tiene una
magnitud reducida. El par de induccin de un devanado amortiguador de jaula de ardilla es
lineal con respecto al deslizamiento cuando ste es pequeo. Como la velocidad es prcticamente
constante en estas condiciones, la potencia de induccin se puede expresar aproximadamente de
la siguiente forma:


m
d
s
s
Pind = Tind m Tn B = Pnind = Ds = D 1
= D
(9.128)
sn
sn
B
dt
Si se incluye la potencia de induccin en la ecuacin diferencial 9.123, se obtiene:
2H d 2 Pp + D d Pe (0 )
+
+
= 0
B dt 2
B dt

(9.129)

La incorporacin del devanado amortiguador modifica los modos de oscilacin de la mquina:


q
P ( )
(Pp + D) (Pp + D)2 8H B e 0
(9.130)
s1,2 =
4H

Cuando se disea el devanado amortiguador de la mquina sincrnica es posible obtener una


caracterstica de la jaula de ardilla que produzca un par de induccin lo suficientemente elevado
como para permitir la anulacin del trmino radical de la expresin 9.130. En estas condiciones
el amortiguamiento es crtico y la respuesta dinmica de la mquina ante pequeas perturbaciones no es oscilatoria. Aun cuando no sea posible obtener una respuesta crtica amortiguada, un

434

IX.10 Efecto del devanado amortiguador

(a) Disposicin fsica de las bobinas

(b) Modelo en coordenadas ortogonales

Figura 9.9 Representacin de los devanados amortiguadores de la mquina sincrnica


amortiguamiento tpico de 20 en por unidad reduce rpidamente las oscilaciones, tal como se
observa en la figura 9.8.
IX.10

E FECTO

DEL DEVANADO AMORTIGUADOR

Al incluir un devanado amortiguador en el rotor de la mquina sincrnica, aumenta la reluctancia del camino principal y se reduce la reactancia del eje directo. La fuerza electromotriz
del campo se debilita por la misma razn. El devanado amortiguado obliga a incrementar las
corrientes de campo para compensar la disminucin de la permeabilidad del circuito magntico.
Por otra parte, este devanado evita en cierta medida que los campos producidos en el estator
corten a los conductores del campo cuando la velocidad del rotor es diferente a la velocidad
sincrnica. Si el flujo principal corta a los conductores de la bobina de campo, pueden inducirse
fuerzas electromotrices tan intensas como para alcanzar la ruptura dielctrica del aislamiento.
La jaula de ardilla o devanado amortiguador, adems de mejorar la respuesta dinmica propia de
la mquina, protege la integridad del aislamiento de la bobina de excitacin, blindando el corte
de estos conductores por el flujo principal.
El devanado amortiguador de la mquina sincrnica puede modelarse mediante dos bobinas
ortogonales cortocircuitadas, una en la direccin del eje directo y la otra en la direccin del
eje cuadratura. De esta forma aparecen nuevos acoplamientos entre las bobinas de la mquina
y los devanados amortiguadores. En la figura 9.9 se presenta un diagrama esquemtico de esta
situacin.
En la direccin del eje directo existe acoplamiento magntico entre el campo, la bobina de la
armadura y el devanado amortiguador ad. En el eje cuadratura, existe acoplamiento entre la
bobina q de armadura y el devanado amortiguador aq. En el eje directo las bobinas se pueden
representar como un transformador de tres devanados y en el eje cuadratura como un transformador de dos enrollados. En la figura 9.11 se representa el circuito equivalente de los cinco
devanados y sus respectivos acoplamientos de transformacin y generacin.

435

Captulo IX Rgimen transitorio de la mquina sincrnica

Figura 9.10 Circuito equivalente de la mquina sincrnica con devanado amortiguador

Las ecuaciones diferenciales de la mquina sincrnica con devanados amortiguadores son:


Ld f p
Ldad p Ldaq
id
vd
Re + Ld p Lq

vq Ld
Re + Lq p
Ld f
Lqad
Lqaq p
iq

v f = Ld f p
0
R f + L f p L f ad p
0
i f (9.131)


iad
vad Ldad p
0
Lad f p Ra + Lad p
0
0
Laqq p
0
0
Ra + Laq p
iaq
vaq

Los devanados amortiguadores se encuentran muy cerca de las bobinas del estator y el acoplamiento entre estos circuitos es muy alto. Durante los primeros instantes de la perturbacin, los
amortiguadores mantienen los enlaces de flujo previos y se reflejan sobre la armadura produciendo las reactancias subtransitorias del eje directo y del eje cuadratura:
L2dad
Lad

(9.132)

L2qaq
Lq = Lq
Laq

(9.133)

Ld = Ld

Las inductancias subtransitorias del eje directo y cuadratura se obtienen con una metodologa
similar a la utilizada para determinar la inductancia transitoria del eje directo. La inductancia
transitoria se define cuando el campo se refleja sobre el eje directo, en las reactancias subtransitorias los devanados amortiguadores se reflejan sobre sus respectivos ejes de acoplamiento
magntico. Como el acoplamiento mutuo entre los amortiguadores y la armadura es muy alto,

las inductancias subtransitorias Ld y Lq , son menores que la inductancia transitoria Ld . En el


436

IX.11 Anlisis subtransitorio aproximado


Tabla 9.2 Rango de los valores de las reactancias subtransitorias

Xd
Xq
0,12 0,18 pu 0,10 0,15 pu
sistema adimensional de unidades estas reactancias se encuentran normalmente en el rango que
se muestra en la tabla 9.2.
En el cdigo 13 se reproduce el listado y los resultados intermedios de un algoritmo desarrollado en el entorno de programacin SCILAB, que permite evaluar el comportamiento de las
corrientes en coordenadas transformadas y primitivas de una mquina sincrnica con devanados
amortiguadores, sometida a un cortocircuito brusco.
IX.11

A NLISIS

SUBTRANSITORIO APROXIMADO

En la seccin IX.8 se desarroll un mtodo aproximado para la evaluacin de la corriente transitoria de la mquina sincrnica de polos salientes sin devanados amortiguadores. Esta aproximacin consiste en suponer que los enlaces de flujo del devanado de campo se mantienen prcticamente constantes durante un cierto tiempo y que los trminos de transformacin asociados a
las bobinas de armadura son despreciables. Si la mquina posee devanados amortiguadores, es
posible considerar una hiptesis similar con respecto a los enlaces de flujo existentes en estas
bobinas. Conservndose prcticamente constantes los enlaces de flujo en los devanados amortiguadores, las fuerzas electromotrices generadas por estos enlaces de flujos tambin se mantienen

constantes. Con esta hiptesis se pueden definir dos fuerzas electromotrices subtransitorias, ed

y eq :
Laq iaq + Lqaq iq
aq

= Ldaq
=
ed = Ldaq iaqequ = Ldaq
Laq
Laq
= Laq iaq

L2qaq
Ldaq Lqaq

iq =
iq = (Lq Lq )iq
Laq
Laq

eq = Lqad iadequ = Lqad

(9.134)

Lad iad + Ldad id + L f ad i f


ad
= Lqad
=
Lad
Lad

L2dad
Lqad Ldad
Lqad L f ad

=
id +
if =
id + Ld f i f = (Ld Ld )id + e f (9.135)
Lad
Lad
Lad
Representando fasorialmente las relaciones 9.134 y 9.135 en la convencin generador se obtiene:

Ed = Ed = j(Xq Xq ) Iq

(9.136)

Eq = jEq = j(Xd Xq )Id + E f = j(Xd Xq )Id + Ve + jXd Id + jXq Iq

E f = Ed + Eq = Ve + jXd Id + jXq Iq

(9.137)
(9.138)

La fuerza electromotriz subtransitoria E f , se mantiene prcticamente constante durante una perturbacin, mientras que los enlaces de flujo de las bobinas amortiguadoras no decaen. La fuerza

electromotriz E f se determina a partir de la condicin de rgimen permanente previa a la per437

Captulo IX Rgimen transitorio de la mquina sincrnica

Algoritmo 13 Anlisis transitorio de la mquina sincrnica con devanados amortiguadores


// ANLISIS TRANSITORIO DE MQUINAS SINCRNICAS CON DEVANADOS AMORTIGUADORES
// p[i=-[L^-1*([R+w*[G)+[L^-1*[v
// [A =-[L^-1*([R+w*[G)
w = 1.0
// velo idad sin rni a w = 1
rd=0.01;rq=0.01;rad=0.03;raq=0.03;rf=0.01;
R = diag([rd rq rad raq rf,0);
// matriz de resisten ias
Ld =1.0;Lq =0.6;Lad=0.90;Laq=0.50;Lf=1.0;
Ldad=0.85;Lqaq=0.40;Ldaq=0.40;Lqad=0.80;Lfad=0.70;Ldf= 0.8;
// matriz de indu tan ias
L = [Ld,0,Ldad,0,Ldf;0,Lq,0,Lqaq,0;Ldad,0,Lad,0,Lfad;0,Lqaq,0,Laq,0;Ldf,0,Lfad,0,Lf;
// matriz de genera in
G = w*[0,-Lq,0,-Ldaq,0;Ld,0,Lqad,0,Ldf;0,0,0,0,0;0,0,0,0,0;0,0,0,0,0;
Lin = inv(L);A =-Lin*(R+G);Ain = inv(A);
// matriz de transi in de estado
[V,g = spe (A);
// autovalores y autove tores
io = [0;0;0;0;1;
// ondi iones ini iales
t = 0:.25:160;
// tiempo (tbase 1/377 s)
e1=exp(g(1,1)*t);e2=exp(g(2,2)*t);e3=exp(g(3,3)*t);e4=exp(g(4,4)*t);e5=exp(g(5,5)*t);
ip =-Ain*Lin*[0;0;0;0;rf*1;
// solu in parti ular p[i=0
ih = io-ip;
// ondi iones homogneas
k= inv(V)*ih;
// oefi ientes homogneos
ke = diag(k,0)*[e1;e2;e3;e4;e5; i = V*ke;
// solu in homognea
for m=1:length(t);
// solu in ompleta
i(1,m)=i(1,m)+ip(1);
// id(t)
i(2,m)=i(2,m)+ip(2);
// iq(t)
i(3,m)=i(3,m)+ip(3);
// ida(t)
i(4,m)=i(4,m)+ip(4);
// iqa(t)
i(5,m)=i(5,m)+ip(5);
// if(t)
end;
s f(0)
plot(t,i(1,:),t,i(2,:),t,i(5,:))
// Figura 9.11(a)
xgrid
s f(1)
plot(t,i(3,:),t,i(4,:))
// Figura 9.11(b)
xgrid
s f(2)
ia = (i(1,:).* os(w*t)-i(2,:).*sin(w*t));
// Transforma in de Park
plot(t,ia,t,i(5,:))
// Figura 9.11( )
xgrid

438

IX.11 Anlisis subtransitorio aproximado

(a) Corrientes directa, cuadratura y de campo

(b) Corrientes de los devanados amortiguadores d y q

(c) Corrientes de campo y de armadura

Figura 9.11 Respuesta transitoria de la mquina sincrnica con devanados amortiguadores ante
un cortocircuito brusco

439

Captulo IX Rgimen transitorio de la mquina sincrnica

Figura 9.12 Fuerza electromotriz subtransitoria E f en el diagrama fasorial


turbacin, construyendo un diagrama fasorial con las impedancias subtransitorias, tal como se
observa en la figura 9.12. A diferencia del caso transitorio, la fuerza electromotriz subtransitoria no est orientada segn el eje cuadratura. Esto se debe a que el enlace de flujo atrapado en
el devanado amortiguador del eje cuadratura induce fuerza electromotriz en el eje directo durante el subtransitorio. Durante el perodo transitorio o en mquinas sincrnicas sin devanados
amortiguadores, el nico enlace atrapado es el de la bobina de campo y slo se induce fuerza
electromotriz en el eje cuadratura tal como se muestra en el diagrama fasorial de la figura 9.7.
La relacin entre la inductancia de los devanados de armadura y su respectiva resistencia es
menor que en el caso de la bobina de campo. Por este motivo, los enlaces de flujo de los devanados amortiguadores decaen ms rpidamente que el enlace de flujo de la bobina de campo.
El proceso subtransitorio desaparece durante los primeros ciclos de la perturbacin y el proceso
transitorio se mantiene por un tiempo ms largo.
El anlisis transitorio aproximado permite disear el sistema de protecciones trmicas de los
devanados y simplificar los anlisis de estabilidad de la mquina sincrnica. Mediante los anlisis subtransitorios aproximados se pueden evaluar los esfuerzos mecnicos originados por las
corrientes en las barras y definir los fusibles limitadores de corriente adecuados para reducir el
nivel de cortocircuito en los bornes de la mquina. El perodo subtransitorio decae muy rpidamente como para tener influencia en las variaciones de velocidad de la mquina sincrnica, por
esta razn en los estudios de estabilidad es una prctica habitual utilizar las fuerzas electromotrices e inductancias transitorias para evaluar los intercambios de potencia elctrica media entre
la masa rotante y el sistema de potencia.
IX.12

D ETERMINACIN

DE LAS INDUCTANCIAS TRANSITORIAS Y

SUBTRANSITORIAS

Durante el cortocircuito brusco de la mquina sincrnica, los enlaces de flujo atrapados en las
bobinas son mantenidos inicialmente por los devanados amortiguadores y por el enrollado de
campo de la mquina, cuando decae el enlace de los devanados amortiguadores el campo contina manteniendo parte de los enlaces de flujo. En la figura 9.13 se ha representado el oscilograma
de la corriente de armadura durante un cortocircuito brusco de la mquina sincrnica en funcin

440

IX.12 Determinacin de las inductancias transitorias y subtransitorias

Figura 9.13 Oscilograma del cortocircuito brusco de la mquina sincrnica


del tiempo. Dependiendo de la posicin angular 0 del rotor en el instante cuando se inicia el
cortocircuito, aparece una componente de corriente continua que decae exponencialmente a medida que la mquina disipa la energa acumulada en los enlaces de flujo atrapados en el instante
inicial de la perturbacin. En el oscilograma se han indicado la envolvente superior, la inferior
y la componente de corriente continua que produce la forma asimtrica del registro.
Para determinar la inductancias y constantes de tiempo transitorias y subtransitorias de la mquina se elimina la componente continua transitoria del oscilograma anterior, restando de la
corriente de armadura total, la semisuma de las envolventes superior e inferior. En la figura 9.14
se presenta el resultado obtenido al realizar esta operacin. Con el grfico simtrico o con el
oscilograma original, se consigue directamente el valor de la corriente de rgimen permanente
que permite evaluar la reactancia del eje directo Xd .
Cuando se elimina el trmino de rgimen permanente y la componente de corriente continua,
toda la informacin necesaria sobre las componentes transitorias y subtransitorias de la corriente de armadura se encuentra en las envolventes del oscilograma. Como las dos constantes de
tiempo transitorias y subtransitorias son diferentes, es posible la separacin de estas dos componentes. La envolvente resultante despus de eliminar las componentes de corriente continua y
de rgimen permanente tiene la siguiente representacin funcional:

i + i = k1 e

+ k2 e

(9.139)

El proceso subtransitorio decae muy rpidamente, por esta razn para un tiempo superior a varias

constantes de tiempo subtransitorio , la envolvente tiende asintticamente hacia una funcin

exponencial que decae con la constante de tiempo transitoria . Si se representa la envolvente en


papel semilogartmico, o se calcula el logaritmo de la expresin 9.139, se obtiene una funcin

441

Captulo IX Rgimen transitorio de la mquina sincrnica

Figura 9.14 Representacin simtrica de la corriente de armadura

cuya asntota es una lnea recta con una pendiente igual al inverso de la constante de tiempo

transitoria :

t
ln(i + i ) ln k2 ; si t >>
(9.140)

La expresin anterior permite identificar entre las envolventes del oscilograma, la constante de

tiempo transitoria y la constante k2 . Restando de la envolvente, la contribucin transitoria,


se obtiene una nueva funcin que depende exclusivamente de la constante de tiempo subtransi
toria . Calculando el logaritmo neperiano de esta ltima funcin se determina el valor de la
constante de tiempo subtransitoria:
h
i
h t i
t
t

(9.141)
ln (i + i ) k2 e
= ln k1 e = ln k1

En la figura 9.15 se representa en papel semilogartmico la envolvente (i +i ) de donde se deter


mina la constante k2 y la constante de tiempo de la asntota lineal de esta curva. Restando esta

componente transitoria de la envolvente se obtiene la funcin i , cuya representacin logartmica


es la recta punteada del diagrama. La pendiente de esta recta determina la constante de tiempo

subtransitoria . Una vez obtenidas las constantes de tiempo transitorias y subtransitorias se


determinan las respectivas inductancias transitorias y subtransitorias:

= d Ld = Re
Re

= d Ld = Re
Re

442

(9.142)

(9.143)

IX.13 Rgimen desequilibrado de la mquina sincrnica

Figura 9.15 Separacin de las constantes de tiempo transitoria y subtransitoria

IX.13

R GIMEN

DESEQUILIBRADO DE LA MQUINA SINCRNICA

Los desequilibrios de los sistemas elctricos de potencia y de las mquinas elctricas se pueden
analizar mediante la transformacin a componentes simtricas. Para este fin es necesario determinar los modelos de la mquina sincrnica de polos salientes en secuencia positiva, negativa y
cero.
El modelo de rgimen permanente de la mquina sincrnica, analizado en el captulo VIII, se
obtiene al excitar la armadura de la mquina mediante un sistema balanceado de corrientes de
secuencia positiva. Por tanto, el diagrama fasorial y sus ecuaciones asociadas modelan la red
de secuencia positiva de la mquina sincrnica de polos salientes. En anlisis aproximados, se
pueden despreciar las diferencias existentes entre la reluctancia del eje directo y cuadratura del
convertidor, simplificando el modelo de secuencia positiva de la mquina, a una fuerza electromotriz E f conectada en serie con la reactancia del eje directo Xd .
El circuito equivalente de secuencia cero est constituido por la reactancia de dispersin de los
devanados del estator X e . Si se alimenta la armadura con un sistema de corrientes de secuencia
cero, la fuerza magnetomotriz resultante en el entrehierro de una mquina simtrica es nula. Por
este motivo, el modelo de secuencia cero de la mquina no tiene fuentes de fuerza electromotriz.

443

Captulo IX Rgimen transitorio de la mquina sincrnica


El modelo de secuencia negativa se obtiene excitando los devanados de armadura de la mquina
con un sistema balanceado de corrientes de secuencia negativa:

2Ie cos t
ia (t) =

2
ib (t) =
(9.144)
2Ie cos( t )
3

4
2Ie cos( t )
ic (t) =
3
Aplicando la transformacin de Park 8.46 a estas corrientes, se obtiene el siguiente resultado:

cos( t 23 )
cos t
cos( t 43 )
id
ia
2
4
iq = 2
sin t sin( t 3 ) sin( t 3 ) ib =
3
1
1
1
i0
ic
2
2
2
=

cos 2 t
3Ie sin 2 t
0

(9.145)

Despreciando las resistencias de las bobinas y reemplazando el resultado anteriores en las ecuaciones diferenciales de la mquina sincrnica 8.54, se obtiene:
vd = Ld

di f
di f
did
Lq iq + Ld f
= 3Ie (2Ld Lq ) sin 2 t + Ld f
dt
dt
dt

(9.146)

diq
+ Ld f i f = 3Ie (Ld 2Lq ) cos 2 t + Ld f i f
(9.147)
dt
di f
did
d
+ Ld f
= (L f i f + Ld f id )
(9.148)
vf = Lf
dt
dt
dt
Determinando de la ecuacin 9.148, la corriente de campo i f en funcin de la corriente del eje
directo id , e introduciendo este resultado en las expresiones 9.146 y 9.147, se obtiene:
vq = Ld id + Lq

if =

Ld f
Ld f
vf
vf
id + t =
( 3Ie cos 2 t) + t
Lf
Lf
Lf
Lf

Ld f

vf
vd = 3Ie (2Xd Xq ) sin 2 t +
Lf

Ld f

vq = 3Ie (2Xq + Xd ) cos 2 t +


v f t
Lf

(9.149)

(9.150)
(9.151)

Aplicando la transformacin inversa de Park a las tensiones vd y vq determinadas en las expresiones 9.150 y 9.151:
r
2
va (t) =
(vd cos t vq sin t) =
3

Ld f
Xd + Xq

3
2Ie sin t + (Xq Xd ) 2Ie sin 3 t +
v f (cos t t sin t)
(9.152)
=
2
2
Lf

444

IX.14 Estabilidad de la mquina sincrnica


Despreciando el trmino de triple frecuencia en la expresin 9.152, se obtiene la impedancia de
secuencia negativa:

Xd + Xq
X =
(9.153)
2
En la expresin 9.153, el trmino dependiente de la tensin del campo v f , no aparece en la red
de secuencia negativa porque produce una fuerza electromotriz de secuencia positiva que se ha
representado previamente en su respectiva red de secuencia. Esto equivale a considerar que el
rotor de la mquina se encuentra en cortocircuito cuando se excita el estator con un sistema
trifsico balanceado de corrientes de secuencia negativa.
Las corrientes de secuencia negativa producen un campo magntico rotatorio que gira a velocidad sincrnica en sentido contrario a la referencia de giro del sistema rotrico. Alternativamente,
y con una frecuencia doble de la sincrnica, el campo magntico rotatorio de secuencia negativa
cruza la reluctancia del eje directo y la del eje cuadratura. Por este motivo, la impedancia de
secuencia negativa es el promedio de las impedancias ofrecidas por estos dos ejes. Como las
bobinas del eje directo y del campo estn acopladas magnticamente, cuando la amplitud de la
fuerza magnetomotriz de armadura est alineada con el eje magntico del rotor, se refleja en

bornes de la armadura, la reactancia transitoria del eje directo Xd . Si la mquina posee devanados amortiguadores, en el eje directo y en el eje cuadratura se reflejan alternativamente las

reactancias subtransitorias Xd y Xq , en este caso la impedancia de secuencia negativa es:

X + Xq
X = d
2

(9.154)

La componente de triple frecuencia obtenida en la expresin 9.154 se debe a que el campo magntico rotatorio de secuencia negativa corta a los conductores del rotor con dos veces la velocidad
sincrnica, originando fuerzas electromotrices y corrientes de doble frecuencia en las bobinas
del rotor. Estas corrientes variando al doble de la frecuencia sincrnica, producen un campo
magntico rotatorio, que visto desde el estator gira a tres veces la velocidad sincrnica. Esta
componente es dbil y no se considera normalmente en los estudios de cortocircuitos desequilibrados clsicos o convencionales. Si es necesaria una precisin mayor en el anlisis, se utiliza el
modelo transitorio completo de la mquina sincrnica en coordenadas dqo f , transformando
las condiciones del desequilibrio en coordenadas primitivas a coordenadas dq0 f mediante la
transformacin de Park y se resuelve posteriormente el sistema de ecuaciones diferenciales con
estas condiciones de contorno impuestas.
IX.14

E STABILIDAD

DE LA MQUINA SINCRNICA

En las secciones precedentes se analiz el comportamiento transitorio electromagntico de la


mquina sincrnica considerando que la velocidad mecnica del rotor es prcticamente constante durante un cierto tiempo. Tambin se plante el problema de las oscilaciones mecnicas
originadas por pequeas perturbaciones elctricas o mecnicas en los ejes del convertidor. En estos casos, las ecuaciones diferenciales que describen el comportamiento del sistema son lineales
y se puede obtener una solucin analtica mediante autovalores y autovectores, la transformada
de Laplace o mediante linealizaciones realizadas en el entorno de un punto de operacin. Sin
embargo, cuando se analiza el comportamiento del sistema electromecnico sometido a grandes

445

Captulo IX Rgimen transitorio de la mquina sincrnica


perturbaciones y desequilibrios, es necesario recurrir a tcnicas no lineales para la solucin de
estos problemas. La tcnica ms utilizada consiste en integrar las ecuaciones diferenciales no
lineales mediante mtodos numricos tales como Euler, Runge-Kutta o Predictor Corrector. En
algunas ocasiones resulta conveniente el empleo de mtodos analticos directos tales como los
criterios de energa o los mtodos de estabilidad de Liapunov, que aun cuando no predicen la
trayectoria temporal de las variables de estado del sistema, s predicen la estabilidad o inestabilidad de la solucin del sistema de ecuaciones diferenciales. El conocimiento preciso de los
lmites de estabilidad de un sistema es generalmente ms importante que la determinacin de
la trayectoria temporal de las variables de estado. Los mtodos directos o analticos simplifican
notablemente este problema.
En los anlisis simplificados de estabilidad de la mquina sincrnica, se considera que el convertidor se encuentra acoplado a una barra infinita. Esto significa que la barra mantiene constante
la tensin y la frecuencia independientemente de la potencia que se inyecta o se extrae de la
misma. En el anlisis electromecnico se puede contemplar la dinmica del sistema mecnico
de regulacin de velocidad asociado con el eje del rotor y la dinmica de la excitatriz del campo
de la mquina. En los anlisis ms simplificados, se supone que la excitatriz es una fuente de
tensin o corriente constante, y que el gobernador de velocidad no es capaz de variar el caudal del fluido energtico durante el proceso dinmico para el caso de los generadores, o que
la carga se mantiene prcticamente constante si el convertidor motoriza un sistema mecnico.
Los anlisis electrodinmicos ms precisos integran simultneamente el conjunto completo de
ecuaciones diferenciales no lineales, incluyendo todos los ejes elctricos y mecnicos, as como
las ecuaciones adicionales introducidas por el gobernador, la excitatriz, el sistema de medidas y
los controladores asociados con la operacin de la mquina.
Cuando se sincroniza la mquina a la red, la fuerza electromotriz producida por el campo se
ajusta a un valor cercano a la tensin de la red, tanto en magnitud como en fase, con la finalidad de obtener una corriente prcticamente nula cuando se cierra el interruptor. Una vez que
la mquina ha sido sincronizada, es necesario incrementar la potencia en el eje para que el estator entregue potencia a la red. Si esto ocurre, manteniendo constante la corriente de campo,
la mquina se desmagnetiza, consumiendo potencia reactiva desde el sistema. En la figura 9.16
se presentan los diagramas fasoriales y la caracterstica potencia elctrica-ngulo de carga, de
una mquina sincrnica de rotor liso despus de su acoplamiento en vaco a la red, para dos
valores de potencia mecnica entregada en el eje. Si se aumenta bruscamente la potencia desde
Pm1 a Pm2 , el escaln de potencia se traduce en un escaln de par mecnico que acelera las masas
rotantes acopladas al eje de la mquina. El ngulo de carga crece desde el valor 1 hasta el valor
2 , correspondiente a la nueva condicin de equilibrio entre la potencia elctrica inyectada a la
red y la potencia mecnica absorbida por el eje. Cuando la mquina alcanza el ngulo de carga
2 , la velocidad del rotor es mayor que la velocidad sincrnica, el ngulo de carga contina creciendo, la mquina entrega ms potencia a la red que la recibida en su eje, por tanto el rotor se
frena. En este proceso la mquina alcanza la velocidad sincrnica y el ngulo de carga mximo,
pero la potencia elctrica entregada a la red es mayor que la potencia mecnica inyectada en el
eje mecnico, por tanto la mquina contina frenndose hasta alcanzar nuevamente el punto de
equilibrio de potencias. En este punto, la velocidad es menor que la velocidad sincrnica y el ngulo de carga sigue disminuyendo hasta que la velocidad del eje alcance nuevamente el valor de
sincronismo. En este momento el ngulo de carga se encuentra en su valor mnimo, la potencia
elctrica es menor que la potencia mecnica y la mquina se acelera nuevamente, repitindose

446

IX.14 Estabilidad de la mquina sincrnica

Figura 9.16 Carga de una mquina sincrnica acoplada a la red en condicin de vaco
todo el proceso indefinidamente o hasta que las prdidas produzcan un efecto amortiguador, tal
como se mostr en la seccin IX.9.
La oscilacin descrita anteriormente se representa analticamente mediante la ecuacin diferencial correspondiente al balance de par en el eje de la mquina:
"
#

2
2
E
V
e
1
d
Ve 1
d m
f
sin 2
= J 2 = Tm Te = Tm
(9.155)
Ta = J
sin
dt
dt
2 Xq Xd
m Xd
d
= m e
dt

(9.156)

En la figura 9.17 se presentan las respuestas temporales de la velocidad del rotor y del ngulo
de carga ante un escaln en el par mecnico de accionamiento. Se observa que las oscilaciones
se mantienen indefinidamente, tal como ocurre en un pndulo sin prdidas. Las oscilaciones de
la velocidad mecnica estn centradas en la velocidad sincrnica y las oscilaciones del ngulo
de carga estn centradas sobre el valor de ngulo 2 correspondiente a la nueva condicin de
equilibrio.
El ngulo de carga en el instante t = 0+ se conserva porque la inercia de la mquina acumula
energa cintica y la energa no puede variar instantneamente, a menos que se disponga de
una fuente de potencia infinita. La energa cintica de una masa rotante se evala a partir de la
siguiente expresin:
1
(9.157)
Wk = J m2
2
Debido a que la energa cintica Wk no puede variar instantneamente sin consumir potencia
infinita, la velocidad m no cambia en el primer instante. El ngulo de carga tampoco puede
variar instantneamente porque se obtiene al integrar la diferencia entre la velocidad mecnica
m y sincrnica e :
Z

(t) =

(m e ) d

(9.158)

447

Captulo IX Rgimen transitorio de la mquina sincrnica

Figura 9.17 Respuesta temporal de la mquina sincrnica sometida a un escaln de potencia en


el eje
Las oscilaciones mecnicas de la mquina sincrnica representadas mediante las ecuaciones
diferenciales 9.155 y 9.156 se mantienen indefinidamente sin la presencia de pares amortiguadores. El sistema es conservativo y todas las oscilaciones se deben al traspaso de energa entre la
inercia de la mquina y el sistema elctrico de potencia. El campo magntico rotatorio contina
girando a la velocidad sincrnica forzado por la barra infinita, pero las oscilaciones mecnicas
son mucho ms lentas4 .
Si se produce un escaln de par mecnico en un punto de operacin ms cercano al par mximo, la energa cintica acumulada en el rotor de la mquina durante la aceleracin, ocasiona
un aumento del ngulo de carga superior al punto de equilibrio. Al sobrepasarse el punto de
equilibrio, el par elctrico es mayor que el par mecnico y la mquina comienza a frenarse. Pero
si la mquina no se frena suficientemente rpido, el ngulo de carga aumenta y puede pasar el
segundo punto de equilibrio, la potencia elctrica es menor que la potencia mecnica y el rotor
contina acelerndose. Cuando se alcanza este punto crtico, se ha perdido el sincronismo entre
la mquina y el sistema elctrico de potencia. Al perder el sincronismo, no es posible entregar
potencia media diferente de cero al sistema y toda la energa entregada en el eje mecnico se
acumula como energa cintica en las masa rotantes. El rotor de la mquina se acelera y si no se
realizan las acciones correctivas necesarias, se produce el fenmeno denominado embalamiento.
Para evitar la prdida de sincronismo es posible incrementar rpidamente la fuerza electromotriz
4

Del orden del segundo o ms por oscilacin.

448

IX.14 Estabilidad de la mquina sincrnica

(a) Condicin de operacin estable

(b) Condicin de operacin inestable

Figura 9.18 Respuesta estable e inestable de la mquina sincrnica sometida a un escaln de


par mecnico
producida por el devanado de campo, inyectando mediante la excitatriz una corriente impulsiva
en este enrollado.
Las reas del grfico de par elctrico en funcin del ngulo de carga representan energa. La
diferencia entre el par elctrico y el par mecnico es el par acelerante sobre la mquina. La
integral del par acelerante sobre la mquina sincrnica en el intervalo comprendido entre dos
ngulos de carga diferentes, es igual a la variacin de energa cintica en el rotor:
Wk =

Z t
0

Pa ( )d =

Z t
0

Ta m d =

Z t
0

d
Ta d =
d

Z (t)
(0)

Ta ( )d

(9.159)

Para que la velocidad de la mquina regrese al valor inicial, es necesario que la evaluacin
de la expresin anterior resulte nula. En este caso no hay variacin neta de energa cintica y
la velocidad final de la mquina es igual a la velocidad inicial. Este mtodo se conoce como
criterio de reas iguales. La energa absorbida por el rotor, para alcanzar el punto de equilibrio,
debe ser regresada a la red elctrica para reducir la velocidad hasta su valor de sincronismo. Si
no es posible frenar la mquina hasta la velocidad sincrnica, el sistema es inestable y pierde
su capacidad de transmitir potencia. En la figura 9.18 se muestra el comportamiento estable e
inestable de la mquina sincrnica sometida a un escaln de par en el eje mecnico.
Las limitaciones operativas de los sistemas mecnicos impiden la aparicin repentina de escalones de par en el eje de la mquina. Es ms frecuente la ocurrencia de perturbaciones de la
red elctrica, tales como cambios sbitos de la tensin en la barra debidos a cortocircuitos bruscos, conexin o desconexin de nuevas cargas a la red, y prdidas o reenganche de las lneas
de transmisin. Estas perturbaciones alteran la caracterstica par elctrico en funcin del ngulo

449

Captulo IX Rgimen transitorio de la mquina sincrnica

Figura 9.19 Trayectoria del ngulo de carga durante el cortocircuito y despus del despeje de la
falla

de la carga de la mquina sincrnica, y mantienen constante la potencia mecnica en el eje del


rotor. El cortocircuito trifsico brusco en bornes de la mquina es una de las perturbaciones ms
severas que pueden aparecer sobre la mquina. Durante el perodo de duracin del cortocircuito,
toda la potencia inyectada en el eje mecnico se convierte en energa cintica, debido a que la
armadura no es capaz de transmitir potencia al sistema elctrico. Cuando desaparece la perturbacin y la mquina comienza a transmitir potencia a la red, el rotor se frena, pero si el ngulo
alcanza el valor mximo max , la velocidad se incrementa nuevamente, perdiendo el sincronismo. El tiempo mximo que es posible mantener el cortocircuito en bornes de la mquina, sin la
prdida del sincronismo una vez recuperada la capacidad de transmisin de potencia al sistema,
se conoce como tiempo crtico de despeje y define los tiempos de actuacin de las protecciones
e interruptores del sistema. En la figura 9.19 se presenta el diagrama par elctrico en funcin
del ngulo de carga de la mquina sincrnica durante el cortocircuito y en el tiempo posterior al
despeje de la falla, considerando que se restituye la capacidad de transmisin inicial.
El criterio de reas iguales permite la determinacin directa de los lmites de estabilidad de
la mquina sincrnica. Este mtodo se puede obtener a partir de una concepcin ms general,
utilizando los teoremas de estabilidad demostrados por Liapunov en el siglo XIX. El teorema de
Liapunov predice que si en un sistema de ecuaciones diferenciales del tipo:
dxi
= fi (x1 , x2 , , xn , t) ; i = 1, 2, , n
dt

(9.160)

existe una funcin derivable V (x1 , x2 , , xn ), denominada funcin de Liapunov, que satisface
en un entorno del origen de coordenadas, las siguientes condiciones:

450

IX.14 Estabilidad de la mquina sincrnica


1. V (x1 , x2 , , xn ) 0, y V (x1 , x2 , , xn ) = 0, solamente cuando xi = 0 ; i = 1, , n,
es decir, cuando la funcin de Liapunov V (x), tiene un mnimo estricto en el origen de
coordenadas;
2. y adems,
estable.

dV
dt

0, cuando t t0 , entonces el punto de equilibrio xi = 0, i = 1, , n, es

Para interpretar este teorema se puede observar que la funcin de Liapunov, encierra a las variables de estado del sistema de ecuaciones diferenciales dentro de una hipersuperficie, si esta
superficie decrece con el transcurso del tiempo, las variables de estado convergen al punto de
equilibrio. Si la hipersuperficie se mantienen constante a medida que transcurre el tiempo, el
sistema converge en un ciclo lmite donde las oscilaciones se mantienen indefinidamente.
Las ecuaciones diferenciales que rigen el comportamiento electromecnico de la mquina sincrnica cuando se desprecian los amortiguamientos y las prdidas, son:
J

d
d m
=J
= Tm Te = Tm k1 sin k2 sin 2
dt
dt

(9.161)

d
d( 0 )
(9.162)
=
= m 0 =
dt
dt
Para encontrar una funcin de Liapunov que cumpla con las condiciones impuestas por el teorema, se multiplica la expresin 9.161 por la velocidad mecnica, y se integran en el tiempo cada
uno de los trminos de potencia obtenidos, agrupndolos en un slo miembro:
V ( , ) =

Z t
0

d
d
d

Z t
0

Tm d +

Z t
0

(k1 sin + k2 sin 2 ) d =

1
1
= J 2 Tm k1 (cos cos 0 ) k2 (cos 2 cos 20 ) = Wk Wm + We (9.163)
2
2
En el intervalo de las variables de estado y , donde la funcin 9.163 es definida positiva
V ( , ) 0, la derivada con respecto al tiempo de la funcin de Liapunov es:


dV
dV d dV d
Tm Te
Tm + Te = 0 0
=
+
= J
(9.164)
dt
d dt
d dt
J
Como la hipersuperficie V ( , ) que encierra la trayectoria de las variables de estado de este
sistema, se mantiene constante a medida que transcurre el tiempo, la respuesta de la mquina es
oscilatoria no amortiguada. El sistema es estable en un ciclo lmite si se cumple que la funcin de
Liapunov 9.163 es definida positiva. Esta funcin de Liapunov coincide con el criterio de reas
iguales, porque fue determinada realizando el balance de energa del sistema. Si se incluye en
las ecuaciones diferenciales los trminos disipativos y se utiliza la misma funcin de Liapunov
indicada anteriormente, se obtiene que la funcin decrece con el tiempo dV
dt 0; y la mquina
alcanza asintticamente el punto de equilibrio = 0, = 0.
Aun cuando los balances totales de energa, determinan generalmente buenas funciones de Liapunov, el mtodo no est restringido en modo alguno a este tipo de funciones. Cualquier funcin
derivable, definida positiva, que se anule en el punto de equilibrio, es una posible funcin de

451

Captulo IX Rgimen transitorio de la mquina sincrnica


Liapunov. Si una funcin con estas caractersticas no satisface la condicin dV
dt 0, no se puede
afirmar que el sistema es inestable, es necesario comprobar otras funciones. Si alguna funcin
cumple con esta propiedad, se garantiza que el sistema es estable o asintticamente estable.
IX.15

D IAGRAMA

DE BLOQUES DE LA MQUINA SINCRNICA

Desarrollando explcitamente los trminos de la ecuacin 9.3 se obtiene la siguiente representacin de la mquina sincrnica en variables de estado:

Rf
Lq
Re
1
1

0
0

L
Ld
Ld f
Ld
Ld f

id
id
vd

Ldd

L
1

iq L
iq

RLqe
0 Lqd f
0 Lq 0 0
q

vq

p
=
+

0 0 1
i0
0 v0
0 RL00
0
0
i0
L0

1
1
Rf
Lq
if
if
vf
Re
0
0

L
L
Ld f

Ld f

Lf

df

(9.165)

De la representacin cannica anterior se puede obtener directamente cuatro funciones de transferencia de primer orden correspondientes a las variables de estado del sistema:
id =

1
Re

Ld
Re p + 1

iq =



Ld f
Ld f
Lq iq + R f
i f + vd
vf
Lf
Lf

1
Re
Lq
Re p + 1



Ld id + Ld f i f + vq

i0 =
1
Rf

"

(9.166)

(9.167)

1
R0

v0
L0
p
+
1
R0

Lf
Lf
Lq L f
Re id + iq + vd + v f
if = L
f
Ld f
Ld f
Ld f
Rf p + 1

(9.168)
#

(9.169)

En la figura 9.20 se presenta el diagrama de bloques construido a partir de las cuatro funciones de transferencia anteriores. En este diagrama las tensiones son las variables de control y las
corrientes son las variables de estado. Simulink del entorno Matlab o Scicos del entorno
Scilab son herramientas grficas que permiten la representacin de estos modelos en diagramas de bloques. La ventaja de estas herramientas reside en la capacidad de realizar modelos
complejos de sistemas sin requerir habilidades de programacin.
IX.16

S UMARIO

1. Durante su operacin, la mquina sincrnica es sometida a diversas condiciones transitorias que afectan su comportamiento y el del sistema de potencia. La evaluacin de estos
procesos dinmicos y transitorios puede en muchos casos ser desacoplada para simplificar el problema y obtener una solucin satisfactoria. Las constantes de tiempo mecnicas

452

IX.16 Sumario

Figura 9.20 Diagrama de bloques de la mquina sincrnica de polos salientes sin devanados
amortiguadores

453

Captulo IX Rgimen transitorio de la mquina sincrnica


y elctricas pueden ser diferentes y esto permite estudiar en algunos casos el problema
electromagntico considerando que la velocidad del rotor es prcticamente constante, o
determinar las variaciones de la velocidad y del ngulo de carga suponiendo que los enlaces de flujo se mantienen congelados entre dos instantes de tiempo, obteniendo de esta
forma la solucin del problema dinmico.
2. La tcnica de los autovalores y los autovalores o la Transformada de Laplace son herramientas capaces de resolver el problema de los transitorios electromagnticos, porque al
considerar que la velocidad mecnica es prcticamente constante 0,98 < m < 1,02, el
modelo resultante de la mquina sincrnica es lineal.
3. Cuando la mquina sincrnica es sometida a un cortocircuito brusco, las corrientes que
circulan por los devanados son varias veces mayores a las calculadas en esta condicin
pero en rgimen permanente. Esto se debe fundamentalmente a que los enlaces de flujo
no pueden variar instantneamente debido a que estn almacenando energa. Las corrientes por las bobinas del estator, del campo y de los devanados amortiguadores si existen,
deben adaptarse para mantener los enlaces de flujo previos a la perturbacin que estaban
siendo forzados por las tensiones aplicadas. En estas condiciones las inductancias se ven
reflejadas en aquellos devanados que estn acoplados y sus valores resultantes se reducen

definiendo las inductancias transitorias Ld y L f o las subtransitorias Ld , Lq y L f , cuando


estn presentes los devanados amortiguadores.
4. Debido a la gran diferencia entre la potencia que fluye en la armadura de la mquina y en
su devanado de campo, la seleccin de una base que permita una representacin adimensional til es ms compleja que en otros modelos del sistema. Una solucin conveniente
consiste en utilizar el flujo comn que acopla el campo con el eje directo como base de flujo. De esta forma las tensiones base del estator y del campo quedan relacionadas a travs
del cociente entre el nmero de vueltas de estas bobinas. Por otra parte seleccionar como
base de potencia la aparente monofsica de la mquina sincrnica, reduce parcialmente el
impacto de la gran diferencia de potencias que fluyen por ambos devanados.
5. Si se considera que los enlaces de flujo en el entrehierro de la mquina se mantienen
constantes durante un tiempo determinado, esto conduce a la posibilidad de analizar los
transitorios electromagnticos mediante mtodos aproximados que representan las variables elctricas mediante fasores. Estos fasores transitorios o subtransitorios permanecen
durante un tiempo que depende de las constantes de tiempo transitorias o subtransitorias
de las bobinas que mantienen los flujos atrapados.
6. Los procesos transitorios aceleran y frenan la mquina sincrnica, produciendo variaciones de la velocidad mecnica alrededor de la velocidad sincrnica y del ngulo de carga
en torno a su punto de equilibrio. Cuando estas oscilaciones son de pequea magnitud
es posible linealizar la ecuacin dinmica que rige su comportamiento. Al linealizar este modelo es posible utilizar la tcnica de polos y ceros para estudiar la estabilidad de
la mquina sincrnica ante pequeas oscilaciones. Ajustando redes compensadoras en el
circuito de campo y diseando apropiadamente los devanados amortiguadores es posible
controlar la frecuencia de estas oscilaciones y reducirlas a cero ms rpidamente.

454

IX.16 Sumario
7. Los devanados amortiguadores de la mquina sincrnica cumplen varias funciones de
gran importancia en la operacin, proteccin y control de la mquina sincrnica. Por una
parte permiten la amortiguacin de las oscilaciones mecnicas produciendo pares de induccin que se oponen a las variaciones de la velocidad mecnica con respecto a la velocidad sincrnica. Tienen la propiedad de ofrecer una proteccin5 al circuito de campo
durante los procesos subtransitorios, evitando que los flujos atrapados en el entrehierro
puedan inducir fuerzas electromotrices destructivas en esta bobina que posee un nmero de vueltas importante. Tambin permiten el arranque de la mquina sincrnica como
motor de induccin al comportarse como un rotor de jaula de ardilla mientras que la mquina no alcanza la velocidad sincrnica. El costo asociado a estas ventajas, consiste en
un incremento sustancial de las corrientes durante los procesos subtransitorios, mientras
que los flujos atrapados entre este devanado y las bobinas del estator no son disipados. La
presencia de los devanados amortiguadores tambin reduce la permeabilidad magntica e
incrementa por tanto los requerimientos de corriente de campo para compensar la cada
de la fuerza magnetomotriz.

8. La estimacin de los parmetros transitorios y subtransitorios de las mquinas sincrnicas


es un tema importante y su solucin requiere la aplicacin de mtodos matemticos de
procesamiento de seales transitorias entre los que destacan la respuesta en frecuencia,
la descomposicin de la seal en el tiempo y la Transformada rpida de Fourier6 entre
otros.

9. El anlisis de los desequilibrios a que son sometidas las mquinas sincrnicas es un problema cuya solucin detallada requiere modelos de gran complejidad pero que puede ser
parcialmente simplificado utilizando la transformacin a componentes simtricas. Aun
cuando la simetra de las ecuaciones de la mquina sincrnica no satisfacen las condiciones requeridas por esta transformacin 7 y los modelos de secuencia no estn desacoplados
su aplicacin aproximada ofrece un herramienta til para establecer rdenes de magnitud
de las corrientes transitorias o subtransitorias durante los desequilibrios.

10. Los anlisis de estabilidad de la mquina sincrnica requieren modelos dinmicos que
permitan evaluar las fluctuaciones de la velocidad y del ngulo de carga en grandes magnitudes. Actualmente la no linealidad de los modelos de la mquina sincrnica y su interrelacin con el sistema elctrico puede ser resuelta por la integracin numrica directa
de las ecuaciones diferenciales que rigen el comportamiento de la mquina8 o las condiciones de estabilidad pueden ser obtenidas aplicando el mtodo de Liapunov, el cual es
independiente de la solucin directa del problema.
5
6
7
8

Esta proteccin se debe a que durante los procesos subtransitorios los devanados amortiguados se comportan
como una jaula de Faraday, aislando parcialmente su interior de los campos electromagnticos externos.
Con mayor precisin el mtodo Short Fast Fourier Transform (SFFT) que permite aplicar la transformada de
Fourier a una seal cuyas frecuencias varan en el tiempo.
Las componentes simtricas desacoplan los sistemas cclicos o simtricos.
Determinacin de la trayectoria de las variables de estado para una perturbacin dada.

455

Captulo IX Rgimen transitorio de la mquina sincrnica


IX.17

E JEMPLO

RESUELTO

Ejemplo: Anlisis transitorio de la mquina sincrnica


Una mquina sincrnica de polos salientes de 5 MVA, 3 kV, 60 Hz, i f n = 100 A,se encuentra acoplada a una barra infinita consumiendo 3 MW con factor de potencia 0,7 inductivo. La reactancia
de dispersin de la mquina es de 0,15 en pu. La reactancia de dispersin de la bobina de campo
es 0,25 en pu. La reactancia del eje directo y cuadratura son 1,0 y 0,7 en pu respectivamente.
La constante de inercia de la mquina en por unidad es de 0,5 s. La mquina posee un devanado
amortiguador que produce el par nominal cundo el deslizamiento alcanza el 1 %. La dispersin
de los devanados amortiguadores es del 7 %. Las prdidas hmicas en el estator alcanzan el
0,1 % y en el rotor el 0,15 %. Determine:
1. Las ecuaciones completas de la mquina, expresando todos los parmetros en un sistema
coherente en por unidad.
2. La corriente instantnea de cortocircuito brusco sin considerar los efectos de los devanados amortiguadores y las resistencias.
3. El valor de la corriente transitoria y subtransitoria de cortocircuito brusco utilizando el
mtodo aproximado.
4. El valor de la corriente subtransitoria si la tensin de la barra infinita se deprime un 15 %,
mantenindose la posicin angular previa.
5. La frecuencia, amplitud y duracin de la oscilacin si la mquina se encuentra en el punto
de operacin definido en el enunciado y se incrementa instantneamente el par mecnico
en un 3 %.9
Solucin:
1. Las ecuaciones completas de la mquina, expresando todos los parmetros en un sistema
coherente en por unidad.
Para expresar las ecuaciones de la mquina en un sistema adimensional de unidades es
necesario determinar las inductancias del eje directo y del campo en valores fsicos. En
el estator se definen como bases la potencia aparente monofsica SB = VB IB , la tensin
base lnea-neutro VB y la velocidad angular base B :
SB =

3
5
rad
MVA ; VB = kV ; B = 2 60 = 377
3
s
3

Se obtienen directamente la corriente base IB , as como la impedancia e inductancia base


del estator ZB y LB respectivamente:
IB =
9

VB
ZB
SB
= 962,25 A ; ZB =
= 1,8 ; LB =
= 4,775 mH
VB
IB
B

Considere la presencia de los devanados amortiguadores.

456

IX.17 Ejemplo resuelto

Las inductancias Ld y Lmd en unidades fsicas se obtienen directamente de los datos del
problema:
Ld = Xd (pu) LB = 4,775 mH ; Lmd = (Xd X e ) LB = 4,058 mH
La inductancia Ld f se obtiene de la expresin de la fuerza electromotriz del campo en la
condicin de vaco:

m
Vn
E f = Vn = Ld f i f Ld f = m = 79,576 mH
if
3
3
La relacin entre la inductancia mutua Ld f y las inductancias de magnetizacin Lmd y Lm f ,
permite determinar el valor fsico de la inductancia de mangnetizacin del campo:
Lm f =

L2d f
Lmd

= 1,5603 H

La corriente base y la tensin base del campo se obtienen de la siguiente forma:


s
Lmd
VB IB
= 49,074 kA ; VBF =
IBF = IB
= 33,962 kV
Lm f
IBF
Las impedancias e inductancias base propias y mutuas son las siguientes:
ZBF =

VBF
VB
VBF
= 692,04 ; ZB FE =
=
= 35,29
IBF
IBF
IB

ZBF
= 1,8356 H ; LB FE = LB EF = 93,6 mH
B
Los parmetros de la mquina en el sistema adimensional de unidades son:
LBF =

Ld (pu) =
L f (pu) =
Lad (pu) =

Ld f
LB FE

Ld
= 1,0 ; Lq (pu) = Xq (pu) = 0,7
LB

Ld f
Lm f
+ X f (pu) = 1,1 ; Ld f (pu) =
= 0,85
LBF
LB EF

+ X a = Xd X e + X a = 0,92 ; Laq (pu) = Xq X e + X a = 0,62

Ldad = Xd X e = 0,85 ; Lqaq = Xq X q = 0,55 ; L f ad = Lm f =


Como las prdidas hmicas del estator son del 0,1 % de la potencia aparente nominal
son 0,3 % de la potencia base monofsica. De igual forma las prdidas en el circuito de
campo son 0,45 % en la base monofsica de potencia. La resitencia de los devanados
amortiguadores se puede determinar del par de induccin al deslizamiento 0,01 con la

457

Captulo IX Rgimen transitorio de la mquina sincrnica


aproximacin siguiente:
Tn

Vth2 s
s Vth2
0,01
Rr
=
= 0,00333 pu
Rr
Tn
3,0

Las resistencias en por unidad, expresadas en la base seleccionada son:


Re = 0,003 pu ; R f = 0,0045 pu ; Ra = 0,00333 pu
El modelo completo de la mquina es:


Ld f p
Ldad p Ldaq
vd
Re + Ld p Lq
vq Ld
R
+
L
p

Lqad
Lqaq p
e
q
df


v f = Ld f p
0
R f + L f p L f ad p
0


vad Ldad p
0
Lad f p Ra + Lad p
0
0
Laqq p
0
0
Ra + Laq p
vaq

Te Tm = 2H

vd
vq
vf
vad
vaq

0,003 + 1,0p
0,7
0,85p
0,003 + 0,7p
0,85
1,0
0,85p
0
0,0045 + 1,1p
0,85p
0
0,85p
0
0,55p
0

0,85p
0,85
id
iq
0,55
0,55p

if
0,85p
0

iad
0,0033 + 0,92p
0
0
0,0033 + 0,62p
iaq

id
iq
if
iad
iaq

d m
d m
= Ld f i f iq + (Ld Lq )id iq + Lqaq iad iq Ldad iaq id Tm = 1,0
dt
dt

2. La corriente instantnea de cortocircuito brusco sin considerar los efectos de los devanados amortiguadores y las resitencias.
La expresin 9.32 determina la corriente instantnea de una mquina sincrnica de polos saliente sometida a un cortocircuito brusco. Para obtener esta corriente se requiere
calcular la fuerza electromotriz del campo E f en las condiciones previas a la perturbacin

y la reactancia transitoria del eje directo Xd :


S=

p
0,6
P
= 0,8571 ; Q = + S2 P2 = 0,6121
=
cos
0,7

P jQ
= 0,6 j0,6121 = 0,8571 134,4
Ve
D = Ve + jXq Ie = 1,4282 j0,42 = 1,4890 16,38
Ie =

Id = |Ie | sin ( ) = 0,8571 sin( ) = 0,7565

E f = D + (Xd Xq ) Id = 1,4890 + 0,3 0,7565 = 1,7159


458

IX.17 Ejemplo resuelto

Xd (pu) = Ld (pu) = Ld
ia (t) =

2E f

L2d f
Lf

= 1,0

0,852
= 0,3432
1,1

"

1 1
1 1
1
1
1
cos( t + 0 ) + ( ) cos(2 t + 0 ) + ( + ) cos 0
2 Xd Xq
2 Xd Xq
Xd

ia (t) = 7,071 cos(377t + 0 ) + 0,817 cos(754 + 0 ) + 0,279 cos 0


3. El valor de la corriente transitoria y subtransitoria de cortocircuito brusco utilizando el
mtodo aproximado.
Las condiciones previas al cortocircuito en la aproximacin transitoria y subtransitoria
son:

Id = Ie sin( ) + = 0,7565 106,38


2
Iq = Ie cos( ) = 0,4030 163,62

E f = Ve + jXd Id + jXq Iq = 1,2190 16,38


L2dad
0,852
Xd = Ld
= 0,2147
= 1,0
Lad
0,92

Xq = Lq

L2qaq
0,552
= 0,7
= 0,2120
Laq
0,62

E f = Ve + jXd Id + jXq Iq = 1,1389 6,44


Durante el cortocircuito la tensin de armadura Ve es cero, las fuerzas electromotrices

transitoria E f y subtransitoria E f se mantienen durante los procesos transitorio y subtransitorios respectivamente y se pueden plantear los siguientes sistemas de ecuaciones:

E f = jXd Id + jXq Iq = 1,2190 16,38

Efd
1,2190 16,38

= 3,5521 106,38 ; Iq =
=0
Id =
=
j0,3432
jXq
jXd
q
2

2
Ie = Id + Iq = 3,5521

Efq

E f = jXd Id + jXq Iq = 1,1389 6,44

Id =

Iq =

Efq

jXd

Efd

jXq

1,121 16, 38
= 5,2250 106,38
j0, 2147

0,1966 106,38
= 0,9273 196,38
j0,2120
q
2

2
Ie = Id + Iq = 5,3067

459

Captulo IX Rgimen transitorio de la mquina sincrnica


4. El valor de la corriente subtransitoria si la tensin de la barra infinita se deprime un 15 %,
mantenindose la posicin angular previa.
En este caso, las condiciones previas a la perturbacin son las mismas, pero ahora la
perturbacin es una reduccin del 15 % de la tensin de armadura. De esta forma se tiene:

E f = Ve + jXd Id + jXq Iq = 1,1389 6,44

0,850 + j0,2147 Id 106,38 + j0,2120 Iq 196,38 = 1,1389 6,44

Id = Id 106,38 = 2,4272 106,38

Iq = Iq 196,38 = 1,7858 196,38 = 1,7858 16,38


q
2

2
Ie = Id + Iq = 3,0134

5. La frecuencia, amplitud y duracin de la oscilacin si la mquina se encuentra en el punto


de operacin definido en el enunciado y se incrementa instantneamente el par mecnico
en un 3 %.
La expresin 9.125 define el comportamiento dinmico de la oscilacin:
(s) =

Tm (0+)
Pe (0 )
2H 2
D
B s + B s +

El coeficiente de amortiguamiento se obtiene del par de induccin nominal del devanado


amortiguador al deslizamiento s = 0,01:
D=

Tn
= 100
sn

La potencia sincronizante se obtiene derivando parcialmente la potencia con respecto al


ngulo de carga en el punto de operacin previo a la oscilacin:
"
#

1
P(0 ) Ve E f
1
2
cos 20 = 2,158

=
cos 0 +Ve

Xq Xd
Xd
Los polos del polinomio del denominador de la expresin anterior determinan las frecuencias naturales y los respectivos amortiguamientos de las oscilaciones de la mquina
sincrnica sometida a pequeas perturbaciones:
q

P ( )

D D2 8H B e 0

8,9378 rad
s

=
s1,2 =
rad
91,06
4H
s

460

IX.18 Ejercicios propuestos


Por lo tanto:
(s) =

IX.18

E JERCICIOS

1
+
B Tm (0 )
Pe (0 )
2H 2
D
B s + B s +

0,03
(s + 8,9378)(s + 91,06)

PROPUESTOS

1. Una mquina sincrnica de polos salientes de 5 kVA, 416V, 60 Hz,i f n = 3 A, se encuentra


acoplada a una barra infinita entregando 3,5 kW con factor de potencia 0,9 capacitivo. La
reactancia de dispersin de la mquina es de 0,1 en pu. La reactancia de dispersin del
rotor en un 50 % mayor. Adems se conoce que:
Xd = 1,0 pu y Xq = 0,7 pu
La constante de inercia de la mquina en por unidad es de 1 s. La mquina posee un devanado amortiguador que produce el par nominal cundo el deslizamiento alcanza el 1 %.
El acoplamiento entre las bobinas del estator y los devanados amortiguadores es del 95 %.

a) Determine la mxima corriente de cortocircuito brusco, considerando que inicialmente la mquina se encuentra en el punto de operacin definido en el enunciado.
b) Compare el resultado anterior con las corrientes transitorias y subtransitorias que se
obtendran por el mtodo aproximado.
c) Determine la frecuencia, amplitud y duracin de la oscilacin si la mquina se encuentra en el punto de operacin definido en el enunciado y se incrementa instantneamente el par mecnico en un 5 %. Considere la presencia de los devanados
amortiguadores.

2. Una mquina sincrnica de polos salientes de 100 MVA, 13 kV, 60 Hz,i f n = 500 A, se encuentra acoplada a una barra infinita de tensin 0,95 en pu. consumiendo 70 MW y entregando 70 MVAR a la red, para lo cual se requieren 849 A de corriente de campo. La
reactancia del eje cuadratura es 0,7 pu. Determine:

a) La corriente instantnea de cortocircuito brusco a partir de las ecuaciones diferenciales que definen el comportamiento de la mquina, si en el instante inicial el rotor
se encuentra a 90 de la posicin del eje magntico de la fase.
b) El comportamiento de la mquina sincrnica si en el mismo punto de operacin
inicial se le desconecta el devanado de campo de forma instantnea.

461

Captulo IX Rgimen transitorio de la mquina sincrnica


c) La corriente transitoria aproximada de la mquina si la tensin de la barra se incrementa instantneamente hasta 1,05 en p.u.

3. Una mquina sincrnica de polos salientes posee las siguientes caractersticas y parmetros:
Sn
100 MVA
Xd
1,2 pu

Vn
10 kV
Xq
0,8 pu

fp
0,8 ind
X e
0,2 pu

f
60 Hz
X f
0,4 pu

vfn
500V
D
30 pu

ifn
1.000 A
H
1,5 s

a) Calcule la frecuencia y amplitud de las oscilacin de esta mquina conectada a una


barra infinita si se incrementa un 5 % el par mecnico de accionamiento.
b) Si la tensin de la barra infinita se deprime un 5 % y la mquina se encontraba previamente en su punto nominal, cul es la corriente transitoria y subtransitoria aproximada?
c) Determine el tiempo crtico que puede permanecer la mquina en cortocircuito si
antes de la perturbacin se encontraba en condiciones nominales, y las mismas se
restablecen posteriormente.
d) Represente el diagrama de bloques completo de la mquina sincrnica.

4. Una mquina sincrnica de polos salientes de 200 MVA, 15 kV,factor de potencia nominal
0,707 y 1,0 kA de corriente nominal de campo, tiene impedancias de 1,0 y 0,6 p.u. en los
ejes directo y cuadratura respectivamente. La mquina se encuentra motorizando una carga de 150 MW con la corriente de campo mxima. Repentinamente el devanado de campo
se cortocircuita en sus bornes. Determine:

a) La corriente en la bobina del campo y en la fase a del estator.


b) La tensin instantnea que aparece en bornes de la bobina de campo, si se abre repentinamente el circuito.

5. Una mquina sincrnica de polos salientes de 100 MVA, 10 kV, 60 Hz, , , i f n = 1 kA, se
encuentra acoplada a una barra infinita entregando 80 MW con factor de potencia 0,9 inductivo. La reactancia de dispersin de la mquina es de 0,2 en pu. La reactancia de
dispersin de la bobina de campo es 0,3 en pu. La reactancia del eje directo y cuadratura
son 0, 9 y 0, 6 en pu respectivamente. La constante de inercia de la mquina en por unidad
es de 0,75 s. La mquina posee un devanado amortiguador que produce el par nominal
cundo el deslizamiento alcanza el 2 %. El acoplamiento entre las bobinas del estator y

462

IX.18 Ejercicios propuestos


los devanados amortiguadores es del 95 %. Determine:

a) La corriente de armadura transitoria y subtransitoria por el mtodo aproximado, si la


tensin de la barra se deprime instantneamente un 10 %.
b) La corriente instantnea despus de la perturbacin definida en la pregunta anterior
sin considerar el efecto de los devanados amortiguadores.
c) La frecuencia, amplitud y duracin de la oscilacin si la mquina se encuentra en el
punto de operacin definido en el enunciado y se incrementa instantneamente el par
mecnico en un 5 %. Considere la presencia de los devanados amortiguadores.

6. Una mquina sincrnica de 5 kVA, 416V , 60 Hz, 1.800 rpm, f pn = 0,8 ind, 2 A de corriente nominal de campo, posee los siguientes parmetros:
Xd
0,9

Xq
0,5

X e
0,18

X f
0,24

Re
0,01

Rf
60

Determine:

a) La corriente instantnea en el campo y en la fase a despus de un cortocircuito brusco, considerando que previo a la perturbacin la mquina estaba entregando la potencia activa nominal a la red con una corriente de campo de 3 A.
b) La corriente transitoria despus de una cada del 10 % de la tensin nominal si la
mquina se encuentra en el punto nominal de operacin, considerando la presencia
de un devanado amortiguador cuyo acoplamiento con la armadura es de 90 %.
c) La corriente subtransitoria despus de una cada del 10 % de la tensin nominal, considerando para las condiciones iniciales el punto de operacin definido inicialmente.
d) Las frecuencias de oscilacin mecnica ante pequeas perturbaciones, considerando
que el devanado amortiguador produce el par nominal con un deslizamiento del 5 %
y que la constante de inercia H es de 1,0 s.

463

Captulo IX Rgimen transitorio de la mquina sincrnica

464

Bibliografa

A DKINS, B. & H ARLEY, R. G., The General Theory of Alternating Current Machines, Chapman and Hall, London, 1975.
A NDERSON , P. & F OUAD , A., Power System Control and Stability, The Iowa State University
Press, USA, 1977.
C ONCORDIA , C., Synchronous Machines: Theory and Performance, General Electric Company,
New York, 1951.
K IMBARK, E. W., Power System Stability: Synchronous Machines, vol. III, Ed. Dover Publications, Inc., New York, 1956.
KOSTENKO, M. P., & PIOTROVSKI, L. M., Mquinas elctricas, vol. II, Editorial Mir, Second
edition, Mosc, 1979.
L ANGSDORF, A. S., Theory of Alternating Current Machinery, Tata McGraw-Hill, Second Edition, New York, 1974.
M C P HERSON , G. & L ARAMORE , R. D., An Introduction to Electrical Machines and Transformers, John Wiley & Sons, Singapur, 1990.
PARK , R. H., Two Reaction Theory of Synchronous machines, Part. 1, AIEE Transactions,
vol. 48, pp. 716-730, 1929.
PARK , R. H., Two Reaction Theory of Synchronous machines, Part. 2, AIEE Transactions,
Vol. 52, pp. 352-355, 1933.
S AY, M. G., Introduction to the Unifield Theory of Electromagnetic Machines, Pitman Press,
London, 1971.
W HITE , D. C. & WOODSON , H. H., Electromechanical Energy Conversion, John Wiley &
Sons, New York, 1959.

465

Captulo IX Rgimen transitorio de la mquina sincrnica

466

ndice alfabtico

ngulo base, 419


ngulo de carga, 363, 364, 432
ngulos elctricos, 107
ngulos mecnicos, 107
acoplamiento mquina-sistema, 387
acoplamiento perfecto, 428
acortamiento de paso, 297
Adams, 348
amortiguamiento, 433
amortiguamiento crtico, 434
amplidina, 181
amplificador rotativo, 181
anillos deslizantes, 203
apertura de interruptores, 408
arco de fuego, 172
arco elctrico, 172
armnicas temporales, 288
armadura, 116, 141, 345
autovalores, 207, 408
autovectores, 207, 408
balance de energa, 49
balance de par, 447
balance de potencia, 23, 29
balance del par, 349
base de flujo, 454
bases, 224
bobinas reales, 348
campo, 16, 345

campo elctrico, 16
campo elptico, 346
campo magntico, 16
campo magntico rotatorio, 121
caracterstica par-deslizamiento, 218
caractersticas normalizadas, 236
carbones, 141
carga aislada, 387
cargas elctricas, 16
centrales elicas, 203
choppers, 176, 318
ciclo de carga, 224
ciclo lmite, 451
circuito equivalente, 376
circuito magntico ideal, 428
circuitos acoplados, 427
clase de aislamiento, 222
coeficiente de friccin, 349, 431
coeficiente de generacin, 153
coeficientes de dispersin, 427
coenerga, 49
coenerga en el campo, 66
colector, 116, 141
componente continua, 441
componentes simtricas, 207, 278, 443, 455
condiciones de contorno, 408
condiciones iniciales, 408
conexin compuesta, 154
conexin derivacin, 154
conexin independiente, 154

467

ndice alfabtico
conexin paralelo, 154
conexin serie, 154
conmutador, 141
conmutador mecnico, 116
conservativa en potencia, 125
conservativo, 52
constante de inercia, 431
constante de tiempo del campo, 417
constante de tiempo transitoria, 430
constantes de tiempo, 408, 454
control par-velocidad, 165
controlador de tensin, 386
controlador de velocidad, 386
convencin generador, 361
convencin motor, 361
conversin, 16
convertidor de frecuencia, 203
convertidor electromagntico elemental, 21
convertidor electromecnico, 49
convertidores magnetohidrodinmicos, 69
coordenadas primitivas, 348, 414, 424
corriente alterna, 345
corriente circulatoria, 148
corriente continua, 69
corriente de campo nominal, 370
corriente de magnetizacin modificada, 310
corriente nominal, 222, 368
corrientes de desplazamiento, 19
corrientes de Foucault, 174
corrientes homopolares, 278
corrientes instantneas, 414
cortocircuito brusco, 412
cortocircuitos, 408
criterio de reas iguales, 450
criterios de energa, 446
curva caracterstica, 25
datos de placa, 368
delga, 143
densidad de corriente, 19
densidades de energa, 15
desequilibrios, 443
deslizamiento, 213, 214, 434
deslizamiento nominal, 222
despeje de la falla, 450
desplazamiento virtual, 74

468

devanado amortiguador, 434


devanado auxiliar, 343
devanado de compensacin, 166
devanado imbricado, 146
devanado ondulado, 146
devanados amortiguadores, 454, 455
diagrama de bloques, 163
diagrama de crculo, 239
diagrama fasorial, 359, 411
dipolos magnticos, 16
dominio de la frecuencia, 408
ecuaciones de borde, 25
ecuaciones de contorno, 25
ecuaciones de frontera, 25
ecuaciones de ligazn, 25
ecuaciones de Maxwell, 18
ecuaciones internas, 25
Edison, 103, 201
efecto Joule, 175
efecto pelicular, 175
eje 0, 355
eje d, 354
eje de la potencia mecnica, 244
eje del par elctrico, 244
eje f, 355
eje mecnico, 103, 110
eje q, 355
ejes, 50
ejes elctricos, 50
ejes mecnicos, 50
embalamiento, 28
energa, 15
energa cintica, 431, 447
enrollado amortiguador, 433
ensayo de rotor bloqueado, 227
ensayo de vaco, 226
entrehierro, 104
envejecimiento, 222, 368
envolvente, 441
equilibrio de fuerzas, 22
Equivalente de Thvenin, 216
escobilla, 143
escobillas, 141, 357
esfuerzos mecnicos, 440
estabilidad, 454

ndice alfabtico
estabilidad de Liapunov, 446
estabilidad dinmica, 407
estator, 103
estimacin de los parmetros, 455
estimacin paramtrica, 229
Euler, 348, 446
excitatriz, 181, 407
factor de potencia nominal, 369
fallas, 408
fasores transitorios, 454
ferrocarriles, 176
flujo de remanencia, 160
flujo remanente, 159
flujos de dispersin, 357
frecuencia, 345
frecuencia naturale, 433
frenado regenerativo, 160, 179, 203
frenado reosttico, 180
freno, 29, 236
fuentes de corriente, 177
fuerza, 16
fuerza electromotriz transitoria, 429
fuerzas electromotrices subtransitorias, 437
funcin de estado, 53
funcin de Liapunov, 450
funcin definida positiva, 451
generador, 21, 29, 152, 236
grado de saturacin, 383
H, 431
hermtica, 125
hermitiana, 125, 127
hipersuperficie, 451
identidad, 209
impedancia de secuencia negativa, 445
impedancia operacional propia, 423
impedancia trmica, 222
inductancia base, 419
inductancia de alisamiento, 178
inductancia del rotor, 350
inductancia transitoria, 436
inductancias subtransitorias, 436, 454
inductancias transitorias, 415, 454
integracin numrica directa, 455

interaccin, 358
interacciones, 16
inversor mecnico, 152
jaula de ardilla, 434
lmites adicionales, 373
lmites de estabilidad, 446, 450
lmites de operacin, 368
lmites trmicos, 368
lnea neutra, 144, 146
leyes de Maxwell, 18
Liapunov, 450
lugares geomtricos, 239, 371, 424
mquina bifsica, 298
mquina de corriente continua, 116
mquina de induccin, 115
mquina elctrica, 50
mquina generalizada, 110, 357
mquina sincrnica, 115
mquina tetrafsica, 298
mquina trifsica, 122
mquinas homopolares, 69
mtodo de Liapunov, 455
mtodo Gauss-Newton, 231
mtodos aproximados, 454
mnimo estricto, 451
mnimos cuadrados, 229
matriz cclica, 210
matriz caracterstica, 409, 412
matriz de impedancia, 414
matriz de inductancias, 111, 349
matriz de par, 111, 349
matriz de resistencias, 111
matriz hermitiana, 355
matriz Hessiana, 231
matriz Jacobiana, 231
matriz simtrica, 209
metadina transformador, 181
metadinamos, 181
metadinas, 181
metros, 176
modelos en secuencia, 443
momento de inercia, 349
motor, 21, 29, 148, 233

469

ndice alfabtico
neutro, 357
no-holonmicos, 126
norte, 107
oscilaciones, 454
oscilaciones automantenidas, 415
oscilaciones mecnicas, 445
oscilograma, 440
pndulo fsico, 433
prdidas, 173
prdidas Joule, 172
prdidas mecnicas, 176, 432
par, 50
par acelerante, 349
par base, 225
par de induccin, 434
par elctrico nominal, 222
par generatriz, 148
par mximo, 218
par motriz, 148
par resistente, 349
par-velocidad, 160
parmetros, 206
pares de polos, 109
paso polar, 109
pequeas oscilaciones, 430, 454
perodo subtransitorio, 440
permeanza, 359
polinomio caracterstico, 410
polos auxiliares, 173
polos salientes, 343
por unidad, 224, 418
potencia, 124
potencia aparente nominal, 369
potencia base, 165, 224
potencia de induccin, 434
potencia nominal, 222
potencia reactiva, 364
potencia sincronizante, 433
Predictor Corrector, 446
principio de conservacin de la energa, 51
principio de los trabajos virtuales, 49, 62, 64
problema dinmico, 454
problema electromagntico, 454
procesamiento de seales, 455

470

puente rectificador, 176


puertos, 50
punto de equilibrio, 432
punto de equilibrio estable, 451
punto de operacin, 220
punto de operacin estable, 221
punto de operacin inestable, 221
rgimen continuo, 68
rgimen permanente, 359, 411
razn de conduccin, 179
reactancia transitoria, 429
receptividad, 180
recta del deslizamiento, 245
reduccin de Krn, 415
reguladores, 407
relacin de Lorenz, 17, 22
relaciones constitutivas, 18
reluctancia, 52, 358
rendimientos, 363
representacin adimensional, 454
respuesta homognea, 409
rotor, 103
rotor cilndrico, 358
rotor liso, 343
rueda de Faraday, 41
Runge-Kutta, 348, 446
saturacin, 169
secuencia cero, 208, 278, 357
secuencia negativa, 208, 278
secuencia positiva, 208, 278
segunda ley de Newton, 24, 66
separatriz, 147
series de Taylor, 433
sincronizador, 388
sincronizar, 345
sistema adimensional, 418
sistema de coordenadas dq0-f, 355
sistema de referencia, 20
sistema elctrico, 388
sistema electromecnico, 408
sistema equilibrado, 359
sistema mecnico, 408
solucin homognea, 413
solucin particular, 411

ndice alfabtico
soluciones no triviales, 410
soluciones temporales, 408
subtransitorio aproximado, 437
sur, 107
trmino de generacin, 67
trmino de transformacin, 67
temperatura mxima, 222
tensin base, 224
tensin nominal, 222, 368
teorema del valor inicial, 423
Tesla, 103, 201
tiempo base, 419
tiempo crtico, 450
tiempo de conmutacin, 170
trabajos virtuales, 74
transductores, 181
transformacin de Clark, 355
transformacin de Park, 354, 411
transformada de Laplace, 408, 411
transformada rpida de Fourier, 455
transitorio aproximado, 428
transitorios electromagnticos, 407, 454
transitorios electromecnicos, 407
tranvas, 176
tringulo de Potier, 386
triple frecuencia, 445
troceadores de tensin, 176
trolebuses, 176
valores nominales, 368
variables dq0, 355
variables primitivas, 355
vectores espaciales, 208, 351
velocidad base, 419
velocidad de operacin, 26
velocidad de sincronismo, 156
velocidad nominal, 370
velocidad sincrnica, 26, 156
vida til, 223
vida media, 222
Ward-Leonard, 316
Whestinghouse, 201
yugo, 53

471

También podría gustarte